Superpage
CNS & pituitary tumors


Copyright: 2005-2024, PathologyOutlines.com, Inc.

NeuroPath related: Jobs, Fellowships, Conferences, Cases, CME, Board Review

Related chapters: CNS nontumor

Editorial Board oversight: Jared T. Ahrendsen, M.D., Ph.D. (last reviewed April 2023), P.J. Cimino, M.D., Ph.D. (last reviewed April 2023), Chunyu Cai, M.D. (last reviewed December 2022)
Page views in 2024 to date: 28

Adamantinomatous craniopharyngioma
Definition / general
  • Adamantinomatous craniopharyngioma is a histologically benign, partially cystic epithelial neoplasm of the suprasellar or sellar region, resembling ameloblastoma or keratinizing and calcifying odontogenic cyst
Essential features
  • Always WHO grade 1
  • Tumor with palisading epithelium, wet keratin and stellate reticulum associated with surrounding gliosis and Rosenthal fibers
  • Shows CTNNB1 mutations and aberrant nuclear expression of beta catenin in up to 95% of cases (Acta Neuropathol Commun 2016;4:20)
Epidemiology
  • Incidence: low
  • Age: bimodal, with peaks at 5 - 15 years and 45 - 60 years; rare neonatal and fetal cases have been reported (Nat Rev Dis Primers 2019;5:75)
  • More common than papillary craniopharyngioma (even in adults)
  • Incidence rates are similar in males and females and between Caucasians and African Americans (Neurosurg Focus 1997;3:e1)
Sites
  • Suprasellar: frequently extends into neighboring structures
  • Rarely intranasal, sphenoid sinus, cerebellopontine angle and pineal region
  • Reference: J Neurol Surg Rep 2016;77:e121
Pathophysiology
  • Possibly arises from neoplastic transformation of ectodermal derived epithelial cell remnants of Rathke pouch and the craniopharyngeal duct
  • Misplaced odontogenic rests along pituitary stalk
  • Reference: Childs Nerv Syst 2005;21:622
Clinical features
  • Insidious, with a delay of approximately 1 - 2 years between initial symptoms and diagnosis (Orphanet J Rare Dis 2007;2:18)
  • Visual disturbances are more frequently observed in children
  • Endocrine deficiencies:
    • Growth hormone (GH) > luteinizing hormone (LH) / follicle stimulating hormone (FSH) > adrenocorticotrophic hormone (ACTH) > thyroid stimulating hormone (TSH)
  • Growth failure and delayed puberty in children
  • Diabetes insipidus (> adults)
  • Headache (most common, due to mass effect or hydrocephalus)
  • Cognitive impairment and personality changes (about 50% of patients)
  • Rare chemical meningitis with cyst rupture and spillage
Diagnosis
  • Best diagnostic clue is preoperative imaging (Front Endocrinol (Lausanne) 2011;2:70)
    • Usually extra-axial and suprasellar
    • Multilobulated and multicystic lesions
    • Variable in size, often > 5 cm
    • Recurrences may be massive
  • MRI:
    • High signal intensity on T1 weighted images
    • Heterogeneous enhancement
    • Fluid levels consistent with cystic components
  • CT:
    • Better than MRI at showing calcifications
Laboratory
  • Same for both types of craniopharyngiomas
  • Full pituitary endocrine workup is usually mandatory (Front Endocrinol (Lausanne) 2011;2:70)
  • Visual acuity and visual field assessment is also performed to show any deficits and rule out papilledema
Radiology description
  • MRI:
    • T1: solid regions are hypo- or isointense, cystic regions are hyperintense
    • Strong heterogeneous enhancement
    • Hyperintense on T2
  • CT:
Radiology images

Images hosted on other servers:

MRI suprasellar mass

Prognostic factors
  • 5 year survival excellent (Front Endocrinol (Lausanne) 2011;2:70)
  • Patients may be left with variable endocrinologic deficiencies
  • Cystic recurrence common after incomplete excision
  • Very rare malignant transformation
Case reports
Treatment
  • Gross total excision or subtotal resection followed by radiation therapy (Front Endocrinol (Lausanne) 2011;2:70)
  • Anatomic location, size, invasion of the nearby structures and the nature of the tumor determine surgical approach
  • Most common indication for surgery is neurologic compromise from tumor mass effect
  • In children, hypothalamic and endocrine dysfunction may develop before visual defects are noticed
  • Radiotherapy is indicated for treatment of residual tumor or recurrence
Gross description
  • Lobular and cystic tumor with calcifications
  • Cysts with dark "motor oil" fluid composed of cholesterol and hemorrhage
  • Irregular tumor interface with adjacent brain
  • Can be densely adherent to brain
Gross images

Images hosted on other servers:

Autopsy image

Microscopic (histologic) description
  • May appear well circumscribed
  • Cords, lobules, nodular whorls and trabeculae of well differentiated squamous epithelium bordered by palisading columnar epithelium
  • Peripheral cells surround looser plumper cells called stellate reticulum
  • Nodules of plump, anucleate squamous cells (ghost cells) and wet keratin
  • Intralobular whorl-like formations (Int Med Case Rep J 2020;13:123)
  • May have degenerative changes with cystic degeneration, calcifications and xanthogranulomatous reactions with giant cells
  • Piloid gliosis and Rosenthal fibers in adjacent brain
  • Rarely, melanin pigment
  • Microscopic brain invasion common with tongues of tumor extending into hypothalamic parenchyma
Microscopic (histologic) images

Contributed by Nelli S. Lakis M.D., M.Sc.
Xanthogranulomatous reaction Xanthogranulomatous reaction Xanthogranulomatous reaction

Xanthogranulomatous reaction

3 components

3 components

Tumor tongues Tumor tongues

Tumor tongues


Nuclear palisading Palisading nuclei

Nuclear palisading

Wet keratin Wet keratin

Wet keratin

Cystic degeneration

Cystic degeneration

Fibrosis

Fibrosis


Inflammatory infiltrate

Inflammatory infiltrate

Adamantinomatous epithelium

Adamantinomatous
epithelium

Stellate reticulum

Stellate reticulum

Beta catenin

Beta catenin

Virtual slides

Images hosted on other servers:

23 year old female

Cytology description
  • Cohesive clumps of well differentiated squamous epithelium (Int Med Case Rep J 2020;13:123)
  • Nodules of anucleate squamous cells
  • Macrophages, amorphous debris and calcifications
Positive stains
Negative stains
Molecular / cytogenetics description
  • Activating mutations of the WNT pathway gene CTNNB1 encoding beta catenin in almost all cases
Sample pathology report
  • Sellar / suprasellar region, suprasellar mass, endoscopic resection:
    • Adamantinomatous craniopharyngioma (see synoptic report)
Differential diagnosis
  • Epidermoid cyst:
    • Uniloculate with thin layer of keratinizing squamous epithelium and keratohyalin granules
  • Papillary craniopharyngioma:
    • No palisading, no wet keratin, no calcifications, no "motor oil" cystic fluid, no xanthogranulomatous reaction
    • Harbors a BRAF V600E mutation and is negative for beta catenin
  • Pilocytic astrocytoma:
    • Much greater cellularity than piloid gliosis, biphasic and may have eosinophilic granular bodies
  • Rathke cleft cyst with squamous metaplasia:
    • Intrasellar, squamous epithelium, as well as ciliated or mucus containing cells, no wet keratin, no calcifications
    • Beta catenin negative nuclei
  • Xanthogranuloma of sellar region:
    • Sellar region cholesterol clefts, lymphoplasmacytic infiltrates, marked hemosiderin deposits, fibrosis, multinucleated giant cells around cholesterol clefts, eosinophilic granular necrotic debris and accumulation of macrophages, no true epithelium and may be associated with Rathke cleft cyst leakage / rupture / hemorrhage (Brain Pathol 2017;27:377)
Board review style question #1

A 16 year old boy comes to the clinic because he has been experiencing headaches and vision disturbances. The headaches have worsened over this past year and also are associated with nausea. His head is imaged and a suprasellar mass with calcifications is identified. The mass is removed surgically and gross histological examination reveals the presence of cystic spaces that are filled with thick dark brown fluid. Which of the following is the most likely diagnosis?

  1. Adamantinomatous craniopharyngioma
  2. Epidermoid cyst
  3. Papillary craniopharyngioma
  4. Rathke cleft cyst
Board review style answer #1
A. Adamantinomatous craniopharyngioma

Comment Here

Reference: Craniopharyngioma-adamantinomatous
Board review style question #2
A 13 year old girl is brought to the clinic because she has been experiencing headaches for the past 8 months. She also complains of visual difficulty. A physical exam reveals the presence of papilledema. The mass is removed surgically and gross histological examination reveals a diagnosis of adamantinomatous craniopharyngioma. Which of the following features is associated with this diagnosis?

  1. BRAF V600E mutation
  2. Lack of nuclear palisading
  3. Nodules of anucleate squamous cells
  4. WHO grade 2
Board review style answer #2
C. Nodules of anucleate squamous cells

Comment Here

Reference: Craniopharyngioma-adamantinomatous

Anaplastic meningioma
Definition / general
  • WHO 2021 definition: a meningioma with overtly malignant cytomorphology that can
    • Resemble a carcinoma, melanoma or high grade sarcoma
    • Display markedly elevated mitotic activity (≥ 12.5 mitoses/mm2, ≥ 20 mitoses/10 high power fields [HPF] of each 0.16 mm2)
    • Harbor a TERT promoter mutation or
    • Have homozygous CDKN2A / CDKN2B deletion
  • CNS WHO grade 3
  • 1 - 3% of meningiomas
  • De novo (primary) or progression from a lower grade (1 or 2) meningioma (secondary) (Neuro Oncol 2018;20:1113)
Essential features
  • Meningioma with overtly malignant cytomorphology that can
    • Resemble a carcinoma, melanoma or high grade sarcoma
    • Display markedly elevated mitotic activity (≥ 12.5 mitoses/mm2, ≥ 20 mitoses/10 HPF of each 0.16 mm2)
    • Harbor a TERT promoter mutation or
    • Have homozygous CDKN2A / CDKN2B deletion
  • CNS WHO grade 3
  • Shows either histologic or immunohistochemical evidence of meningothelial differentiation
    • At least shows focal meningothelial whorls, psammoma bodies or nuclear pseudoinclusions
    • Immunohistochemistry: epithelial membrane antigen (EMA)+ (even focal), SSTR2A+, possible focal CK AE1 / AE3+, STAT6-
  • Recurrence in 50 - 94%
Terminology
  • Also called:
    • Malignant meningioma
    • Meningothelial sarcoma (not preferred)
ICD coding
  • ICD-O: 9530/3 - meningioma, malignant
    Epidemiology
    Sites
    Pathophysiology
    Etiology
    • Risk factors may be similar to risk factors for meningioma: ionizing radiations; neurofibromatosis type 2 (Eur J Epidemiol 2020;35:591)
    Clinical features
    Diagnosis
    • Based on imaging (CT; MRI) / biopsy / resection specimen
    Radiology description
    • MRI: contrast enhancing mass with possible necrotic areas (Neurochirurgie 2021;67:193)
    • MRI: contrast enhancing dural tail sign at the perimeter
    Radiology images

    Contributed by Valeria Barresi, M.D, Ph.D.
     Extra-axial, dural mass

    Extra-axial, dural mass



    Images hosted on other servers:

    MRI

    Prognostic factors
    Case reports
    Treatment
    • Surgery followed by fractioned radiotherapy, experimental chemotherapy or peptide receptor radionuclide therapy (Lancet Oncol 2016;17:e383)
    Gross description
    • Dural based and widely variable in size
    • May be well circumscribed or readily adherent to brain parenchyma
    • Gross necrosis can be present
    Frozen section description
    • Differential diagnosis versus other tumor types: at least focal presence of psammoma bodies, meningothelial whorls or nuclear pseudoinclusions
    • Differential diagnosis versus CNS WHO grade 1 meningiomas: presence of mitoses
    Intraoperative frozen / smear cytology images

    Contributed by Valeria Barresi, M.D, Ph.D.
    Lobular architecture

    Lobular architecture

    Whorls

    Whorls

    Nuclear pseudoinclusions

    Nuclear pseudoinclusions

    Mitoses

    Mitoses


    Whorls

    Whorls

    Psmmoma body

    Psmmoma body

    Mitoses

    Mitoses

    Microscopic (histologic) description
    • May have frank malignant cytology resembling a carcinoma, melanoma or high grade sarcoma
    • Mitotic index: ≥ 12.5 mitoses/mm2, ≥ 20 mitoses/10 HPF of each 0.16 mm2
    • At least focal meningothelial whorls and nuclear pseudoinclusions are useful to establish meningothelial origin
    • Psammoma bodies may be present (World Neurosurg 2021;149:e877)
    • Necrosis and brain invasion may be present
    Microscopic (histologic) images

    Contributed by Valeria Barresi, M.D, Ph.D.
    Mitoses

    Mitoses

    Malignant morphology

    Malignant morphology

    Whorls

    Whorls

    Necrosis

    Necrosis


    EMA

    EMA

    CK AE1 / AE3

    CK AE1 / AE3

    H3K27me3

    H3K27me3

    Cytology description
    Positive stains
    Negative stains
    Molecular / cytogenetics description
    Sample pathology report
    • Brain, parasagittal mass:
      • Diagnosis meningioma, subtype anaplastic, CNS WHO grade 3 (see comment)
      • Comment: Meningothelial neoplasia showing focal meningothelial whorls, patternless architecture, brain invasion and spontaneous necrosis. Mitotic index: 15 mitoses/mm2.
    Differential diagnosis
    Board review style question #1

    A dural based mass is found at the brain convexity. Histological examination shows a tumor with malignant morphology, mitotic index of 15 mitoses/mm2, EMA+, focal CK AE1 / AE3+, SSTR2A+, STAT6-. Which is the most likely diagnosis?

    1. Anaplastic meningioma
    2. Melanoma
    3. Metastasis of carcinoma
    4. Solitary fibrous tumor
    Board review style answer #1
    A. Anaplastic meningioma

    SSTR2A is the most sensitive and specific marker for meningioma, while STAT6 is the most sensitive and specific marker for solitary fibrous tumor. Focal CK AE1 / AE3 immunostaining can be found in anaplastic meningioma, whereas metastatic carcinoma features widespread CK AE1 / AE3 immunostaining.

    Comment Here

    Reference: Anaplastic meningioma
    Board review style question #2
    A mass is found in the left cerebral ventricle. Histological examination shows a tumor with focal meningothelial whorls, nuclear pseudoinclusions, mitotic index of 13 mitoses/mm2, GFAP-, Olig2-, EMA+, CK AE1 / AE3-. Which is the most likely diagnosis?

    1. Anaplastic meningioma
    2. Choroid plexus carcinoma
    3. Ependymoma
    4. Glioblastoma
    Board review style answer #2
    A. Anaplastic meningioma

    Anaplastic meningiomas can also be found in the cerebral ventricles. The presence of meningothelial whorls nuclear inclusions and EMA staining indicate meningothelial derivation. Other entities should be considered in the differential diagnosis, including glioblastoma (which is GFAP+, Olig2+), ependymoma (which is GFAP+, Olig2- and exhibits EMA dot-like staining) and choroid plexus carcinoma (which is more common in children and features widespread, strong CK AE1 / AE3 immunostaining).

    Comment Here

    Reference: Anaplastic meningioma

    Anatomy & histology-pineal gland
    Definition / general
    • Also called epiphysis, pineal body
    • Between superior colliculi at base of brain; 100 - 180 mg
    • Develops at month 2 of gestation as diverticulum in diencephalic roof of third ventricle
    • Replaced by connective tissue after puberty
    • Produces melatonin, which helps regulate circadian rhythms
    Diagrams / tables

    Images hosted on other servers:

    Location in brain

    Gross description
    • Shaped like a pine cone, midline, attached to posterior end of roof of third ventricle in front of cerebellum, 1 cm long, red gray
    Gross images

    Images hosted on other servers:

    Local anatomy (horse)

    Microscopic (histologic) description
    • Loose neuroglial stroma with nests of pineocytes containing well defined neurosecretory (melatonin) granules
    • Also astrocytes
    • Has features of photoreceptors and concretions ("brain sand")
    Positive stains

    Angiocentric glioma
    Definition / general
    • Superficial cerebrocortical tumour with features of infiltrating astrocytoma and ependymoma
    • Relationship to ependymoma is unclear
    Epidemiology
    • Primarily children and young adults
    • Wide age range (2 to 70 years, mean 17 years)
    • Affect both sexes equally
    Clinical features
    • Seizures are characteristic
    Grading
    • WHO grade 1
    • Criteria for higher grade lesions are unclear
    Radiology description
    • Well delineated, solid, hyperintense, nonenhancing cortical lesions
    • Stalk-like extension to adjacent ventricle is diagnostic
    Radiology images

    Images hosted on other servers:

    Axial MR images

    Axial T1 and T2 weighted MR images

    Hyperintense cortically based mass

    Prognostic factors
    • Generally favorable with rare recurrence
    Case reports
    Treatment
    • Surgical excision
    • Role of chemo or radiotherapy is unclear
    Gross description
    • Ill defined, firm
    Microscopic (histologic) description
    • Infiltrative, monomorphous, bipolar spindled cells arranged in angiocentric pattern about cortical blood vessels
    • Also ependymoma-like pseudorosettes, subpial palisading, accumulation of tumor cells, miniature schwannoma-like nodules
    • Usually no mitoses, no vascular proliferation, no necrosis
    • Mitotically active lesions are associated with increased risk of recurrence
    Positive stains
    • EMA (dot-like microlumina, focal surface), variable GFAP
    Differential diagnosis

    Arachnoid cyst
    Definition / general
    Essential features
    • Nonneoplastic, intracranial CSF filled spaces lined by meningothelial cells and an outer collagenous membrane
    • Most primary developmental arachnoid cysts occur in the middle frontal fossa due to the splitting of arachnoid membranes (J Neuropathol Exp Neurol 1981;40:61)
    • Meningothelial cells are positive for epithelial membrane antigen (EMA)
    Terminology
    • Meningeal cyst
    ICD coding
    • ICD-10: G93.0 - cerebral cysts
    • ICD-11: 8D67 - intracranial arachnoid cyst
    Epidemiology
    Sites
    • Arise within both cranial and spinal meninges
    • Most are supratentorial and found in the middle fossa (J Neurosurg 2013;118:222)
    • Other sites include retrocerebellar, convexity, cerebellopontine angle and spinal cord (J Neurosurg 2013;118:222)
    Pathophysiology
    • Primary developmental cysts occur due to the splitting of arachnoid membranes in utero, resulting in abnormal collections of cerebrospinal fluid (CSF) (J Neuropathol Exp Neurol 1981;40:61)
    • Secondary cysts are less common and often occur after trauma, infection or surgery (Case Rep Orthop 2015;2015:250710)
    • Mutation of the FOXC2 gene has been reported in familial forms
    Clinical features
    Diagnosis
    • Computed tomography (CT) and magnetic resonance imaging (MRI) for radiologic assessment
    • Surgical resection is required for a definitive diagnosis
    Radiology description
    • MRI is the diagnostic procedure of choice
    • Arachnoid cysts show low signal intensity on diffusion weighted imaging (DWI) and fluid attenuated inversion recovery (FLAIR) (Tani Girisim Radyol 2003;9:418)
    • No enhancement
    Radiology images

    Contributed by Saman Seyed Ahmadian, M.D.
    T2 FLAIR MRI

    T2 FLAIR MRI

    T2 MRI

    T2 MRI



    Images hosted on other servers:

    T2 MRI

    Case reports
    Treatment
    • Surgery if symptomatic
    Gross description
    Microscopic (histologic) description
    • Cyst wall is composed of a single layer of meningothelial cells and an outer collagenous membrane
    • Meningothelial cells often partially denuded and may not always be recognizable
    • Rare foci of meningothelial hyperplasia with or without psammoma bodies
    • Focal inflammation (rare)
    • Reference: Love: Greenfield's Neuropathology, 9th Edition, 2015
    Microscopic (histologic) images

    Contributed by Saman Seyed Ahmadian, M.D.
    Cyst lining

    Cyst lining

    Meningothelial hyperplasia

    Meningothelial hyperplasia

    Calcification

    Calcification

    No apparent meningothelial cells

    No apparent meningothelial cells

    EMA

    EMA

    Virtual slides

    Images hosted on other servers:
    Arachnoid cyst, resection

    Arachnoid cyst, resection

    Positive stains
    • Meningothelial cells are positive for EMA
    Negative stains
    Electron microscopy description
    Molecular / cytogenetics description
    Sample pathology report
    • Cyst wall, excision:
      • Arachnoid cyst (see comment)
      • Comment: The histologic section shows a cystic lesion composed of a single layer of meningothelial cells with an outer layer of delicate fibrous tissue. The meningothelial cells are positive for EMA by immunohistochemistry, which confirms the diagnosis.
    Differential diagnosis
    Board review style question #1

    A 71 year old patient with altered mental status had a 7.6 cm cystic lesion in the left frontoparietal convexity. The cystic lesion was excised. What immunohistochemistry confirms the diagnosis?

    1. Cytokeratin
    2. EMA
    3. GFAP
    4. Synaptophysin
    Board review style answer #1
    B. EMA. The histologic section shows a cystic lesion with a single layer of meningothelial cells with flattened nuclei and delicate fibrous tissue suggestive of an arachnoid cyst. The meningothelial cells are positive for EMA.

    Comment Here

    Reference: Arachnoid cyst

    Astroblastoma, MN1 altered
    Definition / general
    • Rare ( < 3% of primary brain gliomas), compact glial neoplasm with perivascular pseudorosettes formed of GFAP+ cells arranged around central, often sclerotic, blood vessels
    • Controversial entity - relationship with ependymoma is not clear
    Epidemiology
    • Usually children and young adults, median age 11 years, range 1 - 58 years
    Clinical features
    • Features of astrocytoma and ependymoma; expresses nonfibrillar form of GFAP (so PTAH-)
    Grading
    • No WHO grade assigned for astroblastoma or anaplastic astroblastoma
    Radiology description
    • Discrete supratentorial cerebral, often superficial, contrast enhancing mass
    • Cystic change is common
    Radiology images

    Images hosted on other servers:

    Huge well demarcated mass in frontal lobe

    Prognostic factors
    Case reports
    Treatment
    • Resection (adequate for well differentiated tumors), more aggressive treatment needed for malignant tumors
    Gross description
    • Well circumscribed, peripheral, cerebral hemispheric masses
    • Firm, often cystic
    Microscopic (histologic) description
    • Well circumscribed with discrete pushing borders, occasionally infiltrative in high grade lesions
    • Perivascular pseudorosettes resembling ependymoma but with thick processes from cell body to adventitia of vessel
    • Also vascular hyalinization, little fibrillar background
    • Limit diagnosis to tumors in which these features predominate (other tumors have these features focally)
    • High grade astroblastomas have hypercellular and mitotically active regions, often with vascular proliferation or necrosis with pseudopalisading; rare features are signet ring cells (Neuropathology 2002;22:200)
    Microscopic (histologic) images

    Case #312


    EMA

    GFAP

    Trichrome

    Positive stains
    Negative stains
    Electron microscopy description
    • Abundant intermediate filaments forming bundles in tumor cytoplasm, membrane junctions and external lamina when cells are in contact with collagen fibers (Surg Neurol 1991;35:116)
    Molecular / cytogenetics description
    Differential diagnosis

    Astrocytoma, IDH mutant
    Definition / general
    • IDH1 / IDH2 mutated, diffusely infiltrating glioma, most often with concurrent TP53 or ATRX mutations and without 1p / 19q codeletion
    • Can be graded CNS WHO grade 2, 3 or 4
    Essential features
    • IDH1 codon 132 or IDH2 codon 172 mutated, diffusely infiltrating glioma without 1p / 19q codeletion and usually with TP53 or ATRX mutations
    • In the absence of 1p / 19q codeletion, a component that morphologically resembles oligodendroglioma is compatible with this diagnosis
    • Can be designated CNS WHO grade 2, 3 or 4 depending on presence of mitotic activity, nuclear atypia, pleomorphism, necrosis, microvascular proliferation or CDKN2A / CDKN2B homozygous deletion
    • Significant proliferative activity is consistent with a CNS WHO grade 3 diagnosis
    • Presence of either necrosis, microvascular proliferation or CDKN2A / CDKN2B homozygous deletion is consistent with a CNS WHO grade 4 diagnosis
    Terminology
    • Astrocytoma, IDH mutant, CNS WHO grade 2; previously designated diffuse astrocytoma, IDH mutant
    • Astrocytoma, IDH mutant, CNS WHO grade 3; previously designated anaplastic astrocytoma, IDH mutant
    • Astrocytoma, IDH mutant, CNS WHO grade 4; previously designated glioblastoma, IDH mutant
    • Astrocytoma, IDH mutant, CNS WHO grade 4; historically referred to as secondary glioblastoma
    ICD coding
    • ICD-11:
    Epidemiology
    • Age of diagnosis is typically younger than glioblastoma, IDH wild type, with higher grade tumors occurring more often in older patients (third or fourth decade for grade 2 or 3, versus fifth decade for grade 4) (Acta Neuropathol 2015;129:867)
    • M > F, within grade 2 and grade 3 tumors (N Engl J Med 2015;372:2481)
    • Majority of tumors are sporadic
    • A small percentage are associated with familial cancer syndromes, such as neurofibromatosis, tuberous sclerosis and Li-Fraumeni syndrome (Neuro Oncol 2014;16:896)
    Sites
    Pathophysiology
    • Cell of origin is unknown, although the commonality of IDH mutation across IDH mutant astrocytoma and oligodendroglioma suggests a common histogenesis in these tumors; similarly, single cell sequencing of IDH mutant gliomas suggests this as well (Acta Neuropathol 2009;118:469, Science 2017;355:eaai8478)
    Etiology
    Clinical features
    • Commonly, gradual onset of symptoms
    • May present incidentally in work up for headache or following trauma (Lancet Oncol 2017;18:e315)
    • Seizures are a common symptom in cerebral hemispheric lesions
    • Changes in behavior or personality, especially in frontal lobe tumors
    • Uncommonly, site dependent neurological deficits
    • Manifestations of increased intracranial pressure
    • Often present after a clinical history of a few months of neurologic symptoms
    Diagnosis
    • MRI with contrast is the preferred imaging modality
    • Diagnosis is by biopsy or surgical resection
    Radiology description
    • CT:
      • Expanding, intra-axial, poorly defined mass of low density
      • Variable calcification may be seen
      • Contrast enhancement and central hypodensity due to necrosis, occur with higher grades
    • MRI:
      • T1 hypodensity and T2 hyperintensity
      • T2 hyperintensity with relative FLAIR sequence hypointensity (T2 FLAIR mismatch) is a relatively suggestive indication of IDH mutant astrocytoma (Clin Cancer Res 2017;23:6078)
      • Distortion and enlargement of involved areas, including associated cortical ribbon
      • Contrast enhancement is typically present in higher grade tumors (J Neurooncol 2019;141:327)
      • Ring-like enhancement around central necrosis typical of grade 4
    Radiology images

    Contributed by John DeWitt, M.D., Ph.D. and Meaghan Morris, M.D., Ph.D.

    MRI axial T1 FLAIR

    MRI axial T1 precontrast

    MRI axial T1 postcontrast

    MRI axial T1 FLAIR

    MRI axial T1 postcontrast


    MRI coronal T1 precontrast

    MRI coronal T1 postcontrast

    Bright on FLAIR

    Contrast enhancing

    Prognostic factors
    Case reports
    Treatment
    • Complete resection as extensively as is safely possible (Neuro Oncol 2015;17:868)
    • Chemotherapy (temozolomide) or radiation therapy
    Gross description
    • Ill defined neoplasm with blurring of gray-white junction and expansion of the infiltrated brain areas
    • Variable textures: firm, soft, gelatinous, granular, depending on CNS WHO grade (cellularity, necrosis)
    • Variable microcystic change imparting a spongy appearance
    • Variable calcification with a gritty sensation
    • Typically invades surrounding brain without overt tissue destruction
    • Expands native structures and can give a mass-like appearance on cut section
    • Areas of granularity or softening may be present
    • Soft gray-tan tissue with variable yellow-tan necrotic material
    • Often fragmented
    • Interface of tumor with brain parenchyma is indistinct
    Gross images

    Images hosted on other servers:

    Glioma with cystic spaces in the cerebral hemisphere

    Frozen section description
    • Smear done at the time of frozen section will show astrocytic appearing tumor cells with oblong irregular nuclei with varying degrees of atypia and glial processes
    • High grade nuclear features and mitotic activity may be observed on frozen section but necrosis or microvascular proliferation (features of glioblastoma) should not be present
    • Cellular morphology can be highly variable
      • Often predominantly tumor cells with oval hyperchromatic nuclei in a fibrillary background
      • Variably present, larger cells and pleomorphism
      • Variable quantity of cells with eccentric nuclei and glassy eosinophilic cytoplasm (gemistocytes)
      • Some show predominantly small cells with little pleomorphism and scant cytoplasm
    • Sections are hypercellular showing infiltrating neoplastic cells with edema
    • Variably present mitotic figures, necrosis and microvascular proliferation
    • Vascular thromboses and myxoid background may be present
    • Smear most commonly shows predominantly smaller cells with fine fibrillar processes, elongated nuclei, nuclear atypia and may show mitotic figures
    Intraoperative frozen / smear cytology images

    Contributed by John DeWitt, M.D., Ph.D. and Meaghan Morris, M.D., Ph.D.

    Astrocytoma, IDH mutant, CNS WHO grade 3

    Hypercellular tissue with edema

    Microvascular proliferation

    Smear

    Microscopic (histologic) description
    • Diffusely infiltrating tumor cells with oval to elongated astrocytic nuclei and varying appearance of tumor cytoplasm and fibrillar glial processes (Acta Neuropathol 2015;129:789)
    • At the periphery, tumor cells may infiltrate in a diffuse single cell pattern, often with entrapped neurons and axons
    • Cellular morphology is variable, even within a single tumor
    • Commonly there is a mix of cells with elongated nuclei and fine fibrillar processes, cells with eccentric nuclei and glassy eosinophilic cytoplasm (gemistocytes), larger pleomorphic cells and small cells with scant cytoplasm
    • May show oligodendroglioma-like areas
    • Myxoid background and microcyst formation may be present
    • Variable mitotic activity, cellularity and nuclear atypia depending on CNS WHO grade
    • In small biopsy specimens, the presence of 1 mitosis may be sufficient for a CNS WHO grade 3 diagnosis, while the presence of a few mitotic figures in a large resection would not be sufficient for grade 3 designation (Acta Neuropathol 2020;139:603)
    • Presence of necrosis or microvascular proliferation would be consistent with a CNS WHO grade 4 designation
    Microscopic (histologic) images

    Contributed by Eman Abdelzaher, M.D., Ph.D., John DeWitt, M.D., Ph.D. and Meaghan Morris, M.D., Ph.D.
    Diffuse astrocytoma, NOS: low power Diffuse astrocytoma, NOS: medium power

    CNS WHO grade 2 astrocytoma, NOS

    Subpial accumulation

    Subpial accumulation

    Diffuse astrocytoma, NOS: high power

    CNS WHO grade 2 astrocytoma, NOS

    Perineuronal satellitosis Perineuronal satellitosis

    Perineuronal satellitosis


    Perivascular accumulation

    Perivascular accumulation

    Mitotically active

    Pleomorphic hypercellular astrocytic appearance

    Gemistocytic histology


    Variable morphology and necrosis

    Microvascular proliferation

    Entrapped neurons

    R132H IDH1

    ATRX

    p53


    Ki67

    Olig2

    IDH

    ATRX

    p53

    Ki67

    Cytology description
    Positive stains
    Negative stains
    • ATRX
      • Loss of nuclear ATRX is typical of diffuse astrocytomas, not oligodendrogliomas or reactive gliosis (Front Oncol 2017;7:236)
      • Strong nuclear expression in nonneoplastic vasculature and cells serves as an internal control
    • Keratins (although cocktails may show cross reactivity)
    • CAM5.2, CD45, CD20, MelanA
    Molecular / cytogenetics description
    • IDH1 or IDH2 mutation necessary for the diagnosis, with R132H IDH1 variant seen in > 90% of cases (Acta Neuropathol 2009;118:469)
    • Tumors negative for R132H IDH1 immunohistochemistry require IDH1 / IDH2 sequencing to determine IDH status (Neuro Oncol 2017;19:1640)
    • p53 mutation and ATRX promoter mutation nearly always present (typically absent in oligodendroglioma, IDH mutant and 1p19q codeleted) (N Engl J Med 2015;372:2481)
    • Gain of chromosome 7
    • Usually lack TERT promoter mutations
    • Presence of CDKN2A / CDKN2B homozygous deletion portends a worse prognosis and is sufficient for a CNS WHO grade 4 designation regardless of histologic features present (Acta Neuropathol 2018;136:153)
    Sample pathology report
    • Brain, frontal lobe, biopsy:
      • Integrated diagnosis: astrocytoma, IDH mutant, CNS WHO grade 4
      • Histological diagnosis: astrocytoma with elevated proliferative activity and necrosis
      • WHO histological grade: 4
      • Molecular information:
        • IDH1: mutant (R132H immunohistochemistry)
        • ATRX: nuclear expression lost (consistent with mutant)
        • p53: many positive cells (immunohistochemistry; consistent with mutant)
    Differential diagnosis
    • Normal brain:
      • Beware of thick sections, no nuclear atypia
      • IDH1 negative, p53 negative
    • Demyelinating disease:
      • Not infiltrative, numerous macrophages (CD68 positive)
      • IDH1 negative
    • Pilocytic astrocytoma:
      • Circumscribed and contrast enhancing, histologically compact biphasic architecture (alternating piloid and spongy areas)
      • IDH1 negative
    • Reactive gliosis:
      • Evenly distributed hypertrophic astrocytes, IDH1 negative
    • Glioblastoma, IDH wild type:
      • High grade infiltrative glial neoplasm with astrocytic differentiation, nuclear atypia, pleomorphism, elevated mitotic activity and necrosis or microvascular proliferation
      • IDH mutation is not present by immunohistochemistry or sequencing
    • Oligodendroglioma, IDH mutant and 1p / 19q codeleted:
      • Infiltrating glioma composed of round cells resembling oligodendrocytes, hyperchromatic rounded nuclei, perinuclear halos, fine branching vasculature, scattered calcifications
      • Grade 3 tumors may have increased mitotic activity, variable microvascular proliferation and variable necrosis
      • IDH1 / IDH2 mutation present by immunohistochemistry or sequencing and whole arm codeletion of chromosomes 1p and 19q must be present by molecular testing
      • ATRX alterations and TP53 mutations are typically absent (Acta Neuropathol 2012;124:615, N Engl J Med 2009;360:765)
    • Lymphoma:
      • Parenchymal lymphomas in the central nervous system are typically diffuse large B cell lymphomas, which lack the fine fibrillar cell processes typical of glial cells
      • Diffuse large B cell lymphoma in the central nervous system often shows a perivascular tumor distribution
      • Positive for CD45 and CD20 but negative for GFAP and Olig2 by immunohistochemistry
    • Metastatic disease:
      • Morphology varies by the site of origin
      • However, typically they lack the fine fibrillar processes found in glial tumors
      • Generally are negative for Olig2 and GFAP by immunohistochemistry
    Board review style question #1
    Which of the following is true about IDH mutant astrocytoma?

    1. Codeletion of chromosomes 1p and 19q is a characteristic molecular alteration
    2. IDH mutant astrocytomas have a significantly better prognosis than IDH wild type tumors
    3. IDH protein expression is detected in astrocytic tumors only
    4. Malignant progression to higher grades does not occur
    5. Retained nuclear ATRX is typical of IDH mutant astrocytomas
    Board review style answer #1
    B. IDH mutant astrocytomas have a significantly better prognosis than IDH wild type tumors

    Comment Here

    Reference: Astrocytoma, IDH mutant
    Board review style question #2

    Which of the following is true about the entity in the figure above presenting in a 50 year old man?

    1. If R132H IDH1 immunohistochemistry is negative, no further testing is necessary
    2. If R132H IDH1 immunohistochemistry is positive, ATRX staining of tumor cells is expected to be lost
    3. If R132H IDH1 immunohistochemistry is positive, ATRX staining of tumor cells is expected to be retained
    4. If R132H IDH1 immunohistochemistry is positive, patient survival is similar to glioblastoma, IDH wild type
    5. If R132H IDH1 immunohistochemistry is positive, p53 staining of tumor cells is expected to be weak and scattered
    Board review style answer #2
    B. If R132H IDH1 immunohistochemistry is positive, ATRX staining of tumor cells is expected to be lost. The image is consistent with an infiltrating astrocytoma with significant pleomorphism and mitotic activity. If R132H IDH1 immunohistochemistry is positive, as stated in answer B, the findings are then consistent with astrocytoma, IDH mutant, CNS WHO grade 3. Astrocytoma, IDH mutant tumors often show concurrent ATRX mutation, indicated by loss of staining in tumor cells with ATRX immunohistochemistry.

    Comment Here

    Reference: Astrocytoma, IDH mutant
    Board review style question #3
    You are working up a diffusely infiltrating glial tumor and find tumor cells are positive for R132H IDH1 and have lost ATRX. What histologic or molecular features would be consistent with a diagnosis of astrocytoma, IDH mutant, CNS WHO grade 3?

    1. CDKN2A / CDKN2B homozygous deletion
    2. Microvascular proliferation
    3. Mitoses and pleomorphism in the absence of necrosis, microvascular proliferation or CDKN2A / CDKN2B homozygous deletion
    4. Necrosis
    5. Necrosis, microvascular proliferation or CDKN2A / CDKN2B homozygous deletion
    Board review style answer #3
    C. Mitoses and pleomorphism in the absence of necrosis, microvascular proliferation or CDKN2A / CDKN2B homozygous deletion. The presence of mitoses and pleomorphism in the absence of necrosis, microvascular proliferation or CDKN2A / CDKN2B homozygous deletion is consistent with a diagnosis of astrocytoma, IDH mutant, CNS WHO grade 3. The presence of necrosis, microvascular proliferation or CDKN2A / CDKN2B homozygous deletion would be consistent with astrocytoma, IDH mutant, CNS WHO grade 4. An infiltrating astrocytoma with IDH mutation and no significant proliferative activity, pleomorphism, necrosis, microvascular proliferation or CDKN2A / CDKN2B homozygous deletion would be consistent with astrocytoma, IDH mutant, CNS WHO grade 2.

    Comment Here

    Reference: Astrocytoma, IDH mutant

    Atypical meningioma
    Definition / general
    • A meningioma of intermediate aggressiveness between benign and malignant forms, comprising 5 - 15% of meningiomas
    • WHO grade 2
    • Diagnostic criteria: fulfilling either 1 of 2 major criteria or 3 of 5 minor criteria
      • Major criteria:
        • 4 - 19 mitotic figures/10 high power fields
        • Brain invasion
      • Minor criteria:
        • Increased cellularity
        • Small cells with high N/C ratio
        • Large and prominent nucleoli
        • Patternless or sheet-like growth (loss of lobular architecture)
        • Foci of spontaneous or geographic necrosis
    • Invasion of dura, bone or soft tissue does not affect grading
    • Pleomorphic or atypical nuclei do not affect grade
    • Ki67 is not a true diagnostic criteria; however, it is usually greater than 4% and up to 20%
    Essential features
    • Atypical meningiomas have an intermediate recurrence rate between benign and malignant meningiomas
    • 29 - 52% recur (versus 7 - 25% of classic meningiomas and 50 - 94% of anaplastic meningiomas) (Louis: WHO Classification of Tumours of the Central Nervous System, 4th Edition, 2016)
    • Molecular genetic and epigenetic signatures of atypical meningiomas are becoming increasingly important in predicting prognosis and new targeted therapy in progressive tumors
    ICD coding
    • ICD-10: D32.9 - benign neoplasm of meninges, unspecified
    Epidemiology
    Sites
    • Intracranial, intraspinal or intraorbital
    Pathophysiology
    • Arising from the meningothelial cells or the arachnoid layer
    Etiology
    Clinical features
    • Clinical presentation of atypical and anaplastic meningioma is similar to their benign counterpart
    • Common symptoms include headaches, seizures and focal neurological deficit due to tumor compression (Neurosurg Clin N Am 2016;27:239)
    Diagnosis
    • Diagnose by imaging and pathology of biopsy / resection specimen
    Radiology description
    • Extra-axial mass with dural tail
    • Uniformly contrast enhancing
    • Extensive peritumoral edema is associated with brain invasion (Neuro Oncol 2020 Aug 13 [Epub ahead of print])
    • Several benign meningioma variants, including angiomatous, microcystic, secretory and lymphoplasmacyterich meningiomas may also have prominent peritumoral edema (J Neurooncol 2013;111:49)
    Prognostic factors
    • Extent of surgery and WHO grading
    • DNA methylation profiling may better predict tumor recurrence and prognosis than histologic classification (Lancet Oncol 2017;18:682)
    Case reports
    • 36 and 70 year old women with optic nerve seeding of atypical meningiomas presenting with subacute visual loss (J Neurosurg 2013;119:494)
    • 44 year old man with atypical primary meningioma in the nasal septum with malignant transformation and distant metastasis (BMC Cancer 2012;12:275)
    • Elderly man with metastatic atypical meningioma (J Clin Neurosci 2000;7:69)
    Treatment
    • Gross total resection
    • Postsurgical radiation is often offered for atypical meningiomas, especially after a subtotal resection (J Neurooncol 2013;115:241)
    • Stereotactic radiosurgery
    Gross description
    Frozen section description
    Microscopic (histologic) description
    • May have histology of any grade 1 variant meningioma with increased mitoses (4 - 19/10 high power fields)
      • Mitotic rate is defined as the highest count over 10 consecutive high power fields (1 high power field = 0.16 mm²)
    • May have increased cellularity or areas of small cell collections
    • May have sheet-like growth pattern
    • May have areas of spontaneous necrosis
    • May have macronucleoli
    • Brain invasion is defined as irregular projections of tumor cells into adjacent CNS parenchyma without an intervening layer of leptomeninges at the tumor to brain interphase (Am J Surg Pathol 1997;21:1455)
    Microscopic (histologic) images

    Contributed by Chunyu "Hunter" Cai, M.D., Ph.D.
    Small cell change - meningothelial

    Small cell change - meningothelial

    Small cell change - transitional

    Small cell change - transitional

    Spontaneous necrosis

    Spontaneous necrosis

    Macronuclei

    Macronuclei

    Sheeting

    Sheeting

    Brain invasion - nest

    Brain invasion - nest


    Brain invasion - protrusions

    Brain invasion - protrusions

    Cytology description
    • Squash prep shows similar histology as standard meningioma but may also show occasional mitoses or macronucleoli
    Negative stains
    Molecular / cytogenetics description
    • Majority of atypical meningiomas have loss of NF2 combined with either genome instability (large scale chromosomal alterations) or loss of SMARCB1 (Nat Commun 2018;9:16215)
    • Recurrent losses of chromosome 1p, 6q, 14q,18q and gain of 1q are indicators of poor prognosis (Acta Neuropathol 2017;133:431)
    • Non-NF2 meningiomas are enriched in mutations in TRAF2, KLF4, AKT1 and SMO, most of which are benign and preferentially locate in skull base (Science 2013;339:1077)
    • DNA methylation profiling of meningioma distinguished 6 methylation classes (MCs), benign (ben) 1 - 3, intermediate (int) A and B and malignant (mal)
      • DNA methylation based meningioma classification is reported to better predict tumor recurrence and prognosis than the WHO histological classification (Lancet Oncol 2017;18:682)
    • NF2 mutant atypical meningiomas display increased H3K27me signal and a hypermethylated phenotype due to increased polycomb repressive complex 2 (PCR2) / EZH2 activity (Nat Commun 2018;9:16215)
    Sample pathology report
    • Brain, right frontal lobe mass, excision:
      • Atypical meningioma (see comment)
      • Comment: Section shows a meningioma with predominant meningothelial morphology and rare psammoma bodies. Multiple atypical features are present, including variably increased mitotic index up to 7 mitoses/10HPF (A7), multifocal microscopic necrosis, widespread small cell change, hypercellularity, and sheeted architecture. No macronucleoli or brain invasion is identified. Ki67 proliferation index is 12.7% per 1,000 nuclei count.
    Differential diagnosis
    Board review style question #1

    The arrows in the above image show which of the following?

    1. Blood vessels
    2. Lymphoplasmacytic inflammation
    3. Pseudopalisading necrosis
    4. Small cell change
    Board review style answer #1
    D. Small cell change

    The image shows an atypical meningioma with small cell change, characterized by reduced cytoplasm and increased N/C ratio in these regions. These regions may resemble lymphoplasmacytic inflammation on low power but on high power show nuclei that are similar to adjacent tumor cells.

    Comment Here

    Reference: Atypical meningioma

    Atypical teratoid / rhabdoid tumor
    Definition / general
    • High grade malignant CNS embryonal tumor composed of poorly differentiated cells with a variable number of rhabdoid cells
    • Occurs predominantly in young children; diagnosis is based on demonstrating loss of SMARCB1 (INI1) or SMARCA4 (BRG1)
    • Tumors with similar morphology but that lack characteristic molecular findings are classified as CNS embryonal tumors with rhabdoid features
    Essential features
    • CNS embryonal tumor with a polyimmunophenotype and loss of nuclear SMARCB1 or SMARCA4 expression in tumor cells are required for the diagnosis of atypical teratoid / rhabdoid tumor (AT / RT)
    • Tumors with similar morphology and immunophenotype but that lack a classifiable mutation are classified as CNS embryonal tumors (not elsewhere classified [NEC]) or not susceptible to further analysis (not otherwise specified [NOS])
    ICD coding
    • ICD-O: 9508/3 - atypical teratoid / rhabdoid tumor
    • ICD-11: 2A00.1Y & XH7ZQ4 - other specified embryonal tumors of brain & atypical teratoid / rhabdoid tumor
    Epidemiology
    • Accounts for 1 - 2% of all pediatric brain tumors and is very rare in adults (Front Oncol 2018;8:567)
    • Age typically < 3 years; rare in children aged > 6 years (Neuro Oncol 2014;16:1392)
    • Can occur in cerebral hemispheres, cerebellum or rarely in the spinal cord
    Sites
    • Cerebral hemispheres, cerebellar hemispheres, cerebellopontine angle, brainstem, spinal cord
    • In adults, the most common sites are the cerebral hemispheres and sellar region
    Pathophysiology
    • Mutation or loss of the SMARCB1 locus at 22q11.2 is classic for this tumor (Nature 1998;394:203)
    • Tumors with retained SMARCB1 can have biallelic inactivation and no expression of the SMARCA4 protein (Am J Hum Genet 2010;86:279)
    • Function of SMARCB1 and SMARCA4 and their role in malignant transformation are unclear
    Etiology
    Diagrams / tables

    Images hosted on other servers:
    Features of AT / RT subtypes

    Features of AT / RT subtypes

    Clinical features
    • Depends on age and location
    • In infants, lethargy, vomiting and failure to thrive are common symptoms (J Neurosurg 1996;85:56)
    • If > 3 years, headache and hemiplegia are reported (Childs Nerv Syst 2009;25:707)
    • Cranial nerve palsy (mostly sixth and seventh nerve paresis) may also be present
    • Adult sellar AT / RT often presents with headaches, visual impairment and endocrine disturbances (Mod Pathol 2022;35:1910)
    Diagnosis
    • Diagnosis of AT / RT can be made on biopsy or cytology but staging requires cerebrospinal fluid (CSF) cytology (Radiology 1969;93:1351)
    Laboratory
    • No specific laboratory findings for this entity
    Radiology description
    • Isodense to hyperintense signal intensity on fluid attenuated inversion recovery (FLAIR) images with restricted diffusion
    Radiology images

    Contributed by Chunyu Cai, M.D., Ph.D. (Case #502)
    MRI T1 postsagittal

    MRI T1 postsagittal



    Images hosted on other servers:
    ATRT in a 9 month old

    AT / RT in a 9 month old

    Axial CT head, unenhanced

    Axial CT head, unenhanced

    MRI brain, unenhanced and enhanced

    MRI brain, unenhanced and enhanced

    Prognostic factors
    • Poor; 5 year progression free and overall survival rate is only 60%, even for the favorable group
    • Clinical staging (Radiology 1969;93:1351)
    Case reports
    Treatment
    Gross description
    • Tan-pink to red soft tissue, appears demarcated from adjacent parenchyma
    • Tumors with more mesenchymal tissue appear firm, tan-white
    • Hemorrhage and necrosis can be seen (Autops Case Rep 2020;10:e2020205)
    Gross images

    Images hosted on other servers:
    Posterior fossa, cerebellum and brain stem tumors

    Posterior fossa, cerebellum and brain stem tumors

    Frozen section description
    • Sheets of eosinophilic, embryonal cells with round to oval hyperchromatic nuclei and minimal cytoplasm
    • Rhabdoid cells are larger cells with eccentrically located nuclei and eosinophilic cytoplasm
    Intraoperative frozen / smear cytology images

    Images hosted on other servers:
    Sheets of rhabdoid cells

    Sheets of rhabdoid cells

    Typical cytology

    Typical cytology

    Touch preparation of rhabdoid cells

    Touch preparation of rhabdoid cells

    Microscopic (histologic) description
    • Sheets of densely packed, immature cells with high N:C ratio
    • Diagnostic feature on histology is the presence of cells with rhabdoid features, which includes (Neurooncol Pract 2019;6:163)
      • Well defined cell borders
      • Abundant cytoplasm with eosinophilic inclusions
      • Eccentrically located nuclei containing vesicular chromatin
      • Prominent eosinophilic nucleoli
    • Mitotic figures can be numerous
    • Geographic necrosis can be present
    • Often diverse with a mixed histologic appearance (epithelioid, myxoid, spindled, chondroid) (Am J Surg Pathol 1998;22:1083)
    • Primitive neuroectodermal component is most common (Am J Surg Pathol 2006;30:1462)
    • Mesenchymal and epithelial features are less common
    • Staghorn vasculature prominent in adult sellar AT / RT (Am J Surg Pathol 2017;41:932)
    Microscopic (histologic) images

    Contributed by Nirupama Singh, M.D., Ph.D., Chunyu Cai, M.D., Ph.D. (Case #502) and Geling Li, M.D., Ph.D.
    Vesicular chromatin and prominent nucleoli Vesicular chromatin and prominent nucleoli

    Vesicular chromatin and prominent nucleoli

    Eosinophilic globs

    Eosinophilic globs

    Mitotic figures

    Mitotic figures

    Myxoid area

    Myxoid area


    Chondroid

    Chondroid

    Geographic necrosis

    Geographic necrosis

    Gland

    Gland

    Large pale cell

    Large pale cell

    Rhabdoid cell

    Rhabdoid cell


    Small blue cell

    Small blue cell

    Densely packed immature cells

    Densely packed immature cells

    INI1

    INI1

    Cytology description
    • Embryonal cells with round to oval hyperchromatic nuclei and minimal cytoplasm
    • Rhabdoid cells are larger cells with eccentrically located nuclei and eosinophilic cytoplasm
    Positive stains
    Negative stains
    • Germ cell markers and skeletal muscle differentiation markers (MyoD1, myogenin)
    • Nuclear loss of SMARCB1 (INI1) protein expression is highly sensitive
    Electron microscopy description
    • Rhabdoid cell cytoplasm
      • Spherical, paranuclear, cytoplasmic inclusions
    Electron microscopy images

    Images hosted on other servers:
    Rhabdoid cell

    Rhabdoid cell

    Molecular / cytogenetics description
    • Loss of SMARCB1 (INI1) expression at the protein level is seen in most AT / RTs (Cancer Biol Ther 2009;8:412, Pediatr Dev Pathol 2018;21:6, Neuropathology 2018;38:305)
    • Mutations in SMARCA4 (BRG1) gene are rare
    • Loss of all or part of chromosome 22 is frequent
    • Gene expression and methylation profiling identified 3 potential subgroups that can potentially aid in subgroup based therapies (Neuro Oncol 2020;22:613)
      • ATRT TYR: overexpression of tyrosinase
        • Whole or partial loss of one copy of chromosome 22 accompanied by a point mutation in SMARCB1 on the other allele
        • Majority with infratentorial location
        • Younger patient age (median age at diagnosis: 12 months)
      • ATRT SHH: overexpression of sonic hedgehog and Notch pathways
        • Most display compound heterozygous point mutations in SMARCB1
        • Further subtyped by supratentorial (ATRT SHH1) or infratentorial (ATRT SHH2) localization
      • ATRT MYC: overexpression of MYC oncogene
        • Homozygous, broad loss of SMARCB1 with lower overall DNA methylation levels
        • Older patient age (median age at diagnosis: 27 months)
        • Adult sellar AT / RT: clinically distinct entity, however, DNA methylation is similar to ATRT MYC subgroup (Am J Surg Pathol 2018;42:506)
    Molecular / cytogenetics images

    Images hosted on other servers:
    Methylation array analysis

    Methylation array analysis

    Cluster analysis

    Cluster analysis

    Sample pathology report
    • Brain, suprasellar mass, biopsy:
      • Atypical teratoid rhabdoid tumor (AT / RT), CNS WHO grade 4 (see comment)
      • Comment: Sections of the suprasellar tumor demonstrate patternless sheets of malignant blue cells with round to oval nuclei, vacuolated chromatin and scattered inconspicuous nucleoli. Nuclear profiles are focally irregular. Tumor cells have variable eosinophilic cytoplasm. Scattered mitotic figures and apoptotic cells are noted. Microvasculature is prominent. Necrosis is not seen. INI1 nuclear immunoreactivity is lost in tumor cells.
    Differential diagnosis
    Board review style question #1

    What is the key component of atypical teratoid / rhabdoid tumor (AT / RT)?

    1. Germline BAP1 mutation
    2. Loss of expression of SMARCB1 or SMARCA4
    3. N-MYC amplification
    4. TP53 mutation
    Board review style answer #1
    B. Loss of expression of SMARCB1 or SMARCA4. SMARCB1 (INI1) is the most common and SMARCA4 (BRG1) is more rare (J Neurooncol 2020;150:47). Answer A is incorrect because a BAP1 mutation is not associated with this entity. Answers C and D are incorrect; although they may be seen in certain subgroups, they are not the key component of AT / RTs.

    Comment Here

    Reference: Atypical teratoid / rhabdoid tumor
    Board review style question #2
    Which of the following is the most common age range for presentation of an atypical teratoid / rhabdoid tumor (AT / RT)?

    1. 0 - 3 years
    2. 3 - 20 years
    3. 20 - 50 years
    4. 50 - 80 years
    Board review style answer #2
    A. 0 - 3 years. > 80% of AT / RTs are diagnosed before age 3. Answer C would be correct for the primary adult sellar AT / RT. Answers B and D are not the most common age ranges for this entity.

    Comment Here

    Reference: Atypical teratoid / rhabdoid tumor

    CIC rearranged sarcoma
    Definition / general
    Essential features
    Terminology
    • Undifferentiated round cell sarcoma with CIC-DUX4 fusion
    ICD coding
    • ICD-O: 8803/3 - Small cell sarcoma
    Epidemiology
    Sites
    Etiology
    • CIC-DUX4 fusion oncoprotein potentiates the transcriptional activity of CIC and activates the expression ETV1/4/5, which is a member of the E26 transformation specific (ETS) family of transcription factors (Sci Rep 2020;10:684)
    • MYC amplification in majority of cases (Mod Pathol 2015;28:57)
    Clinical features
    Diagnosis
    Radiology description
    • Large heterogeneous appearing hypermetabolic mass on PET / CT
    Radiology images

    Contributed by Borislav Alexiev, M.D.

    PET / CT

    Prognostic factors
    Case reports
    Treatment
    Gross description
    Gross images

    Contributed by Borislav Alexiev, M.D.

    Soft tissue mass

    Frozen section description
    • Diagnosis of CIC-DUX4 sarcoma on a small tissue fragment without molecular studies would be challenging
    Microscopic (histologic) description
    Microscopic (histologic) images

    Contributed by Borislav Alexiev, M.D.

    Chest wall mass

    Small round cell morphology

    Solid growth pattern

    Spindle cell morphology

    Mitoses

    Myxoid stroma


    CD99 expression

    WT1 expression

    MYC expression

    Cytology description
    • Hypercellular smears, with tumor cells arranged in large groups and singly dispersed
    • Individual cells with high nuclear to cytoplasmic ratio, eccentric round to ovoid nuclei, irregular nuclear contours and small nucleoli (Diagn Cytopathol 2018;46:958)
    • Cytoplasmic vacuoles (Cancer Cytopathol 2016;124:350)
    • Mitotic figures
    • Necrosis
    • Myxoid stromal component
    Electron microscopy description
    • Heterogeneity: in cell density, from tightly packed to loosely unconnected areas (Ultrastruct Pathol 2020;44:237)
    • Polygonal to pleomorphic cells with small processes
    • Round, oval, polygonal or elongated nuclei
    • Abundant glycogen in the cytoplasm and rare cell adhesions
    • No neuroendocrine granules present
    Molecular / cytogenetics description
    Molecular / cytogenetics images

    Contributed by Lawrence J. Jennings, M.D., Ph.D.

    Real time RT-PCR

    Sample pathology report
    • Chest wall mass, excision:
      • CIC-DUX4 associated undifferentiated small round cell sarcoma (see comment)
      • Comment: There is a subcutaneous solid nodular proliferation of small to medium sized round / ovoid and spindle cells with scant amount of amphophilic or lightly eosinophilic cytoplasm. The cells contain medium sized round to oval vesicular nuclei with small nucleoli. High mitotic activity (21 mitoses/10 high power fields) and areas of necrosis are present. Patchy myxoid or edematous stromal changes are seen.
      • Immunohistochemically the cells have strong expression of CD99, WT1 (N terminal) and MYC while are negative for pankeratin AE1 / AE3, EMA, myogenin, S100 and SOX10. INI1 is preserved. The tumor is positive for CIC-DUX4 fusion transcript.
      • This constellation of morphological, immunohistochemical and molecular features strongly supports the diagnosis of CIC-DUX4 associated undifferentiated small round cell sarcoma. It is a sarcoma associated with an aggressive clinical course, with an inferior overall survival compared to Ewing sarcoma.
    Differential diagnosis
    Board review style question #1
    Which of the following is true about CIC-DUX4 rearranged sarcoma?

    1. High metastatic rate, brain most common
    2. Diagnosis always requires clinicopathological and radiological correlation
    3. Prognosis is poor
    4. Tumor is always negative for ERG
    5. Tumor is characterized by cords of epithelioid cells distributed in a desmoplastic stroma
    Board review style answer #1
    C. Prognosis is poor

    Comment Here

    Reference: CIC-DUX4 fusion tumor
    Board review style question #2

    A 65 year old man presented with a left thigh mass. Hematoxylin eosin stains demonstrated proliferation of small to medium sized round / ovoid cells with medium sized round to oval vesicular nuclei and clear or pale eosinophilic cytoplasm. Increased mitotic activity, geographic necrosis and patchy myxoid stromal change were seen. Immunohistochemical stains for CD99, WT1 and DUX4 were positive in tumor cells while all of the following were negative: pankeratin AE1 / AE3, S100, SOX10, myogenin, NY-ESO-1, NKX2.2 and CCNB3. INI1 was retained. Which of the following is most likely the correct diagnosis?

    1. Synovial sarcoma, poorly differentiated
    2. Extraskeletal Ewing sarcoma
    3. Epithelioid sarcoma
    4. BCOR-CCNB3 (Ewing-like) sarcoma
    5. CIC-DUX4 associated undifferentiated round cell sarcoma
    Board review style answer #2
    E. CIC-DUX4 associated undifferentiated round cell sarcoma

    Comment Here

    Reference: CIC-DUX4 fusion tumor

    CNS embryonal tumor, NOS/NEC (pending)
    Table of Contents
    Definition / general
    Definition / general
    [Pending]

    CNS neuroblastoma, FOXR2 activated
    Definition / general
    • CNS neuroblastoma, FOXR2 activated (CNS NB FOXR2) tumors are embryonal neoplasms characterized by poorly differentiated neuroblastic or neuronal differentiation with variable neuropil rich stroma and ganglion cells
    • Characterized by the activation of FOXR2 by structural rearrangements
    Essential features
    • Embryonal tumor with neuronal or neuroblastic differentiation and activation of FOXR2 rearrangement and fusion or a DNA methylation profile aligned with CNS neuroblastoma, FOXR2 activated
    Terminology
    • CNS neuroblastoma, FOXR2 activated
    • Formerly recognized as one of the primitive neuroectodermal tumors of CNS (CNS PNETs)
    ICD coding
    • ICD-O: 9500/3 - neuroblastoma, NOS
    • ICD-10: C71 - malignant neoplasm of brain
    • ICD-11: 2A00.1Y & XH85Z0 - other specified embryonal tumors of brain & neuroblastoma, NOS
    Epidemiology
    Sites
    • Supratentorial
    Pathophysiology
    • Thought to arise from neuroectodermal cells, although the exact cell of origin is not known (J Neuropathol Exp Neurol 2021;80:52)
    • Most frequent genetic alterations are on the transcription factor gene FOXR2 (Cell 2016;164:1060)
    • Although FOXR2 plays a causative role in the formation of CNS embryonal tumors, the exact underlying mechanisms have yet to be determined (Neuro Oncol 2019;21:993)
    Etiology
    • Risk factors have not been reported
    Clinical features
    Diagnosis
    • Embryonal tumor with neuronal or neuroblastic differentiation and activation of FOXR2 rearrangement and fusion or a DNA methylation profile aligned with CNS neuroblastoma, FOXR2 activated
    Laboratory
    • No specific laboratory findings for this entity
    Radiology description
    Radiology images

    Images hosted on other servers:
    T2 weighted and postcontrast T1 weighted axial

    T2 weighted and postcontrast T1 weighted axial

    Prognostic factors
    • Favorable survival for patients with CNS NB FOXR2; higher rate of recurrences in nonirradiated and locally irradiated patients (Neuro Oncol 2021;23:1597)
    Case reports
    Treatment
    Gross description
    • CNS embryonal tumors are often pink, soft, well circumscribed masses; if a prominent desmoplastic component is present, it is often tan-white and firm
    Microscopic (histologic) description
    • Poorly differentiated neuroepithelial cells and neurocytic cells (Cell 2016;164:1060)
      • Uniform small cells with high N:C ratio, oval to round with hyperchromatic nuclei
      • Cells have little to no apparent cytoplasm in a background of neuropil with or without Homer Wright rosettes (J Neuropathol Exp Neurol 2021;80:52)
    • Background of neuropil rich stroma
    • Abundant mitotic activity
    • Palisading necrosis can be present
    • Infiltration of adjacent parenchyma is variable
    Microscopic (histologic) images

    Contributed by Nirupama Singh, M.D., Ph.D., Carmen Perrino, M.D. and P.J. Cimino, M.D., Ph.D.
    Neuroblasts with no cytoplasm

    Neuroblasts with no cytoplasm

    Homer Wright rosettes

    Homer Wright rosettes

    Small tumor cells

    Small tumor cells

    High power pseudorosettes

    Pseudorosettes


    Embryonal cells

    Embryonal cells

    Neurocytic cells

    Neurocytic cells

    Focal ganglion cells

    Focal ganglion cells

    Necrosis

    Necrosis

    MVP

    MVP

    Cytology description
    • Evaluation of cerebrospinal fluid (CSF) for the presence of tumor cells is required for staging (Radiology 1969;93:1351)
    Positive stains
    Negative stains
    Electron microscopy description
    Molecular / cytogenetics description
    • Interchromosomal and intrachromosomal rearrangements in forkhead box R2, leading to increased FOXR2 gene expression
      • FOXR2 locus on chromosome Xp11.21
      • Common fusion partner is JMJD1C locus on chromosome 10q21.3
    • Gain of chromosome 1q is present in most cases (Neuro Oncol 2021;23:1597)
    • High FOXR2 expression can occur in a subset of high grade gliomas and medulloblastomas; however, DNA methylation profiling can distinguish these entities (Cell 2016;164:1060)
    Molecular / cytogenetics images

    Images hosted on other servers:
    Recurrent molecular alterations

    Recurrent molecular alterations

    Videos

    Case presentation of CNS NB FOXR2

    Sample pathology report
    • Brain, suprasellar mass, resection:
      • CNS neuroblastoma, FOXR2 activated, CNS WHO grade 4 (see comment)
      • Comment: Sections of the suprasellar tumor demonstrate small, round, embryonic appearing cells with scant cytoplasm. Necrosis and abundant mitotic activity are appreciated. Immunohistochemical stains for synaptophysin and Ki67 were performed. Synaptophysin is patchy positive. Ki67 proliferation index is ~25%.
      • Molecular pathology findings
        • FOXR2 fusion
        • Gain of chromosome 1q
    Differential diagnosis
    Board review style question #1
    A 2 year old boy presents with seizures. MRI reveals a well circumscribed supratentorial mass with a mixed solid and cystic appearance. Which of the following is the most likely diagnosis?

    1. Ewing sarcoma
    2. Lymphoma
    3. Neuroblastoma
    4. Testicular germ cell tumor
    Board review style answer #1
    C. Neuroblastoma. In this age group, neuroblastoma is the most common of these tumors. Answers A, B and D are incorrect because these diagnoses are less likely in this age group.

    Comment Here

    Reference: Neuroblastoma
    Board review style question #2

    This supratentorial mass is composed of small, round, embryonic appearing cells and Homer Wright rosettes. Whole genome analysis demonstrated a FOXR2 fusion. What is the most likely diagnosis?

    1. CNS neuroblastoma, FOXR2 activated
    2. Embryonal tumors with multilayered rosettes
    3. Ependymoma
    4. Medulloblastoma
    Board review style answer #2
    A. CNS neuroblastoma, FOXR2 activated. This histology along with the molecular findings are consistent with a CNS neuroblastoma, FOXR2 activated. Answers B, C and D are incorrect because although they might be similar morphologically, they will not have a FOXR2 fusion.

    Comment Here

    Reference: Neuroblastoma

    CNS tumor with BCOR internal tandem duplication (pending)
    [Pending]

    CNS tumors NOS/NEC (pending)
    [Pending]

    CNS-gross (pending)
    [Pending]

    CNS-histology (pending)
    [Pending]

    Cauda equina neuroendocrine tumor
    Definition / general
    • Low grade, well circumscribed neuroendocrine neoplasm of cauda equina / filum terminale region
    • CNS WHO grade 1
    • Distinct from paragangliomas and pheochromocytomas outside the central nervous system (CNS) (Acta Neuropathol 2020;140:907)
    Essential features
    • Grade 1, slow growing neuroendocrine neoplasm of cauda equina / filum terminale region
    • Rare entity with a benign clinical course (J Neurooncol 2015;122:539)
    • Generally affects adults
    • Distinct from paragangliomas and pheochromocytomas outside the CNS (Acta Neuropathol 2020;140:907)
    Terminology
    ICD coding
    • ICD-O: 8693/3 - cauda equina neuroendocrine tumor (previously paraganglioma)
    • ICD-11
      • 2A02.0Y & XH1X68 - other specified gliomas of spinal cord, cranial nerves or other parts of the central nervous system & paraganglioma, benign
      • 2A02.0Y & XH0EW6 - other specified gliomas of spinal cord, cranial nerves or other parts of the central nervous system & paraganglioma, NOS
    Epidemiology
    Sites
    Pathophysiology
    • Histogenetically, biologically and molecularly distinct from paragangliomas and pheochromocytomas outside the CNS (Acta Neuropathol 2020;140:907)
    • Arises from specialized neural crest cells in the cauda equina / filum terminale region
    • mRNA analyses revealed that cauda equina paragangliomas overexpress the transcription factor HOXB13 (which is developmentally expressed in the caudal extent of the spinal cord) as opposed to pheochromocytomas / paragangliomas of other regions of the body; this provides circumstantial evidence of their cellular origin (Neuropathol Appl Neurobiol 2021;47:889)
    • Molecular alterations that drive tumorigenesis in cauda equina neuroendocrine tumors are unknown
    Etiology
    Clinical features
    Diagnosis
    • Rarely a clinical or radiological diagnosis
    • Neuroimaging: magnetic resonance imaging (MRI) (modality of choice) and computed tomography (CT) (Neurosurg Rev 2022;45:103)
    • Biopsy
    • WHO essential and desirable diagnostic criteria
      • Essential
        • Well demarcated tumor with zellballen architecture
        • Synaptophysin or chromogranin immunoreactivity in chief cells
        • Cauda equina location
        • Methylation profile of cauda equina neuroendocrine tumor (for unresolved lesions)
      • Desirable
        • S100 positive sustentacular cells
        • Cytokeratin positive chief cells
        • Reticulin silver stain showing typical architecture
    Laboratory
    Radiology description
    • MRI
      • Well circumscribed, intradural extramedullary, sausage shaped mass at cauda equina / filum terminale region (Radiol Case Rep 2019;14:1185)
      • Solid, occasionally partly cystic
      • Generally T1 hypointense to isointense and T2 isointense to hyperintense, with intense post contrast enhancement (Neurosurg Rev 2022;45:103)
      • Although MRI findings are nonspecific and indistinguishable from other solid tumors in cauda equina region, certain characteristic MRI findings could suggest the diagnosis, including a salt and pepper appearance on T2 weighted images related to the hypervascular nature (42.1%), a peripheral hypointense rim (cap sign) on T2 weighted images caused by subcapsular hemosiderin (47.3%) and serpiginous flow voids on all sequences (78.9%) (J Neurooncol 2015;122:539)
      • Perfusion weighted MR images show increased blood flow, consistent with hypervascular tumor (Radiol Case Rep 2019;14:1185)
      • Polar sign may be seen in T1 contrast enhanced and T2 weighted MR images, representing subacute to chronic polar intratumoral hematomas (J Spine Neurosurg 2014;3:4)
    • Spinal angiography reveals a well defined hypervascular lesion with intense early blush that persists well into the arterial and venous phases (silk cocoon appearance), which helps presurgical planning and differentiates CEPs from other spinal tumors (Radiol Case Rep 2019;14:1185, Neurosurg Focus 2015;39:E16)
    • CT: may show scalloping of the vertebral bodies (J Surg Tech Case Rep 2012;4:46)
    Radiology images

    Images hosted on other servers:
    Cauda equina paraganglioma

    MRI

    Intradural enhancing lesion

    Intradural enhancing lesion

    Spinal angiography

    Spinal angiography

    Scalloping of lumbar vertebrae

    Scalloping of lumbar vertebrae

    Prognostic factors
    Case reports
    Treatment
    Clinical images

    Images hosted on other servers:
    Cauda equina paragangliomas

    Intraoperative

    Lesion after durotomy

    Lesion after durotomy

    Hypervascular well marginated mass

    Hypervascular well marginated mass

    Gross description
    • Well circumscribed, delicately encapsulated, oval to sausage shaped vascular mass
    • Smooth surfaced, soft, red-brown and bleeds freely (Histopathology 1997;31:167)
    • Size ranges from 10 mm to 112 mm in greatest dimension
    • May show capsular calcification or cystic change
    • Gross appearance resembles myxopapillary ependymoma
    • If submitted with attached filum terminale, proximal and distal ends should be sampled as surgical margins
    Gross images

    Images hosted on other servers:
    Lesion after en block removal

    Lesion after en block removal

    Hemorrhagic cystic change

    Hemorrhagic cystic change

    Frozen section description
    Frozen section images

    Images hosted on other servers:
    Intraoperative frozen section

    Ependymoma-like features

    Microscopic (histologic) description
    • Well differentiated, encapsulated and richly vascular tumor
    • Organoid (zellballen) architecture: nests or lobules of chief (type I) cells surrounded by sustentacular (type II) cells with intervening delicate capillary and reticulin networks (J Neurooncol 2015;122:539)
    • Chief cells
      • Uniform round or polygonal epithelioid cells
      • Round to oval nuclei with salt and pepper chromatin pattern and inconspicuous nucleoli (J Surg Tech Case Rep 2012;4:46)
      • Finely granular eosinophilic cytoplasm, may be amphophilic or clear (Acta Neuropathol 2020;140:907)
      • Sharp cell borders, particularly around vessels
      • No or mild degenerative nuclear pleomorphism (endocrine atypia), of no prognostic significance
    • Sustentacular cells
      • Spindle shaped cells with attenuated long processes
      • Perilobular; surround chief cell lobules
      • Inconspicuous by routine light microscopy (visible by IHC for S100)
    • Mitotic activity is highly variable (0 - 5/10 high power fields), of no prognostic significance (Acta Neuropathol 2020;140:893)
    • Other common features
    • Less common features
    Microscopic (histologic) images

    Contributed by Eman Abdelzaher, M.D., Ph.D.
    Encapsulated lesion Encapsulated lesion

    Encapsulated lesion

    Zellballen architecture Zellballen architecture

    Zellballen architecture

    Zellballen architecture

    Zellballen architecture

    Salt and pepper chromatin

    Salt and pepper chromatin


    Focal sclerosis

    Focal sclerosis

    Reticulin

    Reticulin

    Synaptophysin Synaptophysin

    Synaptophysin

    S100 positive sustentacular cells

    S100 positive sustentacular cells

    S100 positive chief and sustentacular cells

    S100 positive chief and sustentacular cells


    GFAP negative

    GFAP negative

    GFAP positive sustentacular cells

    GFAP positive sustentacular cells

    EMA

    EMA

    Low Ki67

    Low Ki67

    Cytology description
    Positive stains
    Negative stains
    Electron microscopy description
    • Chief cells
    • Sustentacular cells
      • Electron dense with elongated processes
      • Contain some intermediate filaments
      • Lack neurosecretory granules
    Electron microscopy images

    Images hosted on other servers:
    Numerous perinuclear dense core granules

    Numerous perinuclear dense core granules

    Prominent, rough endoplasmic reticulum

    Prominent rough endoplasmic reticulum and Golgi apparatus

    Molecular / cytogenetics description
    • Distinct DNA methylation and chromosomal copy number profiles as opposed to those of paragangliomas arising from other locations (Acta Neuropathol 2020;140:907)
    • DNA methylation profiling and clustering analysis showed that CEPs are epigenetically distinct from extraspinal paragangliomas, pheochromocytomas, other neuroendocrine tumors and glial or ependymal neoplasms of the spinal cord (Acta Neuropathol 2020;140:907)
    • Copy number analysis revealed diploid genomes in the vast majority of CEPs, whereas extraspinal paragangliomas were mostly aneuploid with recurrent trisomy 1q and monosomies of 1p, 3 and 11, none of which were present in the cohort of CEP (Acta Neuropathol 2020;140:907)
    • RNA and DNA exome sequencing revealed that SDH mutations are absent in cauda equina paragangliomas (Acta Neuropathol 2020;140:893)
    • CEPs are not driven by recurrent oncogenic gene fusions; fusion genes known to be relevant in paragangliomas / pheochromocytomas and associated with poor prognosis (e.g., MAML3 fusions) were not observed in CEPs (Acta Neuropathol 2020;140:893)
    Molecular / cytogenetics images

    Images hosted on other servers:
    DNA methylation cluster analysis of CEPs and other tumors

    DNA methylation cluster analysis of CEPs and other tumors

    Chromosomal copy number analysis

    Chromosomal copy number analysis

    Videos

    Case 8: paraganglioma of the filum terminale

    Sample pathology report
    • Cauda equina mass, total resection:
      • Cauda equina neuroendocrine tumor (cauda equina paraganglioma), CNS WHO grade 1 (see comment)
      • Comment: Encapsulated tumor with zellballen architecture highlighted by reticulin histochemical stain. The tumor is composed of chief cells (synaptophysin positive) arranged in nests surrounded by S100 positive sustentacular cells.
      • Molecular genetics: methylation profile of cauda equina neuroendocrine tumor
    Differential diagnosis
    Board review style question #1

    A 49 year old man presented with lower back pain and sciatica for a duration of 3 months. MRI showed cauda equina intradural extramedullary enhancing vascular mass. The received gross specimen was a delicately encapsulated oval mass with a red-brown cut section. The images shown above depict the histological features of the lesion and synaptophysin immunostain. What is the most likely diagnosis?

    1. Cauda equina paraganglioma
    2. Hemangioblastoma
    3. Myxopapillary ependymoma
    4. Schwannoma
    Board review style answer #1
    A. Cauda equina paraganglioma. The location of the tumor at the cauda equina region together with the histologically distinctive zellballen architecture and synaptophysin positive chief cells are diagnostic of cauda equina paraganglioma. Answer B is incorrect because hemangioblastomas have vacuolated stromal cells and are synaptophysin negative. Answer C is incorrect because myxopapillary ependymomas show characteristic perivascular myxoid change and are synaptophysin negative. Answer D is incorrect because schwannomas do not have a zellballen architecture and are synaptophysin negative.

    Comment Here

    Reference: Cauda equina neuroendocrine tumor
    Board review style question #2
    Which of the following statements about cauda equina paraganglioma is correct?

    1. Encapsulated tumors with zellballen architecture and synaptophysin immunoreactivity
    2. It is commonly functioning with clinical features of catecholamine hypersecretion
    3. It shares common genetic profile with extraspinal paragangliomas and pheochromocytomas
    4. Recurrence rate after total resection is high
    Board review style answer #2
    A. Encapsulated tumors with zellballen architecture and synaptophysin immunoreactivity. Zellballen architecture and synaptophysin positive chief cells are characteristic of cauda equina paraganglioma. Answer B is incorrect because functional activity is extremely rare in cauda equina paraganglioma. Answer D is incorrect because recurrence rate after total resection is low (around 7%). Answer C is incorrect because cauda equina paragangliomas are molecularly distinct from paragangliomas and pheochromocytomas outside the CNS.

    Comment Here

    Reference: Cauda equina neuroendocrine tumor

    Central neurocytoma
    Definition / general
    • Rare, well differentiated, intraventricular neoplasm with neuroepithelial differentiation, typically arising near the foramen of Monro
    Essential features
    Terminology
    • Central neurocytoma
    ICD coding
    • ICD-O:
      • 9506/0 - central neurocytoma benign
      • 9506/1 - central neurocytoma
    • ICD-10:
      • D43.0 - neoplasm of uncertain behavior of brain, supratentorial
      • D43.1 - neoplasm of uncertain behavior of brain, infratentorial
      • D43.2 - neoplasm of uncertain behavior of brain, unspecified
      • D43.9 - neoplasm of uncertain behavior of central nervous system, unspecified
    • ICD-11:
      • 2A00.3 - central neurocytoma of brain
      • XH0C11 - central neurocytoma
    Epidemiology
    Sites
    Pathophysiology
    Etiology
    • Unknown at this time
    Diagrams / tables

    Images hosted on other servers:

    Coronal section diagram

    Clinical features
    Diagnosis
    • Based primarily on histologic and immunophenotypic features (Brain Pathol 1993;3:297)
    • Requires intraventricular localization, oligodendroglioma-like cytology and synaptophysin expression
    • Correlation with radiologic studies (e.g., MRI, CT) is essential; supportive findings include
      • Intraventricular localization (required)
      • Enhancing, multicystic mass
      • MRI: T1 isointense, T2 heterogeneous, FLAIR hyperintense
      • CT: may show calcifications
    • Methylation profiling may aid diagnosis in unresolved cases (J Neurooncol 2022;159:725)
    Radiology description
    • Noncontrast computed tomography (J Clin Neurosci 2013;20:679, Neurosurg Clin N Am 2015;26:11)
      • Highly variable appearance, often with mixed solid and cystic components that appear isodense and hypodense, respectively, to surrounding brain parenchyma
      • Calcifications may be seen, typically partial or punctate
      • Evidence of hydrocephalus or hemorrhage may be visible
    • Magnetic resonance imaging (J Clin Neurosci 2012;19:681, J Clin Neurosci 2013;20:679, Neurosurg Clin N Am 2015;26:11)
      • Classically an intraventricular mass with a multicystic, soap bubble appearance characterized by T1 and T2 isointense solid components and T2 hyperintense, fluid filled cysts
      • Peripheral cyst walls may form spicules and cause undulation of the adjacent lateral ventricle wall (scalloping)
      • Calcifications and flow voids may be seen on T1 sequences
      • Heterogenous contrast enhancement
      • No surrounding peritumoral edema on T2 / FLAIR
    • Proton magnetic resonance spectroscopy (Eur Radiol 2009;19:2049, J Clin Neurosci 2012;19:681, Neurosurg Clin N Am 2015;26:11)
      • Characteristic glycine peak at 3.55 ppm
      • Prominent choline peak
      • Inverted alanine peak
    Radiology images

    Images hosted on other servers:

    Intraventricular mass on MRI

    Calcifications on CT

    T1+C MRI

    Prognostic factors
    Case reports
    Treatment
    Clinical images

    Images hosted on other servers:

    Endoscopic view of intraventricular tumor

    Gross description
    Gross images

    Images hosted on other servers:

    Intraventricular mass

    Frozen section description
    • Sheets of isomorphous, round cells in a fibrillary background
    • Minimal pleomorphism
    • Necrosis or mitotic figures are typically absent
    • Reference: Acta Cytol 2004;48:194
    Intraoperative frozen / smear cytology images

    Contributed by Rebecca Yoda, M.D.

    Cells with monotonous round nuclei

    Intraoperative squash preparation

    Microscopic (histologic) description
    • Neuroepithelial neoplasm composed of uniform, small - medium cells growing in sheets with indistinct cytoplasm (Brain Pathol 1993;3:297)
      • Nuclei are round with regular contours, finely stippled (salt and pepper) chromatin and micronucleoli
      • Perinuclear clearing may be prominent (similar to oligodendrogliomas)
      • Arborizing capillaries
      • Large hyalinized blood vessels
      • Other morphologic features may include
        • Honeycomb-like architecture
        • Patches of fibrillar, neuropil-like matrix mimicking pineocytomatous rosettes
        • Perivascular pseudorosettes
        • Homer-Wright rosettes
        • Ganglioid cells
      • Calcification is usually distributed throughout the tumor and may be prominent
    • Lipomatous differentiation occurs rarely (World Neurosurg 2018;120:214)
    • Hemorrhage is sometimes present; hemosiderin laden macrophages may be seen (Neurosurg Rev 2001;24:48)
    • Designated as atypical central neurocytoma when anaplastic features are seen (J Neurosurg 1992;76:32, Brain Pathol 1993;3:297)
      • Brisk mitotic activity
      • Microvascular proliferation
      • Necrosis
    Microscopic (histologic) images

    Contributed by Rebecca Yoda, M.D.

    Circumscribed border

    Fibrillary matrix

    Perinuclear clearing

    Perivascular pseudorosette

    Neurocytic rosettes


    Linear arrangement

    Calcifications

    Branching capillaries

    Hyalinized vessels

    Lipomatous change


    Necrosis

    Mitotic activity

    Synaptophysin IHC

    NeuN IHC

    GFAP IHC



    Contributed by Nazar M. T. Jawhar, M.D. (Case #119)

    Circumscribed border

    Perivascular pseudorosettes

    Perinuclear clearing

    Rosettes and perivascular pseudorosettes


    Calcification

    Linear cellular arrangement

    Uniform, round cells

    Virtual slides

    Images hosted on other servers:

    Left lateral ventricle tumor

    Cytology description
    • Cerebral spinal fluid cytology may be positive in the presence of disseminated tumor (J Neurosurg 1992;76:32)
      • Crowded, cellular spheres composed of uniform small - medium cells
      • Scant, cyanophilic cytoplasm on Papanicolau stain
      • Neurocytic rosettes may be seen
    • Squash preparations / direct smears (Acta Cytol 2004;48:194, Acta Cytol 2010;54:209)
      • Monotonous, round cells with ill defined cytoplasm and without aggregation or clustering
      • Nuclei tend to have finely granular chromatin and micronucleoli
      • Hemosiderin laden macrophages or reactive astrocytes may be present
    Electron microscopy description
    Electron microscopy images

    Images hosted on other servers:

    TEM showing neuronal features

    Ultrastructure recapitulating neuropil and synapses

    Molecular / cytogenetics description
    Molecular / cytogenetics images

    Images hosted on other servers:

    DNA methylation based classification

    Sample pathology report
    • Brain, intraventricular mass, resection:
      • Central neurocytoma, CNS WHO grade 2
    Differential diagnosis
    Board review style question #1

    A 29 year old woman with an intraventricular brain mass is diagnosed with a central neurocytoma with histologic findings shown above. Which of the following immunohistochemical stains is most likely to be diffusely positive in this neoplasm?

    1. Chromogranin
    2. GFAP
    3. Olig2
    4. Synaptophysin
    Board review style answer #1
    D. Synaptophysin. Synaptophysin expression is the most reliable diagnostic marker for central neurocytoma, which generally shows low to absent immunoreactivity for chromogranin, GFAP and Olig2.

    Comment Here

    Reference: Central neurocytoma
    Board review style question #2
    Which of the following histological features is necessary for the designation of atypical central neurocytoma?

    1. Hypercellularity
    2. Lipomatous differentiation
    3. Macronucleoli
    4. Microvascular proliferation
    Board review style answer #2
    D. Microvascular proliferation: Brisk mitotic activity, necrosis and microvascular proliferation are the 3 histological features necessary for the classification of an atypical central neurocytoma. Answers A and C are incorrect because hypercellularity and macronucleoli are 2 of several criteria used for the diagnosis of an atypical meningioma but are not used to evaluate for atypical central neurocytoma. Answer B is incorrect because lipomatous differentiation may be seen in rare cases of central neurocytoma but does not confer an atypical designation.

    Comment Here

    Reference: Central neurocytoma

    Cerebellar liponeurocytoma
    Definition / general
    • Cerebellar liponeurocytoma is a rare, slow growing tumor of the central nervous system with advanced neuronal or neurocytic differentiation, variable glial differentiation and lipoma-like changes
    • Classified CNS WHO grade 2
    • Typically occurs in the posterior cranial fossa
    • To date, nearly 70 cases have been reported
    Essential features
    • Rare, slow growing tumor with advanced neuronal or neurocytic differentiation and focal lipoma-like changes, typically occurring in the cerebellum
    • Classified as CNS WHO grade 2
    • Extent of surgical resection is the main prognostic factor
    • Histologically composed of monotonous neurocytic cells with rare mitoses and intermingled lipidized cells
    • Diffusely positive for neuronal markers syanptophysin, NeuN and MAP2
    Terminology
    • Lipomatous medulloblastoma (not recommended)
    • Lipidized medulloblastoma (not recommended)
    • Medullocytoma (not recommended)
    • Lipomatous glioneurocytoma (not recommended)
    • Lipidized mature neuroectodermal tumor (not recommended)
    ICD coding
    • ICD-O: 9506/1 - cerebellar liponeurocytoma
    • ICD-11: 2A00.3 & XH2GB0 - central neurocytoma of brain & cerebellar liponeurocytoma
    Epidemiology
    Sites
    Pathophysiology
    • Supposed to originate from GABAergic neurons in the cerebellar ventricular zone, which may aberrantly differentiate into adipocyte-like tumor cells (J Neurooncol 2012;108:513)
    Etiology
    Clinical features
    Diagnosis
    • Based on imaging (computerized tomography, magnetic resonance) / biopsy / resection specimen
    Radiology description
    Radiology images

    Contributed by Valeria Barresi, M.D., Ph.D.
    T1 weighted MRI

    T1 weighted MRI

    T2 weighted MRI

    T2 weighted MRI



    Images hosted on other servers:

    Right cerebellopontine angle mass

    Prognostic factors
    Case reports
    Gross description
    Frozen section description
    • Tumor composed of small, roundish, uniform cells (World Neurosurg 2018;112:18)
    • Infrequent mitoses
    • Microvascular proliferation and necrosis are typically absent
    • Neuropil matrix around vessels may simulate perivascular pseudorosettes of ependymoma
    • Lipid accumulation can be demonstrated using oil red O staining (Neuropathol Appl Neurobiol 1993;19:95)
    Frozen section images

    Contributed by Valeria Barresi, M.D., Ph.D.
    Neurocytic cells

    Neurocytic cells

    Perivascular fibrillary areas

    Perivascular fibrillary areas

    Microscopic (histologic) description
    Microscopic (histologic) images

    Contributed by Valeria Barresi, M.D., Ph.D.
    Lipidized cells Lipidized cells

    Lipidized cells

    Perivascular matrix

    Perivascular matrix

    Synaptophysin immunostaining

    Synaptophysin immunostaining

    NeuN immunostaining

    NeuN immunostaining


    GFAP immunostaining

    GFAP immunostaining

    Olig2 immunostaining

    Olig2 immunostaining

    EMA immunostaining

    EMA immunostaining

    Ki67 immunostaining

    Ki67 immunostaining

    Virtual slides

    Images hosted on other servers:
    Cerebellar liponeurocytoma

    Cerebellar liponeurocytoma

    Cytology description
    • Small, uniform cells with rounded to oval nuclei showing salt and pepper chromatin and scant cytoplasm
    • Some cells show a single, large cytoplasmic vacuole displacing the nucleus to the periphery of the cell, similar to adipocytes (Arch Pathol Lab Med 2012;136:965)
    Cytology images

    Contributed by Valeria Barresi, M.D., Ph.D.
    Sheets of uniform neurocytic cells

    Sheets of uniform neurocytic cells

    Electron microscopy description
    Molecular / cytogenetics description
    Sample pathology report
    • Cerebellum, tumor resection:
      • Cerebellar liponeurocytoma, CNS WHO grade 2 (see comment)
      • Comment: The tumor is composed of monotonous neurocytic cells, with round nuclei, small nucleoli and scant cytoplasm, disposed in a diffuse growth pattern and showing foci of lipidized cells. Only scattered mitoses are present. Necrosis is absent. On immunohistochemistry, tumor cells are diffusely positive for synaptophysin and MAP2 and negative for Olig2 and EMA. Ki67 labeling index is < 5%.
    Differential diagnosis
    • Medulloblastoma:
      • Mainly affects children and young adults
      • Lacks lipidized cells
      • Shows high mitotic index and Ki67 labeling index
      • May show necrosis
      • May occasionally have foamy macrophages that mimic lipomatous differentiation of cerebellar liponeurocytoma
      • Classic subtype is composed of small blue round cells with hyperchromatic nuclei of various shapes
      • Desmoplastic / nodular subtype features reticulin free zones surrounded by densely packed, poorly differentiated, highly proliferative cells with moderately pleomorphic nuclei
      • Large / anaplastic subtype is composed of markedly pleomorphic cells
      • Harbors isochromosome 17q
      • May harbor PTCH, CTNNB and APC mutations
    • Clear cell ependymoma:
      • Shows perivascular pseudorosettes and rosettes
      • Lacks lipidized cells
      • Positive for EMA (dot-like)
      • Diffusely positive for GFAP
      • Negative for synaptophysin
    • Oligodendroglioma, IDH mutant and 1p / 19q codeleted:
      • Rare in the posterior fossa
      • Lacks lipidized cells
      • Positive for Olig2
      • Negative for synaptophysin
      • IDH1 / 2 mutated
      • 1p / 19q codeleted
    Board review style question #1

    The tumor shown above was found in the left cerebellar hemisphere of a 45 year old woman. Only rare mitoses are present. It shows diffuse immunostaining for synaptophysin, focal immunostaining for GFAP and is negative for Olig2 and EMA. What is the most likely diagnosis?

    1. Cerebellar liponeurocytoma
    2. Ependymoma
    3. Medulloblastoma
    4. Oligodendroglioma
    Board review style answer #1
    A. Cerebellar liponeurocytoma. This tumor is by definition located in the cerebellum. It is composed of small neurocytic cells with focal lipidized cells, is positive for synaptophysin and has a low mitotic index. Answer C is incorrect because medulloblastoma is not characterized by lipidized cells and it has a high mitotic index. Answer B is incorrect because ependymoma is GFAP positive and features EMA dot-like staining. Answer D is incorrect because oligodendroglioma is diffusely positive for Olig2 and negative for synaptophysin.

    Comment Here

    Reference: Cerebellar liponeurocytoma

    Chordoid glioma
    Definition / general
    • Rare, discrete, slow growing glial tumor of third ventricle of adults, characterized by chordoid architecture and myxoid background
    • First described in 1998 (J Neuropathol Exp Neurol 1998;57:283)
    • Cells may resemble ependyma of subcommissural organ, present in dorsocaudal third ventricle during embryonic life, which regresses after birth
    Terminology
    • "Glioma" since GFAP+
    Epidemiology
    • Uncommon ( < 50 cases reported)
    • Usually middle aged women (median age 45 years, 63% female)
    Sites
    • Low grade neoplasm arising in third ventricle hypothalamic region
    • Often attached to hypothalamic and suprasellar structures (63%)
    Grading
    • WHO grade 2
    • WHO 2000 lists as "glial tumor of uncertain origin"
    Radiology description
    Radiology images

    Images hosted on other servers:

    Axial CT scan reveals hyperdense mass

    Various images

    Prognostic factors
    Case reports
    Treatment
    Clinical images

    Images hosted on other servers:

    Optic chiasm enlargement

    Gross description
    • Discrete firm mass adherent to wall of third ventricle
    Microscopic (histologic) description
    • Chordoma-like features
    • Clusters and cords of epithelioid cells in mucinous matrix
    • Cells have abundant eosinophilic cytoplasm and round / oval nuclei with indistinct nucleoli
    • Also lymphoplasmacytic infiltrates, Russell bodies, discrete border with peritumoral piloid gliosis
    • May have chondroid metaplasia or papillary formations
    • No / rare mitotic figures, no vascular proliferation, no necrosis
    Positive stains
    Negative stains
    Electron microscopy description
    • Ependymal differentiation
    • Apical pole has microvilli and basal pole has hemidesmosome-like structures connecting cell membranes to basal lamina
    • Also submicroscopic cell body zonation and secretory granules (Am J Surg Pathol 2001;25:401)
    Differential diagnosis
    • Chordoid meningioma: whorls, psammoma bodies and nuclear pseudoinclusions, EMA+, GFAP-, well formed desmosomes, 22q-
    • Chordoma: infiltrates bone, physaliphorous cells, EMA+, diffusely keratin+, S100+, GFAP-, mitochondria rough ER complexes

    Chordoid meningioma
    Definition / general
    • Chordoid meningioma is a histological subtype of meningioma characterized by cords or trabeculae of epithelioid (or spindle) cells embedded in a mucin rich stroma and resembling chordoma
    • It is classified as CNS WHO grade 2, regardless of the presence of worrisome histological features (mitoses, spontaneous necrosis, brain invasion)
    Essential features
    • Histological subtype of meningioma that is characterized by cords or trabeculae of epithelioid (or spindle) cells embedded in a mucin rich stroma and resembling chordoma
    • CNS WHO grade 2
    • Recurrence risk is associated with subtotal resection, high Ki67 labeling index and atypical histological features (mitoses, brain invasion, spontaneous necrosis, small cell with high N:C ratio, hypercellularity, macronucleoli, sheeting)
    • Pure forms exist but it most commonly features intermingled areas of classical meningioma, with syncytial growth pattern, whorls and pseudoinclusions
    • Immunohistochemistry: EMA+, podoplanin+, possible focal cytokeratin+, S100-, brachyury-, GFAP-
    Terminology
    • Chordoid meningioma
    ICD coding
    • ICD-O: 9538/1 - chordoid meningioma
    • ICD-11: XH2LS4 - chordoid meningioma
    Epidemiology
    Pathophysiology
    Etiology
    Clinical features
    Diagnosis
    • Based on imaging (CT, MRI) / biopsy / resection specimen
    Radiology description
    Radiology images

    Contributed by Valeria Barresi, M.D., Ph.D.

    T1 weighted sagittal MRI

    T1 sagittal contrast enhancement

    T2 weighted axial MRI

    FLAIR axial MRI

    Prognostic factors
    Case reports
    Treatment
    Gross description
    Frozen section description
    • Differential diagnosis versus other tumor types: at least focal presence of psammoma bodies, meningothelial whorls or nuclear pseudoinclusions
    • Differential diagnosis versus other meningioma subtypes: cords of cells with eosinophilic cytoplasm in a myxoid background (NMC Case Rep J 2020;7:53)
    Intraoperative frozen / smear cytology images

    Images hosted on other servers:

    Intraoperative histopathological findings

    Vacuolated cells

    Microscopic (histologic) description
    Microscopic (histologic) images

    Contributed by Valeria Barresi, M.D., Ph.D.

    Epithelioid cells

    Spindle cells

    Intermingled typical meningioma


    Mitoses

    Alcian blue

    Podoplanin immunostaining

    Cytology description
    Positive stains
    Molecular / cytogenetics description
    Molecular / cytogenetics images

    Images hosted on other servers:

    FISH 18p, 22q, 14q, 1p

    Sample pathology report
    • Brain, extra-axial convexity mass:
      • Meningioma, subtype chordoid, CNS WHO grade 2 (see comment)
      • Comment: Meningothelial neoplasia showing epithelioid cells with vacuolated, eosinophilic cytoplasm or spindle cells, arranged in cords, within a myxoid matrix.
    Differential diagnosis
    • Chordoma:
      • Most commonly arises in the sacrococcygeal region and is usually extradural and bone invasive
      • Can be intracranial at the clivus, have secondary invasion of the dura or rarely may be extradural
      • Podoplanin negative, brachyury positive and showing extensive staining for CK AE1 / AE3, S100 and EMA
    • Chondrosarcoma:
      • Rarely, may be meningeal
      • Positive for S100 and negative for EMA
    • Chordoid glioma:
      • Rare glioma that is typically localized at the third ventricle
      • GFAP positive
    • Metastatic carcinoma:
      • Metastases of carcinomas may be meningeal
      • They show extensive cytokeratin staining, while chordoid meningioma features only focal staining for keratins
    Board review style question #1

    An extra-axial mass is found at the clivus without bone invasion. Histological examination shows a tumor with eosinophilic epithelioid cells in a myxoid stroma and focal areas with a syncytial growth pattern, whorls and nuclear pseudoinclusions. What is the most likely diagnosis?

    1. Chondrosarcoma
    2. Chordoid meningioma
    3. Chordoma
    4. Metastasis of carcinoma
    Board review style answer #1
    B. Chordoid meningioma. Chondrosarcoma may be meningeal and enter the differential diagnosis with chordoid meningioma. Chordoma is frequently localized at the clivus, though it is usually bone invasive. Even in the absence of immunohistochemistry, the presence of areas with meningothelial classical features (whorls, syncytial growth pattern, nuclear pseudoinclusions) is strongly suggestive of a meningioma.

    Comment Here

    Reference: Chordoid meningioma
    Board review style question #2
    An extra-axial mass is found in the spine. Histological examination shows a tumor composed of eosinophilic epithelioid cells in a myxoid matrix. At immunohistochemistry, it is EMA+, cytokeratin+ (focally), podoplanin+, S100- and brachyury -. Which is the most likely diagnosis?

    1. Chondrosarcoma
    2. Chordoid glioma
    3. Chordoid meningioma
    4. Chordoma
    Board review style answer #2
    C. Chordoid meningioma. Chordoma is EMA+ but also extensively positive for CK AE1 / AE3, S100 and brachyury, while it is negative for podoplanin. Chordoid glioma is localized at the third ventricle and is GFAP+. Chondrosarcoma is S100+ and EMA-.

    Comment Here

    Reference: Chordoid meningioma

    Choroid plexus cyst
    Definition / general
    • Small cyst of choroid plexus containing CSF
    Sites
    • May be present throughout ventricular system but usually in glomus of lateral ventricles
    Clinical features
    • More prevalent in fetuses with chromosomal abnormalities (trisomy 18, trisomy 21, Aicardi syndrome)
    • Common form affect fetuses in 1% of pregnancies; usually asymptomatic, resolves spontaneously by birth but large cysts can cause hydrocephalus
    • Chromosomal abnormalities, specifically trisomy 18, should be considered if cysts are large ( > 1 cm), bilateral or irregular or if maternal age ≥ 32 years (AJR Am J Roentgenol 2009;192:32)
    • In adults, usually asymptomatic, incidental postmortem finding
    Radiology description
    • On CT and MRI, usually show CSF density
    Radiology images

    Images hosted on other servers:

    MR shows small cyst

    Case reports
    Clinical images

    Images hosted on other servers:

    Cyst at foramen of Monro

    Microscopic (histologic) description
    • Cyst wall lined by cuboidal to columnar epithelium with occasional cobblestone appearance typical of normal choroid plexus
    • Some are devoid of epithelial lining
    Positive stains
    Negative stains

    Choroid plexus tumors (papilloma, atypical papilloma, carcinoma)
    Definition / general
    • Choroid plexus tumors are rare neoplasms derived from the choroid plexus epithelium
    • They occur within the ventricle system of the brain
    Essential features
    • Rare intracranial tumor arising in the ventricle, mainly occurring in children
    • 3 histological grades (WHO grade 1, 2, 3): choroid plexus papilloma (CPP), atypical choroid plexus papilloma (aCPP), choroid plexus carcinoma (CPC)
    • Histological classification is based on architecture (preservation of papillary pattern), cellular density, cytology (nuclear pleomorphism), proliferation (mitoses) and necrosis / brain invasion
    • Diagnostically, transthyretin (TTR), KIR7.1, cytokeratin and Ki67 immunohistochemistry are most helpful
    • Based on methylation profiling, these tumors are now categorized in to 3 subtypes
    • In line with other CNS tumors, integrated phenotype genotype diagnosis is preferred, which will guide appropriate management
    Terminology
    • Conventionally, as per current WHO classification, these tumors are termed as choroid plexus papilloma (CPP, WHO grade 1), atypical choroid plexus papilloma (aCPP, WHO grade 2) and choroid plexus carcinoma (CPC, WHO grade 3) (Acta Neuropathol 2016;131:803)
    • Based on recent DNA methylation profiling, they are being classified into 3 prognostically relevant epigenetic subgroups: supratentorial pediatric low risk choroid plexus tumors (CPP and aCPP), infratentorial adult low risk choroid plexus tumors (CPP and aCPP) and supratentorial pediatric high risk choroid plexus tumors (CPP and aCPP and CPC) (J Neurooncol 2020;148:39)
    ICD coding
    • ICD-O:
      • 9390/0 - choroid plexus papilloma, NOS
      • 9390/1 - atypical choroid plexus papilloma
      • 9390/3 - choroid plexus carcinoma
    • ICD-10:
      • D33.2 - benign neoplasm of brain, unspecified
      • C71.9 - malignant neoplasm of brain, unspecified
    Epidemiology
    Sites
    • Lateral ventricle (common in children)
    • Fourth ventricle (distributed across all age groups)
    • Rarely in spinal cord or ectopic sites
    • Multifocal in exceptional circumstances
    Pathophysiology
    • So far, limited literature on pathogenesis or tumorigenesis
    • Genetic or syndromic association present in a proportion of tumors
    • Choroid plexus papilloma and atypical choroid plexus papilloma are genetically similar and are distinct from choroid plexus carcinoma (Clin Cancer Res 2015;21:184)
    • Possible association with simian virus (SV40) reported (Virology 1995;212:710)
    • Possible association with inflammatory pathway described (Acta Neuropathol Commun 2019;7:95)
    Clinical features
    • Hydrocephalus / raised intracranial pressure
    • Headache, vomiting, papilledema
    Diagnosis
    • Clinical features, imaging (CT or preferably MRI)
    • Biopsy with histologic examination for definitive diagnosis
    Radiology description
    • Intraventricular papillary or lobulated lesions on MRI; hypo or isointense on T1, hyper or isointense on T2 and enhanced in postcontrast imaging (Cancer Imaging 2019;19:17)
    Radiology images

    Contributed by Kshitij Mankad, M.D.
    MRI coronal

    MRI coronal

    MRI sagittal

    MRI sagittal

    MRI transverse

    Prognostic factors
    • Historically, histological grade alone has been used for prediction of outcome
    • Recent studies suggest 3 prognostic risk groups based on epigenetics, patient age and tumor location (Neuro Oncol 2016;18:790):
      • Supratentorial pediatric low risk choroid plexus tumors
      • Infratentorial adult low risk choroid plexus tumors
      • Supratentorial pediatric high risk choroid plexus tumors
    • Cerebrospinal fluid spread, drop metastasis and tumor recurrence after treatment are known and when present, carry poorer prognosis (BMJ Case Rep 2012;2012:bcr0120125681)
    Case reports
    Treatment
    • Surgery to achieve gross total resection
    • Adjuvant radiotherapy or chemotherapy, based on the clinical need
    Gross description
    • Papillomas are well circumscribed cauliflower-like masses
    • Cysts, hemorrhages and calcifications may be present
    • Carcinomas are invasive, solid and necrotic
    Gross images

    Images hosted on other servers:

    Fourth ventricle tumor

    Microscopic (histologic) description
    • Choroid plexus papilloma (CPP, WHO grade 1):
      • Papillary (finger-like) architecture, resembling normal choroid plexus
      • Single layer of cuboidal to columnar monomorphic cells
      • Loss of cobblestone surface
      • Mild nuclear pleomorphism, mitotic activity rare (< 2/10 high power fields), lacks necrosis
    • Atypical choroid plexus papilloma (aCPP, WHO grade 2):
      • Higher cellularity relative to CPP
      • Moderate nuclear pleomorphism, blurring of papillary pattern
      • Occasional mitoses (> 2/10 high power fields), with or without necrosis
    • Choroid plexus carcinoma (CPC, WHO grade 3):
      • Frankly malignant
      • High cellularity, hyperchromatic nuclei and nuclear pleomorphism
      • Blurring of papillary pattern and solid arrangement
      • Frequent mitoses (> 5/10 high power fields), necrosis, with or without brain invasion
    Microscopic (histologic) images

    Contributed by Ashirwad Merve, M.B.B.S., Ph.D.
    Benign tumor Choroid plexus papilloma

    Choroid plexus papilloma

    Ki67 in choroid plexus papilloma

    Ki67 in choroid plexus papilloma

    Intermediate grade tumor Atypical choroid plexus papilloma

    Atypical choroid plexus papilloma

    Ki67 in atypical choroid plexus papilloma

    Ki67 in atypical choroid plexus papilloma


    Choroid plexus carcinoma Choroid plexus carcinoma

    Choroid plexus carcinoma

    Ki67 in choroid plexus carcinoma

    Ki67 in choroid plexus carcinoma

    Transthyretin

    Transthyretin

    Cytokeratin

    Cytokeratin

    SMARCB1 (INI1)

    SMARCB1 (INI1)

    Cytology description
    • Intraoperative crush smear: papillary strands with fibrovascular cores
    • Lesional cells exhibit relative cellular crowding and cytological atypia (in relation to normal choroid plexus epithelium)
    Cytology images

    Contributed by Ashirwad Merve, M.B.B.S., Ph.D.
    Intraoperative smear

    Intraoperative smear

    Positive stains
    Negative stains
    Molecular / cytogenetics description
    • The following 3 methylation classes of choroid plexus tumors are recognized based on the German Cancer Research Center (DKFZ) Heidelberg classification:
      • Plexus tumor, subclass adult
      • Plexus tumor, subclass pediatric A
      • Plexus tumor, subclass pediatric B (closely related to methylation cluster 3 described in Neuro Oncol 2016;18:790)
    • TP53 sequencing and testing for germ line TP53 mutation is advisable in children with choroid plexus carcinoma
    Molecular / cytogenetics images

    Contributed by Ashirwad Merve, M.B.B.S., Ph.D.
    Methylation array CNV plot

    Methylation array copy number variation plot

    Sample pathology report
    • Lateral ventricle, tumor, excision:
      • Atypical choroid plexus papilloma (WHO grade 2)
      • Methylation class: Plexus tumor, subclass pediatric B, family class PLEX_T
      • Molecular tests (sequencing): No TP53 mutation
      • Comment: The tumor shows moderate atypia, histologically equivalent to WHO grade 2. The methylation array has classified as subclass B, which is closely related to methylation cluster 3 described in Neuro Oncol 2016;18:790.
    Differential diagnosis
    Board review style question #1

    A 3 month old boy presented with restless crying and vomiting. His head size was larger than expected for his age. Imaging showed an 8 cm, lobulated, enhancing lesion in the right lateral ventricle. Histology is as above. Which of the following is true?

    1. Choroid plexus tumors are common in adults
    2. Recent studies suggest 3 distinct molecular entities based on methylation profiling, patient age and tumor location
    3. This is a choroid plexus carcinoma (WHO grade 3)
    4. TP53 mutation is strongly associated with choroid plexus papilloma
    Board review style answer #1
    B. Recent studies suggest 3 distinct molecular entities based on methylation profiling, patient age and tumor location. An atypical choroid plexus tumor is shown in the image.

    Choroid plexus tumors are rare intracranial tumors arising in the choroid plexus epithelium of the ventricles. They are more common in children. Histologically, they are classified into 3 categories: choroid plexus papilloma (CPP, WHO grade 1), atypical choroid plexus papilloma (aCPP, WHO grade 2) and choroid plexus carcinoma (CPC, WHO grade 3). Although any of these tumors can have CSF spread and recur following treatment, the latter is associated with poor outcome and higher recurrence rate. Recent studies have shown that CPP and aCPP are genetically similar but CPP and aCPP are genetically distinct from CPC. The CPCs are mostly driven by loss of function or mutations of tumor suppressor gene TP53 and may be associated germ line mutations (Li-Fraumeni syndrome). Furthermore, based on methylation profiling, tumor location and age, 3 distinct subgroups with prognostic significance have been described. The advances in understanding of genetics may help deliver appropriate personalized treatment.

    Comment Here

    Reference: Choroid plexus tumors (papilloma, atypical papilloma, carcinoma)
    Board review style question #2
    Which of the following is true regarding the histological differential diagnosis for choroid plexus carcinoma?

    1. Diffuse synaptophysin expression and Homer-Wright rosettes are features of choroid plexus carcinoma
    2. IDH1 is positive in choroid plexus carcinoma
    3. LIN28a is positive in choroid plexus carcinoma
    4. Nuclear INI1 (SMARB1) expression is retained in choroid plexus carcinoma but lost in atypical teratoid / rhabdoid tumor
    Board review style answer #2
    D. Nuclear INI1 (SMARB1) expression is retained in choroid plexus carcinoma but lost in atypical teratoid / rhabdoid tumor

    Choroid plexus carcinomas can mimic other malignant tumors of the childhood, such as CNS embryonal tumors, and it is important to exclude them to guide appropriate treatment. INI1 (SMARCB1) and SMARCA4 immunohistochemistry expression is retained in CPC, which helps to differentiate from aggressive tumor atypical teratoid / rhabdoid tumor (AT / RT). A negative LIN28a differentiates from embryonal tumor with multilayered rosettes (ETMR, bearing C19MC alterations). Diffuse synaptophysin expression with patchy GFAP and presence of Homer-Wright rosettes should raise the possibility of medulloblastoma. Diffuse GFAP expression with possible IDH1 expression (in younger adults) with microvascular endothelial proliferation and necrosis would indicate a high grade glioma / glioblastoma. DNA methylation array and classification of CNS tumors will help in definitive classification. Nevertheless, it is important to have a solid histological diagnosis in case the array fails to confidently classify the tumor.

    Comment Here

    Reference: Choroid plexus tumors (papilloma, atypical papilloma, carcinoma)

    Clear cell meningioma
    Definition / general
    • Rare variant, 0.2 - 0.8% of all meningiomas (J Neurooncol 2007;81:315)
    • WHO grade 2 due to aggressiveness
    • 20 - 60% chance of recurrence even with gross total resection
    • More common in younger patients (mean age 29 years)
    • Predilection for cauda equina and cerebellopontine areas
    Sites
    • Predilection for CP angle and cauda equina
    Radiology description
    • Indistinguishable from classic meningioma
    • Dural based, homogenously enhancing
    Radiology images

    Contributed by Chunyu Cai, M.D., Ph.D. (Case #514)
    MRI

    MRI



    Images hosted on other servers:
    Missing Image

    Various images

    Case reports
    Treatment
    • Gross total resection
    • Postsurgical radiation
    Microscopic (histologic) description
    • Patternless arrangement of clear cells with sometimes distinct cell borders
    • Prominent perivascular and interstitial collagen
    • Little to no mitotic activity
    • May not display any characteristic meningioma features (whorls, psamomma bodies, intranuclear inclusions)
    Microscopic (histologic) images

    Contributed by Chunyu Cai, M.D., Ph.D. (Case #514)
    H&E clear cell

    H&E clear cell

    H&E blocky collagen

    H&E blocky collagen

    IHC SMARCE1

    IHC SMARCE1

    IHC CAIX

    IHC CAIX

    Cytology description
    • Whorled, syncytial architecture composed of spindle to polygonal cells with vacuolated cytoplasm and bland nuclei (Diagn Cytopathol 1998;18:131)
    Positive stains
    Negative stains
    Electron microscopy description
    • Abundant cytoplasmic glycogen
    • Intermediate filaments
    • Interdigitation of cell membranes, desmosomes
    • Occasional cytoplasmic lumina (Ultrastruct Pathol 1999;23:51)
    Differential diagnosis
    Board review style question #1
    Which of the following genetic alterations is characteristic of clear cell meningioma?

    1. AKT1 mutation
    2. BAP1 mutation
    3. NF2 mutation
    4. SMARCE1 mutation
    5. TRAF7 and KLF4 co-mutations
    Board review style answer #1
    D. SMARCE1 mutation. Clear cell meningioma is a rare variant characterized by loss of nuclear SMARCE1 protein expression.

    Comment Here

    Reference: Clear cell meningioma

    Colloid cyst
    Definition / general
    Essential features
    • Benign, unilocular, epithelium lined, mucin filled cyst of third ventricle (eMedicine: Colloid Cysts [Accessed 31 October 2023])
    • Usually adults (20 - 50 years); rare in children
    • Located at anterosuperior third ventricle near foramen of Monro
    • Excellent prognosis
    Terminology
    • Colloid cyst of the third ventricle
    ICD coding
    • ICD-10: G93.0 - cerebral cysts
    Epidemiology
    Sites
    Etiology
    Diagrams / tables

    Images hosted on other servers:
    Mechanisms of sudden death due to colloid cyst

    Mechanisms of sudden death due to colloid cyst

    Typical location of colloid cyst

    Typical location of colloid cyst

    Clinical features
    • Due to its position, causes intermittent obstruction of cerebrospinal fluid (CSF) flow and obstructive hydrocephalus with manifestations of increased intracranial pressure
    • Headache is the most common symptom
    • May also cause nausea, vomiting, blurred vision, gait disturbance, urinary incontinence and personality changes (BMC Neurol 2022;22:397)
    • Sudden impaction (ball valve effect on the foramen of Monro) causes abrupt, transient lower limb paralysis (drop attacks) and rarely, sudden death (Emerg (Tehran) 2015;3:162)
    • May be asymptomatic
    Diagnosis
    • Neuroimaging: computed tomography (CT) and magnetic resonance imaging (MRI)
    • Biopsy
    Radiology description
    • Typical intraventricular location allows confident radiological diagnosis
    • CT: unilocular hyperdense mass at or near the foramen of Monro (AJNR Am J Neuroradiol 2000;21:1470)
    • MRI: spherical, usually nonenhancing, discrete cystic lesion at anterior third ventricle; most are intrinsically bright in precontrast T1 weighted MRI images (AJNR Am J Neuroradiol 2020;41:1833)
    Radiology images

    Contributed by Mohamed Kayed, M.D., Ph.D.
    T1 sagittal MRI

    Small colloid cyst, T1 sagittal MRI

    T1 sagittal MRI

    Small colloid cyst, T2 sagittal MRI

    T1 sagittal MRI

    Large colloid cyst, MRI

    T1 sagittal MRI

    Large colloid cyst, CT



    Images hosted on other servers:
    Extensive chronic hydrocephalus

    Extensive chronic hydrocephalus

    Large colloid cyst

    Large colloid cyst

    T1 hyperintense colloid cyst

    T1 hyperintense colloid cyst

    Prognostic factors
    Case reports
    Treatment
    • Excision (microsurgical or endoscopic) is curative (World Neurosurg 2021:149:e298)
    • Stereotactic aspiration (potential for cyst recurrence)
    • Observation may be reasonable in some stable, asymptomatic cases
    Clinical images

    Images hosted on other servers:
    Intraoperative view

    Intraoperative view of a colloid cyst

    Gross description
    • 1 - 2 cm; larger cysts have been reported
    • Round, unilocular, translucent with thin, glistening wall
    • Cyst filled with clear or turbid viscid mucin that solidifies after fixation (AJNR Am J Neuroradiol 2000;21:1470)
    • Specimens received are often only a wrinkled membrane
    Gross images

    Images hosted on other servers:
    2 cm colloid cyst

    2 cm colloid cyst

    endoscopically excised colloid cyst endoscopically excised colloid cyst

    Endoscopically excised colloid cyst

    Microscopic (histologic) description
    • Hypocellular, fibrous wall lined by simple to pseudostratified columnar epithelium with variable cilia or goblet cells (resembles bronchial epithelium) (Acta Neuropathol 1997;93:271, Diagn Cytopathol 2002;27:27)
    • Cyst lining may be modified by pressure atrophy (become low cuboidal or flattened) or degenerative changes
    • Unlike Rathke cleft and endodermal (enterogenous) cysts, lining epithelium is not prone to squamous metaplasia
    • Fragments of normal choroid plexus are frequently attached to cyst
    • Cyst contents are amorphous and proteinaceous, may show ghosts of desquamated lining cells and eosinophilic filamentous material (degenerated nucleoprotein and phospholipid) resembling infectious organisms (Actinomyces)
    • In chronic lesions, a xanthogranulomatous reaction may occur (Surg Neurol Int 2019:10:169)
    Microscopic (histologic) images

    Contributed by Eman Abdelzaher, M.D., Ph.D.
    fibrous wall and proteinaceous contents

    Fibrous wall and proteinaceous contents

    attached choroid plexus

    Attached choroid plexus

    pseudostratified ciliated cells pseudostratified ciliated cells

    Pseudostratified ciliated cells

    goblet cells

    Goblet cells


    cuboidal lining cuboidal lining

    Cuboidal lining

    xanthogranulomatous reaction xanthogranulomatous reaction xanthogranulomatous reaction

    Xanthogranulomatous reaction


    EMA EMA

    EMA

    S100

    S100

    CD68

    CD68

    Cytology description
    • Epithelial cells, cohesive sheets and individual ciliated cells and goblet cells (Diagn Cytopathol 2002;27:27)
    • Abundant amorphous proteinaceous material with or without Actinomyces-like nucleoprotein arrays
    • Presence of macrophages
    Cytology images

    Images hosted on other servers:
    thick proteinaceous material

    Thick proteinaceous material

    Negative stains
    Electron microscopy description
    • Epithelial nature of lining cells is evident by cytoplasmic tonofilaments and desmosomes
    • 6 cell types: ciliated cells, nonciliated cells with surface microvilli, goblet cells, basal cells, nonspecific small cells and occasional neuroendocrine cells with neurosecretory granules (Acta Neuropathol 1992;83:605)
    • Well formed basal lamina
    Electron microscopy images

    Images hosted on other servers:
    ciliary pattern of colloid cyst

    Ciliary pattern of colloid cyst

    Videos

    Colloid cyst

    Sample pathology report
    • Third ventricular cyst, endoscopic excision biopsy:
      • Colloid cyst
    Differential diagnosis
    • Rathke cleft cyst:
      • Intrasellar or suprasellar location
      • Prone to squamous metaplasia
    • Normal choroid plexus:
      • Sometimes dominant or only epithelial tissue
      • Papillary with cobblestone lining epithelium
      • No ciliated or goblet cells
      • EMA generally negative
    • Choroid plexus papilloma:
      • Papillary with pseudostratified lining epithelium
      • No ciliated or goblet cells
    • Papillary craniopharyngioma with xanthogranulomatous change:
      • Squamous lining
    Board review style question #1

    The brain cyst shown in the above image is strategically located near the foramen of Monro. Which of the following is the correct diagnosis?

    1. Arachnoid cyst
    2. Colloid cyst
    3. Endodermal (enterogenous) cyst
    4. Rathke cleft cyst
    Board review style answer #1
    B. Colloid cyst. Colloid cysts have a stereotypic location near the foramina of Monro, which causes intermittent obstruction of cerebrospinal fluid (CSF) flow and obstructive hydrocephalus. Answer D is incorrect because Rathke cleft cyst is sellar or suprasellar in location. Answer C is incorrect because endodermal (enterogenous) cysts mostly arise in the intraspinal compartment or in the posterior fossa. Answer A is incorrect because an arachnoid cyst is often located in the Sylvian fissure.

    Comment Here

    Reference: Colloid cyst
    Board review style question #2
    Which of the following brain cysts has a columnar cyst lining and is not prone to squamous metaplasia?

    1. Colloid cyst
    2. Endodermal (enterogenous) cyst
    3. Epidermoid cyst
    4. Rathke cleft cyst
    Board review style answer #2
    A. Colloid cyst. The lining epithelium of colloid cysts is not prone to squamous metaplasia. Answers B and D are incorrect because the lining epithelium of Rathke cleft cysts and endodermal (enterogenous) cysts is prone to squamous metaplasia. Answer C is incorrect because epidermoid cysts are lined by keratinized stratified squamous epithelium.

    Comment Here

    Reference: Colloid cyst

    DNA methylation classification of CNS tumors
    Definition / general
    Essential features
    • DNA methylation is a type of epigenetic modification in which a covalent bond in a nucleotide is enzymatically altered to produce a subtly different compound
    • A common example and one of the best studied is cytosine converted to 5 methylcytosine (5mC); this is when a DNA methyl transferase enzyme replaces the hydrogen attached to the carbon atom in position 5 of the cytosine molecule with a methyl group
    • In mammalian genomes, methylation typically occurs at the 5 prime cytosine of a cytosine guanine dinucleotide, which is connected by a phosphodiester bond (CpG) (Trends Genet 2022;38:676)
    Terminology
    • DNA methylation classification
    • Genome wide DNA methylation
    • Whole genome DNA methylation
    • DNA methylation
    • Methylation classifier
    • DNA methylation profiling
    Technologies for measuring DNA methylation
    • Most contemporary technologies for DNA methylation profiling rely on bisulfite conversion of DNA extracted from formalin fixed paraffin embedded tissue
    • Bisulfite treatment of DNA selectively deaminates cytosine and converts it to uracil but leaves 5 methylcytosine unaffected (Nucleic Acids Res 1980;8:4777, Proc Natl Acad Sci U S A 1992;89:1827)
    • Resultant bisulfite treated DNA can then be amplified by polymerase chain reaction, which amplifies uracil and thymine as thymine and 5 methylcytosine as cytosine (Proc Natl Acad Sci U S A 1992;89:1827)
    • Cytosine residues in the PCR products are therefore markers of 5 methylcytosine in the original input DNA (Proc Natl Acad Sci U S A 1992;89:1827)
    • As an alternative to the amplification and sequencing approach, the similar hybridization behavior of uracil and thymine can be exploited to detect cytosine to uracil / thymine polymorphisms in bisulfite treated DNA using DNA hybridization technology (Genome Res 2006;16:383)
    • Bisulfite sequencing
      • Bisulfite treated and PCR amplified DNA can be sequenced by, for example, massively parallel short read sequencing
      • All cytosines in the output sequences are interpreted as 5mC, so genome wide DNA methylation can be identified at nucleotide resolution (Proc Natl Acad Sci U S A 1992;89:1827, Nucleic Acids Res 2001;29:E65)
      • Genome wide bisulfite sequencing is still prohibitively expensive for large scale implementation
    • Early whole genome DNA methylation arrays
      • Illumina GoldenGate DNA Methylation BeadArray; first generation genome wide DNA methylation array, introduced in 2006 (Genome Res 2006;16:383)
      • Illumina Infinium HumanMethylation27K BeadChip; second generation genome wide DNA methylation array, introduced in 2009 (Epigenomics 2009;1:177)
      • Illumina Infinium HumanMethylation450K BeadChip; third generation genome wide DNA methylation array, introduced in 2011 (Genomics 2011;98:288, Epigenetics 2011;6:692)
    • Current whole genome DNA methylation array
      • Illumina Infinium MethylationEPIC BeadChip; fourth generation genome wide DNA methylation array, introduced in 2015
      • Version 1.0 includes 853,307 CpG sites, including 439,562 of the 482,421 CpGs (91%) included in the 450K microarray (Epigenomics 2016;8:389, Nucleic Acids Res 2017;45:e22)
      • Adds 333,265 CpGs located in enhancer regions identified by ENCODE and FANTOM5 (Epigenomics 2016;8:389)
      • Version 2.0 includes more than 935,000 CpG sites
      • Over 186,000 new probes to target known enhancers, super enhancers, CTCF binding domains and open regions of chromatin associated with primary tumors identified by ATAC seq and ChIP seq experiments
      • DNA input requirement: 250 ng
    Utility of whole genome DNA methylation for CNS tumors
    Limitations and potential pitfalls of methylation profiling
    • Like other studies, methylation profiling should be considered in conjunction with other histopathologic, genetic and clinical data in order to reach a diagnosis
    • Misleading methylation profiles can occur (i.e., methylation matches that apparently conflict with histologic, genetic, radiographic or clinical features)
    • Requires a relatively large amount of optimal input DNA (~250 ng)
    • Some methylation classes do not necessarily correspond to a CNS WHO tumor type
    • High confidence methylation classifier score may not be seen in up to 1/3 of cases (Neuro Oncol 2022;24:571, Clin Epigenetics 2019;11:185)
    • Factors that may contribute to low classifier score (no methylation match)
      • Low tumor cell content (low purity of sample)
      • Failed DNA bisulfite conversion
      • Poor quality DNA (e.g., old tissue blocks)
      • Single bioinformatic pipelines are reliant on a single reference dataset
      • Rare tumor types may not be present in reference dataset, so no class exists for them
    Molecular / cytogenetics images

    Contributed by P.J. Cimino, M.D., Ph.D. and Chris Dampier, M.D.
    Methylation copy number plot

    Methylation copy number plot

    Dimensionality reduction of medulloblastoma

    Dimensionality reduction of medulloblastoma



    Images hosted on other servers:
    Methylation based CNS tumor clustering

    Methylation based CNS tumor clustering

    Impact of tumor purity

    Impact of tumor purity

    Workflow in surgical neuropathology

    Workflow in surgical neuropathology

    Heterogeneity of histologic diagnoses

    Heterogeneity of histologic diagnoses

    Sample pathology report
    • Brain, cervical spinal cord, biopsy:
      • Histological classification: glial neoplasm with focal ependymal features
      • CNS WHO grade 2
      • Consensus methylation profiling class: spinal ependymoma
      • Integrated diagnosis: spinal ependymoma, CNS WHO grade 2
    Board review style question #1

    A 7 year old girl undergoes stereotactic needle biopsy for diagnosis of an intracranial intrinsic right parietal lobe mass. There is scant tissue in the biopsy and nucleic acid extraction is expected to yield material for only a single genome wide assay (e.g., cytogenomic array, next generation sequencing, DNA methylation array). Which of these features are specifically detected by DNA methylation array profiling?

    1. Chromosomal copy number alterations
    2. Genomic breakpoints to suggest gene fusions
    3. Methylation class
    4. MGMT promoter methylation status
    5. Single gene mutations (e.g., specific H3 G34 mutation)
    Board review style answer #1
    C. Methylation class is specifically detected by DNA methylation array profiling. Answer D is incorrect because MGMT promoter methylation status can also be detected by methylation specific PCR. Answers A and B are incorrect because chromosomal copy number alterations and genomic breakpoints to suggest gene fusions can also be detected by other modalities such as cytogenetic microarrays, next generation sequencing and fluorescence in situ hybridization. Answer E is incorrect because single gene mutations may be inferred by DNA methylation array but the DNA sequence is not directly detectable.

    Comment Here

    Reference: DNA methylation classification of CNS tumors
    Board review style question #2
    A 5 year old boy undergoes resection of a medulloblastoma from the posterior fossa. DNA methylation profiling of the tumor can determine which of the following?

    1. Medulloblastoma histologic subtype
    2. Medulloblastoma molecular subtype
    3. Mitotic count
    4. Patient age
    5. TP53 mutational status in SHH activated medulloblastoma
    Board review style answer #2
    B. Medulloblastoma molecular subtype. DNA methylation profiling is useful for determining medulloblastoma molecular subtype. Answer D is incorrect because patient age is not discernable by methylation profiling. Answer E is incorrect because TP53 status requires DNA sequencing, not methylation. Answers A and C are incorrect because medulloblastoma histologic subtype and mitotic counts are determined by microscopic examination, not methylation.

    Comment Here

    Reference: DNA methylation classification of CNS tumors

    Dermoid cyst
    Definition / general
    • Developmental lesions arising from embryologic displacement of ectoderm into meninges, ventricles or rarely parenchyma (eMedicine: Imaging in CNS Dermoid Tumors)
    • Benign cyst lined by keratinizing squamous epithelium and cutaneous adnexa
    • Uncommon; much less common than epidermoid cyst
    • Usually affects children
    • Distribution more restricted than epidermoid cysts, usually affects midline (mostly fontanelle or posterior fossa as midline cerebellar lesions or within the 4th ventricle), may affect spinal canal
    • May have sinus tract to skin surface in nasofrontal or occipital regions
    • May rupture and cause chemical meningitis
    Gross description
    • Thick walled with greasy pilosebaceous contents
    Microscopic (histologic) description
    • Fibrous wall lined by keratinizing squamous epithelium with skin adnexa, cyst contains squames, hair, sebum
    • Hair shafts highlighted with polarized light
    • May be hair matrix differentiation (shadow cells)
    • Rupture may cause granulomatous inflammation with foreign body giant cell reaction
    Positive stains
    Differential diagnosis

    Desmoplastic infantile astrocytoma / ganglioglioma
    Definition / general
    • Supratentorial superficially located cystic neuroepithelial tumors of infancy characterized by prominent desmoplasia with neoplastic glial component (desmoplastic infantile astrocytoma, DIA) or neoplastic glioneuronal component (desmoplastic infantile ganglioglioma, DIG)
    • Corresponds to WHO grade 1
    • DIA first described in 1982 by Taratuto et al. (J Neurosurg 1987;66:58)
    • DIG first described in 1987 by VandenBerg et al.
    • Since both lesions have similar radiological and clinical presentation, they are categorized together as desmoplastic infantile astrocytoma / ganglioglioma (DIA / DIG) in the WHO classification
    Epidemiology
    • Incidence: rare ( < 0.1% of CNS tumors)
    • Age and sex:
    Sites
    • Supratentorial
    • Usually frontoparietal
    Clinical features
    • Rapidly increasing head circumference, hydrocephalus, seizures
    Radiology description
    • Supratentorial
    • Superficially located
    • Large, involves more than one lobe
    • Cystic with solid area / mural nodule
    • Enhancing
    • Desmoplastic area exhibits hypointense signal on T2
    Radiology images

    Images hosted on other servers:

    Large right hemispheric mass

    Prognostic factors
    Case reports
    • 3 1/2 month old boy with mixed conventional and desmoplastic infantile ganglioglioma (Mod Pathol 2001;14:720)
    Treatment
    • Gross total resection
    • Chemotherapy if infiltrative or progressive
    • Residual disease may not grow and may spontaneously regress (Neurosurgery 2003;53:979)
    Gross description
    • Large (up to 13 cm), often involves multiple lobes
    • Deep macrocystic portion
    • Superficial solid leptomeningeal portion, firm to hard, focally attached to overlying dura
    Microscopic (histologic) description
    • Well delineated from normal brain
    • Desmoplastic leptomeningeal component
      • Involve the subarachnoid space and extends into Virchow-Robin spaces
      • Neoplastic neuroepithelial cells in desmoplastic spindled stroma arranged in fascicular and storiform patterns with pericellular reticulin deposition lending a mesenchymal appearance
      • Neoplastic neuroepithelial cells:
        • Astrocytic cells:
          • Only component in DIA
          • Spindled or gemistocytic neoplastic astrocytes
        • Neuronal component:
          • Seen in DIG in addition to neoplastic astrocytes
          • Small ganglion cells
          • Uncommonly large ganglion cells or areas resembling ganglioglioma
    • Immature small cell component (unclear prognostic significance)
      • Hypercellular poorly differentiated neuroepithelial cells
      • No desmoplasia
      • May show mitoses, vascular proliferation or necrosis
    • Calcification common, chronic inflammatory cells uncommon
    • Exceptionally, frank anaplastic features are encountered (high mitotic rate, vascular proliferation, palisading necrosis and high proliferation index)
    Cytology description
    • Low cellularity
    • Dispersed or variably sized clusters of large neuronal cells with abundant granular cytoplasm, eccentric hyperchromatic nuclei with undulating nuclear membranes and occasional binucleation, prominent nucleoli
    • Astroglial cells with smaller cytoplasmic rim, nuclear hyperplasia and more prominent irregularities in nuclear membranes
    • May have prominent degenerative changes, foamy macrophages
    • No vascular structures (Cytojournal 2005;2:1)
    Immunohistochemistry & special stains
    Electron microscopy description
    • Astrocytic tumor cells are partly invested by pericellular basal lamina

    Desmoplastic myxoid tumor of the pineal region, SMARCB1 mutant
    Definition / general
    • Pineal region neoplasm with ovoid or spindle shaped cells, containing inactivating alterations of the SMARCB1 / INI1 locus and variable amounts of desmoplasia and myxoid matrix
    Essential features
    • Predominates in adults and adolescents
    • Arises in the pineal region
    • Small to medium sized cells with ovoid to spindle shaped nuclei
    • Variable amounts of desmoplasia and myxoid matrix
    • Inactivating SMARCB1 / INI1 alteration
    Terminology
    • Desmoplastic myxoid tumor of the pineal region, SMARCB1 mutant
    ICD coding
    • ICD-11: 2A00.20 - tumors of the pineal gland or pineal region
    Epidemiology
    Sites
    • Pineal region
    Pathophysiology
    • Precise pathophysiology is unclear
    Etiology
    • Precise etiology is unclear
    Clinical features
    • Sequelae of obstructive hydrocephalus, such as blurred vision, headache, nausea, vomiting, dizziness
    Diagnosis
    • Identification of SMARCB1 / INI1 inactivating alteration and protein loss
    Laboratory
    • Serum and cerebrospinal fluid alpha fetoprotein and beta human chorionic gonadotropin are normal
    Radiology description
    Radiology images

    Contributed by Yue-E Wang, M.S., Ph.D., Jingjing Chen, Ph.D., Wei Wang, Ph.D., An-Li Zhang, M.D., Wenchao Zhou, Ph.D. and Hai-Bo Wu, M.D., Ph.D.

    MRI - sagittal T1

    MRI - sagittal T2

    Prognostic factors
    • Prognostic factors are not yet defined but reported prognosis much better than atypical teratoid / rhabdoid tumor
    • 70% of reported patients are alive without recurrence, with a median follow up of 29.5 months (0 months - 7 years) (Neuro Oncol 2022;24:847)
    Case reports
    Treatment
    • Surgical resection is the treatment of choice
    • Chemotherapy or radiotherapy has been given to a subset of patients (Neuro Oncol 2022;24:847)
    Microscopic (histologic) description
    • Epithelioid to spindle shaped cells embedded in a desmoplastic stroma and loose myxoid matrix
    • Rhabdoid cells may be rare
    • No high grade / malignant features such as brisk mitotic activity and tumor necrosis
    • Reference: Acta Neuropathol 2020;139:277
    Microscopic (histologic) images

    Contributed by Mariana Voudouri, M.D. and George Zanazzi, M.D., Ph.D.

    Epithelioid neoplasm with desmoplasia

    Cellular atypia

    Perivascular tumor cell accumulation

    CD34

    SMARCB1 / INI1



    Contributed by Branavan Manoranjan, M.D., Ph.D., Abdelsimar T. Omar II, M.D., Hai-Bo Wu, M.D., Ph.D., Robert Nordal, M.D. and Yves Starreveld, M.D., Ph.D.

    Microcysts and myxoid matrix

    Trichrome

    Positive stains
    Molecular / cytogenetics description
    • Defined by SMARCB1 / INI1 inactivating alterations
    • DNA methylation profile clusters adjacent to atypical teratoid / rhabdoid tumor - MYC and poorly differentiated chordomas
    • Reference: Acta Neuropathol 2020;139:277
    Sample pathology report
    • Pineal region tumor, resection:
      • Desmoplastic myxoid tumor of the pineal region, SMARCB1 mutant (see comment)
      • Molecular information: SMARCB1 / INI1 loss of nuclear expression
      • Immunohistochemistry: consistent with mutant
      • Comment: A CNS WHO grade has not yet been assigned to this tumor entity.
    Differential diagnosis
    Board review style question #1

    A 35 year old woman presents with progressively worsening blurred vision, headache, nausea and vomiting. MRI shows a pineal region mass with areas of contrast enhancement. A representative hematoxylin and eosin stained image of the tumor is shown. The tumor cells are CD34 positive and exhibit loss of nuclear SMARCB1 / INI1. The diagnosis is consistent with which of the following?

    1. Desmoplastic myxoid tumor of the pineal region, SMARCB1 mutant
    2. Germinoma
    3. Meningioma, CNS WHO grade 1
    4. Pineocytoma, CNS WHO grade 1
    Board review style answer #1
    A. Desmoplastic myxoid tumor of the pineal region, SMARCB1 mutant

    Comment Here

    Reference: Desmoplastic myxoid tumor, SMARCB1 mutant
    Board review style question #2
    A 28 year old man undergoes resection of a pineal region tumor. A representative hematoxylin and eosin stained section reveals spindled to epithelioid tumor cells embedded in a variably myxoid matrix and desmoplastic stroma. The Ki67 proliferation index is 4%. Which gene would you expect to be mutated?

    1. ATRX
    2. DICER1
    3. EWSR1
    4. SMARCB1
    Board review style answer #2

    Diffuse astrocytoma, MYB or MYBL1 altered (pending)
    [Pending]

    Diffuse hemispheric glioma, H3 G34 mutant
    Definition / general
    • Malignant, infiltrative, hemispheric, IDH wild type glioma with a G34R/V mutation in H3F3A
    Essential features
    • Predominates in children and young adults
    • Arises in the cerebral hemispheres
    • Histopathologically heterogeneous, often resembling CNS embryonal tumor
    • Histone H3 G34R/V mutations with concomitant TP53, ATRX and often PDGFRA mutations
    • Poor prognosis: overall survival 12 - 24 months
    Terminology
    • Other names used today or historically that pathologists may be more familiar with:
      • Pediatric glioblastoma
      • Pediatric high grade glioma
      • CNS embryonal tumor
      • Primitive neuroectodermal tumor (PNET)
    ICD coding
    • ICD-10:
      • C71.1 - malignant neoplasm of frontal lobe
      • C71.2 - malignant neoplasm of temporal lobe
      • C71.3 - malignant neoplasm of parietal lobe
      • C71.4 - malignant neoplasm of occipital lobe
      • C71.8 - malignant neoplasm of overlapping sites of brain
      • C71.9 - malignant neoplasm of brain, unspecified
    Epidemiology
    Sites
    • Cerebral hemispheres
    Pathophysiology
    • Histone H3 G34R/V mutations impair SETD2 activity, reducing H3K36me3 to promote a unique gene expression profile that supports tumorigenesis (Proc Natl Acad Sci U S A 2020;117:27354)
    • Histone H3 G34R/V mutations inhibit the neuronal differentiation of GSX2 / DLX expressing interneuron progenitors (Cell 2020;183:1617)
    • PDGFRA signaling may promote gliomagenesis (Cell 2020;183:1617)
    Etiology
    • Precise etiology is unclear, although recurrent histone H3 G34 R/V mutation is present
    • Child with Li-Fraumeni syndrome and a diffuse hemispheric glioma, H3 G34 mutant has been reported (Acta Neuropathol 2021;142:591)
    Clinical features
    • Symptoms related to increased intracranial pressure, such as headache, nausea and vomiting
    • May have site dependent neurological deficits
    • Reference: Brain Tumor Pathol 2017;34:103
    Diagnosis
    • Identification of a histone H3 G34R mutation (more commonly) or histone H3 G34V mutation that is confirmed by sequencing in a diffuse hemispheric glioma
    Radiology description
    • Large, hemispheric, bulky tumors that are T2 hyperintense on MRI (J Neuroradiol 2018;45:316)
    • In one study, MRI showed multifocal lesions in 2 of 8 patients, contrast enhancement in 6 of 8, necrosis in 3 of 8, cysts in 3 of 8, hemorrhage in 1 of 8 and calcifications in 1 of 8 (Clin Nucl Med 2018;43:895)
    Radiology images

    Contributed by Mariana Voudouri, M.D. and George Zanazzi, M.D., Ph.D.

    MRI of posterior left frontal tumor

    Sagittal T1

    Axial T2 / FLAIR

    Axial postcontrast

    Prognostic factors
    Case reports
    Treatment
    • Complete resection that is safely possible, followed by radiation and temozolomide
    Gross description
    • Soft, tan-gray mass within the cortex and subcortical white matter
    • Hemorrhage or necrosis may occur
    Microscopic (histologic) description
    • Tumor cells usually with either astrocytic morphology or CNS embryonal tumor morphology (Acta Neuropathol 2016;131:137)
    • Diffusely infiltrating growth pattern, often with high grade features such as mitotic activity, microvascular proliferation or necrosis
    • Even if high grade features are absent, the presence of an H3 G34R/V mutation confers a CNS WHO grade 4 (J Neuroradiol 2018;45:316)
    Microscopic (histologic) images

    Contributed by Mariana Voudouri, M.D. and George Zanazzi, M.D., Ph.D.

    Hypercellular with clustering

    Area of necrosis

    Marked pleomorphism

    ATRX


    GFAP

    Synaptophysin

    p53

    Positive stains
    Molecular / cytogenetics description
    Sample pathology report
    • Brain tumor, left frontal lobe, resection:
      • Diffuse hemispheric glioma, H3 G34 mutant, WHO grade 4
      • Histological diagnosis: CNS embryonal tumor, NEC, WHO grade 4
      • Molecular information:
        • H3 G34R: positive (immunohistochemistry; consistent with mutant)
        • ATRX: nuclear expression loss (immunohistochemistry; consistent with mutation)
        • p53: positive (immunohistochemistry; consistent with mutation)
        • IDH: negative (R132H immunohistochemistry; consistent with wild type)
    Differential diagnosis
    Board review style question #1

    A 17 year old girl presents with word finding difficulties, right facial droop and right arm weakness. MRI shows a large, left frontal lobe mass in the brain with areas of contrast enhancement. A representative hematoxylin and eosin stained section of the resected tumor is shown. Some tumor cells are GFAP positive and there is abundant p53. There is loss of ATRX without an IDH mutation. The diagnosis is consistent with

    1. Diffuse hemispheric glioma, H3 G34 mutant, WHO grade 4
    2. Diffuse midline glioma, H3 K27 altered, WHO grade 4
    3. Glioblastoma, IDH wild type, WHO grade 4
    4. Pleomorphic xanthoastrocytoma, WHO grade 3
    Board review style answer #1
    A. Diffuse hemispheric glioma, H3 G34 mutant, WHO grade 4

    Comment Here

    Reference: Diffuse hemispheric glioma, H3 G34 mutant
    Board review style question #2
    A 22 year old man undergoes resection of a temporal lobe brain tumor. Targeted next generation sequencing of the tumor reveals a missense mutation exchanging glycine for arginine in the histone H3.3 protein. This patient may be at increased risk for which syndrome?

    1. Li-Fraumeni syndrome
    2. Neurofibromatosis type 1
    3. Neurofibromatosis type 2
    4. Tuberous sclerosis
    Board review style answer #2
    A. Li-Fraumeni syndrome

    Comment Here

    Reference: Diffuse hemispheric glioma, H3 G34 mutant

    Diffuse leptomeningeal glioneuronal tumor
    Definition / general
    • Indolent, low grade neuroepithelial neoplasm that typically shows widespread leptomeningeal and superficial parenchymal CNS dissemination and oligodendroglioma-like cytology. This tumor was added provisionally to the WHO classification scheme of CNS tumors in 2016, and is no longer a provisional entity in the 2021 WHO CNS5 classification
    Essential features
    • Indolent, slow growing tumor with widespread and diffuse leptomeningeal involvement
    • Typically affects children; rare in adults
    • Presents with signs and symptoms of obstructive hydrocephalus
    • Olig2 and S100 positive, monomorphic oligodendroglial-like cells in desmoplastic or myxoid matrix
    • KIAA1549-BRAF fusion, 1p deletion common; 1p19q codeletion and BRAF V600E less common
    • References: Acta Neuropathol 2018;136:239, Acta Neuropathol 2012;124:627
    Terminology
    • Abbreviations: diffuse leptomeningeal glioneuronal tumor (DLGNT)
    • Other historical terms (Acta Neuropathol 2016;131:803):
      • Primary leptomeningeal oligodendrogliomatosis
      • Disseminated oligodendroglioma-like leptomeningeal neoplasm (DOLN)
      • Disseminated oligodendroglial-like neoplasm
      • Diffuse leptomeningeal neuroepithelial tumor
    ICD coding
    • ICD-10: C70.1 - malignant neoplasm of spinal meninges
    Epidemiology
    Sites
    • Widespread, diffuse involvement of spinal and intracranial leptomeninges (Acta Neuropathol 2012;124:627)
    • Within cranium, most common around posterior fossa, brainstem and base of brain
    • May fill ventricles and spread along Virchow-Robin spaces (Acta Neuropathol 2012;124:627)
    • May form cystic or solid tumor lesions in adjacent parenchyma; spinal > intracerebral
    • Spread in subarachnoid space: plaque-like or multinodular pattern (Acta Neuropathol 2012;124:627)
    Pathophysiology
    • Unknown how tumor arises
    Etiology
    Clinical features
    • Often acute, presents with signs and symptoms of increased intracranial pressure (Acta Neuropathol 2012;124:627)
    • Due to obstructive hydrocephalus
    • Nausea, vomiting and headache
    • Opisthotonos
    • Encasement or irritation of spinal or cranial nerves (Acta Neuropathol 2012;124:627)
    • May lead to palsies related to motor, sensory or autonomic function
    • Meningeal signs
    • Ataxia
    • Spinal cord compression
    • Rarely, seizures
    Diagnosis
    • Primarily made on radiologic features
    • Confirmed by tissue biopsy with ancillary immunohistochemistry, cytogenetic (FISH) and molecular testing
    Radiology description
    • MRI shows widespread contrast enhancement and thickening along spinal cord; often extends intracranially to posterior fossa, brainstem and basal brain (J Belg Soc Radiol 2017;101:19)
    • Small cystic or nodular T2 hyperintense lesions along subpial surface of spinal cord or brain
    • Discrete intraparenchymal lesions, most common in spinal cord; sometimes without diffuse spread (Acta Neuropathol Commun 2020;8:95)
    • Obstructive hydrocephalus with associated periventricular T2 hyperintensity
    Radiology images
    Prognostic factors
    Case reports
    Treatment
    Gross description
    • Postmortem examination confirms widespread, diffuse leptomeningeal spread in spinal and intracranial compartments (Brain Tumor Pathol 2018;35:209)
    • Multifocal extension along Virchow-Robin spaces (Acta Neuropathol 2012;124:627)
    • Growth along peripheral nerve roots and infiltration of basal cranial nerves and ganglia
    Microscopic (histologic) description
    • Low to moderate cellularity; diffuse growth or nests in leptomeninges (Brain Tumor Pathol 2015;32:49)
      • Often with desmoplastic or myxoid changes
    • Bland, monomorphous, oligodendroglial-like cells (Brain Tumor Pathol 2015;32:49)
      • With or without perinuclear halo
      • Uniform round nuclei with fine chromatin and inconspicuous nucleoli
    • Some with ganglion or ganglioid cells; may be associated with neuropil
    • Eosinophilic granular bodies (occasional), Rosenthal fibers (rare)
    • Low mitotic activity
    • Anaplasia is rare; associated with more aggressive clinical course (Brain Tumor Pathol 2015;32:49)
      • Nuclear enlargement
      • Atypia
      • Increased mitotic activity
      • Microvascular proliferation
      • Necrosis
    Microscopic (histologic) images

    Contributed by Katherine Schwetye, M.D., Ph.D.
    Oligodendroglial-like appearance

    Oligodendroglial-like appearance

    Tumor adjacent parenchyma

    Tumor adjacent parenchyma

    Bland cytomorphology and nuclear features

    Bland cytomorphology and nuclear features

    S100 positive

    S100 positive

    Ki67 low

    Ki67 low


    These photographs were previously published in Human Pathology, Vol. 70, Schwetye KE et al., "Unusual high-grade features in
    pediatric diffuse leptomeningeal glioneuronal tumor: comparison with a typical low-grade example", p.105-112 Copyright Elsevier (2017).
    Cytology description
    • Cerebrospinal fluid examination
    Positive stains
    Negative stains
    Molecular / cytogenetics description
    Sample pathology report
    • Spine, leptomeninges, biopsy:
      • Diffuse leptomeningeal glioneuronal tumor (see comment)
      • Comment: Hematoxylin and eosin examination of the biopsy material shows a moderately cellular, neoplastic proliferation within the leptomeninges with associated desmoplastic and myxoid change. The cells show relatively monomorphic, bland morphology, round nuclei with perinuclear haloes (reminiscent of an oligodendroglial-like appearance) and inconspicuous nucleoli. Mitotic activity is not identified. Necrosis is not identified. Immunohistochemical stains demonstrate reactivity for S100 and Olig2. Ki67 shows a low (< 2%) index of proliferation. FISH studies confirm the KIAA1549-BRAF rearrangement and deletion of chromosome 1p. Taken together, the radiologic, histomorphologic, immunohistochemical and cytogenetic features support a diagnosis of diffuse leptomeningeal glioneuronal tumor.
    Differential diagnosis
    • Pilocytic astrocytoma:
      • Typically intraparenchymal tumor without primary leptomeningeal spread
      • More predominant Rosenthal fibers
      • Glomeruloid microvascular proliferation does not alone confer worse prognosis
    • Oligodendroglioma:
      • Diffusely infiltrative glial tumor
      • Typically intraparenchymal
      • IDH1 / IDH2 mutations as a requisite feature for diagnosis
    • Macrophage rich reactive / inflammatory process:
      • CD68 positive macrophages
    • Tumors metastatic to the leptomeninges:
      • Clinical history, more likely adults
    Board review style question #1
    Common genetic alterations in the diffuse leptomeningeal glioneuronal tumor include

    1. H3F3 K27M
    2. IDH1 R132H
    3. KIAA1549-BRAF translocation and 1p deletion
    4. PTEN loss
    Board review style answer #1
    C. KIAA1549-BRAF translocation and 1p deletion

    Comment Here

    Reference: Diffuse leptomeningeal glioneuronal tumor
    Board review style question #2

    Based on the image shown above in conjunction with MRI findings of an enhancing, spinal leptomeningeal process without a distinctive intraparenchymal lesion in a 5 year old boy, what is the expected pattern of immunoreactivity?

    1. S100 positive, Olig2 positive, H3F3a K27M positive
    2. S100 positive, Olig2 positive, IDH R132H positive
    3. S100 negative, Olig2 positive, IDH R132H positive
    4. S100 positive, Olig2 positive, IDH R132H negative
    Board review style answer #2
    D. S100 positive, Olig2 positive, IDH R132H negative

    Comment Here

    Reference: Diffuse leptomeningeal glioneuronal tumor

    Diffuse low grade glioma, MAPK pathway altered
    Definition / general
    • Low grade, infiltrative, pediatric glioma with an alteration in a MAP kinase pathway gene such as FGFR1 or BRAF
    • Tumor is IDH wild type, histone H3 wild type and does not have a homozygous deletion of CDKN2A
    Essential features
    • Predominantly in children
    • Arises anywhere in the central nervous system, often in the cerebral hemispheres or posterior fossa
    • Histopathologically heterogeneous, often resembling low grade astrocytoma or oligodendroglioma
    • MAP kinase pathway alteration without alterations in IDH1 / IDH2, histone H3 or homozygous deletion of CDKN2A
    Terminology
    • Pediatric low grade glioma
    • Formerly, pediatric oligodendroglioma or diffuse astrocytoma
    ICD coding
    • ICD-10:
      • D33.0 - benign neoplasm of brain, supratentorial
      • D33.1 - benign neoplasm of brain, infratentorial
      • D43.0 - neoplasm of uncertain behavior of brain, supratentorial
      • D43.1 - neoplasm of uncertain behavior of brain, infratentorial
    Epidemiology
    • Predominantly in children
    • Precise incidence data not yet available
    Sites
    • Cerebral hemispheres, diencephalon, brainstem, cerebellum and spinal cord, with possible site predilections depending on histology and genetic alterations (Cancer Cell 2020;37:569)
    Pathophysiology
    • Missense mutation, intragenic duplication or fusion may constitutively activate a receptor tyrosine kinase, such as FGFR or NTRK, leading to aberrant recruitment of the G protein RAS, which activates the RAF / MEK / ERK cascade (Acta Neuropathol Commun 2020;8:30)
    • Missense mutations in RAS or RAF (especially BRAF V600E) can cause gliomagenesis in the absence of an upstream alteration (Acta Neuropathol Commun 2020;8:30)
    Etiology
    • Precise etiology is unclear
    Clinical features
    • Symptoms related to increased intracranial pressure, such as headache, nausea and vomiting
    • May have site dependent neurological deficits
    • Longstanding epilepsy is common
    Diagnosis
    Radiology description
    • Diffuse lesion, highlighted by T2 FLAIR on MRI, with possible cystic elements
    • May be heterogeneously enhancing on MRI
    Radiology images

    Contributed by Mariana Voudouri, M.D. and George Zanazzi, M.D., Ph.D.

    MRI coronal T2 brainstem tumor

    MRI axial T2 / FLAIR right cerebellar tumor

    MRI axial T1 post contrast right cerebellar flocculus tumor

    Prognostic factors
    • Currently unclear but rarely undergo malignant transformation
    Case reports
    • 9 year old boy with brainstem mass growing slowly over 8 years and low grade astrocytoma containing FGFR2-VPS35 fusion (Cold Spring Harb Mol Case Stud 2020;6:a005660)
    • 15 year old with seizure and temporal lobe mass containing oligodendroglioma-like cells and BRAF V600E mutation (Brain Pathol 2020;30:515)
    • 16 year old boy with developmental delay, germline 7q11.22 deletion and a low grade astrocytoma with BRAF V600E mutation in right temporal lobe (Clin Case Rep 2017;6:274)
    Treatment
    Gross description
    Microscopic (histologic) description
    • General features: infiltrative, glial tumor cells without a neuronal component
    • BRAF V600 altered tumors tend to have densely fibrillary areas and microcalcifications (Acta Neuropathol 2015;130:575)
    • FGFR1 altered tumors tend to have oligodendrocyte-like cells, rare or no mitoses, no necrosis and no microvascular proliferation (Acta Neuropathol 2016;131:833)
    • In a large study excluding NF1 patients, 5.9% of pediatric low grade gliomas exhibited astrocytic differentiation and 3.0% exhibited oligodendroglial differentiation (Cancer Cell 2020;37:569)
    Microscopic (histologic) images

    Contributed by Mariana Voudouri, M.D. and George Zanazzi, M.D., Ph.D.

    Astrocytic morphology

    Pleomorphism

    Oligodendroglial morphology

    GFAP

    ATRX

    Ki67

    Negative stains
    Molecular / cytogenetics description
    Sample pathology report
    • Brain tumor, right frontal lobe, resection:
      • Integrated diagnosis: diffuse low grade glioma, MAPK pathway altered
      • Histological diagnosis: diffuse astrocytoma, low grade
      • Molecular information:
        • IDH1 R132H: negative (immunohistochemistry; consistent with wild type)
        • H3 G34R / V: negative (immunohistochemistry; consistent with wild type)
        • CDKN2A: intact (fluorescence in situ hybridization)
        • BRAF V600E: positive (immunohistochemistry; consistent with mutation)
      • Comment: Sections demonstrate a diffusely infiltrating glioma with moderate cellularity. The tumor is composed of fibrillary astrocytic cells. No mitotic activity, microvascular proliferation or necrosis noted. No Rosenthal fibers or eosinophilic granular bodies are present. A CNS WHO grade has yet to be assigned.
    Differential diagnosis
    Board review style question #1

    A nonenhancing tumor was discovered in the right frontal lobe of a 5 year old boy. Resection revealed fibrillary astrocytic tumor cells without mitotic activity, microvascular proliferation or necrosis. Tumor cells showed immunopositivity for Olig2 and GFAP. What molecular alteration would you expect?

    1. BRAF V600E
    2. Histone H3 G34R
    3. Histone H3 K27M
    4. IDH1 (R132H)
    Board review style answer #1
    Board review style question #2
    An 8 year old girl with lower cranial nerve deficits was found to have a large, infiltrative, minimally enhancing lesion in the left cerebellum, cerebellopontine angle and extending to the thalamus. She underwent a biopsy. Targeted next generation sequencing of the tumor reveals a missense KRAS mutation. What signaling pathway is most likely to be activated in this tumor?

    1. EGFR
    2. MAP kinase
    3. TGF beta
    4. Wnt
    Board review style question #2

    Diffuse midline glioma, H3 K27 altered
    Definition / general
    • WHO 2021 definition: diffuse midline glioma, H3 K27-altered, is an infiltrative midline glioma with loss of H3 p.K28me3 (K27me3) and usually, either an H3 c.83A>T p.K28M (K27M) substitution in one of the histone H3 isoforms (H3.1, H3.2 or H3.3), aberrant overexpression of EZHIP or an EGFR mutation (CNS WHO grade 4)
    Essential features
    • Predominant in children but also occurs in adults
    • Arises in the midline; the brain stem, thalamus and spinal cord are the most common locations
    • High grade features, such as mitotic activity, microvascular proliferation and necrosis, may be seen but are not necessary for the diagnosis
    • Diffusely infiltrative of both adjacent and distant brain structures
    • Poor prognosis with 2 year survival rate of < 10%
    Terminology
    • Brain stem and pontine lesions previously termed brain stem glioma and diffuse intrinsic pontine glioma (DIPG)
    ICD coding
    • ICD-10: C71.9 - malignant neoplasm of brain, unspecified
    Epidemiology
    • Official incidence data are not available; this is because, historically, infiltrative gliomas arising in the midline have not been distinguished from other infiltrative gliomas in large registries
    • M = F
    • Median age at diagnosis is 5 - 11 years, with tumors that arise in the pons occurring at a younger age (~7 years) than those that arise in the thalamus (~11 years) (Neuro Oncol 2014;16:iv1)
    • Those harboring EGFR amplifications, containing mutations in isoform histone H3.3 or resulting from EZHIP overexpression, typically occur at 7 to 8 years of age, while those with K27M mutations in isoform histone H3.1 or H3.2 tend to occur in younger patients (~5 years) (Cancer Cell 2017;32:520)
    Sites
    • Most commonly located in the spinal cord, pons and thalamus
    • Occasionally arises in the cerebellum
    Pathophysiology
    • No genetic susceptibility is known; however, rare cases of H3 K27-altered tumors have been seen in patients with genetic tumor syndromes such as NF1, Li-Fraumeni syndrome or mismatch repair deficiency
    Etiology
    Clinical features
    • Patients typically present with evidence of cerebrospinal fluid obstruction or brain stem dysfunction, such as cranial nerve abnormalities, ataxia and long tract signs that develop over a short period of time (Front Oncol 2012;2:205)
    • Tumors that arise in the thalamus are often associated with motor weakness and gait disturbance (Neuro Oncol 2011;13:680)
    Diagnosis
    • Typically diagnosed by imaging (MRI) and stereotactic biopsy; resection is often not possible given the involvement of critical brain stem structures
    Grading
    • Grade 4 of 4
    • Presence of H3 K27-alteration in an infiltrative glioma arising in the midline is sufficient for a grade 4 designation, even in the absence of necrosis or microvascular proliferation (Acta Neuropathol 2014;128:573)
    Radiology description
    • On MRI, typically T1 hypointense and T2 hypertintense
    • Contrast enhancement (typically < 25% of tumor volume), hemorrhage or necrosis may be seen (J Neurooncol 2011;105:119)
    • Tumors arising in the pons typically present as large expansile masses that occupy > 67% of the pons (Front Oncol 2012;2:205)
    • Exophytic component may be present
    • Infiltration into adjacent structures, such as the cerebellar peduncles, midbrain or medulla is frequent
    Radiology images

    Contributed by Rawia Mubarak Mohamed, M.D. and Najla Saleh Ben Gashir, M.D. (Case #477)
    Case #477

    Tumor in thalamic region



    Images hosted on other servers:

    CT - axial noncontrast: pontine tumor

    MRI - axial T2: pontine tumor

    MRI - sagittal T1: pontine tumor

    MRI - sagittal T2: pontine tumor

    MRI - axial T2: pontine tumor

    Prognostic factors
    Case reports
    Treatment
    Gross description
    • Infiltrative nature causes enlargement and distortion of involved anatomical structures
    • Focal discoloration and softening, indicating hemorrhage or necrosis, may be present (Neurol Med Chir (Tokyo) 2003;43:375)
    Microscopic (histologic) description
    • Tumor cells typically have an astrocytic morphology and may be small and monomorphic to (occasionally) large and pleomorphic (Neurol Med Chir (Tokyo) 2003;43:375)
    • Show an infiltrative growth pattern with tumor cells diffusely growing among native neurons and invading into adjacent structures
    • Occasionally, an oligodendroglial-like pattern with halos may be seen
    • Some cases will not show mitoses, necrosis or microvascular proliferation consistent with a WHO grade 2 histologic appearance; however, in the presence of H3 K27-alteration, WHO grade 4 is warranted given the aggressive nature of these tumors (Acta Neuropathol 2014;128:573)
    Microscopic (histologic) images

    Contributed by John DeWitt, M.D., Ph.D.

    Diffusely infiltrating astrocytic tumor cells

    Necrosis


    H3 K27M

    R132H-IDH1

    ATRX

    p53



    Contributed by Chunyu Cai, M.D., Ph.D.
    Clusters of tumor cell seeding

    Clusters of tumor cell seeding

    H3K27M

    H3 K27M

    Midline glioma H3K27

    H3K27Me3



    Contributed by Rawia Mubarak Mohamed, M.D. and Najla Saleh Ben Gashir, M.D. (Case #477)

    High grade infiltrating glial cells


    Olig2

    H3 K27M

    Positive stains
    Negative stains
    • R132H-IDH1
    • Keratins (although cocktails may show cross reactivity with GFAP)
    • H3K27me3: H3 K27-altered tumors show loss of H3K27me3 staining, a finding that by itself is not specific, however, may be helpful in identifying diffuse midline gliomas with H3 p.K28I (K27I) mutation or EZHIP overexpression, which are negative for H3K27M IHC but show loss of nuclear H3K27me3 expression
    • Reference: Brain Pathol 2016;26:569
    Molecular / cytogenetics description
    • Various molecular techniques including sequencing, expression arrays and methylation patterns are consistent with 4 distinct subtypes of diffuse midline gliomas that are defined by oncohistone alterations:
      • H3.3 c.83A>T p.K28M (K27M) mutant
      • H3.1 or 3.2 c.83A>T p.K28M (K27M) mutant
      • H3 wild type with aberrant overexpression of EZHIP
      • EGFR mutation or amplification
    • Additional mutations seen in diffuse midline glioma, H3 K27-altered (Nat Genet 2014;46:451, Nat Genet 2014;46:457, Nat Genet 2014;46:444, J Neurosurg 1999;90:833):
      • TP53 (~50%)
      • PDGFRA amplification (~30%)
      • CDK4 / 6 or CCND1 - 3 amplification (~20%)
      • ACVR1 mutation (~20%)
      • PPM1D mutation (~15%)
      • MYC / PVT1 amplification (~15%)
      • ATRX mutation (~15%)
      • CDKN2A / B homozygous deletion (< 5%)
    Sample pathology report
    • Brain, pons, biopsy:
      • Integrated diagnosis: diffuse midline glioma, H3 K27-altered, WHO grade 4 of 4 (see comment)
        • Histological diagnosis: astrocytoma with proliferative activity
        • CNS WHO grade 4 of 4
        • Molecular information:
          • IDH: negative (R132H immunohistochemistry; consistent with wild type)
          • ATRX: nuclear expression retained (immunohistochemistry; consistent with wild type)
          • p53: negative (immunohistochemistry; consistent with wild type)
          • H3K27M: positive (immunohistochemistry; consistent with mutant)
      • Comment: The specimen consists of core biopsy specimens of white matter with moderately atypical infiltrating astrocytic tumor cells. There is no evidence of vascular proliferation or necrosis. Scattered mitoses are seen. Although the histologic grade is that of a grade 3 astrocytoma, the positivity for H3K27M is consistent with a diagnosis of diffuse midline glioma, H3 K27-altered, which are considered grade 4 lesions due to their historically aggressive clinical behavior
    Differential diagnosis
    • Astrocytoma, IDH mutant, CNS WHO grade 2:
      • Grade 2 histology with IDH mutation present
    • Astrocytoma, IDH mutant, CNS WHO grade 3:
      • Grade 3 histology with IDH mutation present
    • Astrocytoma, IDH mutant, CNS WHO grade 4:
      • Grade 4 histology with IDH mutation present or lower grade histology with IDH mutation and homozygous deletion of CDKN2A / CDKN2B
    • Glioblastoma, IDH wild type:
      • Astrocytoma lacking IDH or H3 K27M mutations with either grade 4 histology or any of the following molecular alterations: EGFR amplification, TERT promoter mutation, gain of chromosome 7 / loss of chromosome 10
      • Tumors do not show loss of H3 K27 trimethylation
    • Posterior fossa (PFA) ependymoma (Childs Nerv Syst 2017;33:1047, Nat Commun 2019;10:2146):
      • Will show loss of H3K27Me3 (trimethyl mark) as in diffuse midline glioma, H3 K27-altered and rare cases also contain H3 K27 mutations
      • Found in the posterior fossa where diffuse midline glioma, H3 K27-altered may also occur
      • Differentiated by ependymal differentiation (round to oval glial tumor cells with perivascular pseudorosettes) as opposed to astrocytic differentiation of diffuse midline glioma, H3 K27-altered
      • Molecular studies including sequencing and methylation profiling could be considered in histologically ambiguous cases
    Board review style question #1

    This tumor was found arising in the pons of a 7 year old boy. The presence of which molecular alteration would warrant a WHO grade 4 designation?

    1. 1p / 19q codeletion
    2. H3 K27M mutation
    3. IDH1 mutation
    4. KIA1549-BRAF fusion
    5. TP53 mutation
    Board review style answer #1
    Board review style question #2
    A work up of a biopsy of a diffusely infiltrating glial tumor from an 11 year old girl reveals sequencing results that show a methionine substitution for a lysine at position 27 in one of the genes encoding histone H3 (H3 K27M). Where is this tumor most likely arising from?

    1. Cerebellar hemisphere
    2. Frontal lobe
    3. Lateral ventricle
    4. Temporal lobe
    5. Thalamus
    Board review style answer #2
    Board review style question #3
    Which statement about diffuse midline glioma, H3 K27-altered is true?

    1. Despite its name, it is typically not found in the midline
    2. It may lack high grade histologic features but is still considered grade 4
    3. The prognosis varies based on the histologic features
    4. This diagnosis includes midline gliomas that are diffusely infiltrating but have not been tested for the H3 K27M mutation
    Board review style answer #3
    B. It may lack high grade histologic features but is still considered grade 4. This tumor is considered grade 4 regardless of the histologic features and all of these tumors are considered to have a dismal prognosis. Most of these tumors appear in the midline. Molecular or IHC testing to confirm one of the diagnostic alterations or loss of H3 K27 trimethylation is required for this diagnosis.

    Comment Here

    Reference: Diffuse midline glioma, H3 K27-altered

    Diffuse pediatric type high grade glioma, H3 wildtype and IDH wildtype (pending)
    [Pending]

    Dysembryoplastic neuroepithelial tumor
    Definition / general
    • Cortex based glioneuronal neoplasm that is often located in the mesial temporal lobe of adolescents and young adults and associated with medically refractory epilepsy, usually with activating mutations of FGFR1, CNS WHO grade 1 (Neurosurgery 1988;23:545)
    Essential features
    Terminology
    • Dysembryoplastic neuroepithelial tumor
    • Simple dysembryoplastic neuroepithelial tumor
    • Complex dysembryoplastic neuroepithelial tumor
    ICD coding
    • ICD-O: 9413/0 - dysembryoplastic neuroepithelial tumor
    • ICD-11: 2A00.21 - mixed neuronal glial tumors
    • ICD-11: XH0H76 - dysembryoplastic neuroepithelial tumor
    Epidemiology
    Sites
    Pathophysiology
    • Activating alteration in FGFR1 (receptor tyrosine kinase) > activation of MAPK pathway > dysregulated cell growth, proliferation and differentiation (Acta Neuropathol 2016;131:847)
    Etiology
    Diagrams / tables

    Images hosted on other servers:

    Relative distribution of LEATs

    Proposed modification of LEAT classification

    Nodular appearing neoplasm

    Clinical features
    Diagnosis
    • Cortical glioneuronal tumor with presence of specific glioneuronal component
    • FGFR1 alteration or DNA methylation profile can help in challenging cases
    • Preferable to make diagnosis in context of early onset focal epilepsy
    Radiology description
    Radiology images

    Images hosted on other servers:

    Representative imaging features in adolescent

    Typical imaging features

    10 year old girl with seizure

    Prognostic factors
    Case reports
    Treatment
    Clinical images

    Images hosted on other servers:

    Surgical resection of epileptogenic tumor

    Gross description
    • Usually poorly demarcated (Epileptic Disord 2002;4:99)
    • Located predominantly in gray matter and subcortical white matter
    • May contain solid, mucoid or cystic components
    Gross images

    Images hosted on other servers:

    Frontobasal surgical specimen

    Microscopic (histologic) description
    • Growth pattern
    • Pathognomonic appearance
      • Bundles of axons lined by small oligodendroglia-like cells form columns oriented perpendicularly to the cortical surface with intervening cytologically normal neurons floating in a myxoid matrix (Neurosurgery 1988;23:545)
    • Absence of the following:
      • Dysplastic ganglion-like cells
      • High cellularity
      • Necrosis
      • Perivascular lymphoid infiltrates
      • Eosinophilic granular bodies
    • Simple form
      • Pathognomonic component alone
    • Complex form
    Microscopic (histologic) images

    Contributed by P.J. Cimino, M.D., Ph.D. and Chris Dampier, M.D.
    Nodular growth pattern

    Nodular growth pattern

    Specific glioneuronal (pathognomonic) component

    Specific glioneuronal (pathognomonic) component

    Floating neuron

    Floating neuron

    Cystic component

    Cystic component

    GFAP

    GFAP

    IDH1 p.R132H

    IDH1 p.R132H

    Cytology description
    • Smear preparation (alcohol fixed, H&E stained) (Acta Cytol 2022;66:142)
      • Uniform round cells
      • Mucinous and fibrillary background
    Cytology images

    Images hosted on other servers:

    Smear preparations

    Partially arranged as columnar structures

    Fine neuritic processes

    Positive stains
    Negative stains
    Molecular / cytogenetics description
    Molecular / cytogenetics images

    Contributed by P.J. Cimino, M.D., Ph.D. and Chris Dampier, M.D.
    SCNA plot

    SCNA plot

    Videos

    Dysembryoplastic neuroepithelial tumor (DNET)

    Sample pathology report
    • Brain, temporal lobe, biopsy:
      • Dysembryoplastic neuroepithelial tumor, CNS WHO grade 1
      • Positive for FGFR1 alteration
    Differential diagnosis
    Board review style question #1

    The tumor shown above is from the mesial temporal lobe of a child. What is the most common underlying genetic alteration in this entity?

    1. BRAF V600E mutation
    2. FGFR1 alteration
    3. IDH1 R132H
    4. NF1 loss
    5. TP53 mutation
    Board review style answer #1
    B. FGFR1 alteration. TP53 mutations are uncommon in dysembryoplastic neuroepithelial tumor (E). IDH1 R132H is the most common mutation in IDH mutant astrocytomas and oligodendrogliomas but should be negative in dysembryoplastic neuroepithelial tumor (C). BRAF p.V600E mutation has been reported in dysembryoplastic neuroepithelial tumor but methylation profiling indicates such tumors are probably a distinct entity (A). NF1 loss may occur in dysembryoplastic neuroepithelial tumor but is rare (D).

    Comment Here

    Reference: Dysembryoplastic neuroepithelial tumor
    Board review style question #2
    Which of the following is true of dysembryoplastic neuroepithelial tumors?

    1. Associated with NF2
    2. Associated with seizures
    3. Harbor several cytogenetic alterations
    4. Most frequently occur in adults
    5. Show diffuse CD34 immunoreactivity
    Board review style answer #2
    B. Associated with seizures. Dysembryoplastic neuroepithelial tumor should not show significant CD34 immunopositivity (E). Dysembryoplastic neuroepithelial tumor usually occurs in children (D). Dysembryoplastic neuroepithelial tumor is generally without chromosomal losses or gains, resulting in a flat copy number profile (C). Dysembryoplastic neuroepithelial tumor has rarely been associated with NF1 but not NF2 (A).

    Comment Here

    Reference: Dysembryoplastic neuroepithelial tumor

    Dysplastic gangliocytoma of the cerebellum
    Definition / general
    Etiology
    • Due to or associated with germline PTEN mutation (Am J Hum Genet 2003;73:1191)
    • PTEN mutations are identified in all adult onset cases but not in childhood onset cases
    • Associated with Cowden disease (OMIM: Cowden Syndrome 1; CWS1), an autosomal dominant disorder with trichilemmomas, hamartomas, intestinal polyposis, palmoplantar keratoses, oral papillomas; increased incidence of breast, GU, CNS and thyroid tumors; due to abnormalities in 10q23
    Clinical features
    • Rare, < 200 cases described
    • Usually young adults, may present in childhood
    • Presents with dysmetria, headache, ataxia, mass effect
    Radiology description
    • Unilateral cerebellar affection with distorted architecture and enlarged folia
    • Characteristic striped appearance on FLAIR and T2 W MRI images
    Case reports
    Prognosis and treatment
    • Surgical resection if symptomatic
    • Good prognosis; recurs in 25% of cases but no malignant potential
    Gross description
    • Discrete region of hypertrophy and coarse gyral pattern
    Microscopic (histologic) description
    • Relative preservation of cerebellar architecture
    • Diffuse enlargement of internal granular layer and molecular layers, with replacement of internal granular layer by dysplastic ganglion cells of different sizes and axonal hypermyelination of molecular layer
    • Clear vacuoles in white matter and molecular layer
    • Common: calcification and ectatic vessels
    • No mitotic figures, no necrosis, no endothelial proliferation
    Immunohistochemistry & special stains
    • Dysplastic ganglion cells are synaptophysin+ with loss of PTEN protein expression; also are phosphorylated AKT+ and phosphorylated S6+ (indicates AKT / mTOR pathway activation)
    Differential diagnosis

    Embryonal tumor with multilayered rosettes
    Embryonal tumor with abundant neuropil and true rosettes, C19MC altered
    Definition / general
    • CNS embryonal tumor with biphasic architecture
    • Aggressive CNS embryonal tumor with multilayered rosettes (ETMR), amplicon C19MC altered and C19MC unaltered or not tested (Acta Neuropathol 2016;131:803)

    Essential features
    • Belongs to ETMR, a new subclassification of embryonal tumors
    • Embryonal tumor with abundant neuropil and true rosettes (ETANTR), ependymoblastoma and medulloepithelioma share molecular similarity and comprise a single clinicopathological entity
    • Overexpression of LIN28A protein in C19MC altered embryonal tumors
    • Poor prognosis

    ICD coding
    • ICD-10: C71.9 - malignant neoplasm of brain, unspecified

    Epidemiology
    • M = F
    • Mostly < 2 years of age

    Sites
    • Supratentorial in both hemispheres (70%)
    • Infratentorial in cerebellum and brain stem (30%)

    Clinical features
    • Signs and symptoms of increased intracranial pressure (headache, nausea and visual disturbances)
    • Focal neurologic signs (ataxia and weakness)

    Grading
    • WHO grade 4

    Diagnosis

    Radiology description
    • Contrast enhancing large tumor masses on CT and MRI

    Case reports

    Treatment

    Microscopic (histologic) description
    • Biphasic architecture
    • Dense clusters of small cells with round / polygonal nuclei, scanty cytoplasm and indistinct cell bodies along with large, paucicellular, fibrillary / neuropil like areas along with neoplastic neurocytic and ganglion cells
    • Mitosis and apoptotic bodies in hypercellular area
    • Multilayered rosettes

    Positive stains

    Negative stains

    Molecular / cytogenetics description
    • Amplicon at 19q13.42 is sensitive and specific diagnostic marker for medulloepithelioma with C19MC alteration (Acta Neuropathol 2014;128:279)
    • Embryonal tumor with abundant neuropil and true rosettes (ETANTR), ependymoblastoma and medulloepithelioma share molecular similarity and comprise a single clinicopathological entity
    Ependymoblastoma, C19MC altered
    Definition / general
    • Aggressive CNS embryonal tumor with multilayered rosettes (ETMR) and amplicon C19MC upregulation
    • Rare tumor of children aged < 4 years
    • Other tumors in this group are
      • Embryonal tumor with abundant neuropil and true rosettes
      • Medulloepithelioma
    • References: Acta Neuropathol 2016;131:803, Acta Neuropathol 2014;128:305

    Essential features
    • Belongs to ETMR with or without alteration of C19MC, a new subclassification of embryonal tumors
    • Rosettes are frequent and characteristic of embryonal tumors
    • Ependymoblastoma lacks a neuropil-like matrix and ganglion cell elements
    • Overexpression of LIN28A protein in C19MC altered embryonal tumors
    • Poor prognosis

    ICD coding
    • ICD-O: 9478/3 - embryonal tumor with multilayered rosettes C19MC altered

    Epidemiology
    • M = F
    • Mostly < 2 years of age

    Sites
    • Supratentorial in both hemispheres (70%)
    • Infratentorial in cerebellum and brain stem (30%)

    Clinical features
    • Signs and symptoms of increased intracranial pressure (headache, nausea and visual disturbances)
    • Focal neurologic signs (ataxia and weakness)

    Diagnosis

    Radiology description
    • Contrast enhancing large tumor masses on CT and MRI

    Case reports

    Treatment

    Gross description
    • Well circumscribed
    • Grayish pink with areas of necrosis and hemorrhage

    Microscopic (histologic) description
    • Rosettes are frequent and characteristic of ETMR
    • Sheets and clusters of poorly differentiated cells
    • Lacks neuropil-like matrix and ganglion cells
    • Rosettes are intermixed with small to medium sized embryonal cells with high N:C ratio

    Positive stains

    Negative stains

    Molecular / cytogenetics description
    Medulloepithelioma, C19MC not altered
    Definition / general
    • CNS embryonal tumor with prominent pseudostratified neuroepithelium that resembles the embryonic neural tube and poorly differentiated neuroepithelial cells
    • Aggressive CNS embryonal tumor with multilayered rosettes (ETMR) and amplicon C19MC altered, C19MC unaltered or C19MC not tested
    • Significant proportions of medulloepithelioma have not shown C19MC alterations (Acta Neuropathol 2016;131:803)

    Essential features
    • Belongs to ETMR, a new subclassification of embryonal tumors, some of which have C19MC alterations
    • C19MC altered embryonal tumors overexpress LIN28A protein
    • Poor prognosis

    ICD coding
    • ICD-10: C71.9 - malignant neoplasm of brain, unspecified

    Epidemiology
    • M = F
    • Mostly < 2 years of age

    Sites
    • Supratentorial in both hemispheres (70%)
    • Infratentorial in cerebellum and brain stem (30%)

    Clinical features
    • Signs and symptoms of increased intracranial pressure (headache, nausea and visual disturbances)
    • Focal neurologic signs (ataxia and weakness)

    Grading
    • WHO grade 4

    Diagnosis

    Radiology description
    • Contrast enhancing large tumor masses on CT and MRI

    Case reports

    Treatment

    Microscopic (histologic) description
    • Neoplastic pseudostratified neuroepithelium resembling embryonic neural tube, with papillary, tubular and trabecular arrangements
    • Sheets of poorly differentiated cells with hyperchromatic nuclei and high N:C ratio
    • Multilayered rosettes may be seen
    • Periodic acid-Schiff positive external limiting membrane
    • No cilia or blepharoplasts on luminal surface of tubules
    • Abundant mitotic figures
    • Rarely mesenchymal differentiation and melanin pigment

    Positive stains

    Negative stains

    Molecular / cytogenetics description

    Enterogenous cyst
    Definition / general
    • Cyst lined by simple columnar epithelium (with or without cilia or mucus globules)
    • Rare, benign; probably of malformative nature
    Terminology
    • Also called neurenteric, enteric, enterogenic, bronchogenic, respiratory cyst
    Sites
    • Usually intraspinal, rarely intracranial
    • Favored sites: posterior fossa and spinal cord (usually anterior to cord, intradural, extramedullary)
    • All ages are affected: infants, children and adults
    Radiology description
    • Like the closely related Rathke cleft cyst and colloid cyst, cyst is often bright on precontrast images
    Case reports
    Treatment
    • Complete excision; occasionally is adherent to adjacent structures
    • Recurrence may occur
    Gross description
    • Usually 1 cm or less, simple, thin walled, filled with opalescent mucinous material
    Microscopic (histologic) description
    • Columnar epithelium resting on collagen layer
    • Goblet cells often present
    • Variable cilia, may have squamous metaplasia
    • More complex lesions replicate gastrointestinal or respiratory epithelium (Appl Immunohistochem Mol Morphol 2004;12:230)
    Positive stains
    Negative stains
    Electron microscopy description
    • Well developed stereocilia, distinct basal cells, thin basement membrane
    Differential diagnosis

    Ependymoma
    Definition / general
    • Circumscribed neuroepithelial tumor with histological and molecular evidence of ependymal differentiation that may arise in the supratentorial region, posterior fossa or spinal cord
    Essential features
    • 2021 CNS WHO classification has reclassified ependymal tumors according to their location and methylation profiles
    • Some traditional histological subtypes, including the papillary, clear cell, tanycytic and anaplastic subtypes, have been discontinued
    • Supratentorial ependymal tumors include supratentorial ependymoma ZFTA fusion positive (ST-ZFTA), supratentorial ependymoma, YAP1 fusion positive (ST-YAP1) or supratentorial subependymoma (ST-SE)
    • Posterior fossa ependymal tumors include posterior fossa ependymoma group A (PFA), group B (PFB) and posterior fossa subependymoma (PF-SE)
    • Spinal ependymal tumors include spinal ependymoma (SP-EP), spinal ependymoma, MYCN amplified (SP-MYCN), spinal subependymoma (SP-SE) and myxopapillary ependymoma (SP-MP)
    • Common pathology features across all ependymoma subtypes include perivascular pseudorosettes, ependymal rosettes, positive GFAP and S100 cytoplasmic positivity, dot-like EMA perinuclear reactivity and negative Olig2 nuclear reactivity
    ICD coding
    • ICD-10
      • C71.0 - malignant neoplasm of cerebrum, except lobes and ventricles
      • C71.1 - malignant neoplasm of frontal lobe
      • C71.2 - malignant neoplasm of temporal lobe
      • C71.3 - malignant neoplasm of parietal lobe
      • C71.4 - malignant neoplasm of occipital lobe
      • C71.5 - malignant neoplasm of cerebral ventricle
      • C71.7 - malignant neoplasm of brain stem
      • C71.9 - malignant neoplasm of brain, unspecified
      • C72.9 - malignant neoplasm of central nervous system, unspecified
      • D33.2 - benign neoplasm of brain, unspecified
    Epidemiology
    • Ependymal tumors comprise 1.9% of all primary CNS tumors and 6.9% of primary glial neoplasms, with incidence rates of ~0.4/100,000 in the U.S. (Neuro Oncol 2015;17:iv1)
    • ST-ZFTA, ST-YAP1 and PFA predominantly occur in children, while SP-MYCN predominantly occurs in middle aged adults; other subtypes (ST-SE, PF-SE, PFB, SP-EP, SP-MP, SP-SE) can occur in all ages (Acta Neuropathol 2024;147:24)
    • M:F = 1.38:1 (Acta Neuropathol 2024;147:24)
    Sites
    • Typically associated with ventricular lining in the spinal cord or cerebrum but can occur intraparenchymally / paraventricularly, particularly in supratentorial tumors
    • Spinal ependymomas commonly in cervical and thoracic spine
    • Spinal myxopapillary ependymoma predominantly in conus medullaris and filum terminale
    • Posterior fossa ependymomas around fourth ventricle
    • References: Childs Nerv Syst 2003;19:270, Crit Rev Oncol Hematol 2007;63:81
    Pathophysiology
    • Single cell RNA sequencing across all major molecular ependymoma groups revealed hierarchical cellular populations, including undifferentiated neural stem cells, radial glia cells and more differentiated cells towards ependymal, astrocytic and neuronal lineages (Cancer Cell 2020;38:44)
    • Proportion of undifferentiated or less differentiated cells correlates with poor prognosis and increased recurrence
    • Aberrant radial glia-like cells are potential cells of origin for supratentorial ependymoma, with ZFTA::RELA fusions and neural stem cell-like cells the origin of posterior fossa ependymoma (Cancer Cell 2020;38:44)
    • Spinal ependymoma likely arises from mature adult ependymal cells due to their highly similar transcriptomic profiles through single cell RNA sequencing (Acta Neuropathol 2024;147:22)
    Etiology
    • Unknown
    Clinical features
    • Spinal cord ependymomas present with pain, weakness, sensory loss or radiculopathy, depending on level affected
    • Intracerebral ependymomas present with obstructive hydrocephalic symptoms as well as features of mass effect, neurologic deficits or seizures, depending on location
    • Reference: Pediatr Neurosurg 2019;54:98
    Diagnosis
    • Diagnosis is made by integrating histologic features, tumor location and molecular findings (Neuro Oncol 2021;23:1231)
    Radiology description
    • Commonly discrete, well circumscribed mass with avid uniform contrast enhancement
    • Hyperdense on computed tomography and T2 hyperintense on magnetic resonance imaging
    • Reference: Childs Nerv Syst 2009;25:1203
    Radiology images

    Contributed by Chunyu Cai, M.D., Ph.D.
    MRI cervical spine ependymoma

    MRI cervical spine ependymoma

    MRI posterior fossa ependymoma

    MRI posterior fossa ependymoma

    MRI supratentorial ependymoma

    MRI supratentorial ependymoma



    Images hosted on other servers:
    MRI: ependymoma of fourth ventricle MRI: ependymoma of fourth ventricle

    MRI: ependymoma of fourth ventricle

    Prognostic factors
    • Spinal ependymoma (SPEP), generally indolent
      • Recent studies suggest the presence of an NF2 mutation may confer a higher risk of recurrence
    • Spinal ependymoma, MYCN amplified (SPMYCN) has poor prognosis
    • Ependymomas with ZFTA fusions
      • 6% of ependymomas occur in posterior fossa and those occur exclusively in children and have worse prognosis
      • Supratentorial ependymoma with ZFTA fusion and combined homozygous CDKN2A / CDKN2B losses have worse prognosis
    • Supratentorial ependymomas with YAP1 fusions have good prognosis
    • Of posterior fossa ependymomas
      • PFA is characterized by H3K27 trimethyl loss on IHC and has worse prognosis than PFB
      • Gain of 1q and loss of 6q are additional risk factors that predict even worse outcomes in PFA
    • Complete resection is a predictor of good prognosis in PFA, PFB, PF-SE and SP-MP but not in ST-ZFTA
    • References: Acta Neuropathol 2024;147:24, Brain Pathol 2020;30:863, Acta Neuropathol 2024;147:22
    Case reports
    Treatment
    • Gross total resection if possible, with or without adjuvant radiotherapy
    • Multiple pathway inhibitors (ALK, EGFR, ErbB2, FAK, MEK, mTOR, TEAD, VEGF) are in clinical trials for NF2 associated tumors (Acta Neuropathol 2024;147:22)
    Gross description
    • Gray-red tumors with or without cystic degeneration, hemorrhage or necrosis
    • Intracranial tumors are well circumscribed
    • Ependymomas of fourth ventricle are typically exophytic
    Gross images

    Images hosted on other servers:
    Missing Image

    Arising from floor of fourth ventricle

    Frozen section description
    • Cellular tumor with sharp border with brain parenchyma
    • Perivascular pseudorosettes, true ependymal rosettes and lumina (J Neurosurg Spine 2018;30:133)
    Intraoperative frozen / smear cytology images

    Contributed by Chunyu Cai, M.D., Ph.D.
    Ependymoma frozen section

    Fibrillary perivascular pseudorosette

    Perivascular pseudorosette

    Perivascular pseudorosette

    Cytology ependymoma

    Fibrillary cytoplasm

    Microscopic (histologic) description
    • Cellular tumor with typically sharply circumscribed borders; may be infiltrative
    • Monomorphic round to oval cells with speckled chromatin
    • Perivascular pseudorosettes, true ependymal rosettes, lumina and fibrillar areas
    • May see gemistocyte-like cells and hypercellular nodules, particularly in posterior fossa tumors
    • Can have nonpalisading necrosis, areas of cystic or myxoid degeneration, calcifications, degenerative atypia, neuronal differentiation and rarely metaplastic elements
    • Morphologic subtypes have no clinicopathological significance and include papillary, clear cell and tanycytic
    • Utility of histological grading is debated; the 2021 WHO still recommend assigning either WHO grade 2 or grade 3 to an ependymoma, according to its histopathological features as part of the integrated diagnosis
    • Myxopapillary ependymoma are now assigned WHO grade 2 in the 2021 WHO CNS tumor classification
    • Subependymoma remains a WHO grade 1 (Neuro Oncol 2021;23:1231)
    Microscopic (histologic) images

    Contributed by Chunyu Cai, M.D., Ph.D., Maria Martinez-Lage, M.D. and Eman Abdelzaher, M.D., Ph.D.
    Perivascular pseudorosettes

    Perivascular pseudorosettes

    Ependymal rosettes

    Ependymal rosettes

    True ependymal rosettes

    True ependymal rosettes

    Ependymoma canal

    Ependymoma canal

    H&E

    Tanycytic ependymoma


    EMA showing dot-like and ring-like positivity

    EMA showing dot-like and ring-like positivity

    GFAP

    GFAP

    EMA

    EMA

    MAP2

    MAP2

    H3K27me IHC PFB

    H3K27me IHC PFB

    Virtual slides

    Images hosted on other servers:
    Ependymoma

    Ependymoma

    Cytology description
    • Spindle shaped cells with oval to elongated nuclei and delicate fibrillary cytoplasm with occasional intracytoplasmic lumina, which can be arranged around blood vessels
    • Occasional nuclear grooves and inclusions can be seen (J Pathol Transl Med 2019;53:104)
    Positive stains
    Negative stains
    Electron microscopy description
    • Features of ependymal cells
      • Cilia with a 9+2 microtubular pattern
      • Blepharoblasts and microvilli of the lumina
      • Zipper-like junctional complexes of lateral aspect
      • Lack of basement membrane
    • Reference: CNS Oncol 2014;3:49
    Molecular / cytogenetics description
    • Molecular groups based on fusion and methylation profiling (Cancer Cell 2015;27:728)
      • Supratentorial ependymoma, ZFTA fusion positive (formerly C11orf95::RELA fusion tumors), comprises 60% of supratentoral ependymomas and portends a poor prognosis
      • Supratentorial ependymoma, YAP1 fusion positive, has a better prognosis
      • Posterior fossa ependymoma, group A
      • Posterior fossa ependymoma, group B
      • Spinal ependymomas with NF2 mutations
      • Spinal ependymoma, MYCN amplified
    • Molecular characterization of histopathological ependymoma variants (Acta Neuropathol 2020;139:305)
      • Tanycytic ependymomas were mostly located in spinal cord; DNA methylation match to spinal or myxopapillary ependymoma
      • Clear cell ependymomas were mostly supratentorial; DNA methylation match to RELA fusion positive ependymomas
      • Papillary ependymomas match to posterior fossa group B or myxopapillary ependymoma
    Videos

    Ependymal tumors by Dr. Rodriguez

    Sample pathology report
    • Brain, fourth ventricular mass, excision:
      • Posterior fossa group B ependymoma, WHO grade 2 (see comment)
      • Comment: Sections show an ependymal neoplasm with perivascular pseudorosettes and multiple foci of well formed ependymal canals. Solid, papillary and clear cell components are variably present. No necrosis, microvascular proliferation or mitotic figures are identified. Tumor cell nuclei are oval and uniform. Focal areas of hemosiderin laden macrophages are present.
      • Immunohistochemically, a subset of the tumor cells express GFAP. EMA is negative. MIB1 proliferation index is < 1%. H3K27me nuclear expression is retained in over 95% of tumor cells.
      • The collective findings of floor of fourth ventricle location, adult age and retained H3K27me nuclear expression in tumor cells are most consistent with posterior fossa group B (PFB) ependymoma. Correlation with NGS result is recommended.
    Differential diagnosis
    Board review style question #1
    A biopsy of an enhancing, well circumscribed mass located in the paraventricular white matter of the parietal lobe shows cells with oval nuclei and speckled chromatin forming perivascular pseudorosettes and true rosettes with lumina. Fluorescence in situ hybridization reveals a ZFTA::RELA fusion. What is the best diagnosis?

    1. Astrocytoma, IDH mutant
    2. Oligodendroglioma, IDH mutant and 1p / 19q codeleted
    3. Posterior fossa ependymoma
    4. Supratentorial ependymoma, YAP1 fusion positive
    5. Supratentorial ependymoma, ZFTA fusion positive
    Board review style answer #1
    E. Supratentorial ependymoma, ZFTA fusion positive is the correct answer because the ZFTA::RELA (previously named C11orf95::RELA) fusion defines this tumor entity. Answers A, B, C and D are incorrect because the ZFTA::RELA fusion is exclusively seen in ependymoma and has not been seen in any other CNS tumors.

    Comment Here

    Reference: Ependymoma
    Board review style question #2

    Which immunohistochemical stain can help differentiate posterior fossa group A (PFA) ependymomas from group B (PFB)?

    1. EMA
    2. H3K27me3
    3. Ki67
    4. L1CAM
    5. S100
    Board review style answer #2
    B. H3K27me3 is the correct answer because the H3K27 trimethylation status distinguishes PFA from PFB. PFA has loss of the H3K27 trimethylation and therefore, loss of nuclear staining on H3K27me3 staining, while PFB group ependymoma tumor cells have retained nuclear expression. Answers A, C, D and E are incorrect because they don't distinguish H3K27 trimethylation status.

    Comment Here

    Reference: Ependymoma

    Epidermoid cyst
    Definition / general
    • Benign cyst lined by keratinized stratified squamous epithelium and lacking skin adnexa (eMedicine: Brain Epidermoid Imaging)
    • More common than dermoid cyst (1% of intracranial masses)
    Terminology
    • Also called epidermoid or epidermoid tumor
    Sites
    • Occurs throughout neuriaxis
    • Favored site: cerebellopontine angle
    • Other sites: brainstem, cranial diploe, intraspinal, pineal gland, sella, temporal lobe
    • Rarely undergoes malignant degeneration
    Radiology description
    • Discrete, extra-axial with signal characteristics reflecting keratinous contents
    • Diffusion MRI helps distinguish from arachnoid cysts
    Radiology images

    Images hosted on other servers:

    Cerebellopontine angle

    Case reports
    Gross description
    • Well defined round mass, thin walled, pearly white
    Microscopic (histologic) description
    • Fibrous wall lined by keratinizing squamous epithelium; contains anucleate squames but no skin adnexae and no hair
    Positive stains
    Differential diagnosis

    Extraventricular neurocytoma
    Definition / general
    Radiology description
    • Circumscribed, solitary and 57% cystic
    Prognostic factors
    • Poor prognostic factors: incomplete excision, atypical features, high cell proliferation rates; also older patient age
    Case reports
    Treatment
    • Total excision prevents recurrence
    Microscopic (histologic) description
    • Sheets, clusters, ribbons or rosettes of monotonous tumor cells with round and regular vesicular nuclei and distinct nucleoli embedded in matrix of fine neuropil
    • Ganglion cells in 66%
    • Atypical if necrosis, vascular proliferation and 3+ mitotic figures/10 HPF
    Positive stains
    • Synaptophysin (strong), GFAP (46%; also is staining of entrapped astrocytes at periphery)
    Electron microscopy description
    • Neuritic type processes with microtubules, dense core granules
    Differential diagnosis
    • Ependymoma
    • Oligodendroglioma: no ganglion cell differentiation, no salt and pepper chromatin and no neuropil islands; usually more infiltrative, synaptophysin-

    Gangliocytoma & ganglioglioma
    Definition / general
    • Uncommon, well differentiated, slow growing glioneuronal tumors composed predominantly of neoplastic mature ganglion-like cells (gangliocytoma) or a mixture of neoplastic ganglion-like cells and atypical glial cells (ganglioglioma)
    Essential features
    • Ganglioglioma and gangliocytoma are considered discreet tumor entities by the 2021 WHO classification of CNS tumors
    • Neoplastic ganglion cells often display dysplastic / dysmorphic features
    • Characterized by molecular alterations resulting in activation of the MAPK pathway (BRAF V600E is most common)
    • Commonly associated with epilepsy
    ICD coding
    Epidemiology
    Sites
    Pathophysiology
    Etiology
    • No known risk factors or environmental exposures for ganglioglioma or gangliocytoma
    Clinical features
    Diagnosis
    • Magnetic resonance imaging (MRI) of brain
    • Stereotactic biopsy can be performed, especially in sites unamenable to surgical excision
    Radiology description
    • Circumscribed, cystic mass with enhancing mural nodule
    • Classically, well delineated, T1 hypointense, T2 hyperintense cyst with enhancing mural nodule (Radiographics 2002;22:1177)
    • Variable calcifications and enhancement
    Radiology images

    Contributed by Jared T. Ahrendsen, M.D., Ph.D.

    Temporal lobe mass

    Prognostic factors
    Case reports
    Treatment
    Gross description
    • Well demarcated, variably solid and cystic mass forming lesion
    • Lacks areas of hemorrhage and necrosis
    Frozen section description
    • Well demarcated mass but can show brain infiltration at the tumor edge
    • Admixture of neuron-like cells and atypical glial cells (ganglioglioma) or predominantly neuron-like cells (gangliocytoma)
    • Variable microcalcifications, perivascular lymphocytes, eosinophilic granular bodies
    • Reference: Acta Cytol 2022;66:142
    Intraoperative frozen / smear cytology images

    Contributed by Jared T. Ahrendsen, M.D., Ph.D.

    Low grade glioneuronal tumor

    Admixture of mildly atypical glial cells

    Microscopic (histologic) description
    • Ganglioglioma:
    • Gangliocytoma:
      • Generally similar to that of ganglioglioma except lacking atypical neoplastic glial cells
      • Composed almost entirely of large, dysmorphic ganglion cells (as described above), often arranged in irregular clusters
    Microscopic (histologic) images

    Contributed by Jared T. Ahrendsen, M.D., Ph.D.

    Well circumscribed brain lesion

    Mixed glioneuronal tumor

    Ganglioglioma with binucleate neurons

    Well demarcated mass

    Dysplastic ganglion-like cells in gangliocytoma


    Olig2 IHC

    CD34 IHC

    Synaptophysin IHC

    Virtual slides

    Images hosted on other servers:
    Circumscribed glioneuronal tumor

    Circumscribed glioneuronal tumor

    Positive stains
    Negative stains
    • Ganglion-like cells often lack NeuN immunoreactivity (which is positive in nonneoplastic neurons)
    • Negative for IDH1 R132H staining
    Electron microscopy description
    • Not routinely used for clinical / diagnostic purposes
    • Ultrastructural characteristics of ganglioglioma include:
      • Ganglion-like cells with numerous dense core granules within the perikarya and cellular processes and abundant rough endoplasmic reticulum (Cancer 1997;79:989)
    Molecular / cytogenetics description
    • Ganglioglioma:
    • Gangliocytoma:
      • No signature molecular profile published to date
    Videos

    Neuronal and glioneuronal tumors
    by Dr. Fausto Rodriguez (PathCast)

    Sample pathology report
    • Brain mass, temporal lobe, resection:
      • Ganglioglioma, BRAF V600E mutant, WHO grade 1
      • Histologic classification: ganglioglioma
      • WHO grade: 1
      • Molecular information: BRAF V600E
    Differential diagnosis
    Board review style question #1

    A 15 year old patient presents with chronic seizures that are refractory to medical management. Magnetic resonance imaging (MRI) of the brain reveals a cystic lesion with an area of solid enhancement located in the medial temporal lobe. Surgical resection was performed, revealing the histology shown above. What is the most likely diagnosis?

    1. Ganglioglioma
    2. Glioblastoma
    3. Oligodendroglioma
    4. Pilocytic astrocytoma
    5. Subependymal giant cell astrocytoma
    Board review style answer #1
    A. Ganglioglioma. This clinical scenario describes classic features of ganglioglioma: a young patient with refractory seizures who presents with a cystic mass with enhancing mural nodule and histology that demonstrates a biphasic tumor with admixed neoplastic glial cells and dysmorphic ganglion-like cells. Other common histologic findings include perivascular lymphocytes, calcifications and eosinophilic granular bodies. Glioblastoma (answer B) is a high grade, infiltrating neoplasm composed of markedly atypical glial cells with increased mitoses, necrosis and microvascular proliferation. Oligodendroglioma (answer C) generally occurs in adults with brain imaging that shows an infiltrative process and histologic examination that reveals tumor cells with perinuclear halos (fried egg appearance). Pilocytic astrocytoma (answer D) usually occurs in the cerebellum or optic pathway and is characterized by bipolar glial cells with hair-like (piloid) processes, Rosenthal fibers, eosinophilic granular bodies and degenerative atypia. Subependymal giant cell astrocytoma (answer E) is a tumor found almost exclusively in patients with tuberous sclerosis; it is generally located near the ventricles and is composed of large, atypical, multinucleated astrocytes without a population of neoplastic ganglion-like cells.

    Comment Here

    Reference: Gangliocytoma & ganglioglioma
    Board review style question #2
    A 10 year old patient with chronic epilepsy underwent surgical resection of a temporal lobe mass. The histology was compatible with ganglioglioma, WHO grade 1. What is the most likely molecular alteration present in this tumor?

    1. BRAF V600E mutation
    2. H3 K27M mutation
    3. IDH1 R132H mutation
    4. NF1 mutation
    5. NF2 mutation
    Board review style answer #2
    A. BRAF V600E mutation. This is the most common alteration observed in ganglioglioma. H3 K27M mutation (answer B) is a characteristic molecular alteration of diffusely infiltrating midline gliomas (such as diffuse intrinsic pontine glioma). IDH1 R132H (answer C) is the most common IDH mutation, which occurs in oligodendrogliomas and infiltrating astrocytomas. NF1 alterations (answer D) are commonly seen in pilocytic astrocytomas of the optic pathway (i.e., optic pathway glioma). NF2 mutations (answer E) are common in meningiomas and schwannomas

    Comment Here

    Reference: Gangliocytoma & ganglioglioma

    Germ cell tumors
    Choriocarcinoma
    Definition / general
    • Prognosis poorer than germinoma - median survival 22 months in cases with high hCG levels (J Neurooncol 2004;66:225)
    • Serum levels of hCG are helpful

    Case reports

    Microscopic (histologic) description
    • Syncytiotrophoblasts (large multinucleated cells) and cytotrophoblasts

    Positive stains

    Differential diagnosis
    Embryonal carcinoma
    Definition / general
    • Prognosis poorer than germinoma
    • May be associated with precocious puberty
    • Characterized by rapid and bulky growth and spread to liver and lungs
    • 60% have metastases at presentation

    Treatment
    • Chemotherapy
    • Radiation therapy
    • Surgery

    Positive stains
    Germinoma
    Definition / general
    • Most common intracranial germ cell neoplasm
    • Often teenagers and young adults
    • Two - thirds male
    • May be mixed with other germ cell tumors
    • May derive from ectopic rests, transformation of resident germ cells or migration of germ cells late in development
    • Most common site is pineal region
    • Also anterior or posterior third ventricle, rarely fourth ventricle
    • Rarely associated with dysgenetic syndromes
    • Immunostains useful because biopsy is often small
    • Relatively good prognosis (5 - 10 year survival is 75 - 95%) versus 25 - 40% for nongerminoma germ cell tumors (Pediatr Neurol 2002;26:369)
    • Very sensitive to radiotherapy and chemotherapy; nongerminomatous germ cell tumors are less radiosensitive
    • Metastases may be due to surgical displacement of tumor
    • Spinal cord metastases occur in 10 - 15% of patients

    Staging / staging classifications
    • T1: smaller than 5 cm in diameter and located in the suprasellar, intrasellar or pineal region
    • T2: larger than 5 cm in diameter and located in the perisellar region
    • T3: may be smaller than 5 cm in diameter but invades and encroaches on the third ventricle
    • T4: extends into the anterior, middle or posterior fossa

    • N: not indicated for CNS tumors

    • M0: no evidence of gross subarachnoid or hematogenous metastasis
    • M1: microscopic tumor cells found in the CSF
    • M2: gross nodular seeding in the ventricular system or cranial subarachnoid spaces
    • M3: gross nodular seeding in the spinal subarachnoid spaces
    • M4: metastasis outside the cerebrospinal axis

    Case reports

    Treatment
    • Resection difficult due to high collagen content
    • Radiation therapy helpful

    Gross description
    • Soft, gray-pink, homogenous
    • Variable encapsulation
    • Usually poorly circumscribed and infiltrative

    Microscopic (histologic) description
    • Resembles seminoma / dysgerminoma
    • Large, epithelioid cells with abundant PAS+ cytoplasm, large, round nuclei and irregular and pleomorphic nuclei
    • May have prominent nests of lymphocytes with occasional granulomatous inflammation that may obscure tumor cells (Neurol Med Chir (Tokyo) 2005;45:415)
    • Lymphocytes may smear in small biopsies
    • Frequent mitotic activity and necrosis
    • Syncytiotrophoblasts in 14%
    • Less anaplasia than embryonal carcinoma
    • No cells intermediate in size between lymphocytes and large germinoma cells

    Cytology description
    • Loose fragments or single large pleomorphic and polygonal cells with vacuolated cytoplasm, enlarged oval nuclei and prominent nucleoli
    • Frequent mitotic figures, naked nuclei, foamy background
    • Also smashed lymphoid cells with streaking

    Positive stains

    Negative stains

    Electron microscopy description
    • Glycogen in cytoplasm, sparse cytoskeletal elements, prominent nucleoli

    Differential diagnosis

    Additional references
    Teratoma
    Definition / general
    • Tissue derived from ectoderm, endoderm and mesoderm (at least 2 of 3 germinal layers)
    • Usually well differentiated / grade I of IV
    • Incidence varies with age: 30 - 50% of congenital brain tumors, 2% of brain tumors in infants and children, 0.5% at all ages
    • Congenital cases are usually fatal; may replace cerebral hemispheres (Pediatr Pathol 1987;7:333)
    • Mature teratomas:
      • Have well differentiated tissue from all three germinal layers, including neuroectoderm, epithelium (solid, cystic, glandular or tubular), cartilage or other mesenchymal elements, glial and neuronal tissue
    • Immature teratomas:
      • Have less differentiated tissue from any of the three germinal layers
      • 50% with intracranial tumors die within one year
      • Pineal teratomas are more common in males but saccrococcygeal teratomas are more common in females
      • Must sample thoroughly for correct diagnosis

    Terminology

    Prognostic factors
    • Poor prognostic factors: tissue resembling medulloepithelioma, neuroblastoma, retinoblastoma or ependymoblastoma

    Case reports

    Treatment
    • Newborns: complete surgical excision (difficult)

    Additional references
    Yolk sac tumor
    Definition / general
    • Rare intracranial tumor, usually in pineal or suprasellar regions
    • Also called endodermal sinus tumor
    • Prognosis poorer than germinoma (median survival 2 years or less)

    Case reports

    Gross description
    • Usually large

    Microscopic (histologic) description
    • Tubulopapillary structures with vacuolated cuboidal cells, cystic spaces with eosinophilic hyaline bodies and Schiller-Duval bodies

    Positive stains

    Additional references

    Glioblastoma, IDH wild type
    Definition / general
    • An aggressive, infiltrating, astrocytic glioma that lacks mutations in IDH1, IDH2 and histone H3 genes and is:
      • Histologically defined by brisk mitotic activity and microvascular proliferation or necrosis
      • Or molecularly defined by the presence of TERT promoter mutation, EGFR gene amplification or copy number changes in the form of combined gain of chromosome 7 and loss of chromosome 10 (Acta Neuropathol 2018;136:793, Acta Neuropathol 2018;136:805)
    Essential features
    • Characterized by diffusely infiltrative growth pattern with nuclear atypia and either:
      • Mitotic activity, necrosis or microvascular proliferation or
      • Molecularly defined by the present of TERT promoter mutation, EGFR amplification or combined gain of chromosome 7 and loss of chromosome 10
    • Lacks mutations in IDH1 / IDH2 and histone H3 genes
    • Various morphologic subtypes have been recognized (giant cell, small cell, epithelioid, sarcomatous / gliosarcoma) with similar prognosis
    • Primitive neuronal component has increased rate of cerebrospinal fluid dissemination (Brain Pathol 2009;19:81)
    Terminology
    • Glioblastoma, IDH wild type
    • Glioblastoma multiforme (not recommended)
    • Diffuse astrocytoma with molecular features of glioblastoma (no longer recommended)
    ICD coding
    • ICD-O: 9440/3 - glioblastoma, NOS
    • ICD-11: 2A00.00 & XH5571 - glioblastoma of brain & glioblastoma, IDH wild type
    Epidemiology
    Sites
    • Most commonly in supratentorial regions (frontal, temporal, parietal and occipital lobes), with highest incidence in the frontal lobes
    • Most often centered in subcortical white matter
      • Many cases show infiltration into cortex and across the corpus callosum with spread to contralateral hemisphere
    • Rare cases reported in the cerebellum and spinal cord (World Neurosurg 2013;80:e237, J Neurosurg Spine 2010;13:67, Neurooncol Adv 2021;3:vdab154)
    Pathophysiology
    • Cell of origin undetermined
      • Some studies suggest a variety of CNS cell types can undergo malignant transformation with features of glioblastoma (GBM) (oligodendrocyte precursor cells, neural precursor cells, astrocytes and neurons)
    • Sequencing of human glioblastomas suggests that a neural precursor cell in the subventricular zone may be the cell of origin (Nature 2018;560:243)
    Etiology
    Clinical features
    • Symptoms are dependent on tumor location
    • May present with signs of increased intracranial pressure (i.e., headaches, nausea, emesis), seizures or focal neurologic deficits (visual field defects, hemiparesis, aphasias, etc.) (BMJ 2021;374:n1560)
    Diagnosis
    Radiology description
    • MRI: T2 / fluid attenuated inversion recovery (FLAIR) bright infiltrative lesion(s) with postcontrast T1 showing irregular peripheral rim enhancement with central necrosis
    • May cross the corpus callosum
    • May be multifocal
    • Lack of contrast enhancement may be observed in molecularly defined glioblastoma
    • Certain subtypes (i.e., gliosarcoma, epithelioid, giant cell) may appear well circumscribed (Neurosurg Rev 2021;44:3335, Eur Radiol 2019;29:429, Surg Pathol Clin 2020;13:249)
    Radiology images

    Contributed by Bharat Ramlal, M.D.

    T2 FLAIR MRI

    T1 postcontrast MRI

    Prognostic factors
    Case reports
    Treatment
    • Surgical resection where possible in younger patients (≤ 70 years old) and patients with good performance status, followed by radiotherapy with concurrent and adjuvant temozolomide (TMZ)
    • Tumor treating fields (TTFields / Optune) under investigation - alternating electric field therapy using low intensity energy to stop glioma proliferation; relatively recent treatment option with rare reports showing favorable outcomes (Front Oncol 2020;10:477)
    Gross description
    • Ill defined whitish gray mass with areas of hemorrhage and necrosis
    • Can expand gyri and cross the corpus callosum
    Frozen section description
    • Hypercellular infiltrative lesion with variable morphology
      • Infiltration often difficult to assess on frozen sections but entrapped neurons may be useful
    • Nuclear hyperchromasia and nuclear elongation, possible giant cells
    • Mitotic activity
    • Necrosis; geographic or pseudopalisading
    • Microvascular proliferation
    Intraoperative frozen / smear cytology images

    Contributed by Bharat Ramlal, M.D.

    Markedly hypercellular neoplasm

    Nuclear atypia and palisading tumor cells

    Pleomorphic, astrocytic tumor cells

    Intraoperative smear preparation

    Microscopic (histologic) description
    • Infiltrating, hypercellular astrocytic neoplasm often with hyperchromatic, elongated nuclei and irregular nuclear membranes
    • Typically mitotically active, though not required if molecular criteria are met
    • Microvascular proliferation or necrosis is required for a histologic diagnosis of GBM
      • Microvascular proliferation: multilayered, small caliber vessels with glomeruloid appearance (J Neuropathol Exp Neurol 1992;51:488)
      • Necrosis: can be geographic or pseudopalisading with neoplastic cells surrounding central necrosis
    • Variable cell morphology: undifferentiated / primitive neuronal cells, astrocytic, gemistocytic, oligodendroglial-like, small cell, lipidized, granular, epithelioid, giant cells, mesenchymal metaplasia and epithelial metaplasia
      • Primitive neuronal cells (embryonal): markedly increased cellularity composed of cells with high N/C ratio, brisk mitotic activity with apoptotic bodies, nuclear molding, sometimes with neuroblastic rosettes
        • Typically has conventional infiltrating astrocytic component, which is morphologically distinct
        • Loss of glial markers, expression of neuronal markers (synaptophysin)
        • Higher risk of CSF dissemination but similar survivals as classic GBM
        • Associated with MYC amplifications
      • Astrocytic: fibrillary, elongated processes
      • Gemistocytic: abundant eosinophilic cytoplasm with eccentric nuclei
      • Oligodendroglial-like: cells with small, round nuclei with perinuclear clearing in a vascular background
      • Small cell change: monomorphic cells with small, round or angulated, hyperchromatic nuclei and brisk mitotic activity
      • Lipidized / xanthomatous cells: cells with abundant foamy cytoplasm
        • Be sure to exclude pleomorphic xanthoastrocytoma
      • Granular cells: large cells with small nuclei and abundant granular cytoplasm
        • PAS positive
        • May be CD68 positive but negative for CD163
      • Epithelioid: large eosinophilic cells with prominent nucleoli
        • May resemble rhabdoid cells with more eccentric nuclei
        • May be immunoreactive to cytokeratins but negative for CAM5.2
        • May be more sharply demarcated with less infiltration
        • Often associated with BRAF V600E mutations, usually in younger individuals (Discov Med 2018;26:51, Int J Clin Exp Pathol 2020;13:1529)
      • Giant cell: well circumscribed tumors composed of markedly pleomorphic and bizarre cells, including multinucleated tumor cells
      • Mesenchymal / sarcomatous: may be well circumscribed; corresponds to cellular differentiation along various lineage; sarcomatous (spindled and fibroblastic), osseous, chondroid or myogenic differentiation (see Gliosarcoma)
      • Epithelial metaplasia: rare but may include squamous or adenomatous differentiation
        • Keratin pearls, epithelial whorls: CK5/6 positive
        • Glandular structures
    Microscopic (histologic) images

    Contributed by Bharat Ramlal, M.D. and Meaghan Morris, M.D., Ph.D.

    Hypercellular neoplasm

    Nuclear atypia and brisk mitotic activity

    Atypical astrocytic tumor cells

    Glomeruloid microvascular proliferation

    Pseudopalisading necrosis


    Marked nuclear pleomorphism

    Primitive neuronal component

    Adenoid morphology

    Epithelioid glioblastoma


    Granular cell morphology

    Small cell glioblastoma


    IDH1 R132H

    GFAP

    Olig2

    Ki67

    Synaptophysin

    Cytology description
    • Intraoperative smears may show marked cellularity, with moderate to markedly pleomorphic astrocytic / gemistocytic cells with fine fibrillar glial processes (Diagn Cytopathol 1986;2:312)
    • May show necrosis
    • Mitotic figures can be observed
    Positive stains
    Negative stains
    Electron microscopy description
    • Bundles of cytoplasmic filaments 80 - 90 angstroms in diameter (Am J Pathol 1973;73:589)
    • Pleomorphic nuclei and prominent nucleoli with nuclear infoldings and cytoplasmic invaginations (intranuclear pseudoinclusions)
    Molecular / cytogenetics description
    Sample pathology report
    • Brain, right frontal lobe, biopsy:
      • Glioblastoma, IDH wild type, CNS WHO grade 4
      • Histologic diagnosis: glioblastoma
      • Grade: 4
      • Molecular information: EGFR amplified, negative for IDH1 / IDH2 and histone H3 alterations
    Differential diagnosis
    • Astrocytoma, IDH mutant:
      • Younger individuals, presence of IDH1 / IDH2 gene mutations by immunohistochemistry or sequencing
      • Often have ATRX mutation (loss of expression) and p53 mutations (nuclear overexpression)
    • Diffuse hemispheric glioma, H3 G34 mutant:
      • Older adolescents and young adults (age 11 - 30) with hemispheric mass
      • May have classic GBM morphology or primitive neuronal / embryonal morphology
      • Characterized by histone H3 G34 mutation on sequencing
      • Classically lack Olig2 expression, show ATRX loss of expression (mutant) and have p53 mutations (nuclear overexpression)
    • Diffuse midline glioma, H3 K27 altered:
      • Midline tumor (brainstem, thalamus, spinal cord, less often basal ganglia or cerebellum)
      • Most positive for histone H3K27M mutant protein (nuclear)
      • All show loss of histone H3K27 trimethylation (H3K27me3)
    • Diffuse pediatric type high grade glioma, H3 wild type and IDH wild type:
      • Typically in a child or young adult
      • Frequent PDGFRA or EGFR alterations or MYCN amplification
      • Methylation profiling may be helpful in difficult cases
    • Pleomorphic xanthoastrocytoma:
      • Prominent xanthomatous cells
      • Often well circumscribed
      • Lower grade lesions have no necrosis and low mitotic activity
      • Eosinophilic granular bodies (EGBs), Rosenthal fibers and perivascular lymphocytic cuffing
      • Often BRAF V600E positive IHC / sequencing and CD34 positive
      • Pericellular reticulin deposition
    • Lymphoma:
      • More monotonous and discohesive with perivascular cuffing of tumor cells
      • Typically large B cell morphology
      • Positive for lymphoid markers (CD45, CD20, etc.)
      • Negative GFAP and Olig2
    • Tumefactive multiple sclerosis / demyelinating diseases:
      • Macrophage rich lesions (CD163 positive) with perivascular lymphoplasmacytic inflammation
      • Loss of myelin (LFB) with axonal preservation (neurofilament)
      • Creutzfeldt cells: astrocytic cells with nuclear fragmentation may mimic mitotic figures
      • Astrocytes have a reactive (fibrillary) appearance, which can be highlighted by GFAP IHC
    • Abscess:
      • Abundant necrosis with mixed acute and chronic inflammation
      • Peripheral granulation tissue and fibrosis
      • Neutrophilic inflammation predominates
      • Microorganisms may be identified
    Board review style question #1

    Which histologic subtype of glioblastoma, IDH wild type (pictured above) is important to diagnose due to increased risk of cerebrospinal fluid (CSF) dissemination?

    1. Epithelioid
    2. Gemistocytic
    3. Primitive neuronal component
    4. Sarcomatoid
    Board review style answer #1
    C. Primitive neuronal component. Glioblastomas with a primitive neuronal component have an increased risk of CSF spread and potential for distant metastasis in rare case reports. As such, this particular morphologic finding may be important to bring to the attention of the clinical team. A primitive neuronal component in glioblastoma can be diagnosed by identifying a morphologically distinct area with high cellularity, high mitotic activity, loss of expression of glial markers and expression of neuronal markers such as synaptophysin.

    Comment Here

    Reference: Glioblastoma, IDH wild type
    Board review style question #2
    BRAF V600E mutations are usually identified in low grade gliomas but are also present in a subset of glioblastomas with which histological features?

    1. Epithelioid
    2. Gemistocytic
    3. Giant cell
    4. Oligodendrolial-like features
    Board review style answer #2
    A. Epithelioid. BRAF V600E mutations are more frequently encountered in epithelioid glioblastomas and may allow for treatment with targeted therapies (Int J Mol Sci 2018;19:1090).

    Comment Here

    Reference: Glioblastoma, IDH wild type

    Glioependymal / ependymal cyst
    Definition / general
    • Rare, benign intraparenchymal and often paraventricular cyst lined by simple epithelium or glial tissue, S100+ or GFAP+, resting on neuroglia (J Neuroradiol 1995;22:48)
    • Usually intracranial, not midline but may affect spinal cord; may affect adult cerebellum or represent burned out pilocytic astrocytoma
    • Not in communication with ventricle or CSF spaces
    • Rarely ruptures and causes meningitis
    • Cyst lined by glial tissue
    Radiology images

    Images hosted on other servers:

    MR: cystic lesion of right frontal lobe

    Case reports
    Gross description
    • Resembles arachnoid cyst
    Gross images

    Images hosted on other servers:

    Ependymal cyst

    Microscopic (histologic) description
    • Simple columnar or cuboidal cells, often ciliated, resting on neuroglia; no fibrous capsule
    • Alternatively, wall lined by gliosis, Rosenthal fibers present, variable hemosiderin; no epithelial lining
    Positive stains
    Negative stains
    Electron microscopy description
    • Neuroepithelial origin
    Differential diagnosis

    Glioma overview
    Definition / general
    • Most common form of intra-axial primary tumors of the central nervous system (CNS)
    • Includes adult and pediatric type diffuse gliomas, circumscribed gliomas and ependymal tumors
      • Note: pediatric type gliomas occur more frequently in children and adolescents; however, they can also occur in young adults (and vice versa)
      • Glioneuronal and neuronal tumors will not be discussed here
    • Increasing importance of molecular diagnostics for glioma classification (IDH1 / IDH2, histone H3, etc.)
    Essential features
    • Group of primary CNS neoplasms with glial differentiation
    • Range in biologic aggressiveness from WHO grade 1 (benign) to WHO grade 4 (malignant)
    • Changes with the 2021 CNS WHO (5th edition) (Neuro Oncol 2021;23:1231)
      • Arabic numbers are now used instead of Roman numerals for grading (i.e., WHO grade 1, 2, 3, 4 instead of WHO grade I, II, III, IV)
      • Simplified classification of adult type diffuse gliomas into 3 groups
        • Glioblastoma, IDH wildtype, WHO grade 4
        • Astrocytoma, IDH mutant, WHO grade 2 - 4
        • Oligodendroglioma, IDH mutant and 1p / 19q codeleted, WHO grade 2 - 3
      • Additional molecular surrogates for WHO grade 4 designation (i.e., grade 4 designation should be applied even without the presence of grade 4 histology) (Neuro Oncol 2021;23:1231)
        • Glioblastoma, IDH wildtype
          • Polysomy 7 and monosomy 10 (+7 / -10)
          • EGFR amplification
          • TERT promoter mutation
        • Astrocytoma, IDH mutant
          • CDKN2A / CDKN2B homozygous deletion
    Terminology
    • Intra-axial: within the brain or spinal cord parenchyma
    • Extra-axial: not within the brain or spinal cord parenchyma (e.g., meningioma)
    • Intramedullary: within the spinal cord parenchyma
    • Extramedullary: not within the spinal cord parenchyma (e.g., schwannoma)
    • Supratentorial: above tentorial membrane - cerebrum
    • Infratentorial: below tentorial dura - brainstem, cerebellum, spinal cord
    ICD coding
    • ICD-O
      • 9400/3 - astrocytoma, IDH mutant, grade 2
      • 9401/3 - astrocytoma, IDH mutant, grade 3
      • 9445/3 - astrocytoma, IDH mutant, grade 4
      • 9450/3 - oligodendroglioma, IDH mutant and 1p / 19q codeleted, grade 2
      • 9451/3 - oligodendroglioma, IDH mutant and 1p / 19q codeleted, grade 3
      • 9440/3 - glioblastoma, IDH wildtype
      • 9421/1 - diffuse astrocytoma, MYB or MYBL1 altered
      • 9431/1 - angiocentric glioma
      • 9413/0 - polymorphous low grade neuroepithelial tumor of the young
      • 9421/1 - diffuse low grade glioma, MAPK pathway altered
      • 9385/3 - diffuse midline glioma, H3 K27 altered
      • 9385/3 - diffuse hemispheric glioma, H3 G34 mutant
      • 9385/3 - diffuse pediatric type high grade glioma, H3 wildtype and IDH wildtype
      • 9385/3 - infant type hemispheric glioma
      • 9421/1 - pilocytic astrocytoma
      • 9421/3 - high grade astrocytoma with piloid features
      • 9424/3 - pleomorphic xanthoastrocytoma
      • 9384/1 - subependymal giant cell astrocytoma
      • 9444/1 - chordoid glioma
      • 9430/3 - astroblastoma, MN1 altered
      • 9391/3 - supratentorial ependymoma, NOS
      • 9396/3 - supratentorial ependymoma, ZFTA fusion positive
      • 9396/3 - supratentorial ependymoma, YAP1 fusion positive
      • 9391/3 - infratentorial ependymoma, NOS
      • 9396/3 - posterior fossa group A (PFA) ependymoma
      • 9396/3 - posterior fossa group B (PFB) ependymoma
      • 9391/3 - spinal ependymoma, NOS
      • 9396/3 - spinal ependymoma, MYCN amplified
      • 9394/1 - myxopapillary ependymoma
      • 9383/1 - subependymoma
    • ICD-11
      • 2A00.0Y& XH6PH6 - other specified gliomas of brain & astrocytoma, NOS
      • 2A00.0Y& XH2HK4 - other specified gliomas of brain & diffuse astrocytoma, IDH mutant
      • 2A00.0Y& XH7K31 - other specified gliomas of brain & oligodendroglioma, IDH mutant and 1p / 19q codeleted
      • 2A00.0Y& XH5571 - glioblastoma of brain & glioblastoma, IDH wildtype
      • 2A00.0Y& XH41C5 - other specified gliomas of brain & angiocentric glioma
      • 2A00.2Y - other specified tumors of neuroepithelial tissue of brain
      • 2A00.0Y - other specified gliomas of brain
      • 2A00.0Y& XH7692 - other specified gliomas of brain & diffuse midline glioma, H3 K27M mutant
      • 2A00.0Y& XH29Q5 - other specified gliomas of brain & pilomyxoid astrocytoma
      • 2A00.0Y& XH99U2 - other specified gliomas of brain & pleomorphic xanthoastrocytoma
      • 2A00.0Y& XH1L48 - other specified gliomas of brain & subependymal giant cell astrocytoma
      • 2A00.0Y& XH9HV1 - other specified gliomas of brain & chordoid glioma
      • 2A00.0Y& XH1DC5 - other specified gliomas of brain & astroblastoma
      • 2A00.0Y & XH1511 - other specified gliomas of brain & ependymoma, NOS
      • 2A00.0Z & XH1511 - other and unspecified neoplasms of brain or central nervous system & ependymoma, NOS
      • 2A00.0Y& XH15U1 - other specified gliomas of brain & myxopapillary ependymoma
      • 2A00.0Y& XH8FZ9 - other specified gliomas of brain & subependymoma
    Epidemiology
    • CNS tumors account for only 2% of all malignant neoplasms (Neuro Oncol 2021;23:iii1)
      • 40% with neuroepithelial origin
    • In children, CNS tumors are the leading cause of cancer related death (NCHS Data Brief 2016;257:1, Neuro Oncol 2018;20:iv1)
    • Children and adolescents are more likely to acquire pediatric type diffuse gliomas and certain circumscribed gliomas, whereas adults are more likely to acquire adult type diffuse gliomas, though there is considerable overlap (Neuro Oncol 2021;23:iii1)
    Sites
    • Brain
    • Spinal cord
    Pathophysiology
    • Generally, not well understood
    • Most glial tumors are likely derived from populations of CNS progenitor cells, including neural progenitor cells, glial progenitor cells, oligodendrocyte precursor cells, etc. (Cells 2021;10:621)
    Etiology
    • In most cases, there is not a strong causal connection
    • History of prior craniospinal radiation exposure is a known risk factor for glioma development (J Natl Cancer Inst 2006;98:1528, Neurooncol Adv 2021;3:vdab109)
    • Gliomas in hereditary cancer disorders (Neuro Oncol 2010;12:104, Curr Neurol Neurosci Rep 2021;21:64, Neurooncol Pract 2021;8:375)
      • Neurofibromatosis type 1: pilocytic astrocytoma (including optic pathway glioma), high and low grade gliomas, which can be difficult to classify
      • Neurofibromatosis type 2: ependymomas, most commonly spinal
      • Tuberous sclerosis: subependymal giant cell astrocytoma (SEGA)
      • Noonan syndrome: pilocytic astrocytoma
      • Enchondromatosis (Ollier disease, Maffucci syndrome): diffuse gliomas (IDH mutant)
      • Li Fraumeni syndrome: grade 4 gliomas in younger patients, including diffuse pediatric type high grade glioma, H3 wildtype and IDH wildtype; IDH mutant astrocytomas in young adults (Acta Neuropathol Commun 2023;11:3)
      • Lynch syndrome (Turcot syndrome): glioblastoma
    Clinical features
    • Clinical presentation highly variable, depending on patient age, tumor location and tumor type (Acta Neurol Scand 2015;131:88)
      • Common presenting symptoms include seizure, cognitive disorder, headache, nausea / vomiting, focal neurologic deficit
    Diagnosis
    • Combination of clinical findings, imaging (especially magnetic resonance imaging [MRI]) and tissue based biopsy / resection (Lancet Oncol 2017;18:e315)
    Radiology description
    • Highly variable, depending on location and tumor type (Radiology 2022;304:494, Childs Nerv Syst 2009;25:1203)
      • Circumscribed gliomas
        • Well circumscribed, variably enhancing, occasionally seen as a cystic mass with enhancing mural nodule
      • Diffuse low grade gliomas
        • Expansile but usually without enhancement
        • T2 FLAIR mismatch sign is a specific (but not sensitive) indicator in adults for astrocytoma, IDH mutant
        • Calcifications and cystic components can occasionally be seen in oligodendroglioma, IDH mutant and 1p / 19q codeleted
      • Diffuse high grade gliomas
        • Expansile and generally with variable enhancement
        • Glioblastoma often displays a characteristic ring enhancement with surrounding vasogenic edema and diffusion restriction
      • Ependymoma
        • Often conspicuous on T2 weighted FLAIR sequences and usually visible on unenhanced T1 weighted images
    Radiology images

    Contributed by Jared T. Ahrendsen, M.D., Ph.D.
    Glioblastoma MRI

    Glioblastoma MRI

    Glioma MRI

    Glioma MRI

    Optic pathway glioma MRI

    Optic pathway glioma MRI

    Spinal ependymoma MRI

    Spinal ependymoma MRI

    Prognostic factors
    • CNS WHO grade (BMC Cancer 2020;20:35)
      • WHO grade 1: generally benign; potentially curable with resection
      • WHO grade 2 - 3: intermediate prognosis; survival generally many years with eventual tumor progression
      • WHO grade 4: malignant; survival limited to months
      • Even low grade tumors can cause significant morbidity and mortality if located in delicate parts of the brain, such as the brainstem
    • Other factors that influence prognosis include patient age, comorbid conditions, location of tumor, presence of hydrocephalus, extent of surgical resection, response to adjuvant therapy and molecular features of the tumor (BMC Cancer 2020;20:35, Ann Palliat Med 2021;10:863)
    Case reports
    Treatment
    Frozen section description
    • Can be useful in some cases to determine circumscribed versus infiltrative growth patterns
    • Helpful to identify high grade features, such as mitotic activity, microvascular proliferation and necrosis
    • Due to freezing artifact, nuclear atypia is difficult to assess on frozen section (see Cytology description)
    • References: Surg Pathol Clin 2015;8:27, Semin Diagn Pathol 2002;19:192
    Intraoperative frozen / smear cytology images

    Contributed by Jared T. Ahrendsen, M.D., Ph.D.
    Glioma with astrocytic differentiation

    Glioma with astrocytic differentiation

    Glioma smear preparation

    Glioma smear preparation

    Microscopic (histologic) description
    • Diffuse gliomas
      • Variably hypercellular with individual tumor cells infiltrating the brain parenchyma
      • Diffuse infiltration results in ill defined margin with adjacent brain tissue
      • Secondary structures of Scherer, indicative of infiltrative growth
        • Perineuronal satellitosis by tumor cells
        • Perivascular condensation by tumor cells
        • Subpial accumulation of tumor cells
        • Spread of tumor cells along white matter tracks
      • Variable nuclear atypia, including nuclear enlargement, irregular nuclear contours, hyperchromasia, multinucleation, etc.
      • Some histologic features are suggestive of particular diagnostic entities (see individual chapters for more complete descriptions)
    • Circumscribed astrocytic neoplasms
      • Pilocytic astrocytoma
        • Circumscribed, biphasic (compact and loose) astrocytic neoplasm with piloid cellular processes, bipolar tumor cells, low proliferation and often Rosenthal fibers or eosinophilic granular bodies
      • High grade astrocytoma with piloid features
        • Histopathologic features vary considerably and molecular testing is generally required for diagnosis
      • Pleomorphic xanthoastrocytoma
        • Predominantly circumscribed astrocytoma with pleomorphic nuclei, occasional cells with bubbly (xanthomatous) cytoplasm, eosinophilic granular bodies, perivascular lymphocytes and reticulin rich background
      • Subependymal giant cell astrocytoma (SEGA)
        • Circumscribed, moderately cellular astrocytic neoplasm composed of polygonal cells with abundant eosinophilic cytoplasm and often multinucleated cells
      • Chordoid glioma
        • Glial neoplasm composed of chords / ribbons of epithelioid cells in a variably myxoid background
      • Astroblastoma, MN1 altered
        • Glial neoplasm characterized by the presence of astroblastic perivascular pseudorosettes: radially oriented tumor cells with thick cellular processes oriented towards a central blood vessel
    • Ependymal tumors
      • Ependymoma
        • Circumscribed neoplasm characterized by monomorphic round tumor cells, often arranged around blood vessels to form perivascular pseudorosettes
      • Subependymoma
        • Round, uniform tumor cells arranged in clusters / nests within a dense fibrillary background
        • Microcystic changes and calcifications are common
      • Myxopapillary ependymoma
        • Glioma with papillary structures and often prominent myxoid background
    • Reference: Neuro Oncol 2021;23:1231
    Microscopic (histologic) images

    Contributed by Jared T. Ahrendsen, M.D., Ph.D.
    Circumscribed glioma

    Circumscribed glioma

    Pilocytic astrocytoma

    Pilocytic astrocytoma

    Pleomorphic xanthoastrocytoma (PXA)

    Pleomorphic xanthoastrocytoma (PXA)

    Glioma with infiltrating edge

    Glioma with infiltrating edge


    Diffuse glioma entrapped neurons

    Diffuse glioma entrapped neurons

    High grade infiltrating glioma

    High grade infiltrating glioma

    Perivascular pseudorosettes

    Perivascular pseudorosettes

    GFAP positive glioma

    GFAP positive glioma

    Virtual slides

    Images hosted on other servers:
    Glioblastoma, not otherwise specified

    Glioblastoma, not otherwise specified

    Diffuse astrocytoma, not otherwise specified

    Diffuse astrocytoma, not otherwise specified

    Oligodendroglioma, not otherwise specified

    Oligodendroglioma, not otherwise specified

    Ependymoma, not otherwise specified

    Ependymoma, not otherwise specified

    Subependymoma

    Subependymoma

    Cytology description
    • Enlarged, variably hyperchromatic nuclei, often with irregular nuclear contours
    • Fine, fibrillary cytoplasmic processes are characteristic of glioma cells
    Positive stains
    Negative stains
    Molecular / cytogenetics description
    • Adult type diffuse gliomas
      • Astrocytoma, IDH mutant (J Neuropathol Exp Neurol 2016;75:4, N Engl J Med 2015;372:2481, Acta Neuropathol 2018;136:153)
        • By definition, presence of IDH1 / IDH2 mutation and intact chromosomes 1p and 19q
        • Co-occurring ATRX and TP53 mutations are common
        • Presence of CDKN2A / CDKN2B homozygous deletion associated with poor outcome and sufficient for WHO grade 4 designation, irrespective of histologic grade
      • Oligodendroglioma, IDH mutant and 1p / 19q codeleted (N Engl J Med 2015;372:2481, CNS Oncol 2015;4:287)
        • By definition, presence of IDH1 / IDH2 mutation and whole arm deletion of chromosomes 1p and 19q
        • Co-occurring CIC and TERT promoter mutations are common
      • Glioblastoma, IDH wildtype (Acta Neuropathol 2018;136:793, N Engl J Med 2005;352:997)
        • By definition, IDH1 / IDH2 wildtype
        • Characteristic molecular findings
          • TERT promoter mutation
          • EGFR amplification
          • Polysomy 7 and monosomy 10 (+7 / -10)
          • Presence of any of these 3 molecular alterations is sufficient for WHO grade 4 designation in an adult type, IDH wildtype diffuse glioma, irrespective of histologic grade
        • MGMT promoter methylation associated with better response to alkylating chemotherapy (temozolomide)
        • Many other co-occurring molecular and cytogenetic alterations can be encountered
    • Pediatric type diffuse low grade gliomas
      • Diffuse astrocytoma, MYB or MYBL1 altered (Acta Neuropathol 2019;137:683)
        • Defined by the presence of MYB or MYBL1 fusion, most common fusion partners include PCDHGA1, MMP16 and MAML2
        • Typically not MYB::QKI fusion, which characterizes angiocentric glioma (see below)
      • Angiocentric glioma (Acta Neuropathol 2019;137:683, Nat Genet 2016;48:273)
        • Characterized by alteration of MYB on chromosome 6q, most commonly MYB::QKI fusion
      • Polymorphous low grade neuroepithelial tumor of the young (PLNTY) (J Neuropathol Exp Neurol 2021;80:821)
        • Characterized by MAPK pathway activating alterations, including BRAF V600E and fusions involving FGFR2 and FGFR3
      • Diffuse low grade glioma, MAPK altered (Nat Genet 2013;45:602)
        • Defined by the presence of a MAPK pathway activating alteration, in addition to wildtype IDH and H3 as well as absence of CDKN2A / CDKN2B homozygous deletion
    • Pediatric type diffuse high grade gliomas
      • Diffuse midline glioma, H3 K27 altered (Acta Neuropathol 2015;130:815)
        • Defined by loss of H3 K27 trimethylation with presence of H3 K27 mutation, EGFR amplification or EZHIP overexpression in a diffusely infiltrating glioma located along the midline
        • Can be alternately defined by loss of H3 K27 trimethylation with a methylation profile of diffuse midline glioma in a diffusely infiltrating glioma located along the midline
      • Diffuse hemispheric glioma, H3 G34 mutant (Acta Neuropathol 2016;131:137)
        • Defined by the presence of H3 G34 mutation in a diffusely infiltrating glioma located in a cerebral hemisphere
      • Diffuse pediatric type high grade glioma, H3 wildtype and IDH wildtype (Acta Neuropathol 2017;134:507)
        • Diffuse glioma lacking IDH and H3 alterations; often with PDGFRA alteration, EGFR alteration or MYCN amplification
      • Infant type hemispheric glioma (Nat Commun 2019;10:4343)
        • Characterized by the presence of a receptor tyrosine kinase abnormality, especially fusions involving ROS1, MET1, ALK or NTRK family gene
    • Circumscribed astrocytic neoplasms
      • Pilocytic astrocytoma (Cancer Res 2008;68:8673, Nat Genet 2013;45:602)
        • Cerebellar tumors commonly harbor KIAA1549::BRAF fusion (7q34 duplication); other MAPK pathway alterations can also be encountered
      • High grade astrocytoma with piloid features (Acta Neuropathol 2018;136:273)
        • Presence of a MAPK pathway alteration along with CDKN2A / CDKN2B homozygous deletion, amplification of CDK4 or ATRX mutation
        • DNA methylation profiling is only definitive method for diagnosis
      • Pleomorphic xanthoastrocytoma (Hum Pathol 2019;86:38, Brain Pathol 2019;29:85)
        • Characterized by the presence of both BRAF V600E and CDKN2A / CDKN2B homozygous deletion
      • Subependymal giant cell astrocytoma (SEGA) (Oncotarget 2017;8:95516)
        • Seen almost exclusively in patients with tuberous sclerosis (germline mutations in TSC1 or TSC2)
      • Chordoid glioma (Nat Commun 2018;9:810)
        • Characterized by PRKCA D463 mutation
      • Astroblastoma, MN1 altered (Brain Tumor Pathol 2019;36:112)
        • Defined by presence of MN1 alteration, most commonly MN1::BEND2 fusion
    • Ependymal tumors
      • Ependymoma (Cancer Cell 2015;27:728, Brain Pathol 2020;30:863)
        • Supratentorial ependymoma characterized by ZFTA fusion or YAP1 fusion
        • Posterior fossa ependymoma characterized by unique methylation patterns
        • Spinal ependymoma characterized by NF2 mutation or MYCN amplification
      • Subependymoma (Neuro Oncol 2018;20:1616)
        • Recurrent losses of chromosome 19 and partial losses of chromosome 6
      • Myxopapillary ependymoma (Neuro Oncol 2018;20:1616)
        • Recurrent gains of chromosome 16 and losses of chromosome 10
      • Note: ependymomas are defined by both anatomic location as well as molecular features
    • Many of these tumors form distinct clusters on DNA methylation profiling (Nature 2018;555:469)
    Sample pathology report
    • Brain, left frontal lobe mass, resection:
      • Integrated diagnosis: glioblastoma, IDH wildtype, WHO grade 4
      • Histopathologic classification: glioblastoma
      • CNS WHO grade: 4
      • Molecular information: TERT promoter mutation, EGFR amplification

    • Brain, right parietal mass, resection:
      • Integrated diagnosis: supratentorial ependymoma, not otherwise specified (NOS)
      • Histopathologic classification: ependymoma
      • CNS WHO grade: 3
      • Molecular information: genetic material extracted from FFPE tissue was of insufficient quality or quantity for molecular testing
    Differential diagnosis
    • Reactive gliosis (ischemic, inflammatory, demyelination, infectious, etc.):
      • Reactive astrocytes tend to have less dense cellularity, regularly spaced cells, absent mitotic activity and lack of molecular alterations characteristic of gliomas (i.e., negative for IDH, TP53, ATRX, BRAF, H3, etc.)
    • Glioneuronal tumors, especially in the case of a circumscribed astrocytic neoplasm:
      • Generally display some degree of neuronal differentiation, including dysmorphic ganglioid cells or expression of neural antigens (such as synaptophysin)
    • See individual topics for additional differential considerations for each tumor type
    Board review style question #1

    During autopsy, an incidental mass is discovered in the fourth ventricle. The histopathology is shown in the photomicrograph above. What is the most likely diagnosis?

    1. Astrocytoma, IDH mutant, WHO grade 2
    2. Pilocytic astrocytoma
    3. Reactive astrogliosis
    4. Subependymoma
    Board review style answer #1
    D. Subependymoma. The photomicrograph shows round, uniform tumor cells arranged in clusters / nests within a dense fibrillary background. This is highly characteristic histomorphology for subependymoma. Answer C is incorrect because reactive gliosis generally does not form a mass lesion and is characterized by relatively evenly spaced astrocytes with abundant eosinophilic cytoplasm. Answer A is incorrect because astrocytoma, IDH mutant, WHO grade 2 is an infiltrating neoplasm not commonly encountered in the fourth ventricle. Answer B is incorrect because pilocytic astrocytoma is more commonly encountered in the cerebellum or optic pathway and is characterized by bipolar tumor cells, biphasic tumor background (compact areas and cystic areas) and the presence of Rosenthal fibers, eosinophilic granular bodies and degenerative atypia.

    Comment Here

    Reference: Glioma overview
    Board review style question #2
    A 6 year old girl presents with altered mental status. Brain imaging reveals an expansile mass centered in the pons. Biopsy is performed and demonstrates a diffuse glioma with rare mitotic activity. There is no microvascular proliferation or necrosis observed. Immunohistochemistry (IHC) for H3K27me3 shows loss of nuclear expression and H3K27M IHC is positive. By current WHO standards, what is the grade of this tumor?

    1. 1
    2. 2
    3. 3
    4. 4
    Board review style answer #2
    D. 4. By current WHO classification, a diffuse glioma involving a midline structure that harbors H3K27M alteration is WHO grade 4 and is associated with malignant clinical behavior. Therefore, by definition, Answers A, B and C are incorrect. WHO grade 1 neoplasms (A), including pilocytic astrocytoma, are clinically benign; although these neoplasms can cause significant clinical morbidity if located in neuroanatomically exquisite locations. WHO grade 2 and 3 neoplasms (B and C), such as oligodendroglioma, IDH mutant and 1p / 19q codeleted, represent tumors with intermediate clinical behavior.

    Comment Here

    Reference: Glioma overview

    Gliosarcoma
    Definition / general
    • Rare, classic variant of glioblastoma (GBM), WHO grade 4
    • Biphasic glial and mesenchymal differentiation
    Essential features
    • De novo: primary gliosarcoma (GS); secondary GS develops in previously resected or irradiated glioblastoma
    • Biphasic glial and mesenchymal differentiation
    • Clinical and genetic similarities to glioblastoma; likewise, treatment and management is similar to that of glioblastoma
    • Overall, an aggressive tumor with slightly worse prognosis than glioblastoma (Neuro Oncol 2009;11:183)
    Terminology
    • Gliosarcoma (GS)
    • Classic variant of glioblastoma (GBM)
    ICD coding
    • ICD-10: C71.9 - malignant neoplasm of brain, unspecified
    Epidemiology
    Sites
    • Majority are supratentorial, with tendency towards peripheral localization and dural attachment (Eur Radiol 2019;29:429)
    • Temporal > parietal > frontal > occipital lobes; rarely involve occipital region or posterior fossa (Neuro Oncol 2009;11:183)
    • Can involve and invade into overlying meninges, bone and extracranial soft tissue (Case Rep Radiol 2016;2016:1762195)
    • Mesenchymal / sarcomatous component is associated with propensity to invade and metastasize
    • Gliosarcoma has greater propensity for extracranial metastasis than other CNS tumors, including glioblastoma (J Neurooncol 2007;83:39)
    • Most common sites of extracranial metastases include lung, liver and lymph node but are also reported in adrenal gland, spleen, kidney, oral mucosa, skin, bone marrow and bone
    • Metastatic tumor deposits contain sarcomatous with or without glial components
    Pathophysiology
    • Similar genetic alterations (p53 mutations, p16 deletions and PTEN mutations) both in glial and sarcomatous components support monoclonal origin (Am J Pathol 2000;156:425)
    Etiology
    • Prior history of radiation for CNS tumor is a known risk factor
    Clinical features
    • Presents similarly to glioblastoma
    • Symptoms and signs depend on tumor location, increased intracranial pressure
    • Common symptoms include headache, nausea, vomiting, seizures, dysphasia, dizziness, cognitive dysfunction, altered mental status and personality changes
    • Common signs include focal neurological deficits, visual field defects and papilledema
    • Reference: Cancer 2010;116:1358
    Diagnosis
    • Based on clinical history, imaging, histology, immunohistochemical and molecular features
    Radiology description
    Radiology images

    Contributed by Nicolas Kostelecky, M.D.
    Sagittal T1 postcontrast

    Sagittal T1 postcontrast

    Coronal T1 postcontrast

    Coronal T1 postcontrast



    Images hosted on other servers:
    Spectrum of radiologic phenotypes (n = 4)

    Spectrum of
    radiologic
    phenotypes
    (n = 4)

    Heterogeneous, solid / cystic, with rim enhancement

    Heterogeneous, solid / cystic, with rim enhancement

    Solid / cystic with leptomeningeal involvement

    Solid / cystic with leptomeningeal involvement

    Cystic / solid; temporoparietal region

    Cystic / solid; temporoparietal region

    Before surgery

    Before surgery

    Prognostic factors
    Case reports
    Treatment
    Gross description
    • Usually well circumscribed, lobular mass in superficial location, weakly attached to dura (Neurosurgery 1990;26:261)
    • Glial component is soft with foci of yellowish necrosis or red-brown recent and remote hemorrhage
    • Sarcomatous component is firm and well circumscribed, facilitating separation from adjacent brain tissue
    Microscopic (histologic) description
    • Mixed tumor of biphasic differentiation with glial and sarcomatous components
    • Proportions of glial and sarcomatous components vary; may show mosaic pattern
    • Glial component typically shows typical features of glioblastoma, including pleomorphic astrocytic cells with nuclear atypia, mitosis, pseudopalisading necrosis and microvascular proliferation
    • Sarcomatous component shows densely packed, spindle shaped cells with nuclear atypia; can show a variety of morphologies: fibrosarcoma, osteosarcoma, chondrosarcoma, angiosarcoma, rhabdosarcoma or undifferentiated pleomorphic sarcoma
    Microscopic (histologic) images

    Contributed by Irfan Yasin, M.B.B.S., Katherine Schwetye, M.D., Ph.D. and Miguel Guzman, M.D.
    Glial and sarcomatous component

    Glial and sarcomatous component

    Sarcomatous component Sarcomatous component

    Sarcomatous component

    Herringbone architecture

    Herringbone architecture

    Severe atypia

    Severe atypia


    GFAP GFAP

    GFAP

    Reticulin Reticulin

    Reticulin

    Virtual slides

    Images hosted on other servers:
    H&E; intermixed glial and sarcomatous elements

    H&E; intermixed glial and sarcomatous elements

    Cytology description
    Positive stains
    Negative stains
    • Olig2, unlike other glioblastomas
    Molecular / cytogenetics description
    • Most common genetic aberrations in gliosarcoma similar to those of primary glioblastoma: PTEN and PIK3 mutations, p53 mutation and p16 deletion (Cancers (Basel) 2019;11:284)
    • Among 19 commonly altered genes in gliosarcoma: most frequent are TERT promoter (92%), PTEN (66%), TP53 (60%) and NF1 (41%) (Sci Rep 2021;11:18009)
    • Altered genes that are potentially targetable by FDA approved therapies: BRAF, EGFR, CDKN2A, NF1 and PTEN (Sci Rep 2021;11:18009)
    • Less frequent pathologic EGFR alteration in gliosarcoma as compared to glioblastoma (BMC Neurol 2021;21:231)
    • MGMT methylation and IDH1 mutation are rare events in primary gliosarcoma (J Neurooncol 2018;137:303)
    • Gliosarcomas overexpress mesenchymal proteins, indicating epithelial mesenchymal transition (EMT) (Brain Pathol 2012;22:670)
    Sample pathology report
    • Brain, cerebrum, right, mass, excision:
      • Gliosarcoma, IDH wildtype, WHO grade 4 (see microscopic description and comment)
      • Microscopic examination shows highly pleomorphic, glial and spindled tumor cells with pseudopalisading necrosis and microvascular proliferation. The spindle cell component has herringbone architecture. The more pleomorphic glial elements show high cellularity. Tumor is remarkable for abundant atypical mitosis. There are areas of geographical necrosis.
      • IHC: The glial elements are positive for GFAP. Reticulin is markedly increased in the spindle cell areas with concomitant negativity for GFAP. This dual immunostaining pattern with strong reactivity of glial component for GFAP and reticulin positivity in mesenchymal component supports a diagnosis of glioblastoma.
      • Molecular: PCR based analysis of IDH1 and IDH2 genes (exon 4) shows wildtype phenotype. Analysis for MGMT gene promoter hypermethylation shows no hypermethylation.
    Differential diagnosis
    • Glioblastoma with meningeal invasion:
      • Lacks alternating areas of glioma and sarcoma
    • Oligosarcoma (Acta Neuropathol 2022;143:263):
      • Recurrence after oligodendroglioma or de novo
      • Belongs to IDH mutant and 1p19q codeleted category in 2021 WHO
      • Smooth muscle differentiation by proteomic profiling
      • Embedded in a dense network of reticulin fibers
      • Frequent p53 accumulation, positivity for SMA and CALD1, loss of Olig2, gain of H3K27me3, CDKN2A / CDKN2B deletions, loss of NF1 and gain of YAP1
      • Frequent mutations in IDH, TERT promoter, FUBP1, CIC, NF1 and TP53
    • Primary sarcomas of the CNS:
      • Lack glial differentiation
    • Metastatic sarcomas:
      • Clinical history of a primary sarcoma elsewhere
      • Lack glial differentiation
    Board review style question #1

    A 58 year old man with a medical history that is significant for resection and treatment (including radiotherapy) of a glioblastoma (IDH wildtype) 2 years prior, presents with a headache. MRI shows a heterogeneously enhancing mass associated with the dura near his prior resection site. The tumor is resected and shows the above on H&E and immunostains. Which of the following is most likely true of this tumor?

    1. It is a metastatic sarcoma
    2. It is a recurrent glial malignancy, this time with sarcomatous differentiation: secondary gliosarcoma
    3. Most likely hypermethylated in the MGMT gene promoter region
    4. WHO grade is 3, since we do not see necrosis
    Board review style answer #1
    B. It is a recurrent glial malignancy, this time with sarcomatous differentiation: secondary gliosarcoma

    Comment Here

    Reference: Gliosarcoma
    Board review style question #2
    Gliosarcoma shows which of the following similarities to primary glioblastoma?

    1. Frequent EGFR alteration and MGMT hypermethylation
    2. Frequent IDH mutations and loss of ATRX expression
    3. Frequent infratentorial and spinal cord localization
    4. Slightly increased incidence in males
    Board review style answer #2
    D. Slightly increased incidence in males

    Comment Here

    Reference: Gliosarcoma

    Granular cell tumor (pending)
    [Pending]

    Grossing, features to report & staging
    Definition / general
    • Evaluate the arachnoid, gray and white matter
    • Section, if possible, from the arachnoid through the gray to the white matter
    • Sample interface between the lesion and normal appearing brain
    • EM helpful for poorly differentiated or unusual tumors (clear cell ependymoma), toxic - metabolic disease, infection; recommended to save tissue for EM in challenging cases
    • Infarcts: section perpendicular to surface of brain, submit sections of arachnoid, gray and white matter; save gray matter for EM if considering CADASIL or mitochondrial disease
    • Hematopoietic lesions: also B5, cell culture media (for flow cytometry), touch preparations, flash frozen tissue for molecular analysis
    • Infectious disorders: recommended that surgeon obtain cultures from sterile operating field
    • Toxic - metabolic disorders: recommend fixation of gray and white matter in formalin, glutaraldehyde, frozen tissue bank

    • Margins: usually impossible to assess due to piecemeal nature of resection or infiltrative growth pattern; may be able to determine for childhood pilocytic astrocytoma of cerebellum, cerebral dysembryoplastic neuroepithelial tumor and meningioma

    • Safety: recommended that surgeon contact lab if patient is HIV+ (use cytology, not frozen section) or may have Creutzfeldt-Jakob disease (decline frozen sections since cannot decontaminate cryostat); decontaminate cryostat used for HIV+ tissue by replacing blade, removing frozen debris and wiping surfaces with 10% bleach in water; for CJD specimens that are sectioned, recommended to place cassettes in formalin for 24 hours, then transfer to 100% formic acid for one hour, then return to fresh formalin for 48 hours, then process; label cassettes with pencil (formic acid dissolves ink), wrap needle biopsies in lens paper, handled embedded tissue as if infectious, dispose of blade, section waste, wipe microtome and cutting station with bleach, incinerate all towels, gloves and waste

    Gross features of various disease entities
    • Abnormal mass: solid tissue not identifiable as gray or white matter
    • Cyst wall: flat tissue from 0 to 3 mm thick - compare to surgeon's impression or radiographs
    • Gliosis: often yellow or gray and firm
    • Necrosis: soft, friable, often cavitates
    • Semiliquid or gelatinous masses: usually tumors, including oligodendroglioma, lymphoma and pituitary adenoma
    • Vascular malformation: vessels larger than usual aggregate of arachnoid vessels, often associated with hemorrhage, may involve meninges or parenchyma
    Intraoperative consult
    • Indications: (a) determine if lesional tissue is present, (b) determine if adequate sampling, (c) provide preliminary information to assist neurosurgeon (see below) and (d) perform special techniques (culture, B5 for lymphoma, touch preparations)
    • Need not give definitive diagnosis
    • Should not grade astrocytic glial neoplasm unless is clearly a glioblastoma (since tumors often have variable grades)
    • For some tumors, attempts at total resection are made (meningioma, schwannoma, solitary metastases, cysts, ependymoma, hemangioblastoma, cerebellar pilocytic astrocytoma, craniopharyngioma) so intraoperative consultation may be helpful
    • Recommended to assess undefined lesions by both frozen section and cytologic preparation
    • Cytologic preparation: adds fine nuclear detail, reveals glial type processes or epithelioid features of carcinomas; shows discohesiveness associated with pituitary adenoma, oligodendroglioma, medulloblastoma or lymphoma; may be more accurate than frozen section (Stereotact Funct Neurosurg 1995;65:187)
    • Recommended to obtain touch preparations (touch glass slide to wet tissue, fix before it dries, then stain)
    • Artifacts: long empty cavities in parenchyma (due to cryostat) vs. microcysts which contain cells and eosinophilic proteinaceous material)
    • References: Arch Pathol Lab Med 2005;129:1635, Arch Pathol Lab Med 1997;121:481, Mod Pathol 1988;1:378, J Neurol Neurosurg Psychiatry 1988;51:332, Arch Pathol Lab Med 2005;129:1653
    Features to report
    • Specimen type
    • Specimen size (greatest dimension, additional dimensions are optional)
    • Tumor site
    • Tumor size (greatest dimension, additional dimensions are optional)
    • Histologic type
    • Histologic grade (WHO I - IV, other, cannot be determined, not applicable)
    • Margins (involved, uninvolved, cannot be assessed, not applicable)
    • Results of additional studies performed (electron microscopy, cytogenetics, molecular testing, immunohistochemistry)
    • References: Arch Pathol Lab Med 2001;125:1162

    Staging
    • No TNM classification exists for CNS tumors (Edge: AJCC Cancer Staging Manual, 8th Edition, 2017) for these reasons:
      • For T component, histology and location are more important than tumor size
      • For N component, brain and spinal cord tumors do not propagate in lymphatics, so lymph nodes are not staged
      • For M component, metastatic disease is rarely seen (except for CSF seeding in some pediatric tumors) and extremely rare outside of the CNS
      • WHO classification and WHO grading systems should be used
      • Generally favorable prognostic factors: younger age, total resection versus subtotal resection; higher Karnofsky performance scale

    Hemangioblastoma
    Definition / general
    • Benign, slowly growing, highly vascular neoplasm containing neoplastic stromal cells which usually involves the cerebellum, brainstem or spinal cord
    • Generally associated with loss or inactivation of the VHL gene, with frequent occurrence in von Hippel-Lindau (VHL) disease (Am J Clin Pathol 1997;107:459)
    Essential features
    • Circumscribed, enhancing solid cystic mass, most frequently located in the cerebellum and observed as a cyst with enhancing mural nodule (AJNR Am J Neuroradiol 1992;13:1343)
    • Composed of neoplastic stromal cells with foamy cytoplasm within a background network of numerous small vessels
    • Associated with loss of function of VHL, with frequent occurrence in von Hippel-Lindau (VHL) disease (Am J Clin Pathol 1997;107:459)
    • Other neoplasms associated with VHL may be histologic mimics (e.g. clear cell renal cell carcinoma, paraganglioma, pheochromocytoma)
    • WHO grade 1
    Terminology
    • Hemangioblastoma
    • Capillary hemangioblastoma (not recommended)
    • Lindau tumor (not recommended)
    • Angioblastoma (not recommended)
    ICD coding
    • ICD-O: 9161/1 - hemangioblastoma
    • ICD-10:
      • D33.0 - benign neoplasm of brain, supratentorial
      • D33.1 - benign neoplasm of brain, infratentorial
      • D33.2 - benign neoplasm of brain, unspecified
      • D33.3 - benign neoplasm of cranial nerves
      • D33.4 - benign neoplasm of spinal cord
      • D33.7 - benign neoplasm of other specified parts of central nervous system
      • D33.9 - benign neoplasm of central nervous system, unspecified
    • ICD-11:
      • XH6810 - hemangioblastoma
      • 2F7Y - neoplasms of uncertain behavior of other specified site
    Epidemiology
    Sites
    Pathophysiology
    • VHL loss of function implicated in most cases
      • Normal VHL protein (pVHL)
        • Regulates cell cycle (Proc Natl Acad Sci U S A 1998;95:993)
        • Regulates cellular hypoxia signaling via the HIF (hypoxia inducible factor) complex, which under hypoxic conditions leads to enhanced levels of growth factors (e.g. vascular endothelial factor, platelet derived growth factor, transforming growth factor alpha) (Nat Rev Cancer 2015;15:55)
      • Pseudohypoxia hypothesis
      • VHL syndrome (Eur J Hum Genet 2011;19:617)
        • Autosomal dominant tumor predisposition syndrome resulting from inactivating germline mutation in the VHL tumor suppressor gene
        • Biallelic VHL inactivation
    Etiology
    Diagrams / tables

    Images hosted on other servers:

    VHL regulation of the HIF pathway

    Clinical features
    Diagnosis
    • Based on histologic and immunophenotypic features (Acta Neuropathol 2013;125:333)
    • Clinical history of VHL syndrome or evidence of genetic alterations leading to VHL inactivation is supportive
    Laboratory
    Radiology description
    Radiology images

    Images hosted on other servers:

    T1+C MRI posterior fossa

    Prognostic factors
    Case reports
    Treatment
    Clinical images

    Images hosted on other servers:

    Cystic mass in medulla

    Multiple spinal masses

    Gross description
    • Solid or cystic with mural nodule (J Neurosurg 2003;98:82)
    • Well circumscribed and pseudoencapsulated
    • Highly vascular
    • Red with yellow / orange cut surface regions attributed to lipid content
    Gross images

    Contributed by Rebecca Yoda, M.D.

    Hemangioblastoma cut surfaces



    Images hosted on other servers:

    Variegated red-yellow tumor

    Hemangioblastoma in situ

    Intraoperative frozen / smear cytology images

    Contributed by P.J. Cimino, M.D., Ph.D.
    Moderate hypercellularity

    Moderate hypercellularity

    Smear preparation

    Microscopic (histologic) description
    • Architecture (Acta Neuropathol 2013;125:333)
      • Composed of neoplastic stromal cells arranged between numerous small vessels
      • Compact, generally noninfiltrative growth with variable lobularity
    • Stromal component
      • Mild nuclear pleomorphism with degenerative atypia
      • Clear, foamy cytoplasm with lipid containing vacuoles
      • Rare hyaline globules
    • Vascular component
      • Numerous thin walled vessels
      • Well demarcated border with neoplasm
    • Other findings
      • Mitotic figures rare to absent
      • Intratumoral hemorrhage
      • Mast cells
      • Cyst-like spaces
      • Adjacent parenchyma may have piloid gliosis with Rosenthal fibers
      • Extramedullary hematopoiesis (~15%) (Neurosurgery 1991;29:34)
    • Histologic variants
    Microscopic (histologic) images

    Contributed by P.J. Cimino, M.D., Ph.D.

    Circumscribed border

    Vascular and stromal cells

    Foamy cytoplasm

    Large branching vessels

    Hyalinized stroma

    Extensive hemorrhage


    Nuclear atypia

    Inhibin

    CAIX

    CD31

    PAX8

    Virtual slides

    Images hosted on other servers:

    Hemorrhagic tumor

    Highly vascular tumor

    Cytology description
    • Generally resistant to smear (squash preparation), resulting in larger tissue clumps
    • Successful cytologic smear preparations include the following (Acta Cytol 1998;42:1104):
      • Cytoplasm
        • Foamy
        • Indistinct cytoplasmic borders
      • Nuclei
        • Mildly pleomorphic
        • Hyperchromatic, speckled chromatin
        • Nuclear grooves
      • Hemosiderin
      • Clumping of cohesive cells often obscures cytologic features
    Electron microscopy description
    Electron microscopy images

    Images hosted on other servers:

    EM lipid droplets

    Molecular / cytogenetics description
    Molecular / cytogenetics images

    Images hosted on other servers:

    VHL next generation sequencing

    Recurrent loss of chromosome 3

    DNA methylation based classification

    Idiogram of VHL mutation

    Videos

    Hemangioblastoma MRI

    Hemangioblastoma histopathology

    Sample pathology report
    • Brain, cerebellum, excision:
      • Hemangioblastoma, WHO grade 1
    Differential diagnosis
    Board review style question #1

    A 47 year old man presents with ataxia. MRI reveals a solitary cerebellar cyst with an enhancing mural nodule. A representative histologic photo of the surgical specimen is shown. On immunohistochemical stains, stromal cells are negative for AE1 / AE3 and PAX8. Which of the following IHC stains is most likely to be positive in stromal cells?

    1. CAM5.2
    2. CD31
    3. Inhibin alpha
    4. PAX2
    5. RCC marker
    Board review style answer #1
    C. Inhibin alpha is positive in hemangioblastoma stromal cells. CAM5.2 (answer A) is negative in hemangioblastoma and positive in epithelial neoplasms. CD31 (answer B) is negative in hemangioblastoma stromal cells and positive in vascular endothelial cells. PAX2 (answer D) is negative in hemangioblastoma and positive in renal cell carcinoma. RCC marker (answer E) is negative in hemangioblastoma and positive in renal cell carcinoma.

    Comment Here

    Reference: Hemangioblastoma
    Board review style question #2

    A 32 year old woman is found to have multiple enhancing intra-axial mixed solid cystic masses in the cerebellum and spinal cord as well as numerous bilateral renal cysts and a renal mass. A biopsy of a cerebellar mass is shown. A germline mutation is most likely to be present in which gene?

    1. FLCN
    2. NF2
    3. PKD1
    4. TSC1
    5. VHL
    Board review style answer #2
    E. VHL. Von Hippel-Lindau syndrome manifestations may include multiple hemangioblastomas, renal cysts and renal cell carcinoma.

    1. FLCN - Birt-Hogg-Dubé syndrome (increased risk of renal cysts and tumors; no CNS involvement)
    2. NF2 - neurofibromatosis type 2 (increased risk of various lesions primarily affecting the nervous system, including schwannoma, meningioma, ependymoma; no renal involvement)
    3. PKD1 - autosomal dominant polycystic kidney disease (associated with bilateral renal cysts and berry aneurysms; no other CNS involvement)
    4. TSC1 - tuberous sclerosis (renal manifestations may include angiomyolipoma and cysts; CNS manifestations may include cortical tubers and subependymal giant cell astrocytoma; no known association with hemangioblastoma)

    Comment Here

    Reference: Hemangioblastoma

    High grade astrocytoma with piloid features (HGAP)
    Definition / general
    • Circumscribed astrocytic glioma with high grade piloid or glioblastoma-like histological features and distinct DNA methylation profile
    Essential features
    • Recently recognized entity, included in the 2021 WHO classification of CNS tumors
    • Circumscribed astrocytic glioma occurring throughout the entire CNS, mostly in the posterior fossa
    • High grade histological features associated with piloid or glioblastoma-like morphology
    • Distinct DNA methylation profile and MAPK pathway gene alterations, often associated with homozygous deletion (HD) of CDKN2A / 2B or ATRX alterations
    • CNS WHO grade currently not assigned (clinical behavior roughly similar to CNS WHO grade 3)
    Terminology
    • Anaplastic astrocytoma with piloid features (not recommended by CNS WHO 2021)
    ICD coding
    • ICD-O: 9421/3 - high grade astrocytoma with piloid features
    • ICD-11: 2A00.0Y & XH6PH6 - other specified gliomas of brain & astrocytoma, NOS
    Sites
    Pathophysiology
    • Cell of origin is currently unknown
    • 3 pathways are probably involved in the pathogenesis (Acta Neuropathol 2018;136:273)
      • MAPK pathway gene alterations are likely the initiating event (driver mutation)
      • Deregulation of retinoblastoma tumor suppressor protein cell cycle pathway by CDKN2A / 2B inactivation (homozygous deletion) or occasionally CDK4 amplification
      • Activation of telomere maintenance by ATRX alterations and rarely TERT promoter mutations
    • Numerous chromosomal alterations and frequent structural aberrations (Acta Neuropathol 2018;136:273)
      • Partial gains of 12q and 17q
      • Losses of 1p and 8p
      • Partial losses of 14 and 19q
    Etiology
    Diagrams / tables
    Not relevant to this topic
    Clinical features
    • Clinical signs and symptoms depend on tumor location
    Diagnosis
    • Magnetic resonance imaging (MRI), followed by stereotactic brain biopsy or surgical resection
    • CNS WHO 2021 diagnostic criteria
      • Essential
        • Histological aspect of an astrocytic glioma and
        • DNA methylation profile of HGAP
      • Desirable
        • MAPK pathway gene alteration
        • CDKN2A / 2B HD or CDK4 amplification
        • ATRX mutation or loss of nuclear expression
        • Anaplastic histological features
    Laboratory
    Not relevant to this topic
    Radiology description
    Radiology images

    Contributed by Antonio d’Amati, M.D.
    T2 MRI of brain

    T2 MRI of brain

    T1 MRI of brain

    T1 MRI of brain

    Coronal FLAIR MRI of brain

    Coronal FLAIR MRI of brain

    Sagittal FLAIR MRI of brain

    Sagittal FLAIR MRI of brain

    Prognostic factors
    • Prognostic data are currently limited to a single retrospective study (Acta Neuropathol 2018;136:273)
      • 5 year overall survival: 50%
      • No prognostic association with high grade histological features (e.g., mitoses or necrosis)
      • No prognostic association with MGMT promoter methylation status
    • CNS WHO grade currently not assigned (clinical behavior roughly similar to CNS WHO grade 3)
    Case reports
    Treatment
    Clinical images
    Not relevant to this topic
    Gross description
    • Mostly well demarcated solid lesion
    • Areas of cystic degeneration, necrosis or hemorrhage may be variably present
    Gross images

    Contributed by Antonio d’Amati, M.D.
    Cerebellar surgical resection

    Cerebellar surgical resection

    Frozen section description
    Not relevant to this topic and unknown at this time (no studies currently published in literature)
    Frozen section images
    Not relevant to this topic and unknown at this time (no studies currently published in literature)
    Microscopic (histologic) description
    • Histological features are very variable and not sufficient for diagnosis, without additional molecular testing
    • Astrocytic glioma, with moderate cell density
    • Predominantly solid / circumscribed tumor growth but possible invasion into the adjacent parenchyma
    • Growth patterns (Acta Neuropathol 2018;136:273)
      • Glioblastoma-like
      • Pleomorphic xanthoastrocytoma-like
      • Pilocytic astrocytoma-like (cells with hair-like piloid cytoplasmic processes)
    • Eosinophilic granular bodies or Rosenthal fibers (~30% of cases) (Acta Neuropathol 2018;136:273)
    • High grade histological features
    Microscopic (histologic) images

    Contributed by Francesca Gianno, M.D., Ph.D.
    Piloid features

    Piloid features

    Spindle cells

    Spindle cells

    Sheets of cells

    Sheets of cells

    Rosenthal fibers Rosenthal fibers

    Rosenthal fibers


    Eosinophilic granular bodies

    Eosinophilic granular bodies

    Pleomorphic cells

    Pleomorphic cells

    Myxoid background Myxoid background

    Myxoid background

    GFAP

    GFAP


    Olig2

    Olig2

    ATRX

    ATRX

    Low Ki67

    Low Ki67

    High Ki67

    High Ki67

    p16

    p16

    Virtual slides
    To be added
    Cytology description
    Not relevant to this topic
    Cytology images
    Not relevant to this topic
    Immunofluorescence description
    Not relevant to this topic
    Immunofluorescence images
    Not relevant to this topic
    Positive stains
    Negative stains
    Electron microscopy description
    • Not routinely used for diagnostic purposes
    Electron microscopy images
    Not relevant to this topic
    Molecular / cytogenetics description
    Molecular / cytogenetics images

    Contributed by Simone Minasi, Ph.D. and Francesca Romana Buttarelli, Ph.D.
    CDKN2A homozygous deletion

    CDKN2A homozygous deletion

    KIAA1549::BRAF fusion

    KIAA1549::BRAF fusion



    Images hosted on other servers:
    TSNE analysis of DNA methylation data

    DNA methylation data

    H&E stainings and copy number plots

    Copy number plots

    Videos
    Not relevant to this topic (no videos found)
    Sample pathology report
    • Brain, cerebellum, resection:
      • Integrated diagnosis: high grade astrocytoma with piloid features (HGAP)
      • Histologic diagnosis: astrocytic glioma, with high grade histologic features
      • Immunohistochemical results
        • GFAP+
        • Olig2+
        • IDH1 R132H-
        • Loss of ATRX nuclear expression
        • Ki67: 10%
      • CNS WHO grade: not assigned
      • Molecular information
        • DNA methylation class: HGAP (score 0.99)
        • NF1 mutation
        • CDKN2A homozygous deletion
    Differential diagnosis
    Board review style question #1

    A 43 year man has a cerebellar mass. Histologic features are shown in the image above. Immunohistochemistry revealed IDH1 R132H negativity and loss of ATRX nuclear expression. Molecular analyses only revealed NF1 mutation and CDKN2A homozygous deletion. What is the most likely diagnosis?

    1. Ganglioglioma
    2. Glioblastoma, IDH wild type
    3. High grade astrocytoma with piloid features (HGAP)
    4. Pilocytic astrocytoma
    5. Pleomorphic xanthoastrocytoma
    Board review style answer #1
    C. High grade astrocytoma with piloid features (HGAP). The image shows an astrocytic glioma, with piloid aspects and high grade histologic features. Answer D is incorrect because pilocytic astrocytomas with high grade histologic features (anaplastic PA) are uncommon in adult population; moreover, ATRX loss and CDKN2A homozygous deletion exclude this diagnosis. Answers B and E are incorrect because glioblastoma, IDH wild type and pleomorphic xanthoastrocytoma are uncommon in cerebellar locations and usually show preserved ATRX expression. Answer A is incorrect because ganglioglioma shows different clinical, histological, immunohistochemical and molecular features.

    Comment Here

    Reference: High grade astrocytoma with piloid features (HGAP)
    Board review style question #2
    Which of the following is an essential molecular criterion to confirm a diagnosis of high grade astrocytoma with piloid features (HGAP)?

    1. ATRX alterations
    2. CDK4 amplification
    3. KIAA1549::BRAF fusions
    4. Specific DNA methylation profile
    5. TP53 mutations
    Board review style answer #2
    D. Specific DNA methylation profile. According to 2021 CNS WHO criteria, demonstration of DNA methylation profile is mandatory to confirm a diagnosis of HGAP. Answers A, B and C are incorrect because while ATRX alterations, KIAA1549::BRAF fusions and CDK4 amplification are reported at variable frequency in HGAP, these only represent desirable criteria. Answer E is incorrect because TP53 mutations are not commonly associated with HGAP.

    Comment Here

    Reference: High grade astrocytoma with piloid features (HGAP)

    High grade astrocytoma with piloid features (HGAP) (pending)
    [Pending]

    Histiocytic tumors
    Erdheim-Chester disease
    Histiocytic sarcoma
    Juvenile xanthogranuloma
    Langerhans cell histiocytosis
      Definition / general
    • See also Lymph node topic
    • Rare lytic skull lesion or parasellar mass
    • Children and young adults
    • May arise from primary histiocytic proliferation with secondary atrophy or from demyelination and gliosis of unknown origin (J Child Neurol 2000;15:150)
    Rosai-Dorfman disease
      Definition / general
    • See also Lymph node topic
    • Also called sinus histiocytosis with massive lymphadenopathy
    • Often (25 - 43%) has extranodal involvement
    • Rare (100 cases/year in US) and isolated CNS findings are extremely rare
    • Benign but may be associated with systemic disease
    • Children and young adults (mean age 41 years, range 22 - 63 years)
    • CNS tumors usually associated with dura
    • References: Mod Pathol 2001;14:172, Clin Neuropathol 2005;24:112 (isolated CNS disease)
    Microscopic (histologic) images

    Case #488

    Rosai-Dorfman disease of CNS

    Board review style question #1
      In distinguishing CNS Rosai-Dorfman disease (RDD) from other histiocytoses, what is the correct immunophenotype for RDD?

    1. S100-, CD1a-
    2. S100+, CD1a+
    3. S100+, CD1a-
    4. S100-, CD1a+
    Board review style answer #1
    C. S100+, CD1a-

    Explanation: CNS Rosai-Dorfman disease forms lymphohistiocytic masses with prominent pale S100+, CD1a- histiocytes. Answer A (S100-, CD1a-) is the staining pattern of histiocytes in Erdheim-Chester disease. Answer B (S100+, CD1a+) is the staining pattern of histiocytes in Langerhans cell histiocytosis.

    Comment Here

    Reference: Histiocytic tumors

    Infant type hemispheric glioma (pending)
    [Pending]

    Intracranial mesenchymal tumor, FET::CREB fusion positive (pending)
    [Pending]

    Intravascular
    Definition / general
    • Extranodal large B cell lymphoma characterized by lymphoma cells, predominantly within lumina of blood vessels, especially capillaries, with the exclusion of larger arteries and veins
    • Few to no circulating lymphoma cells in peripheral blood
    Essential features
    • Rare, mature B cell lymphoma limited to intravascular spaces
    • Most common in skin and central nervous system
    • Classical variant most common clinical presentation (neurological and cutaneous manifestations)
    • Neoplastic cells are located in lumina of small to intermediate sized vessels
    • Lymphoma cells are large with prominent nucleoli and vesicular chromatin
    • Pan B cell markers positive, as well as BCL2 and MUM1
    • Aggressive behavior with short overall survival
    Terminology
    • Intravascular large B cell lymphoma (ILBCL)
    • Malignant angioendotheliomatosis, angioendotheliomatosis proliferans syndrome, intravascular lymphomatosis, angioendothelilotropic lymphoma (all obsolete)
    ICD coding
    • ICD-O: 9712/3 - intravascular large B cell lymphoma
    Epidemiology
    Sites
    • Selective growth within lumina of small blood vessels, particularly capillaries
    • Extranodal sites, including bone marrow, with sinusoidal or perivascular involvement
    • Most common sites of involvement: skin, nervous system, kidneys, lungs, endocrine glands
    • Lymph node involvement rare (Blood 2018;132:1561)
    Pathophysiology
    • Localization to blood vessel lumens partially explained by lack of CD29 (β1 integrin) and CD54 (ICAM1), both of which are important for transvascular lymphocyte migration (Blood 2018;132:1561)
    Etiology
    • Unknown
    Clinical features
    • Classic / western form:
      • Clinical presentation ranges from a few mild symptoms (fever of unknown origin, pain, organ specific local symptoms) to severe symptoms (B symptoms and signs of multiorgan failure) (Cancer Sci 2021;112:3953)
      • CNS involvement (present in 35%) with heterogeneous symptoms, including sensory and motor deficits or neuropathies, meningoradiculitis, paresthesias, hemiparesis, seizures and altered mental state
      • Skin lesions (present in 40%) are heterogeneous, including painful indurated erythematous eruptions, cellulitis, peau d’orange, small red palpable spots, nodules with or without ulceration, tumors and erythematous and desquamated plaques (Blood 2018;132:1561)
      • Lymphadenopathy typically absent
      • May present with disseminated intravascular coagulation leading to critical bleeding after biopsy (Cancer Sci 2021;112:3953)
    • Cutaneous variant:
      • 25% of patients
      • More frequent in western countries
      • Patients usually younger, with a median age of 59
      • Single or multiple skin lesions; no other sites involved
      • Disease progression is less aggressive and associated with better overall survival (Blood 2018;132:1561)
    • Hemophagocytic syndrome associated form:
      • Presents in Asian populations with multiorgan failure, hepatosplenomegaly and pancytopenia
      • Bone marrow infiltration common
      • Skin and CNS involvement are rare
      • Rapid aggressive onset and progression with median survival of 2 - 8 months (Blood 2018;132:1561)
    Diagnosis
    Laboratory
    • Anemia, leukopenia, thrombocytopenia and unexplained hypoxemia are most frequently observed
    • Low levels of serum albumin
    • High LDH and ferritin (Br J Haematol 2019;187:328)
    Radiology description
    • Lung involvement presents with ground glass appearance and nodules (Blood 2018;132:1561)
    • Organ enlargement (liver, spleen, kidney and adrenal gland) is common (Cancer Sci 2021;112:3953)
    • Brain MRI shows hyperintense lesions in pons, nonspecific white matter lesions, infarct-like lesions or meningeal enhancement
    • PET CT can detect bone marrow involvement (Br J Haematol 2019;187:328)
    Prognostic factors
    • Median overall survival of 105 months, with chemotherapy 135 months
    • 5 year survival of approximately 50 - 60% (Br J Haematol 2019;187:328)
    Case reports
    Treatment
    Microscopic (histologic) description
    • Lymphoma cells can be found in any organ vessel lumina (Br J Haematol 2019;187:328)
    • Lymphoma cells are large with high nuclear to cytoplasmic ratio with single or multiple prominent nucleoli and scant cytoplasm
    • Morphologic spectrum from centroblasts to immunoblasts / plasmablasts, including rare forms with anaplastic morphology
    • Different growth patterns:
      • Discohesive - lymphoma cells are preferentially within the central portion of the blood vessels with a free floating appearance
      • Cohesive pattern - lymphoma cells almost completely fill the lumen to the point that assessment of vascular structure tends to be difficult
      • Marginating pattern - less frequent, lymphoma cells preferentially adhere to endothelium, leaving the central portion of the lumen free (Blood 2018;132:1561)
    • Infiltration patterns in bone marrow:
      • Pure intrasinusoidal infiltration - neoplastic cells are confined within the intrasinusoidal spaces
      • Intrasinusoidal infiltration with extravasation - neoplastic cells proliferate within intrasinusoidal space but with extravasation
      • Diffuse interstitial infiltration - neoplastic cells proliferate diffusely within bone marrow (Br J Haematol 2019;187:328)
    Microscopic (histologic) images

    Contributed by Kathryn Gibbons, M.D.
    ILBCL involving skin and subcutaneous tissue

    ILBCL involving skin and subcutaneous tissue

    Large lymphoma cells in small vessels

    Large lymphoma cells in small vessels

    ILBCL involving the brain parenchyma

    ILBCL involving the brain parenchyma

    Vessel with involvement by ILBCL

    Vessel with involvement by ILBCL


    Large lymphoma cells in the vessels

    Large lymphoma cells in the vessels

    CD20

    CD20

    BCL6

    BCL6

    MUM1

    MUM1

    Virtual slides

    Images hosted on other servers:

    Lymphoma cells in blood vessels of subcutaneous adipose tissue

    Lymphoma cells limited to lumina of blood vessels

    Positive stains
    Negative stains
    Molecular / cytogenetics description
    Sample pathology report
    • Skin, left thigh, punch biopsy:
      • Intravascular large B cell lymphoma (see comment)
      • Comment: Biopsy shows large atypical CD20 positive B cells present within vascular spaces in the subcutaneous fat. These cells coexpress CD5, BCL6 and MUM1 and are negative for CD10 and CD3. Overall, these findings are consistent with intravascular large B cell lymphoma.
    Differential diagnosis
    Board review style question #1

    The image shown above is from a brain biopsy in a patient with fever and seizures. Large cells are strongly positive for CD20. What is the correct diagnosis?

    1. CNS vasculitis
    2. Intravascular large B cell lymphoma
    3. Marginal zone lymphoma
    4. Plasmablastic lymphoma
    Board review style answer #1
    B. Intravascular large B cell lymphoma

    Comment Here

    Reference: Intravascular large B cell lymphoma
    Board review style question #2
    Which of the following combinations of immunohistochemical stains is most consistent with immunophenotypic profile of intravascular large B cell lymphoma?

    1. CD20+, CD3-, CD5-, CD10+, BCL6+, BCL2+, c-MYC+
    2. CD20+, CD5+, Cyclin D1+
    3. CD20+, PAX5+, MUM1+, BCL2+
    4. CD20-, CD2+, CD3+, CD5-, CD7+, CD56+
    Board review style answer #2
    C. CD20+, PAX5+, MUM1+, BCL2+

    Comment Here

    Reference: Intravascular large B cell lymphoma

    Lymphomatoid granulomatosis
    Definition / general
    • Lymphomatoid granulomatosis is an angiocentric and angiodestructive lymphoproliferative disease involving extranodal sites composed of Epstein–Barr virus (EBV) positive B cell admixed with reactive T cells, which usually predominate (Blood 2016;127:2375)
    Essential features
    • Angiocentric and angiodestructive lymphoproliferative disease involving extranodal sites composed of EBV positive B cell admixed with reactive T cells
    Terminology
    • Angiocentric immunoproliferative lesion
    ICD coding
    • ICD-O: 9766/1 - lymphomatoid granulomatosis
    • ICD-O: 9766/3 - lymphomatoid granulomatosis, grade 3
    Epidemiology
    Sites
    Pathophysiology
    • Lymphomatoid granulomatosis due to the predominance of T cells on pathologic examination, was initially thought to be a T cell disorder (Am J Med 1982;72:467)
      • Subsequently, lymphomatoid granulomatosis was determined to be a B cell lymphoproliferative disorder secondary to EBV, with a prominent angiocentric T cell infiltrate (Mod Pathol 1990;3:435)
    • Lymphomatoid granulomatosis has a complex relationship with the host’s underlying immune function and defective immune surveillance of EBV infected B cells, particularly a functional defect in CD8+ cytotoxic T cells, is hypothesized to lead to disease development (Blood 1996;87:4531)
    • Particular study suggests that immunologic deficits are likely preexistent and that a quantitative or qualitative defect in mainly CD8+ cytotoxic T cells may be prerequisite for disease (Cancer 1982;49:2070)
    Etiology
    • Resembles an EBV driven lymphoproliferative disorder (Blood 1996;87:4531)
    • Also common in many immunodeficiency states, such as AIDS, allogenic organ transplant, Wiskott-Aldrich syndrome and X linked lymphoproliferative syndrome (Cancer Surv 1997;30:233)
    • Normal counterpart: mature B cell, transformed by EBV (Blood 2016;127:2375)
    Clinical features
    Diagnosis
    • Combination of light microscopy and immunohistochemical markers or in situ hybridization for EBV coded small RNA (EBER)
    Radiology description
    • Chest Xray may show single or multiple nodules with poorly defined margins
    • CT scans show well defined and poorly defined nodules throughout both lungs (AJR Am J Roentgenol 2000;175:1335)
    Radiology images

    Contributed by Nicholas Joseph Dcunha, M.B.B.S., M.D. and Elanthenral Sigamani, M.B.B.S., M.D.

    Mass in lung (CT scan)

    Prognostic factors
    • Prognostic features are difficult to determine due the rarity of the disease
    Case reports
    Treatment
    • In a prospective NCI study, patients with low grade lymphomatoid granulomatosis received primary therapy with IFN-α while patients with high grade lymphomatoid granulomatosis received primary therapy with DA-EPOCH-R (dose adjusted etoposide, prednisone, vincristine, cyclophosphamide, doxorubicin and rituximab) every 3 weeks for up to 6 cycles of therapy (Blood 2018;132:785)
    Gross description
    Microscopic (histologic) description
    • Angiocentric and angiodestructive polymorphous lymphoid infiltrate (Cancer 1979;43:360)
    • Lymphocytic vasculitis with infiltration of the vessel wall; may also show infarct like tissue necrosis or fibrinoid necrosis of vessel wall (Blood 1997;90:4099)
    • Infiltrate is predominantly composed of lymphocytes with admixed plasma cells, immunoblasts and histiocytes; background lymphocytes; may show atypia but are not overtly neoplastic
    • Lymphomatoid granulomatosis is composed of variable number of EBV positive B cells admixed with inflammatory background (Am J Surg Pathol 2010;34:e35)
    • EBV positive cells resemble immunoblasts or are multinucleated; may show atypia
    • Classic Reed-Sternberg cells are absent
    • Well formed granulomas are typically not seen in lungs and most other extranodal sites except skin where granulomatous reaction might be seen in subcutaneous tissue (Am J Surg Pathol 2001;25:1111)
    • Grading is based on the relative number of EBV positive B cells to the reactive lymphocyte background (Am J Surg Pathol 2015;39:141)
      • Grade I: EBV positive cells are < 5 per high power field; absent or rare large transformed cells on light microscopy; necrosis if present is focal
      • Grade II: EBV positive cells are 5 - 50 per high power field; few large transformed cells on light microscopy; necrosis is more commonly seen
      • Grade III: EBV positive cells are > 50 per high power field; large transformed cells are easily seen on light microscopy; large areas of necrosis are common
    Microscopic (histologic) images

    Contributed by Nicholas Joseph Dcunha, M.B.B.S., M.D. and Elanthenral Sigamani, M.B.B.S., M.D.

    Lymphocytic vasculitis with angioinvasion (lung)

    CD20 in tumor cells and CD3

    CD30 and EBV LMP1 IHC


    Lymphocytic vasculitis with angioinvasion (skin)

    CD20 and CD30 in tumor cells; CD3 and CD4 in reactive T cells

    Lymphocytic vasculitis with angioinvasion (CNS)

    CD20 and EBV LMP1 in tumor cells; CD3 in reactive T cells

    Positive stains
    Molecular / cytogenetics description
    Sample pathology report
    • Lung tissue, bronchoscopic biopsy:
      • Lymphomatoid granulomatosis, grade I, biopsies from left lung, upper and lower lobes (see comment)
      • Comment: The small reactive lymphoid cells are CD3 positive. The scattered large cells are positive for CD20 and CD79a and show faint membrane positivity for EBV LMP1. The MIB1 proliferation index is low (approximately 10 - 20%). Special stains for microorganisms (AFB TB, PASD, GMS) are negative.
      • Microscopic description: Dense interstitial and diffuse infiltrates of small lymphoid cells with round nuclei, clumped chromatin and scattered large lymphoid cells with round nuclei, visible nucleoli and moderate amounts of cytoplasm. Admixed with these are a few aggregates of histiocytes and occasional eosinophils. The lymphoid cells display angioinvasion with transmural infiltration by lymphocytes which adhere to the endothelium with focal luminal occlusion. There are extensive areas of necrosis with necrotic outlines of blood vessels. The adjacent lung tissue shows extensive type II pneumocyte hyperplasia.
    Differential diagnosis
    Board review style question #1
    What cell is neoplastic in a case of lymphomatoid granulomatosis?

    1. B lymphocytes
    2. Histiocytes
    3. NK cells
    4. T lymphocytes
    Board review style answer #1
    A. B lymphocytes

    Comment Here

    Reference: Lymphomatoid granulomatosis
    Board review style question #2

    Which of these findings is essential for the diagnosis of lymphomatoid granulomatosis?

    1. Angioinvasion by tumor cells
    2. Granulomatous reaction
    3. Necrosis
    4. Reed-Sternberg-like cells
    Board review style answer #2
    A. Angioinvasion by tumor cells

    Comment Here

    Reference: Lymphomatoid granulomatosis

    MALT lymphoma of the dura (pending)
    [Pending]

    Medulloblastoma
    Definition / general
    • 4 genetically defined groups and 4 histologically defined groups (Nature 2017;547:311)
      • Genetically defined: (a) WNT (for wingless integration site) activated, (b) SHH (sonic hedgehog) activated (either TP53 mutant or TP53 wild type), non-WNT / non-SHH, either (c) medulloblastoma group 3 or (d) medulloblastoma group 4 (Nature 2017;547:311)
      • Histologically defined: classic, desmoplastic / nodular, medulloblastoma with extensive nodularity and large cell / anaplastic medulloblastoma (Neuropathol Appl Neurobiol 2002;28:257)
    Essential features
    ICD coding
    • ICD-10:
      • C71 - malignant neoplasm of brain
      • C71.6 - malignant neoplasm of cerebellum
    Epidemiology
    • Medulloblastoma is the most common CNS embryonal tumor of childhood and is second only to pilocytic astrocytoma for all intracranial neoplasms (Acta Neuropathol 2012;123:473)
    • Classic medulloblastoma (Acta Neuropathol 2012;123:473)
      • Non-WNT / non-SHH tumors or WNT activated
      • Midline location
    • Desmoplastic / nodular medulloblastoma (Acta Neuropathol 2012;123:473)
      • Cerebellar hemispheres and midline
      • Bimodal age distribution
      • Gorlin syndrome: nevoid basal cell carcinoma syndrome
      • SHH activated
      • Favorable outcome in young children compared to nondesmoplastic
    • Medulloblastoma with extensive nodularity (Acta Neuropathol 2012;123:473)
      • Closely related to desmoplastic medulloblastoma
      • SHH activated
      • Predominantly occurs in infants
      • Excellent outcome
    • Large cell / anaplastic medulloblastoma (Acta Neuropathol 2012;123:473)
      • Undifferentiated cells with marked nuclear pleomorphism, prominent nucleoli, cell wrapping and high mitotic count and apoptotic counts
      • Associated with all genetic variants but more common in SHH activated and in group 3
    Sites
    • Fourth ventricle or cerebellar parenchyma
    Clinical features
    • Signs and symptoms of increased intracranial pressure (headache, nausea, vomiting)
    Diagnosis
    • Diagnosis of medulloblastoma requires histologic examination of tumor tissue and molecular genetic analyses
    • Integrated pathology diagnosis that includes both morphologic subtype and molecular subgroup along with any additional molecular data is recommended (Pediatr Dev Pathol 2022;25:23)
    Radiology description
    Prognostic factors
    • 3 prognostic subgroups (high risk, low risk and standard risk) depending on (Neuro Oncol 2021;23:1163)
      • Clinical findings: age at diagnosis, presence or absence of metastases at diagnosis, extent of resection of the tumor (Neuro Oncol 2021;23:1163)
      • Histopathological findings: histological group (large cell / anaplastic = poor prognosis; desmoplastic / nodular = good prognosis), presence of tumor cells in the cerebrospinal fluid (Neuro Oncol 2021;23:1163)
      • Biological / molecular findings: immunohistochemical / genetic subgroup (non-WNT / non-SHH = groups 3 and 4 having a poor prognosis; WNT activated = good prognosis), SHH activated TP53 mutation (associated with a poorer prognosis), MYC / MYCN amplification (associated with a poorer prognosis) (Neuro Oncol 2021;23:1163)
    Case reports
    Treatment
    Gross images

    Images hosted on other servers:
    Cerebellar tumor extending into fourth ventricle

    Cerebellar tumor extending into fourth ventricle

    Microscopic (histologic) description
    • Classic medulloblastoma (Neuropathol Appl Neurobiol 2002;28:257)
      • Small, blue, round cell tumor
      • Syncytial arrangement of densely packed undifferentiated cells (embryonal cells)
      • Mitosis with apoptotic bodies
      • Homer Wright rosettes
    • Desmoplastic / nodular medulloblastoma (Neuropathol Appl Neurobiol 2002;28:257)
      • Densely packed, undifferentiated cells with hyperchromatic and pleomorphic nuclei that produce dense intercellular reticulin fiber network with nodular reticulin free zones
    • Medulloblastoma with extensive nodularity (Neuropathol Appl Neurobiol 2002;28:257)
      • Expanded lobular architecture due to reticulin free nodular zones becoming enlarged and rich in neuropil-like tissue
    • Large cell / anaplastic medulloblastoma (Neuropathol Appl Neurobiol 2002;28:257)
      • Anaplasia with marked nuclear pleomorphism, high mitotic count and high apoptotic count
      • Nuclear molding and cell wrapping
    • Medulloblastomas with melanotic or myogenic differentiations (Neuropathol Appl Neurobiol 2002;28:257)
    Microscopic (histologic) images

    Contributed by Arnault Tauziede-Espariat, M.D., John DeWitt, M.D., Ph.D., Meaghan Morris, M.D., Ph.D.,
    Nirupama Singh, M.D., Ph.D. and Eman Abdelzaher, M.D., Ph.D.


    Genetically defined

    WNT (for wingless integration site) activated
    YAP1 immunoreactivity

    YAP1 immunoreactivity

    Beta catenin immunoreactivity

    Beta catenin immunoreactivity



    SHH (sonic hedgehog) activated (either TP53 mutant or TP53 wild type)
    Sheets of undifferentiated cells

    Sheets of undifferentiated cells

    Vague nodular formation

    Vague nodular formation

    Vague nodular formation with reticulin

    Vague nodular formation with reticulin

    Neuron specific enolase (NSE)

    Neuron specific enolase (NSE)

    Ki67

    Ki67

    Synaptophysin

    Synaptophysin


    Beta catenin

    Beta catenin

    INI1

    INI1

    YAP1 immunoreactivity

    YAP1 immunoreactivity

    GAB1 immunoreactivity

    GAB1 immunoreactivity

    OTX2 immunoreactivity

    OTX2 immunoreactivity



    Histologically defined

    Classic
    Homer Wright rosettes Homer Wright rosettes

    Homer Wright rosettes



    Desmoplastic / nodular medulloblastoma

    Desmoplastic medulloblastoma

    Pale nodular islands


    Pale islands formed of uniform cells

    Tumor in the subarachnoid space

    Desmoplastic / nodular medulloblastoma

    Desmoplastic / nodular medulloblastoma

    Reticulin fibers network

    Reticulin fiber network



    Medulloblastoma with extensive nodularity
    Extensive nodularity Extensive nodularity

    Extensive nodularity



    Large cell / anaplastic medulloblastoma
    Apoptotic bodies, prominent nucleoli

    Apoptotic bodies, prominent nucleoli

    Cell wrapping

    Cell wrapping

    Anaplasia in medulloblastoma

    Anaplasia in medulloblastoma



    Special morphology
    Melanocytic differentiation

    Melanocytic differentiation

    Myogenic differentiation

    Myogenic differentiation

    Cytology description
    • Clusters of tumor cells with hyperchromatic nuclei
    Cytology images

    Contributed by Arnault Tauziede-Espariat, M.D.
    Cytological findings

    MGG staining

    Positive stains
    Negative stains
    Molecular / cytogenetics description

    Genetic subgroups Immunohistochemical markers
    β catenin
    GAB1
    YAP1
    OTX2 p75 NGFR Filamin A
    WNT activated + n - + n and c / - + n - + c
    Non-WNT / non-SHH (groups 3 and 4) - - - + n - -
    SHH activated - (only c and m) + c + n and c - + c + c
    c = cytoplasmic, m = membranous, n = nuclear
    Molecular / cytogenetics images

    Contributed by Arnault Tauziede-Espariat, M.D.
    <i>MYCN</i> amplification

    MYCN amplification

    Sample pathology report
    • Brain, cerebellar tumor, resection:
      • Classic medulloblastoma (see comment)
      • Comment: Medulloblastoma, WNT activated, CNS WHO Grade 4
      • Molecular pathology findings
        • CTTNB1 p.Ser33Cys mutation
        • Monosomy 6
    Board review style question #1

    The tumor shown above is found in the cerebellum in a child. What is the diagnosis?

    1. Anaplastic / large cell medulloblastoma
    2. Classic medulloblastoma
    3. Desmoplastic / nodular medulloblastoma
    4. Myomedulloblastoma
    Board review style answer #1
    C. Desmoplastic / nodular medulloblastoma

    Comment Here

    Reference: Medulloblastoma
    Board review style question #2
    Which of the following is true of medulloblastomas?

    1. Medulloblastomas, SHH activated show YAP1 and OTX2 immunoreactivities
    2. Medulloblastomas, WNT activated show YAP1 and GAB1 immunoreactivities
    3. Medulloblastomas, non-WNT / non-SHH show OTX2 expression
    4. Beta catenin cytoplasmic expression is specific of medulloblastomas, WNT activated
    5. p53 overexpression is specific of medulloblastomas, SHH activated
    Board review style answer #2
    C. Medulloblastomas, non-WNT / non-SHH show OTX2 nuclear expression. Answers A and D are incorrect because medulloblastomas, SHH activated show YAP1 and GAB1 immunoreactivities and beta catenin nuclear expression is specific of medulloblastomas, WNT activated. Answer B is incorrect because medulloblastomas, WNT activated show YAP1 immunoreactivity without GAB1 expression. Answer E is incorrect because p53 overexpression is evidenced in cases of Li-Fraumeni syndrome. Medulloblastomas, SHH activated are frequent in Li-Fraumeni syndrome but other histomolecular subtypes are possible.

    Comment Here

    Reference: Medulloblastoma

    Melanocytic tumors / melanoma
    Definition / general
    • Melanocytes are a normal yet sparse cell of the leptomeninges, most often seen over the anterior / lateral cord, brainstem, base of brain
    • Can give rise to rare primary intracranial melanocytic tumors
    • WHO recognizes three categories of primary CNS melanocytic lesions: diffuse melanocytosis, melanocytoma, malignant melanoma
    • Diffuse melanocytosis: strongly associated with neurocutaneous melanosis, a rare congenital syndrome with giant congenital pigmented skin nevi and high rate of CNS melanoma usually presenting before age 2 (Semin Cutan Med Surg 2004;23:138)
    • Melanocytoma: less than 0.1% of brain tumors, arises at any age
    • Melanoma: incidence of 0.005 cases per 100,000; reported in ages 15 - 71 with a peak in the fourth and fifth decade
    Sites
    • Diffuse melanocytosis can involve infra or supratentorial leptomeninges but has highest frequency in cerebellum, brain stem, temporal lobes
      • Involves subarachnoid space and superficial cortex
    • Melanocytomas can occur in any area of meninges; however, have a predilection for cervical and thoracic spinal cord (intradural, extramedullary) and Meckel cave
    • Melanomas can also occur anywhere within meninges but have predilection for spinal cord, posterior fossa, Meckel cave
    Clinical features
    • Diffuse melanocytosis presents most commonly with features of neurocutaneous melanosis - congenital nevi, hydrocephalus, mass effect and neuropsychiatric symptoms
    • Melanocytoma and melanoma present with mass effect / cord compression symptoms
    Radiology images

    Images hosted on other servers:

    Diffuse melanocytosis

    MRI of thoracic area

    Abnormal signal in the left temporal lobe

    Prognostic factors
    • Diffuse melanocytosis: poor prognosis even when histologically benign
    • Melanocytoma: good prognosis with resection, rarely transforms into malignant melanoma
      • Can be called intermediate grade when Ki67 and mitoses are present but no obvious melanoma
    • Malignant melanoma: poor prognosis (6 years in spine) yet better than metastatic melanoma to CNS (6 months) (J Neurosurg 1987;66:47)
    Case reports
    Treatment
    • Gross total resection
    • Adjuvant chemoradiation therapy for malignant melanoma
    Gross description
    • Usually solitary, well demarcated, dural based with black or reddish brown discoloration
    Microscopic (histologic) description
    • Melanocytoma:
      • Solitary, circumscribed lesions - do not invade adjacent structures
      • Nests (reminiscent of whorls) of relatively uniform cells with variable melanin pigment
      • Bland, oval nuclei with eosinophilic nucleoli
      • Mitoses no more than 1/10 HPF
    • Malignant melanoma:
      • Hypercellular sheets or nests of spindled or epithelioid cells
      • May have significant pleomorphism
      • Atypical mitoses (5/10 HPF)
      • Invasion of adjacent structures or necrosis may be seen
      • Prominent nucleoli
    Microscopic (histologic) images

    Contributed by Rana Al-Zaidi, M.B.B.S.


    55 year old man with nausea, vomiting and sudden loss of consciousness and 4 cm temporal lesion - primary CNS melanoma



    Contributed by Jesse Kresak, M.D.

    Melanocytoma

    Melanocytoma: S100

    Melanocytoma: MelanA

    Melanocytoma: HMB45

    Cytology images

    Images hosted on other servers:

    Melanoma cells in CSF

    Positive stains
    Negative stains
    Electron microscopy description
    • Melanosomes present, no junctions, no desmosomes
    Differential diagnosis
    Additional references

    Meningeal cyst
    Definition / general
    • Also called diverticulum
    • Usually spinal
    Radiology images

    Images hosted on other servers:

    Various images (figures 1A - 4C)

    Case reports
    Microscopic (histologic) description
    • Lined by fibrous tissue resembling dura, with or without arachnoid inner membrane

    Meningioangiomatosis
    Definition / general
    • Rare, benign, hamartomatous lesion of children and young adults characterized by leptomeningeal and meningovascular proliferation
    • Presents with seizures and headaches (or asymptomatic)
    • Mean age 21 - 28 years, range 1 - 70 years
    • 25% associated with neurofibromatosis II
    • May be associated with other developmental anomalies (Clin Neuropathol 2013;32:37, Pediatr Dev Pathol 2014;17:292)
    Radiology description
    Case reports
    Treatment
    Gross description
    • Thick and opaque leptomeninges with abnormal vessels but no evidence of neoplasia
    Microscopic (histologic) description
    • Often with plaque or rind-like meningothelial proliferation at cortical surface with tongue-like projections into underlying gray matter, predominantly in a perivascular pattern
    • Also increased cortical vascularity
    • Often leptomeningeal calcification
    • Variable psammoma bodies and osteoid
    • May have entrapped reactive glial tissue or neurons
    • Also variable neurofibrillary tangles
    • No / rare mitotic activity, no necrosis, no marked pleomorphism
    Cytology description
    • Numerous thin walled capillaries, bland spindle cells, occasional large cells with prominent nucleoli, variable meningothelial whorls and neurons (Acta Cytol 2001;45:1069)
    Positive stains
    Negative stains
    Differential diagnosis
    • Atypical meningioma: increased mitotic activity and either increased cellularity, high N/C, prominent nucleoli, sheet-like growth or necrosis
    • Desmoplastic infantile astrocytoma: prominent desmoplastic stroma with neuroepithelial cells
    • Invasive meningioma: infiltrative growth into brain parenchyma with destruction, no prominent vascular component
    • Schwannoma: encapsulated, Schwann cells in palisading or myxoid patterns, strong S100+, EMA-
    • Vascular malformation: Sturge-Weber syndrome or arteriovenous malformation, no perivascular meningothelial elements

    Meningioma
    Definition / general
    • Most common primary CNS tumor, arising from arachnoid cap cells associated with dura mater or choroid plexus, accounting for 36% all CNS tumors (Neuro Oncol 2015;17:iv1)
    • Growing along external surface of brain, spinal cord or rarely, within the ventricular system
    Essential features
    • 3 grades exist based on WHO criteria:
      • Most are slow growing WHO grade 1 (benign)
      • 20 - 25% are WHO grade 2 (increased likelihood of recurrence)
      • 1 - 6% are WHO grade 3 (malignant with metastatic potential)
    • While there are 15 WHO recognized morphological meningioma variants, this topic focuses more on WHO grade 1 variants
      • WHO grade 1 meningiomas have low mitotic index (< 4/10 high power field), no brain invasion and < 3 atypical features (necrosis, small cell change, sheeted architecture, macronuclei and hypercellularity)
    ICD coding
    • ICD-10:
      • D32.9 - benign neoplasm of meninges, unspecified
      • D32.0 - benign neoplasm of cerebral meninges
      • D32.1 - benign neoplasm of spinal meninges
      • C70 - malignant neoplasm of meninges
      • C70.0 - malignant neoplasm of cerebral meninges
      • C70.1 - malignant neoplasm of spinal meninges
      • C70.9 - malignant neoplasm of meninges, unspecified
      • D42 - neoplasm of uncertain behavior of meninges
      • D42.0 - neoplasm of uncertain behavior of cerebral meninges
      • D42.1 - neoplasm of uncertain behavior of spinal meninges
      • D42.9 - neoplasm of uncertain behavior of meninges, unspecified
    Epidemiology
    • Mainly adults, mean age of 65
    • F > M
      • 66% of cerebral meningiomas occur in women; 90% of spinal cord meningiomas occur in women (Neuro Oncol 2012;14:v1)
    • Rare cases may occur in pediatric population, which are typically associated with genetic syndromes or childhood radiation (Acta Neuropathol 2021;142:873)
    Sites
    • Extra-axial mass growing along external surface of brain, spinal cord, rarely within the ventricular system or outside of the CNS (ectopic)
      • Common CNS sites:
        • Convexity
        • Skull base
        • Falx and tentorium
        • Spinal cord
    • Ectopic:
    Pathophysiology
    • Meningiomas driven by chromosome 22q alterations (e.g. NF2, SMARCB1) arise in neural crest cell derived meninges, including convexities, falx, tentorium and spinal cord
    • Meningiomas driven by hedgehog signaling pathway, PI3K signaling, TRAF7, KLF4 and POLR2A arise in the mesodermal derived meninges of the midline and paramedian anterior, central and ventral posterior skull base (Oncogene 2021;40:875)
    Etiology
    Clinical features
    Grading
    • Grade 1 variants
      • Angiomatous:
        • Vascular component should exceed 50% of total tumor area
        • Can further divide into microvascular and macrovascular variants
        • Mean Ki67 index is ~2%
        • Does not recur if entirely resected
        • Differential diagnosis includes hemangioblastoma for the microvascular variant and vascular malformations for the macrovascular variant (Am J Surg Pathol 2004;28:390)
        • Genetically characterized by polysomy of chromosome 5, 13 and 20 (Oncotarget 2014;5:10596)
      • Fibroblastic:
        • Spindle cell neoplasm with few or no meningothelial nests or whorls; often has thick bundles of collagen
        • Resembles schwannoma or solitary fibrous tumor but is positive for meningothelial markers SSTR2A and EMA and negative for STAT6
        • NF2 mutated meningiomas are predominantly fibroblastic or transitional and more commonly located in convexity, falx or tentorium (Nat Genet 2013;45:285, Oncogene 2021;40:875)
      • Lymphoplasmacyte rich:
      • Meningothelial:
        • Most common variant
        • Lobulated architecture, often contains meningothelial whorls
        • Syncytial cells with indistinct cell membranes, eosinophilic cytoplasm
        • Round uniform nuclei, intranuclear pseudoinclusions common
        • May have sparse psammoma bodies
        • Meningiomas with SMO or AKT1 mutations are predominantly meningothelial and more commonly located in the midline or paramedian skull base (Nat Genet 2013;45:285, Oncogene 2021;40:875)
      • Metaplastic:
      • Microcystic:
        • Cells have elongated processes and vacuolated cytoplasm that resembles microcysts
        • May have prominent nuclear pleomorphism but usually low mitotic index
        • Often rich in vasculature and overlaps with angiomatous meningioma
        • Small areas of classic meningothelial nests may be present
        • Microcystic areas are weakly but diffusely positive for hypoxic marker carbonic anhydrase IX (J Neuropathol Exp Neurol 2019;78:1081)
      • Psammomatous:
        • Found in spinal region
        • Numerous psammoma bodies, intervening meningothelial cells hard to find
      • Secretory:
        • Eosinophilic round secretions (pseudopsammoma bodies) positive for CEA and PAS
        • Genetically characterized by combined KLF4 and TRAF7 mutations (Acta Neuropathol 2013;125:351)
      • Transitional:
        • Mixed meningothelial and fibroblastic features
        • Usually prominent whorls, psammoma bodies and clusters of syncytial cells
        • NF2 mutated meningiomas are predominantly fibroblastic or transitional (Nat Genet 2013;45:285)
    • Grade 2 variants
    • Grade 3 variants
    • Upcoming 2021 WHO CNS tumor classification emphasizes that the criteria defining atypical or anaplastic (i.e. grade 2 and 3) meningioma should be applied regardless of the underlying subtype (Neuro Oncol 2021;23:1231)
    Diagnosis
    • Diagnose by imaging and pathology of biopsy / resection specimen
    Radiology description
    • Extra-axial mass with dural tail
    • Uniformly contrast enhancing
    • Extensive peritumoral edema is usually associated with brain invasion (Neuro Oncol 2021;23:324)
      • With the caveat that several special grade I variants (angiomatous, microcystic, secretory and lymphoplasmacyte rich) can have prominent peritumoral edema (J Neurooncol 2013;111:49)
    Radiology images

    Contributed by Chunyu Cai, M.D., Ph.D.

    MRI benign meningioma

    MRI atypical meningioma

    MRI en plaque meningioma

    MRI secretory meningioma

    Prognostic factors
    Case reports
    Treatment
    • Observation, if asymptomatic
    • Gross total resection is usually curative
    • Postoperative radiation if incompletely excised or WHO grade 2 or 3 (Neurosurg Focus 2015;38:E3)
    Gross description
    Gross images

    Images hosted on other servers:
    Missing Image

    Tumor displaces brain without invading

    Compression of underlying cortex

    Resected tumor

    Adherent to falx cerebri

    Microscopic (histologic) description
    • Arachnoid plane exists between meningioma and CNS parenchyma
    • Histology varies by variant (see Grading)
    Microscopic (histologic) images

    Contributed by Chunyu Cai, M.D., Ph.D.

    Meningothelial meningioma:

    Meningothelial meningioma

    Meningothelial meningioma syncycial

    Meningothelial meningioma SSTR2


    Fibrous meningioma:

    Fibrous meningioma

    Fibrous meningioma SSTR2a



    Transitional meningioma:

    Transitional meningioma


    Angiomatous meningioma:

    Angiomatous meningioma macrovascular

    Angiomatous meningioma microvascular


    Microcystic meningioma:

    Microcystic meningioma vacuolated cytoplasm

    Microcystic meningioma nuclear pleomorphism



    Clear cell meningioma:

    Clear cell meningioma

    Clear cell meningioma SMARCE1


    Lymphoplasmacyte rich meningioma:

    Lymphoplasmacyte
    rich meningioma

    Lymphoplasmacyte
    rich meningioma
    CD3

    Lymphoplasmacyte
    rich meningioma
    SSTR2


    Secretory meningioma:

    Secretory meningioma

    Secretory meningioma pCEA

    Secretory meningioma EMA


    Other meningioma:

    Metaplastic meningioma

    Chordoid meningioma

    Psammomatous meningioma

    Virtual slides

    Contributed by Andrey Bychkov, M.D., Ph.D.

    Meningioma

    Cytology description
    Cytology images

    Contributed by Chunyu Cai, M.D., Ph.D.

    Streaked cytoplasm

    Psammoma body

    Cellular whorls



    Images hosted on other servers:

    Meningothelial tumor

    Positive stains
    Negative stains
    Molecular / cytogenetics description
    • Fibrous, transitional and psammomatous morphologies are associated with NF2 mutation, while meningothelial, secretory and microcystic variants are non-NF2 (Oncogene 2021;40:875)
    • Majority of atypical meningiomas have loss of NF2 combined with either genome instability (large scale chromosomal alterations) or loss of SMARCB1 (Nat Commun 2017;8:14433)
    • Non-NF2 meningiomas are enriched in mutations in TRAF2, KLF4, AKT1 and SMO, most of which are benign and preferentially located in the skull base (Science 2013;339:1077)
    • Clear cell meningioma has been linked to SMARCE1 mutation (J Pathol 2014;234:436)
    • Nearly 100% of secretory meningioma contain TRAF7 / KLF4 comutations, mutually exclusive to NF2 (Science 2013;339:1077)
    • Angiomatous meningiomas contain multiple chromosome alterations, particularly gains of 5 and 20 (100% and 89%); blood vessels are nonneoplastic in origin (Oncotarget 2014;5:10596)
    • DNA methylation profiling of meningioma distinguished 6 methylation classes (MCs) in adults, benign (ben) 1 - 3, intermediate (int) A and B and malignant (mal)
      • DNA methylation based meningioma classification is reported to better predict tumor recurrence and prognosis than the WHO histological classification (Lancet Oncol 2017;18:682)
    • Pediatric meningiomas are mostly associated with genetic syndromes or childhood radiation
      • Global DNA methylation profile grouped separately from adult meningiomas and forms 3 groups: group 1 (clear cell and papillary meningiomas), 2A (enriched in NF2 driven high grade meningiomas) and 2B (enriched for rhabdoid, chordoid and other non-NF2 driven high grade meningiomas)
    Videos

    Molecular classification of meningioma

    Sample pathology report
    • Brain, inferior skull base mass (resection):
      • Meningioma, transitional (WHO grade 1) (see comment)
      • Comment: The tumor is a transitional meningioma. There is no evidence of necrosis, brain invasion, sheet-like growth or increased cellularity. There are no prominent nucleoli or small cells with a high nuclear to cytoplasmic ratio. Mitotic figures are not identified (0/10 high power fields). No brain is present for evaluation of brain invasion.
    Differential diagnosis
    Board review style question #1

    A 57 year old man presented with an intracranial convexity based tumor with histology shown in the above image. What is the best set of immunostains to determine the diagnosis?

    1. p53 and EMA
    2. S100 and CD34
    3. SMA and SOX10
    4. SSTR2a and STAT6
    Board review style answer #1
    D. SSTR2a and STAT6. The image shows a low grade spindle cell neoplasm. Within the CNS, the main differential considerations are fibrous meningioma, solitary fibrous tumor / hemangiopericytoma and schwannoma. Schwannomas are more common in the cerebellopontine angle and spinal cord but not convexity, so the main differential diagnosis in this case is meningioma versus solitary fibrous tumor. Among the choices, SSTR2a is the most sensitive and specific marker for meningioma and STAT6 is the most sensitive and specific marker for solitary fibrous tumor.

    Comment Here

    Reference: Meningioma
    Board review style question #2
    Which of the following typically low grade meningioma variants can have prominent peritumoral edema?

    1. Angiomatous, microcystic, meningothelial
    2. Psammomatous, secretory, fibrous
    3. Secretory, angiomatous, metaplastic
    4. Secretory, microcystic, lymphoplasmacyte rich
    5. Transitional, microcystic, meningothelial
    Board review style answer #2
    D. Secretory, microcystic, lymphoplasmacyte rich. The low grade meningioma variants that may have prominent peritumoral edema are secretory, angiomatous, microcystic and lymphoplasmacyte rich.

    Comment Here

    Reference: Meningioma

    Metastases
    Definition / general
    • Most common brain tumors in community hospitals are metastases (50% of intracranial tumors in hospitalized patients)
      • Primary is usually known: melanoma or carcinoma (lung, breast, kidney and GU), occasionally germ cell tumor
      • Common with choriocarcinoma
      • Common presentation is adult with seizures or ataxia
      • Dural metastases are often from breast and prostate, but also unusual primaries; may have survival of 2+ years (Arch Pathol Lab Med 2001;125:880)
    • Metastases to vertebral column: often prostate, breast or hematopoietic neoplasm
    • Metastases with unknown primary: lung, colon, kidney
    • Multiple lesions are suggestive of metastasis vs. primary CNS tumor
    • Usually to cerebrum, but no distinct patterns

    Meningeal carcinomatosis:
    • Represents 4-8% of metastatic brain tumors
    • Diffuse spread of tumor in subarachnoid space
    • Associated with carcinoma of lung and breast and ALL
    • Poor prognosis
    • Repeat lumbar punctures and immunocytochemistry may be helpful in differentiating from aseptic meningitis (Arch Pathol Lab Med 2000;124:759)
    Prognostic factors
    • Favorable prognostic factors: single metastasis, younger age, surgical resection of metastasis, primary in lung (non-small cell), breast, melanoma, renal cell or ovary
    Case reports
    Meningeal carcinomatosis:
    Gross description
    • Sharply demarcated lesion at gray-white matter junction, surrounded by edema
    Gross images

    Images hosted on other servers:

    Lung metastasis

    Microscopic (histologic) description
    • Epithelial cells with discrete cell boundaries, pushing margin except for small cell carcinoma (infiltrative)
    Microscopic (histologic) images

    Case #222

    Alveolar soft part sarcoma (H&E)


    MyoD1

    PASD

    TFE3

    Vimentin



    Contributed by Ankur Sangoi, M.D. (Case #425)

    Metastatic lung adenocarcinoma, ALK rearranged



    Metastatic lung adenocarcinoma, ALK rearranged



    Napsin A

    TTF1

    ALK break-apart FISH images (ALK gene in RED)

    Cytology images

    Images hosted on other servers:

    Metastatic GI carcinoma (signet ring cells)

    Immunostains
    Recommended stain panels

    • General: TTF1 (lung and thyroid, Hum Pathol 2002;33:642), cytokeratin and CAM 5.2, CK7, CK20 and PSA
    • Breast carcinoma: CK7+, CAM5.2+, CK20-, TTF1-
    • Lung adenocarcinoma: CK7,+ CAM5.2+, TTF1+, CK20-
    • Colon carcinoma: CK20+, CK7-, TTF1-
    • Renal cell carcinoma: RCC+, CAM5.2+, vimentin+
    Differential diagnosis

    Miscellaneous rare CNS lymphomas
    Immunodeficiency associated CNS lymphomas
    [Pending]
    Other low grade B cell lymphomas of the CNS
    [Pending]
    T cell and NK / T cell lymphoma
    [Pending]

    Multinodular and vacuolating neuronal tumor (MVNT) (pending)
    [Pending]

    Myxopapillary ependymoma
    Definition / general
    • CNS WHO grade 2 glial neoplasm with a variably papillary architecture characterized by a radial arrangement of spindled or epithelioid to cuboidal cells around blood vessels with perivascular myxoid change (intermediate layer of Alcian blue positive myxoid stroma) and mucin rich microcyst formation (Neuro Oncol 2021;23:1231)
    Essential features
    • Glioma with papillary architecture and perivascular myxoid change or at least focal myxoid microcysts
    • Immunoreactive for glial fibrillary acidic protein (GFAP)
    • For unresolved tumors, DNA methylation profile is aligned with myxopapillary ependymoma
    ICD coding
    • ICD-O: 9394/1 - myxopapillary ependymoma
    • ICD-11: 2A00.0Y & XH15U1 - other specified gliomas of brain & myxopapillary ependymoma
    Epidemiology
    Sites
    Pathophysiology
    Etiology
    • Unknown
    Clinical features
    • Chronic lower backache
    • Sciatica
    • Sensory or motor deficits leading to impotence, urinary and fecal incontinence, myelopathy
    Diagnosis
    Radiology description
    • Well circumscribed, sharply demarcated, ovoid, contrast enhancing mass in the cauda equina / conus / filum terminale region
    • Hyperintense on T1 and T2 weighted MRI with intense contrast enhancement
    Radiology images

    Contributed by Ahmed Ijaz Gilani, M.B.B.S., M.D., Ph.D.
    MRI T1 with contrast

    MRI T1 with contrast

    MRI T2

    MRI T2

    Prognostic factors
    • Relatively favorable prognosis with 10 year survival rates > 90.6% (J Neurooncol 2016;126:165, Neuro Oncol 2015;17:588)
    • Often resistant to complete removal due to locally advanced growth or cerebrospinal fluid borne seeding of the thecal sac or more rostral neuroaxis; this may be seen in > 50% of pediatric cases (World Neurosurg 2020;136:e245, J Neurooncol 2016;126:165)
    • Many patients live with this disease but require repeated operations and adjuvant therapy
    • Tumors arising in the conus have a poorer prognosis than tumors in the cauda equina as the former adhere strongly to the spinal cord and are less amenable to resection
    • Radiotherapy improves progression free survival (Neuro Oncol 2015;17:588)
    • Anaplastic histology may increase risk of aggressive behavior (Brain Pathol 2019;29:75)
    Case reports
    Treatment
    • Surgical excision is the mainstay of treatment
    • Often lifted cleanly from the operative bed; others dissected piecemeal but still in toto
      • Low risk of recurrence in either of the above scenarios
    • Radiotherapy reserved for subtotally resected tumors
    • Some authors recommend radiotherapy for tumors excised completely but piecemeal (Mod Pathol 1997;10:304, Neurosurgery 1992;30:202, Cancer 1985;56:883)
    Gross description
    • Often encapsulated, soft in consistency, pink to tan-gray
    • May be grossly gelatinous or show hemorrhage and cystic changes
    Gross images

    Contributed by Moneil Patel, M.D. (Case #111)
    On telfa pad

    On telfa pad

    Frozen section description
    • Intraoperative squash / smear preparation and frozen section reveals epithelioid to spindled tumor cells arranged around microcysts and forming papillary structures with perivascular myxoid change; these features are diagnostic in the appropriate clinical setting
    Microscopic (histologic) description
    • Most common pattern is radial arrangement of cuboidal to epithelioid elongated glial tumor cells around hyalinized fibrovascular (central, often hyalinized blood vessels) cores in a papillary configuration
    • Accumulation of basophilic myxoid material around blood vessels (myxoid stroma) and in microcysts
    • Myxoid material is highlighted by PAS and Alcian blue positive staining
    • In cases composed of confluent sheets of epithelioid cells with little or no papillary structures, PAS and Alcian blue positivity is useful in reaching a correct diagnosis
    • Fascicular growth and spindle cells are common
    • Pleomorphic tumor giant cells can be seen
    • Occasionally tumor cells show distinctive eosinophilic balloons; these are PAS positive spherules that demonstrate spiculated reticulin staining (Am J Surg Pathol 1996;20:1091)
    • Uncommon examples reported as anaplastic myxopapillary ependymomas show hypercellularity and reduced mucin in association with at least 2 of the following features: ≥ 5 mitoses / 10 high power field, Ki67 labeling index ≥ 10%, microvascular proliferation, spontaneous necrosis (Brain Pathol 2019;29:75)
    Microscopic (histologic) images

    Contributed by Ahmed Ijaz Gilani, M.B.B.S., M.D., Ph.D. and Aisha Hassan Memon, M.B.B.S.
    Radial arrangement

    Radial arrangement

    Epithelioid tumor cells

    Epithelioid tumor cells

    Papillary structures

    Papillary structures

    Microcystic pattern

    Microcystic pattern

    Hyalinized fibrovascular core with myxoid matrix

    Hyalinized fibrovascular core with myxoid matrix


    Alcian blue

    Alcian blue

    GFAP

    GFAP

    Olig2

    Olig2

    EMA

    EMA

    Ki67

    Ki67

    Cytology description
    • Cytology usually recapitulates the histologic findings; myxoid matrix and cells show nuclear uniformity and process formation
    • Epithelioid to spindle cells
    • Arrangement of tumor cells around blood vessels forms papillary structures with perivascular myxoid change
    • Tumor cells are arranged around myxoid microcysts
    Cytology images

    Contributed by Ahmed Ijaz Gilani, M.B.B.S., M.D., Ph.D. and Brittany Pakalniskis, M.D. (Case #399)
    Stromal globules

    Stromal globules

    Papillary structures and rosettes

    Papillary structures and rosettes

    Stromal globules (Diff-Quik stain) Stromal globules (Diff-Quik stain)

    Stromal globules (Diff-Quik stain)

    Stromal globules (Pap stain) Stromal globules (Pap stain)

    Stromal globules (Pap stain)

    Positive stains
    Negative stains
    Electron microscopy description
    • Zipper-like intercellular junctional complexes, occasional cilia, surface and intraluminal microvilli are typical features of ependymal differentiation
    • In addition, interdigitating cell processes and microtubular aggregates bond by endoplasmic reticulum
    Molecular / cytogenetics description
    • Unique DNA methylation profile (however, histologically classic ependymomas may also cluster on DNA methylation with myxopapillary ependymomas)
    • Prognostic significance of DNA methylation profile of myxopapillary ependymomas in context of uncharacteristic histological features (little myxoid change, pseudorosetting, spindle cells or tanycytic features) is not known
    • Recurrent gain of chromosome 16 and losses of chromosome 10 have been noted (Acta Neuropathol 2020;139:305, Cancer Cell 2015;27:728, Neuro Oncol 2018;20:1616)
    Sample pathology report
    • Lower spinal mass, resection specimen:
      • Myxopapillary ependymoma, CNS WHO grade 2
    Differential diagnosis
    • Schwannoma:
      • Arises from nerve roots (myxopapillary ependymomas usually arise in the filum terminale)
      • Usually, distinctive histological appearance from ependymoma; however, a paucicellular ependymoma with fascicular architecture may simulate schwannoma
      • No Antoni A or B areas are present in ependymomas
      • There are no nuclear or cytoplasmic inclusions in ependymomas
      • Myxopapillary ependymomas have a delicate rather than densely collagenized capsule seen in schwannomas
      • Schwannomas show pericellular reticulin and extensive immunostaining for laminin and type IV collagen
      • Both tumors are S100 and GFAP positive
      • Schwannomas show more intense staining for S100 compared with GFAP; the reverse is true for myxopapillary ependymomas
    • Paraganglioma:
      • More often arises from nerve roots than ependymomas
      • Presence of zellballen pattern or ganglion cells confirms the diagnosis of paraganglioma
      • Chromogranin positive (myxopapillary ependymomas are negative)
    • Chordoma:
      • Mostly extradural
      • Mucinous matrix of myxopapillary ependymoma may simulate chordoma; however, mucin distribution in chordoma is diffuse and tends not to be confined in perivascular spaces
      • Chordomas are strongly immunoreactive for low molecular weight keratins and EMA; negative for GFAP
    • Metastatic adenocarcinoma:
      • Glandular differentiation, goblet or signet cells with intracytoplasmic mucin supports the diagnosis of metastatic adenocarcinoma
      • Metastatic carcinomas are keratin positive and GFAP negative
    • Myxoid chondrosarcoma:
      • Myxoid or chondroid matrix with lacunar-like spaces supports the diagnosis of myxoid chondrosarcoma
      • Myxoid chondrosarcoma cells would be negative for GFAP
    Board review style question #1
    A 22 year old man was diagnosed with a myxopapillary ependymoma in the filum terminale of the spinal cord. The diagnosis was confirmed by histological examination and immunohistochemical staining. What is the most important IHC stain expressed by this tumor that distinguishes it from the other lesions included in the differential diagnosis?

    1. CD99
    2. GFAP
    3. Keratin
    4. Olig2
    5. S100
    Board review style answer #1
    B. GFAP. Diffuse immunoreactivity for GFAP distinguishes myxopapillary ependymoma from metastatic carcinoma, paraganglioma, schwannoma, chordoma and myxoid chondrosarcoma (Appl Immunohistochem Mol Morphol 2013;21:485).

    Comment Here

    Reference: Myxopapillary ependymoma
    Board review style question #2

    In the 5th edition of the 2021 WHO classification for CNS, which histological grade does myxopapillary ependymoma correspond to?

    1. Grade 1
    2. Grade 2
    3. Grade 3
    4. Grade 4
    5. Unassigned
    Board review style answer #2
    B. Grade 2. The Consortium to Inform Molecular and Practical Approaches to CNS Tumor Taxonomy update 6 recommends that spinal myxopapillary ependymomas be diagnosed based on histological criteria but notes that these tumors have a clinical outcome similar to classic ependymoma and should be assigned WHO grade 2 rather than grade 1 assigned in 2016 WHO CNS (Brain Pathol 2020;30:844).

    Comment Here

    Reference: Myxopapillary ependymoma

    Oligodendroglioma, IDH mutant and 1p / 19q codeleted
    Definition / general
    • CNS WHO 2021 definition: diffusely infiltrating glioma with IDH1 or IDH2 mutation and codeletion of chromosome arms 1p and 19q (CNS WHO grade 2 or 3)
    Essential features
    • Diffusely infiltrating glial neoplasm with IDH1 or IDH2 mutation and 1p / 19q whole arm codeletion (both features are required for diagnosis)
    • Morphology resembles nonneoplastic oligodendrocytes with round monotonous nuclei and perinuclear halos
    • Chicken wire vasculature, microcalcifications and microcysts are characteristic (Neuro Oncol 2014;16:1244)
    • Astrocytic differentiation does not preclude diagnosis if molecular features are present
      • Small gemistocytes (mini gemistocytes) with rounded bellies of eosinophilic, eccentrically placed cytoplasm are occasionally seen, especially in grade 3 tumors (Acta Neuropathol 1984;64:265)
    • Presence of other atypical features (including multinucleated giant cells, sarcomatous features, neurocytic differentiation or ganglion-like cells) does not preclude a diagnosis of oligodendroglioma if the requisite molecular features are present (Acta Neuropathol 2010;120:237, J Neuropathol Exp Neurol 2002;61:947, Neuropathology 2014;34:323)
    ICD coding
    • ICD-O:
      • 9450/3 - oligodendroglioma, NOS
      • 9451/3 - oligodendroglioma, anaplastic
    Epidemiology
    • Most epidemiologic data is based on histologic, rather than molecular, classification of oligodendroglioma
    • Incidence of 0.23 cases per 100,000 population in the United States (Neuro Oncol 2019;21:v1)
      • Incidence of CNS WHO grade 3 oligodendroglioma is 0.11
    • Of all brain tumors in the United States:
      • 0.9% are oligodendroglioma WHO grade 2
      • 0.4% are oligodendroglioma WHO grade 3
    • Peak incidence in fourth and fifth decades of life (Neuro Oncol 2020;22:iv1)
    • Rare in infants and children (Am J Surg Pathol 2014;38:1058)
    • Slight male predominance (Neuro Oncol 2019;21:v1)
    Sites
    • Infiltrative neoplasm involving the white and gray matter
    • Can occur anywhere in the neuraxis; most common locations (Neuro Oncol 2020;22:iv1):
      • Frontal lobes: 59%
      • Temporal lobes: 14%
      • Parietal lobes: 10%
      • Occipital lobes: 1%
    • Rarely observed in midline structures, brainstem, cerebellum or spinal cord
    • Leptomeningeal spread occasionally observed, particularly at recurrence (Neurology 2019;92:e2483)
    Pathophysiology
    • Cell (or cells) of origin for oligodendroglioma remains unknown
    • IDH mutation is likely the initiating event (driver mutation), which precedes 1p / 19q codeletion (Adv Anat Pathol 2015;22:50, Biomed Res Int 2014;2014:540236)
    • IDH mutations give rise to metabolic alterations, with increased production of 2-hydroxyglutarate (2HG)
    • Increased 2HG inhibits histone demethylation, causing a hypermethylation phenotype in neoplastic cells: glioma CpG island methylated phenotype (G CIMP) (Nature 2012;483:479, Acta Neuropathol 2013;125:621)
    Etiology
    Clinical features
    Grading
    • WHO grade 2:
      • Well differentiated tumor lacking anaplastic features (brisk mitotic activity, microvascular proliferation, necrosis)
    • WHO grade 3:
      • Prominent anaplastic features (necrosis, microvascular proliferation or brisk mitotic activity) are compatible with anaplastic oligodendroglioma, IDH mutant and 1p / 19q codeleted, WHO grade 3
      • Strict mitotic activity criteria do not currently exist
      • Some authors suggest ≥ 6 mitotic figures per 10 high power fields in resection specimens for grade 3 designation (J Neuropathol Exp Neurol 2001;60:248)
      • Fewer mitotic figures might be sufficient for grade 3 designation in small biopsy specimens if other anaplastic features (vascular proliferation or necrosis) or significant nuclear atypia are present
      • CDKN2A homozygous deletion may serve as a molecular marker of CNS WHO grade 3 in IDH mutant and 1p / 19q codeleted oligodendrogliomas (Neuro Oncol 2019;21:1519)
    Diagnosis
    • Magnetic resonance imaging (MRI), followed by stereotactic brain biopsy or surgical resection
    • Methods to detect IDH gene mutation:
    • Methods to detect 1p / 19q codeletion:
      • Fluorescent in situ hybridization (FISH)
      • Array comparative genomic hybridization
      • Polymerase chain reaction (PCR)
    Radiology description
    Radiology images

    Contributed by Jared T. Ahrendsen, M.D., Ph.D. and John DeWitt, M.D., Ph.D.
    MRI, brain: T1 post

    MRI, brain: T1 post

    MRI, brain: T2

    MRI, brain: T2

    MRI: frontal lobe tumor with cystic change MRI: frontal lobe tumor with cystic change

    MRI: frontal lobe tumor with cystic change



    Images hosted on other servers:

    Axial flair frontal lobe tumor

    Axial flair large temporal lobe tumor

    Prognostic factors
    Case reports
    Treatment
    • Gross total resection, if possible
    • Adjuvant chemotherapy (temozolomide) and radiotherapy
      • Given to patients with symptomatic or progressive tumors, tumors with CNS WHO grade 3 histology or those with large postoperative residual tumor
    • References: Crit Rev Oncol Hematol 2008;66:262, Lancet 2005;366:985
    Gross description
    • Variably well defined, gray-pink mass
    • Mucoid change can give a gelatinous consistency
    • Areas of cystic degeneration, calcifications, hemorrhage or necrosis can be seen
    Frozen section description
    • Moderately cellular, diffusely infiltrating neoplasm
    • Glia with mild to moderate nuclear atypia
    • Round nuclei with speckled chromatin
    • Calcifications, perineuronal satellitosis or perivascular accumulation of tumor cells may be seen
    • Will not see perinuclear halos on frozen section or smear preparations
    • Anaplastic features (necrosis, vascular proliferation, mitoses) may be seen in WHO grade 3 tumors
    • Reference: J Cytol 2011;28:147
    Intraoperative frozen / smear cytology images

    Contributed by Jared T. Ahrendsen, M.D., Ph.D.
    Intraoperative smear preparation

    Intraoperative smear preparation

    Intraoperative frozen section

    Microscopic (histologic) description
    • Closely packed cells with small, round, monotonous nuclei (slightly larger than a normal oligodendrocyte)
    • Perinuclear clearing (fried egg appearance)
      • Formalin fixation artifact
      • Will not be seen on frozen sections or smear preparations
    • Network of thin walled, branching blood vessels (chicken wire vasculature)
    • Microcalcifications (calcospherites) are characteristic
    • Presence of perineural, perivascular or subpial aggregates of tumor cells (secondary structures of Scherer)
    • Occasional mitoses and moderate nuclear atypia are still consistent with grade 2 designation (J Neuropathol Exp Neurol 2001;60:248)
    • Not uncommon to find well differentiated / fibrillary astrocytic morphology (Acta Neuropathol 1984;64:265)
    • Features of CNS WHO grade 3 oligodendroglioma:
      • Presence of microvascular proliferation
      • Presence of necrosis
      • Presence of brisk mitotic activity
        • Strict mitotic figure cutoffs do not currently exist; some authors suggest ≥ 6 mitoses per 10 high power fields for WHO grade 3 designation in tumors without necrosis or vascular proliferation (Neuro Oncol 2014;16:1244, Neuro Oncol 2016;18:888)
    Microscopic (histologic) images

    Contributed by Jared T. Ahrendsen, M.D., Ph.D. and John DeWitt, M.D., Ph.D.
    Diffusely infiltrating glial neoplasm

    Diffusely infiltrating glial neoplasm

    Microcalcifications

    Microcalcifications

    Perinuclear halos Perinuclear halos

    Perinuclear halos

    Chicken wire vasculature

    Chicken wire vasculature


    Secondary structures of Scherer

    Secondary structures of Scherer

    Diffusely infiltrating glial neoplasm

    Cellularity and vascular proliferation

    Diffusely infiltrating glial neoplasm

    Focal necrosis

    Diffusely infiltrating glial neoplasm

    Brisk mitotic activity

    Mixed astrocytic histology


    IDH1 R132H

    IDH1 R132H

    p53

    ATRX

    Virtual slides

    Images hosted on other servers:

    Oligodendroglioma,
    resection

    Oligodendroglioma,
    IDH1 R132H,
    resection

    Positive stains
    Negative stains
    Electron microscopy description
    • Not routinely used for diagnostic purposes
    Molecular / cytogenetics description
    Sample pathology report
    • Brain, frontal lobe, left, tumor, resection:
      • Integrated diagnosis: oligodendroglioma, IDH1 R132H mutant and 1p / 19q codeleted
      • Histologic diagnosis: oligodendroglioma
      • CNS WHO grade: 3
      • Molecular information:
        • IDH1 R132H mutation
        • 1p / 19q codeletion
        • TERT promotor mutation
    Differential diagnosis
    • Astrocytoma, IDH mutant:
      • Lacks 1p / 19q codeletion and TERT promotor mutations
      • Harbors p53 or ATRX mutations
        • p53 expression strong and diffuse in tumor cells
        • ATRX expression lost in tumor cells
    • Other tumors with oligo-like morphology:
    • Macrophage rich lesions:
      • Stain positive with macrophage markers
    Board review style question #1

    A 42 year old man presents to the emergency room with new onset seizures. Brain magnetic resonance imaging (MRI) reveals a nonenhancing infiltrative mass lesion in the right frontal lobe. A biopsy is performed, shown in the image above. What molecular features are most likely present?

    1. BRAF V600E mutation
    2. EGFR amplification
    3. IDH mutation and 1p / 19q codeletion
    4. IDH, p53 and ATRX mutations
    5. Polysomy 7 and monosomy 10 (+7 / -10)
    Board review style answer #1
    C. IDH mutation and 1p / 19q codeletion. The image shows an oligodendroglioma, which is defined by the presence of IDH mutation and 1p / 19q codeletion.

    Comment Here

    Reference: Oligodendroglioma, IDH mutant and 1p / 19q codeleted
    Board review style question #2
    Which of the following is a common genetic alteration in oligodendroglioma, IDH mutant and 1p / 19q codeleted?

    1. ATRX mutation
    2. BRAF V600E
    3. EGFR amplification
    4. p53 mutation
    5. TERT promotor mutation
    Board review style answer #2
    E. TERT promotor mutation. TERT promotor mutations are commonly observed in oligodendroglioma.

    Comment Here

    Reference: Oligodendroglioma, IDH mutant and 1p / 19q codeleted

    Papillary craniopharyngioma
    Definition / general
    • Suprasellar (usually) epithelial encapsulated neoplasm with well differentiated nonkeratinizing squamous epithelium and papillary fibrovascular stroma
    • BRAF V600E mutation in almost all cases
    Essential features
    • WHO grade 1
    • Encapsulated tumor with well differentiated nonkeratinizing squamous epithelium and papillary fibrovascular stroma
    Terminology
    • Suprasellar papillary squamous epithelioma
    • Ciliated craniopharyngioma
    • Ciliated and goblet cell craniopharyngioma
    Epidemiology
    Sites
    • Suprasellar or intraventricular (third ventricle)
    Pathophysiology
    Clinical features
    • Visual disturbances (Orphanet J Rare Dis 2007;2:18)
    • Obstructive hydrocephalus
    • Mental / personality changes
    • Hyperprolactinemia (pituitary stalk effect)
    • Diencephalic syndrome (rare)
    Diagnosis
    • Lesions usually solid (J Neurosurg 1995;83:206)
    • If cystic, may have mural nodule
    • No calcifications on CT scan
    • MRI: contrast enhancing solid or cystic mass
    Laboratory
    • Full pituitary endocrine workup is usually mandatory
    • Visual acuity and visual field assessment is also performed to show any deficits and rule out papilledema
    Radiology description
    • More spherical in outline and usually lacks the prominent cystic component (Childs Nerv Syst 2019;35:169, J Neurosurg 1995;83:206)
    • Solid or contains a few smaller cysts
    • May have cyst and mural nodule configuration
    • Tends to displace adjacent structures
    • MRI:
      • T1 weighted images: 85% of cysts are hypointense
      • T1 weighted images: solid component iso to hypointense
      • Vividly contrast enhancing
    • CT:
      • Cysts small and insignificant
      • Near cerebrospinal fluid (CSF) density
      • Solid component near soft tissue density
      • Vivid enhancement
      • Calcifications very rare
    Radiology images

    Contributed by Chunyu "Hunter" Cai, M.D., Ph.D.
    T1 precontrast sagittal

    T1 precontrast sagittal

    T1 precontrast coronal

    T1 precontrast coronal



    Images hosted on other servers:

    Suprasellar mass

    Solid suprasellar mass with small cystic component

    Preoperative MRI

    Prognostic factors
    Case reports
    Treatment
    Gross description
    Microscopic (histologic) description
    Microscopic (histologic) images

    Contributed by Nelli S. Lakis, M.D., M.Sc. and Chunyu "Hunter" Cai, M.D., Ph.D.
    Papillary architecture

    Papillary architecture

    Fibrovascular cores

    Fibrovascular cores

    Stroma

    Stroma

    H&E

    Lymphocytic inflammation

    BRAF V600E immunostain

    BRAF V600E immunostain

    Virtual slides

    Images hosted on other servers:

    54 year old man with papillary craniopharyngioma

    Cytology description
    • Sheets of epithelial cells
    • Individual nucleated squamous cells
    Positive stains
    Negative stains
    Molecular / cytogenetics description
    • BRAF V600E mutation in 95% of cases
    Sample pathology report
    • Sellar / suprasellar region, suprasellar mass, endoscopic resection:
      • Papillary craniopharyngioma (see comment)
      • Comment: BRAF V600E immunostain shows positive cytoplasmic reactivity, supporting / confirming the diagnosis of papillary craniopharyngioma.
    Differential diagnosis
    • Adamantinomatous craniopharyngioma:
      • Irregular, infiltrative borders
      • Complex architecture
      • Wet keratin
      • Calcifications
      • Peripheral palisading
      • Loose stellate reticulum
      • Positive nuclear beta catenin in keratin whorls (may be focal)
    • Epidermoid cyst:
      • Uniloculate with thin layer of keratinizing squamous epithelium and keratohyalin granules
    • Rathke cleft cyst with squamous metaplasia:
      • Usually cystic without solid component, squamous epithelium with ciliated or mucus containing cells
    Board review style question #1

    A 56 year old man is being evaluated due to headaches and visual disturbances. A brain MRI shows a 5 cm solid and cystic suprasellar lesion. The lesion is resected and a histopathologic examination reveals fibrovascular cores and abundant well differentiated squamous epithelium. Which of the following is a feature of the most likely diagnosis?

    1. BRAF V600E mutation
    2. Most commonly occurs in children
    3. Nuclear palisading
    4. Nuclear staining with beta catenin
    Board review style answer #1
    A. BRAF V600E mutation

    Comment Here

    Reference: Papillary craniopharyngioma
    Board review style question #2
    A 43 year old woman is seen in the clinic due to visual disturbances. A physical exam is unremarkable except for visual field testing, which reveals a bitemporal hemianopia. An MRI of the cranium is performed and reveals the presence of a solid and cystic mass in the suprasellar region. The lesion is resected and histologic examination reveals abundant well differentiated squamous epithelium overlying fibrovascular cores, membranous staining of beta catenin and presence of the BRAF V600E mutation. Which of the following is the most likely diagnosis?

    1. Adamantinomatous craniopharyngioma
    2. Dermoid cyst
    3. Papillary craniopharyngioma
    4. Rathke cleft cyst
    Board review style answer #2
    C. Papillary craniopharyngioma

    Comment Here

    Reference: Papillary craniopharyngioma

    Papillary glioneuronal tumor
    Definition / general
    • Rare, low grade biphasic neoplasm characterized by pseudopapillae lined by glial cells with interpapillary neuronal elements
    • Favorable outcome suggests that it corresponds to WHO grade 1
    • First described by Komori et al. (Am J Surg Pathol 1998;22:1171)
    Epidemiology
    • Age: young adults
    • Sex: no gender predilection
    Sites
    • Cerebral hemisphere
    • > Temporal lobe
    Radiology description
    • Supratentorial, discrete, contrast enhancing
    • Often cystic with occasional cyst mural nodule pattern
    Radiology images

    Images hosted on other servers:

    Heterogeneously enhancing mass in pineal gland

    Case reports
    Treatment
    • Complete surgical resection
    • Favorable clinical outcome
    Microscopic (histologic) description
    • Biphasic pattern with glial and neuronal components
    • Glial component: small uniform glial cells line pseudopapillae with hyalinized vascular cores; no atypia or mitosis
    • Neuronal component: interpapillary zone is occupied by neurocytes with uniform nuclei and occasional perinuclear halos; occasionally ganglion and ganglioid cells are admixed
    • Vascular proliferation and necrosis are exceptional
    • The surrounding brain tissue shows gliosis
    Cytology description
    • Nonspecific, uniform cells
    Immunohistochemistry & special stains
    Electron microscopy description
    • Three cell types: astrocytic, neuronal, glioneuronal progenitor cells
    Differential diagnosis

    Papillary glioneuronal tumor
    Definition / general
    • Low grade glioneuronal tumor with a biphasic pattern composed of pseudopapillary glial structures and interpapillary neuronal components; also with PRKCA gene fusion (mainly SLC44A1::PRKCA fusion)
    • Corresponds to WHO grade 1
    Essential features
    • Rare, low grade glioneuronal neoplasm with biphasic histological and immunophenotypic pattern characterized by pseudopapillae lined by GFAP positive glial cells with interpapillary synaptophysin positive neuronal elements (PMID: 26899971, PMID: 35512827, PMID: 26671581)
    • First described by Komori et al. (Am J Surg Pathol 1998;22:1171)
    • PGNT is characterized by PRKCA gene fusion (mainly SLC44A1::PRKCA fusion) and a distinctive methylation profile
    • Supratentorial, well delineated and often cystic tumors with predilection to young adults
    • Favorable prognosis with gross total resection
    Terminology
    • Not recommended by WHO: pseudopapillary ganglioneurocytoma, pseudopapillary neurocytoma with glial differentiation
    ICD coding
    • ICD-O: 9509/1 - papillary glioneuronal tumor
    • ICD-11: 2A00.21 & XH3XU4 - mixed neuronal glial tumors & papillary glioneuronal tumor
    Epidemiology
    • Rare (PMID: 37787321)
    • Age: predilection for young adults (median age at diagnosis is 16 years (range: 6 - 54 years) (PMID:30759284, PMID: 34038055 PMID: 28823671)
    • No gender predilection (PMID:30759284, PMID: 35512827)
    Sites
    • Supratentorial, with predilection for temporal and frontal lobes (PMID:30759284, PMID: 28823671, PMID: 35512827, PMID:26671581)
    • Propensity for the ventricular system, with > 60% of tumors in a periventricular or intraventricular location (PMID:30759284, PMID: 28823671 PMID: 26671581)
    • It can occur anywhere in the cerebrum, cerebellum, or brainstem (PMID: 28823671)
    Pathophysiology
    • PRKCA gene fusion is the hallmark of PGNT, mainly SLC44A1::PRKCA fusion; less frequently NOTCH1::PRKCA fusion (PMID:30759284, PMID:26671581)
    • A reciprocal translocation t(9;17)(q31 ;q24) results in the canonical SLC44A1::PRKCA fusion with consequent generation of a constitutively expressed oncoprotein (PMID:22725730)
    • PRKCA encodes protein kinase C alpha (PKCA) involved in MAPK signaling pathway (PMID:26671581)
    Etiology
    • Unknown
    Diagrams / tables
    NA
    Clinical features
    • Principal manifestations include headache and seizures and mass effects (PMID: 26899971, PMID: 34038055)
    • Occasionally asymptomatic (PMID: 35512827)
    • Hemorrhagic onset has been reported (PMID:16771176, PMID:22717600)
    Diagnosis
    • Neuroimaging: MRI and CT
    • Biopsy: diagnosis should be heavily weighted towards molecular findings because morphological analyses frequently result in mistyping (PMID:30759284)
    • WHO diagnostic criteria
      • WHO essential diagnostic criteria
        • Biphasic histological and immunophenotypic pattern with pseudopapillary glial lining and interpapillary neuronal components
        • PRKCA gene fusion (mostly SLC44A1::PRKCA)
        • For unresolved lesions: methylation profile of papillary glioneuronal tumor
      • WHO desirable diagnostic criteria
        • Well delineated, solid and cystic tumor
    Laboratory
    NA
    Radiology description
    • MRI
      • Supratentorial, well delineated
      • Cystic or solid, some cases are cystic with a mural nodule (PMID: 26899971, PMID: 34038055, PMID: 35512827, PMID: 22725730)
      • Solid component is hypo- to isointense on T1 weighted images and iso- to hyperintense on T2 weighted images (PMID: 35512827)
      • Contrast enhancing (PMID: 26899971)
      • No or only minimal peritumoral edema and little mass effect (PMID: 35512827)
      • No diffusion restriction (PMID: 35512827)
    Radiology images

    Contributed by Eman Abdelzaher, M.D., Ph.D.
    MRI, axial T1

    MRI, axial T1

    MRI, axial T2

    MRI, axial T2

    MRI, sagittal T1 with contrast

    MRI, sagittal T1 with contrast

    Axial T1 with contrast

    Axial T1 with contrast



    Images hosted on other servers:
    Contrast enhancement and ventricular association

    Left temporal lobe mass, MRI

    Left temporal lobe mass, MRI

    Contrast enhancement and ventricular association

    Prognostic factors
    • Outcome is usually favorable, with a reported median 5 year progression free survival > 80% (PMID: 30759284)
    • Gross total resection constitutes the main prognostic factor; male sex and low Ki67 proliferation index were also reported as positive prognostic indicators (PMID: 28823671, PMID: 35512827)
    • Recurrence was reported in one molecularly confirmed case, while it was documented in 11.8 % of morphologically diagnosed cases, with higher rates associated with incomplete resection (PMID:30759284, PMID: 24553727)
    • Increased Ki67 proliferation index (> 20%) or anaplastic features with tumoral progression or dissemination have been reported in morphologically diagnosed cases but have not been described in PRKCA fused cases (PMID:28823671, PMID:19119904, PMID: 24553727)
    Case reports
    • 14 year old boy with intraventricular papillary glioneuronal tumor (PMID: 37787321)
    • 16 year old girl with suprasellar papillary glioneuronal tumor mimicking craniopharyngioma (PMID: 32695553)
    • 38 year old man with papillary glioneuronal tumor growing slowly for 26 years (PMID: 35854961)
    • 48 year old woman with papillary glioneuronal tumor masquerading as malignant brain tumor (PMID: 37621982)
    • 64 year old man with papillary glioneuronal tumor with an excessive angiomatous component (PMID: 29336376)
    Treatment
    • Gross total resection is optimal (PMID: 34038055)
    • Radiation therapy and chemotherapy are used rarely, on a case by case basis (PMID: 28823671)
    Clinical images

    Images hosted on other servers:
    Right occipital lobe tumor

    Right occipital lobe tumor

    Left frontal lobe tumor

    Left frontal lobe tumor

    Gross description
    • Well circumscribed, solid or cystic lesions
    • Calcification and hemorrhage may be observed
    Gross images
    NA
    Frozen section description
    NA
    Frozen section images
    NA
    Microscopic (histologic) description
    • Characterized by a biphasic pattern created by a glial pseudopapillary component and an interpapillary neuronal component with variable proportions of both components (PMID: 26899971, PMID:9777979)
    • Glial component is composed of a single or pseudostratified layer of small cuboidal glial cells with round nuclei and scant cytoplasm, lining pseudopapillary structures (present in 93% of molecularly confirmed PGNTs) with hyalinized vascular cores (PMID:30759284)
    • Neuronal component is composed of uniform neurocytes or medium sized neurons disposed in a neuropil background (PMID:9777979)
    • Ganglion cells (21.4%), microcalcifications (35.7%) may be seen (PMID:30759284)
    • Rarely, eosinophilic granular bodies may be encountered (PMID:9777979, PMID:26671581)
    • Occasional mitoses may be seen (PMID: 30759284)
    • Microvascular proliferation and necrosis are absent (PMID:30759284, PMID:9777979)
    • Anaplastic features were rarely reported in non-molecularly confirmed cases (PMID: 19119904, PMID:19157003)
    • Peritumoral reactive gliosis
    Virtual slides
    NA
    Cytology description
    • Nonspecific
    • May exhibit vague papillary structures with hyalinized cores lined by bland tumor cells (PMID: 37787321, PMID: 22020040)
    • Loosely cohesive clusters and singly scattered tumor cells with mildly pleomorphic round to oval nuclei with stippled chromatin, small nucleoli and scant to moderate cytoplasm (PMID: 33991783, PMID: 37787321, PMID: 22020040)
    Immunofluorescence description
    NA
    Immunofluorescence images
    NA
    Positive stains
    • Biphenotypic immunophenotype: glial in pseudopapillary structures and neuronal in interpapillary areas
    • GFAP: positive in glial cells ensheathing pseudopapillae and in scattered cells in the interpapillary areas (PMID: 26899971, PMID: 22725730)
    • S100: positive in glial cells (PMID: 35512827, PMID: 22725730)
    • Olig2: positive in glial cells; in some cases, oligodendrocyte-like cells (Olig2+ and GFAP-) may be seen (PMID:15906048)
    • Synaptophysin: positive in neuronal component (PMID: 26899971)
    • a href="https://www.pathologyoutlines.com/topic/stainsneun.html">NeuN: positive in neuronal component (PMID: 26899971)
    • NFP / neurofilament: mostly confined to ganglioid / ganglion cells
    • Ki67: generally low (does not exceed 2%) but elevated proliferative activity (ranging from 10% to > 50%) has been reported in non-molecularly confirmed cases (PMID:24709681, PMID:19157003 PMID: 34038055)
    Negative stains
    • Chromogranin A: variable (PMID:9777979, PMID:26671581)
    • CD34: may be focally positive (PMID:26671581)
    • p53 (PMID:26671581)
    • H3-K27 (PMID:26671581)
    • BRAF V600E (PMID:26671581)
    • EMA (PMID: 32695553)
    • IDH1 R132H (PMID: 32695553, https://doi.org/10.3171/CASE21266)
    Electron microscopy description
    • Three cell types: astrocytic, neuronal, glioneuronal progenitor cells
    Differential diagnosis

    Papillary meningioma
    Definition / general
    • WHO grade 3
    • Aggressive, high rate of recurrence, may metastasize
    • Often occurs in younger patients
    • May be difficult to recognize as meningothelial
    • Helps to refer to imaging or surgeon to verify a dural based lesion
    Sites
    • Most often supratentorial
    • However, can occur in posterior fossa and spinal cord
    Radiology description
    • MRI may show irregular tumor borders, heterogenous tumoral enhancement and peritumoral edema suggesting a high grade meningioma
    Radiology images

    Images hosted on other servers:

    Right frontal papillary meningioma

    Large, lobulated, irregular mass

    Mass with a broad base

    Postoperative shows severe leptomeningeal seeding

    Avidly enhancing meningioma

    Preoperative sagittal and axial adolinium enhanced MR images

    Prognostic factors
    • Complete surgical resection is the most favorable prognostic indicator (Neurosurgery 2013;73:777)
    • Low MIB1 index and retention of progesterone receptor (PR) have also been reported as markers of improved prognosis
    Case reports
    Treatment
    Gross description
    • Dural based
    • May be well circumscribed or readily adherent to brain parenchyma
    • Size can vary widely
    Microscopic (histologic) description
    • Papillary structures are best appreciated at edges of tumor
    • Papillary features may be focal but at least 50% of tumor should have papillary features to call papillary meningioma, WHO grade 3
    • Cross sections through papillae may give an ependymoma-like histology with perivascular pseudorosettes
    • May have areas of classic meningioma or may be difficult to appreciate any meningothelial features (whorls, psammoma bodies, nuclear pseudoinclusions)
    • Often has features of atypical meningioma: hypercellularity, brain invasion, increased mitoses, prominent nucleoli
    Cytology description
    • Not distinct from a classic meningioma smear
    • However, may appreciate papillary clusters from low power
    Cytology images

    Images hosted on other servers:

    Meningioma smear

    Positive stains
    • EMA (variable), NSE
    • PR (in a subset; helpful if positive, not ruled out if negative), vimentin
    Negative stains
    Electron microscopy description
    Electron microscopy images

    Images hosted on other servers:

    Bundles of intermediate filaments


    Papillary tumor of pineal region
    Definition / general
    • First described in 2003 (Am J Surg Pathol 2003;27:505)
    • 4 of 6 were women, ages 19 - 53 years
    • May derive from ependymal cells of subcommissural organ (also chordoid glioma)
    Radiology description
    • Well circumscribed pineal mass
    Microscopic (histologic) description
    • Epithelial-like growth with vessels covered by a layer of tumor cells
    • Cells are large, columnar / cuboidal with clear cytoplasm, round or infolded nuclei at basal pole of tumor cells
    • May have rosettes
    Positive stains
    Negative stains
    Electron microscopy description
    • Abundant rough endoplasmic reticulum with distended cisternae filled with secretory product
    • Also microvilli and perinuclear intermediate filaments
    Differential diagnosis

    Pilocytic astrocytoma
    Definition / general
    • Circumscribed, well differentiated astrocytic neoplasm with piloid (hair-like) processes, most commonly occurring in children and young adults, WHO grade 1 (Acta Neuropathol 2016;131:803)
    Essential features
    Terminology
    • Pilocytic astrocytoma
    • Juvenile pilocytic astrocytoma (not recommended)
    • Spongioblastoma (not recommended)
    • Pilomyxoid astrocytoma (subtype)
    • Optic nerve glioma
    ICD coding
    • ICD-O: 9421/1 - pilocytic astrocytoma
    • ICD-11: 2A00.0Y - other specified gliomas of brain
    • ICD-11: XH29Q5 - pilomyxoid astrocytoma
    Epidemiology
    Sites
    • Found throughout neuraxis
    • Most common sites: cerebellum (42%), supratentorial (36%), optic pathway / hypothalamus (9%), brainstem (9%), spinal cord (2%) (J Neurosurg 2003;98:1170)
    • Infratentorial location most common in childhood (J Neurooncol 2017;131:163)
    Pathophysiology
    • Mutation in MAPK pathway component → pathway activation → transcription factor activation → increased cell growth and proliferation
    • Biallelic inactivation of NF1 in the setting of neurofibromatosis type 1 (Genome Res 2013;23:431)
    Etiology
    • Most cases are sporadic
    • Germline mutations in MAPK pathway genes, including FGFR1, NF1, PTPN11 and RAF1
    Diagrams / tables

    Images hosted on other servers:

    MAPK pathway mutations

    Clinical features
    • Neurological deficits depending on tumor location
    • Nonlocalizing signs
      • Macrocephaly
      • Headache
      • Endocrinopathy
      • Increased intracranial pressure
    • Optic pathway tumors in 15% of patients with neurofibromatosis type 1 (Ophthalmology 1984;91:929)
    Diagnosis
    • Diagnosis may be made on histologic features alone (Acta Neuropathol 2015;129:775)
    • When microscopic findings are limited / ambiguous, molecular testing may be necessary to assess for gene fusions and other alterations
    Radiology description
    • Well demarcated enhancing mass (Insights Imaging 2014;5:387)
    • Cystic mass with enhancing mural nodule (~66% of cases), solid or heterogeneous mixed solid - cystic
    • Fusiform mass in optic pathway
    Radiology images

    Contributed by P.J. Cimino, M.D., Ph.D.
    T1 postcontrast MRI

    T1 postcontrast MRI

    Prognostic factors
    Case reports
    Treatment
    Clinical images

    Images hosted on other servers:

    Intraoperative surgical image

    Exophthalmos - optic nerve glioma

    Orbital pilocytic astrocytoma

    Gross description
    • Usually well circumscribed (Acta Neuropathol 2015;129:775)
    • Commonly cystic
    • May have myxoid or mucoid appearance
    • May contain calcifications or hemosiderin staining
    Gross images

    Images hosted on other servers:

    Intraocular pilocytic astrocytoma

    Cerebellar tumor

    Intraoperative frozen / smear cytology images

    Contributed by P.J. Cimino, M.D., Ph.D.
    Atypical piloid glial cells

    Atypical piloid glial cells

    Smear (squash) preparation

    Smear (squash) preparation

    Microscopic (histologic) description
    • Growth pattern
      • Predominantly solid / circumscribed; often limited peripheral infiltration
      • Frequent extension into subarachnoid space
    • Biphasic appearance
      • Compact fibrillar portions: elongated nuclei, bipolar piloid processes, Rosenthal fibers
      • Loose microcystic portions: round to oval nuclei, cobweb-like processes, eosinophilic granular bodies
    • Occasional "pennies on a plate" multinucleated cells
    • Oligodendroglioma-like areas may be present
    • Glomeruloid vessels
    • Regressive / degenerative changes
      • Degenerative atypia
      • Vascular hyalinization
      • Infarct-like necrosis
      • Calcification
      • Perivascular lymphocytes
    • Anaplasia in a minority of cases
    • Pilomyxoid astrocytoma subtype
      • Variant with angiocentric arrangement of monophasic bipolar tumor cells in a myxoid background
    Microscopic (histologic) images

    Contributed by P.J. Cimino, M.D., Ph.D.
    Biphasic appearance

    Biphasic appearance

    Compact fibrillary area

    Compact fibrillary area

    Cystic area

    Cystic area

    Rosenthal fibers

    Rosenthal fibers

    Eosinophilic granular bodies

    Eosinophilic granular bodies

    Microvascular proliferation

    Microvascular proliferation


    Multinucleated cells

    Multinucleated cells

    Hyalinized vessels

    Hyalinized vessels

    Extensive regressive features

    Extensive regressive features

    Oligodendroglial-like cytomorphology

    Oligodendroglial-like
    cytomorphology

    Mitosis

    Mitosis

    GFAP piloid processes

    GFAP piloid processes


    Rosenthal fiber - GFAP

    Rosenthal fiber - GFAP

    ATRX retained nuclear immunoreactivity

    ATRX retained nuclear immunoreactivity

    Increased Ki67 with anaplasia

    Increased Ki67 with anaplasia

    Masson trichrome - EGBs

    Masson trichrome - EGBs

    Virtual slides

    Images hosted on other servers:
    Pilocytic astrocytoma (grade I)

    Pilocytic astrocytoma (grade I)

    Cytology description
    • Smear preparation (alcohol fixed, H&E stained) (Cancer Cytopathol 2015;123:331)
      • Mildly atypical glial cells with long piloid processes
      • Ovoid nuclei with delicate chromatin
      • Rosenthal fibers
      • Eosinophilic granular bodies
    Negative stains
    Electron microscopy description
    • Rosenthal fibers: amorphous, electron dense elements surrounded by intermediate filaments within astrocytic processes (J Pathol Transl Med 2015;49:427)
    • Eosinophilic granular bodies: round body, electron dense homogeneous material
    Electron microscopy images

    Images hosted on other servers:

    Eosinophilic granular bodies

    Molecular / cytogenetics description
    • Activating alterations in components of MAPK pathway (Nat Genet 2013;45:927, Nat Genet 2013;45:602)
      • > 70% KIAA1549-BRAF fusion due to 7q34 tandem duplication
        • ~90% cerebellar cases
        • ~50% supratentorial cases
      • ~5% other BRAF fusion
      • ~5% BRAF V600E
      • ~8% NF1 loss
      • < 5% FGFR1 mutation
      • < 5% FGFR1 fusions / internal tandem duplication
      • ~2% NTRK fusions
      • PTPN1 mutation (rare)
      • KRAS mutation (rare)
      • RAF1 fusions (rare)
    • Chromosomal polysomies (5, 6, 7, 11, 15) detected by comparative genomic hybridization (CGH) (J Neuropathol Exp Neurol 2006;65:1049, Br J Cancer 2000;82:1218)
    • Infratentorial / supratentorial tumors show distinct gene expression and DNA methylation signatures (Acta Neuropathol 2013;126:291)
    • H3 K27M mutation (rare) (Brain Pathol 2019;29:126)
    • Negative for IDH1 / IDH2 exon 4 mutations
    • Usually negative for TP53 mutations
    Molecular / cytogenetics images

    Images hosted on other servers:

    KIAA1549-BRAF fusion by FISH

    Copy number variation plot

    NTRK2 fusion by next generation sequencing

    NF1 biallelic inactivation

    Pilocytic astrocytoma methylation classifier

    Videos

    Pilocytic astrocytoma

    Rosenthal fibers in pilocytic astrocytoma

    Sample pathology report
    • Brain, cerebellum, biopsy:
      • Pilocytic astrocytoma, WHO grade 1 (see comment)
      • Comment: Positive for KIAA1549-BRAF fusion.
    Differential diagnosis
    Board review style question #1

    The above tumor is from the cerebellum of a child. What is the most common underlying genetic alteration in this entity?

    1. TP53 mutation
    2. IDH1 R132H
    3. BRAF V600E mutation
    4. BRAF fusion involving KIAA1549
    5. NF1 loss
    Board review style answer #1
    D. BRAF fusion involving KIAA1549. TP53 mutations (answer A) are uncommon in pilocytic astrocytoma. IDH1 R132H (answer B) is the most common mutation in IDH mutant astrocytomas and oligodendrogliomas but should be negative in pilocytic astrocytomas. BRAF V600E mutation (answer C) and NF1 loss (answer E) occur in pilocytic astrocytomas at lower frequencies.

    Comment Here

    Reference: Pilocytic astrocytoma
    Board review style question #2
    Which of the following is true of pilocytic astrocytomas?

    1. Show diffuse p53 immunoreactivity
    2. Most frequently occur in adults
    3. Demonstrate reticulin rich foci
    4. Associated with neurofibromatosis 2
    5. Potentially curable with resection
    Board review style answer #2
    E. Potentially curable with resection. Pilocytic astrocytomas should not show significant p53 immunopositivity (answer A) and they usually occur in children (answer B). Reticulin rich foci are a more typical feature of pleomorphic xanthoastrocytoma or gliosarcoma (answer C). In addition, pilocytic astrocytomas are associated with NF1 (answer D).

    Comment Here

    Reference: Pilocytic astrocytoma

    Pilomyxoid astrocytoma
    Definition / general
    Essential features
    • A variant of pilocytic astrocytoma with distinct histologic and clinical features
    • Predominantly affects infants and young children
    • Preferentially located in the hypothalamic / chiasmatic region
    • Histologically characterized by small monomorphous bipolar cells, a myxoid background and perivascular arrangement
    • Generally has more aggressive behavior than pilocytic astrocytoma due to local recurrence and cerebrospinal fluid dissemination
    Terminology
    • Pilomyxoid astrocytoma (PMA)
    ICD coding
    • ICD-O: 9425/3 - pilomyxoid astrocytoma
    Epidemiology
    Sites
    Pathophysiology
    Etiology
    Diagrams / tables

    Images hosted on other servers:

    BRAF testing in pediatric low grade gliomas

    Histologic features

    Clinical features
    Diagnosis
    • MRI with contrast
    • Definitive diagnosis requires tissue biopsy
    Radiology description
    • MRI findings (Neurol Res 2008;30:945, MedGenMed 2004;6:42):
      • Well circumscribed
      • The majority (84.6%) are solid
      • T1: isointense or hypointense signal
      • T2 / FLAIR: hyperintense signal
      • No diffusion restriction
      • Homogeneous or heterogeneous enhancement pattern
      • No peritumoral edema
      • Craniospinal imaging is advisable to detect dissemination
    Radiology images

    Images hosted on other servers:

    Chiasmatic / hypothalamic pilomyxoid astrocytoma in a 3 month old infant:
    DWI: no sign of restricted diffusion
    T2w: homogeneous hyperintense signal
    T1w: hypointense signal
    T1w+ contrast: solid mass with marked enhancement

    Prognostic factors
    Case reports
    Treatment
    Gross description
    Frozen section description
    Frozen section images

    Images hosted on other servers:

    Perivascular bipolar cells in a mucoid background

    Microscopic (histologic) description
    Microscopic (histologic) images

    Contributed by Eman Abdelzaher, M.D., Ph.D.

    Angiocentric pattern

    Monomorphous cells in a myxoid background

    Cytology description
    Positive stains
    Negative stains
    Electron microscopy description
    • Bipolar tumor cells with elongated thin processes extending to rest upon the basal lamina of blood vessels
    • Abundant intermediate (glial) filaments can be seen in some cells
    • Apical surfaces may show microvilli, blebs and occasional cilia
    • Vesicles and coated pits, as well as dense core granules and synaptoid complexes may be seen (Adesina: Atlas of Pediatric Brain Tumors, 2010)
    Electron microscopy images

    Images hosted on other servers:

    Bipolar, thin processes
    extending to basal
    lamina of vessels;
    dense core granules

    Molecular / cytogenetics description
    Sample pathology report
    • Brain, suprasellar region, biopsy:
      • Pilomyxoid astrocytoma
      • Ancillary tests:
        • Strong positive GFAP staining
        • Focally positive for synaptophysin
        • Ki67 proliferation index: 3%
    Differential diagnosis
    Board review style question #1

    A 10 month old boy presents with a solid enhancing hypothalamic mass on MRI. Biopsy shows a highly myxoid tumor with perivascular arrangement of monomorphous cells. What is the most likely genetic alteration in this tumor?

    1. BRAF duplication / fusion
    2. H3F3A K27M mutation
    3. IDH1 and IDH2 mutations
    4. SMARCB1 / INI1 mutations
    Board review style answer #1
    A. BRAF duplication / fusion. The histology shows pilomyxoid astrocytoma. BRAF duplication / fusion is a frequent event in this tumor. IDH1 / 2 mutations are frequent in diffusely infiltrating gliomas. SMARCB1 / INI1 mutations are the hallmark of atypical teratoid / rhabdoid tumor. H3F3A K27M mutation is associated with diffuse midline gliomas, H3 K27M mutant.

    Comment Here

    Reference: Pilomyxoid astrocytoma
    Board review style question #2
    A 13 month old infant presented with failure to thrive. MRI showed a suprasellar enhancing mass. Intraoperative frozen section showed monomorphous bipolar tumor cells in an abundant myxoid background. Which of the following statements describes this tumor?

    1. Has a favorable prognosis with no reported risk of recurrence
    2. Preferentially located in the cerebellum with rare cases involving other sites
    3. Shares common genetic alterations with pilocytic astrocytoma
    4. Tends to affect adults
    Board review style answer #2
    C. Shares common genetic alterations with pilocytic astrocytoma. Pilomyxoid astrocytoma is considered a variant of pilocytic astrocytoma. They may be part of a single disease spectrum. They share common genetic alterations (BRAF duplication / fusion), some pilomyxoid astrocytomas mature into classic pilocytic astrocytomas over time and intermediate forms exist. Pilomyxoid astrocytomas tend to affect young children and are preferentially located in the hypothalamic / chiasmatic region. They are well known to exhibit a more aggressive behavior compared to pilocytic astrocytoma.

    Comment Here

    Reference: Pilomyxoid astrocytoma

    Pineal cyst
    Definition / general
    • Glial cyst within the pineal gland
    • Present in 2 - 3% of adults
    • May resemble a cystic tumor
    • Usually incidental finding, affect all age groups
    • Minority of cases, usually in young adults, are large enough to compress aqueduct and cause symptoms
    • Fine needle aspiration may provide rapid diagnosis (Cancer 2005;105:80)
    Radiology description
    • Midline, well circumscribed, 1 - 3 cm
    Case reports
    Gross description
    • Smooth external surface and inner lining
    Gross images

    Images hosted on other servers:

    Pineal cyst

    Microscopic (histologic) description
    Three layered cyst:
    • Inner lining is glial tissue which is hypocellular, highly fibrillated with many Rosenthal fibers; hemosiderin is common
    • Glial layer is surrounded by compressed, often calcified, pineal parenchyma
    • Outer layer is leptomeningeal connective tissue (Ann Diagn Path 1997;1:11)
    Cytology description
    • Small, uniform polygonal cells
    Differential diagnosis

    Pineal parenchymal tumor of intermediate differentiation
    Definition / general
    • Features intermediate between pineocytoma and pineoblastoma
    • 10% of pineal parenchymal tumors
    • Occasionally associated with CNS or extraneural metastasis but otherwise difficult to predict prognosis
    • Better survival than pineoblastoma
    Microscopic (histologic) description
    • Marked hypercellularity but relatively bland nuclear features
    • No / mild nuclear pleomorphism
    • No / rare pinocytomatous rosettes
    • Minimal mitotic activity
    Molecular / cytogenetics description
    Additional references

    Pineoblastoma
    Definition / general
    • Grade 4 of 4
    • Second most common pineal gland tumor after germ cell tumor
    • Usually age 20 years or less
    • Frequent CNS metastases or spinal seeding, which is the main cause of death
    • 5 year survival is 58%
    • Poor prognostic factors:
      • 7+ mitotic figures/10 HPF
      • Presence of necrosis
      • No neurofilament staining
    Case reports
    Treatment
    • Surgery, variable radiation therapy
    • Prognosis is usually poor
    Gross description
    • Located in pineal gland
    • May appear well demarcated from surrounding brain tissue but usually infiltrates into surrounding structures
    Microscopic (histologic) description
    • Sheets of densely packed cells with high grade (anaplastic / undifferentiated) features including high N/C ratio with minimal cytoplasm and large hyperchromatic nuclei
    • Also necrosis, frequent mitotic figures
    • Focal nuclear molding
    • Homer Wright or Flexner-Wintersteiner rosettes
    • May have lower grade features of pineocytoma and pineal parenchymal tumor of intermediate differentiation
    • Often infiltrates into surrounding structures
    Cytology images

    Images hosted on other servers:

    Small blue cell neuroepithelial malignant tumor

    Positive stains
    Negative stains
    Electron microscopy description
    • Occasional cytoplasmic dense core granules
    • Short immature cell processes
    • Occasional junctional complexes
    • No definite synapses
    Electron microscopy images

    Images hosted on other servers:
    Missing Image

    Presence of abundant filopodia and rare intercellular junctions but no definitive synapses

    Differential diagnosis

    Pineoblastoma
    Definition / general
    • Malignant embryonal neoplasm of the pineal gland (CNS WHO grade 4)
    Essential features
    • Malignant embryonal neoplasm (CNS WHO grade 4)
    • High mitotic index
    • Located in pineal region
    Terminology
    • Pineoblastoma
    ICD coding
    • ICD-O: 9362/3 - pineoblastoma
    • ICD-10: C75.3 - malignant neoplasm of pineal gland
    • ICD-11: 2A00.20 & XH1ZH1 - tumors of the pineal gland or pineal region & pineoblastoma
    Epidemiology
    • Pineal tumors are rare, representing < 1% of all CNS neoplasms (Neuro Oncol 2023;25:iv1)
    • Pineoblastoma represents ~30 - 35% of all pineal parenchymal tumors
    • Pineoblastoma typically arise in children / adolescents, with the overall median age of 6 years, varying depending on the molecular subtype (Acta Neuropathol 2021;141:771)
      • miRNA processing altered 1: 8.5 years old
      • miRNA processing altered 2: 11.6 years old
      • RB1 altered: 2.1 years old
      • MYC / FOXR2 activated: 1.3 years old
    Sites
    Pathophysiology
    Not relevant
    Etiology
    Diagrams / tables
    Not relevant
    Clinical features
    Diagnosis
    • Brain imaging, usually magnetic resonance imaging (MRI)
    • Requires tissue for definitive diagnosis, with molecular studies performed to further subclassify
    Laboratory
    Not relevant
    Radiology description
    Radiology images

    Contributed by Jared T. Ahrendsen, M.D., Ph.D.
    Axial MRI T1 postcontrast

    Axial MRI T1 postcontrast

    Axial MRI T2

    Axial MRI T2

    Prognostic factors
    • Young patient age, disseminated disease, extent of surgical resection and extent of adjuvant therapy are all important prognostic factors (Cancer 2012;118:173, Clin Transl Oncol 2012;14:827)
    • Staging (based on the Chang system for medulloblastoma) (Radiology 1969;93:1351)
      • M0: no evidence of gross subarachnoid or hematogenous metastases
      • M1: microscopic tumor cells in CSF
      • M2: gross nodular disease seen in cerebral / cerebellar subarachnoid space or in third / lateral ventricles
      • M3: gross nodular disease seen in spinal subarachnoid space
      • M4: metastases outside of the CNS
    • Clinical outcomes vary by molecular subgroup (Acta Neuropathol 2021;141:771)
      • miRNA processing altered 1: intermediate prognosis
      • miRNA processing altered 2: favorable prognosis
      • RB1 altered and MYC / FOXR2 activated: poor prognosis
    Case reports
    Treatment
    • Following surgical resection, patients typically receive adjuvant craniospinal irradiation (CSI) and chemotherapy (Acta Neuropathol 2020;139:259)
    Clinical images
    Not relevant
    Gross description
    • Soft, friable, tan-pink to tan-gray mass
    • May see areas of hemorrhage or necrosis (Cancer 1980;45:1408)
    Gross images
    N/A
    Frozen section description
    • Densely cellular, poorly differentiated neoplasm with numerous mitoses, necrosis, hyperchromasia and high N:C ratio
    Intraoperative frozen / smear cytology images

    Contributed by Jared T. Ahrendsen, M.D., Ph.D., Nitin R. Wadhwani, M.D. and Rachel A. Multz, M.D.
    Small round blue cells

    Small round blue cells

    Malignant embryonal cells

    Malignant embryonal cells

    H&E smear preparation

    Smear preparation

    Diff-Quik smear preparation

    Diff-Quik smear preparation



    Images hosted on other servers:
    Diff-Quik

    Diff-Quik

    Microscopic (histologic) description
    • Hypercellular, poorly differentiated, small round blue cell tumor (embryonal morphology)
      • High N:C ratio, nuclear molding, brisk mitotic activity, necrosis
    • Homer-Wright rosettes (eosinophilic material as false lumen) and less commonly Flexner-Wintersteiner rosettes (true lumen formation)
    • May or may not see invasion of adjacent brain structures
    • Reference: Brain Pathol 2000;10:49
    Microscopic (histologic) images

    Contributed by Jared T. Ahrendsen, M.D., Ph.D.
    Pineoblastoma with necrosis

    With necrosis

    Pineoblastoma histology Pineoblastoma histology

    Histology

    Homer-Wright pseudorosettes

    Homer-Wright pseudorosettes


    Mitotic activity

    Mitotic activity

    Synaptophysin positive

    Synaptophysin positive

    Variably chromogranin positive

    Variably chromogranin positive

    High Ki67

    High Ki67

    Virtual slides
    Not available
    Cytology description
    • As CSF cytology is used as part of the staging criteria for patients with pineoblastoma, recognizing tumor cells on cytology specimens is crucial
    • Tumor cells will show high N:C ratio, brisk mitotic activity and necrosis (Cytopathology 2023 Dec 15 [Epub ahead of print])
    • Sheet-like architecture and the presence of Homer-Wright rosettes may or may not be appreciated by cytology
    Cytology images
    Moved to frozen
    Immunofluorescence description
    Not relevant
    Immunofluorescence images
    Not relevant
    Positive stains
    Negative stains
    Electron microscopy description
    • No longer used in routine clinical practice
    Electron microscopy images
    N/A
    Molecular / cytogenetics description
    • 4 distinct subgroups of pineoblastoma have been identified from DNA methylation profiling (Acta Neuropathol 2020;139:223, Acta Neuropathol 2020;139:243, Acta Neuropathol 2021;141:771)
      • miRNA processing altered 1
        • Germline DICER1 variant or alterations in DICER1, DROSHA and DGCR8 (loss of function) by molecular testing
        • Gains of chromosomes 7 and 12, loss of chromosomes 16 and 22q seen on cytogenetics
      • miRNA processing altered 2
        • Germline DICER1 variant or alterations in DICER1 and DROSHA (loss of function) by molecular testing
        • Loss of chromosomes 8, 14q, 16 and 20 seen on cytogenetics
      • RB1 altered
        • Germline RB1 variant, somatic loss of function RB1 mutation or gain of miR17 / 92 seen on molecular testing
        • Gains of chromosomes 1q and 6p and loss of chromosome 16 seen on cytogenetics
      • MYC / FOXR2 activated
        • FOXR2 overexpression or MYC amplification seen on molecular testing
        • Gains of 8q and loss of 16q seen by cytogenetics
    Molecular / cytogenetics images

    Images hosted on other servers:
    Overview of pineoblastoma subtypes

    Overview of pineoblastoma subtypes

    Videos
    Not relevant
    Sample pathology report
    • Brain, pineal tumor, resection:
      • Pineoblastoma, miRNA processing altered 1, CNS WHO grade 4
    Differential diagnosis
    Board review style question #1

    A 5 year old boy presents with an intracranial mass that appears to arise adjacent to the splenium of the corpus callosum. A resection is performed and shows the tumor morphology above. The tumor cells are positive for synaptophysin, NSE and INI1 and negative for GFAP and Olig2. What is the most likely molecular alteration in this tumor?

    1. Biallelic loss of SMARCB1
    2. DICER1 mutation
    3. H3 K27M mutation
    4. IDH1 R132H mutation
    5. SHH pathway activation
    Board review style answer #1
    B. DICER1 mutation. The microscopic image shows a tumor of the pineal region with high N:C ratio, nuclear molding and numerous apoptotic bodies. The morphologic and anatomic features of the tumor, combined with the patient’s young age, indicate that this tumor is most likely a pineoblastoma. DICER1 mutations can be seen in both the miRNA processing altered 1 and miRNA processing altered 2 subgroups of pineoblastoma. Answer D is incorrect because IDH1 R132H mutations are typically seen in adult type diffusely infiltrative gliomas and patients are usually young adults. They will be positive for GFAP. Answer A is incorrect because biallelic loss of SMARCB1 is seen in atypical teratoid / rhabdoid tumors (AT / RT), which can be seen in young children and can show some morphologic overlap with pineoblastoma. These tumors will be variably positive for GFAP and typically show loss of INI1 by immunohistochemistry. Answer E is incorrect because SHH pathway activation is seen in a subset of medulloblastoma. They typically arise in the posterior fossa of younger children but can show significant morphologic overlap with pineoblastoma. Answer C is incorrect because H3 K27M alterations are seen in diffuse midline gliomas, which are positive for Olig2 and can show variable positivity for GFAP.

    Comment Here

    Reference: Pineoblastoma
    Board review style question #2
    A 5 year old boy presents with symptoms of increased intracranial pressure. Brain imaging reveals a mass centered in the pineal region and biopsy reveals a diagnosis of pineoblastoma. Which of the following immunostains is most specific for this diagnosis?

    1. Beta catenin
    2. CRX
    3. H3 K27M
    4. INI1
    5. LIN28A
    Board review style answer #2
    B. CRX. CRX is a transcription factor involved in retinal and pineal lineage differentiation. It is often diffusely positive in pineoblastoma and other pineal parenchymal tumors. Answer A is incorrect because aberrant nuclear beta catenin expression is observed in adamantinomatous craniopharyngioma and WNT activated medulloblastoma. Answer D is incorrect because loss of nuclear INI1 expression is commonly seen in atypical teratoid / rhabdoid tumors (AT / RT). Answer E is incorrect because LIN28A expression is commonly observed in embryonal tumors with multilayered rosettes (ETMR) and a subset of other tumors. Answer C is incorrect because H3 K27M is generally encountered in diffuse midline glioma.

    Comment Here

    Reference: Pineoblastoma

    Pineocytoma
    Definition / general
    • Grade 1 of 4
    • Mostly adults age 25 - 35 years
    • Slow growing
    • Average survival 7 years
    Gross description
    • Well circumscribed, gray, hemorrhagic
    Microscopic (histologic) description
    • Similar to normal pineal glands well differentiated cells but hypercellular
    • Fibrovascular stroma highlights expansive lobules of tumor cells with uniform round nuclei
    • Pinocytomatous rosettes (large, loose, Homer Wright-like rosettes with central fibrillar zones surrounded by neoplastic cells with round nuclei)
    • May have features of neuronal differentiation (ganglion cells)
    • Noninfiltrative, no / rare mitotic figures, no necrosis, no / minimal atypia
    Differential diagnosis

    Pituicytoma
    Definition / general
    • Pituicytomas are tumors that arise from the pituicytes of the neurohypophysis and pituitary infundibulum
    • 4 subtypes
    • Pituicytoma derives from light and dark normal pituicytes, oncocytic pituicytoma derives from oncocytic pituicytes, the granular cell subtype derives from granular pituicytes and the ependymal subtype derives from the elongated ependymal pituicytes (Am J Surg Pathol 2013;37:1694)
      • These tumors and cells are characterized by the expression of the transcription factor TTF1
    Essential features
    • Pituicytoma belongs to the group of TTF1 expressing tumors of the neurohypophysis and infundibulum
    • Histologic features include fascicles of spindle cells or epithelioid cells expressing TTF1, S100 protein and GFAP
    • Surgical resection is curative; excellent outcome
    Terminology
    • Not recommended
      • Infundibuloma
      • Low grade glioma of sellar and suprasellar region
      • Pilocytic astrocytoma
      • Spindle cell oncocytoma of adenohypophysis
      • Granular cell myoblastoma / schwannoma / neuroma
      • Pituitary ependymoma / sellar ependymoma
    ICD coding
    Epidemiology
    • Classic pituicytoma represents ~0.07% of all primary sellar neoplasms (Eur J Endocrinol 2007;156:203)
    • ~50% of posterior pituitary tumors (World Neurosurg 2021;145:148)
    • Usually in adults: 40 - 60 years
    • No obvious male or female predominance
    • Exceptional in children (3% of posterior pituitary tumors)
    Sites
    • Sella and suprasellar region
    Pathophysiology
    Etiology
    • Unknown
    Clinical features
    • Main signs and symptoms at onset (World Neurosurg 2021;145:148)
      • Headache
      • Visual field defects
      • Hypopituitarism
      • Occurrence of polydipsia and polyuria (diabetes insipidus) is rare
      • Apoplexy is exceptional
      • Pituicytoma can also present in association with primary tumors of adenohypophyseal cells (collision tumors)
    Diagnosis
    • Evidence of a sellar and suprasellar (or suprasellar only) lesion involving the infundibulum
    • Usually no excess of pituitary hormones
    • Diagnosis is made on histology
    • Reference: World Neurosurg 2021;145:148
    Laboratory
    • Low serum levels of pituitary hormones
    • Increase in prolactin due to pituitary stalk compression
    • Occasional increase in growth hormone or adrenocorticotrophic hormone (ACTH) / peripheral cortisol due to coexistence of a pituitary neuroendocrine tumor
    • References: World Neurosurg 2021;145:148, Endocrine 2020;70:15
    Radiology description
    • Can be suspected when exclusively suprasellar and the anterior hypophysis is spared
    • Indistinguishable from other clinically nonfunctioning primary sellar tumors when it involves the sellar and suprasellar region
    • MRI features: isointense on T1 weighted sequences and often inhomogeneous intensity on T2 weighted sequences; homogenous or heterogeneous contrast enhancement
    • Reference: World Neurosurg 2021;145:148
    Radiology images

    Contributed by Federico Roncaroli, M.D.
    Sagittal postcontrast MRI

    Sagittal postcontrast MRI

    Prognostic factors
    • Behaves as a benign tumor when completely resected
    • Recurrence can occur when incompletely excised
    • Oncocytic tumors have higher frequency of recurrence, possibly due to rich vascularity and adhesion
    • Radiation therapy has been reported for recurrent / residual disease
    • Metastasis has not been reported
    • 1 recurrent case showed nuclear atypia and mitoses (Pathology 2011;43:389)
    Case reports
    Treatment
    Gross description
    • Usually white, gray or light yellow soft tissue fragments; hemorrhagic changes can be seen
    Frozen section description
    • Fascicles of spindle cells in fibrillary background reminiscent of glioma
    • Possible ill defined whorls reminiscent of sellar meningioma
    • Bipolar cells similar to schwannoma
    Microscopic (histologic) description
    • Sheets and short fascicles of spindle, bipolar tumor cells; storiform pattern can be present
    • Variants distinguished based on cytologic features
      • Oncocytic tumors have epithelioid cells with oncocytic cytoplasm
      • Granular cell variants have large cells with granular cytoplasm
      • Ependymal variants have elongated cells with ependymal cell-like features
    • Mixed morphologies are not uncommon
    • Rarely, focal chronic inflammation
    • Absent pericellular reticulin fibers
    • Absent Rosenthal fibers, granular bodies and hyalinized blood vessel walls
    • Lack of Herring bodies
    • Usually negligible mitotic activity
    • Rarely, increased cellularity, severe nuclear atypia and high mitotic count
    • References: Am J Surg Pathol 2000;24:362, Am J Surg Pathol 2013;37:1694
    Microscopic (histologic) images

    Contributed by Federico Roncaroli, M.D. and Sylvia L. Asa, M.D., Ph.D.
    Fascicles of spindle cells

    Fascicles of spindle cells

    Ill defined whorls

    Ill defined whorls

    Schwannoma-like features

    Schwannoma-like features

    Mild nuclear atypia

    Mild nuclear atypia

    Oncocytic pituicytoma

    Oncocytic pituicytoma


    Oncocytic pituicytoma

    Oncocytic pituicytoma

    Granular cell pituicytoma

    Granular cell pituicytoma

    GFAP expression

    GFAP expression

    TTF1 nuclear expression

    TTF1 nuclear expression

    Cytology description
    • Aggregates of spindle cells with fibrillary to fine cytoplasm, usually thin walled, delicate vessels and neuropil-like background; features can be reminiscent of meningioma (Diagn Cytopathol 2020;48:342)
    Positive stains
    Negative stains
    Electron microscopy description
    • Elongated or oval tumor cells
    • Focal accumulations of intermediate filaments
    • No interdigitating cell membranes
    • Oncocytic variant has abundant mitochondria rich in lamellar cristae
    • No secretory granules
    • References: Neurosurgery 2004;54:753, Am J Surg Pathol 2013;37:1694
    Molecular / cytogenetics description
    Sample pathology report
    • Pituitary gland / sellar region, surgical excision:
      • Pituicytoma (subtype if relevant) (WHO grade 1) (see comment)
      • Comment: Pituicytoma belongs to the group of TTF1 expressing tumors of the posterior hypophysis and infundibulum. It rarely recurs when incompletely removed; follow up is recommended.
      • Immunohistochemically, the tumor cells show TTF1 nuclear expression, diffuse vimentin and S100 expression. Immunostains for keratins, synaptophysin, chromogranin and biomarkers of adenohypophyseal neuroendocrine cells are negative.
    Differential diagnosis
    Board review style question #1

    A middle aged man presented with hypopituitarism. Imaging showed a sellar and suprasellar enhancing lesion. The microscopic image above is from the surgical specimen. Which of the following immunostains is typically positive in these tumor cells?

    1. Chromogranin
    2. Cytokeratin CAM5.2
    3. Neurofilament
    4. TTF1
    5. Vasopressin
    Board review style answer #1
    D. TTF1. TTF1 is positive since this is pituicytoma, a tumor derived from posterior pituitary cells known as pituicytes, which derive from the medial basal hypothalamus, which expresses TTF1. Answers A and C are incorrect because these are not neural or neuroendocrine cells; therefore, they are negative for chromogranin and neurofilament. Answer B is incorrect because they are not epithelial and therefore are negative for cytokeratins, unlike adenohypophyseal cells. Answer E is incorrect because vasopressin is produced by neurons of the hypothalamus, not by these modified glial cells.

    Comment Here

    Reference: Pituicytoma
    Board review style question #2
    Which of the following is correct regarding pituicytoma?

    1. Patients present with headaches, visual field disturbances and hypopituitarism
    2. Patients present with syndrome of antidiuresis (SIAD)
    3. Surgery is not curative
    4. These tumors are not responsive to radiotherapy
    Board review style answer #2
    A. Patients present with headaches, visual field disturbances and hypopituitarism. Answer B is incorrect because patients with pituicytoma do not have features of vasopressin excess / antidiuresis but may have diabetes insipidus. Answers C and D are incorrect because surgery can be curative if complete resection is possible but if not, radiotherapy can provide control of growth.

    Comment Here

    Reference: Pituicytoma

    Pituitary hyperplasia
    Definition / general
    Essential features
    • Nodular or diffuse hyperplasia of polymorphic acini with intact reticulin, without effacement of gland architecture
    • MRI essentially shows symmetrical and diffuse enlargement, similar intensity to gray matter (Pituitary 1999;1:169)
    ICD coding
    • ICD-10: E23.7 - disorder of the pituitary gland, unspecified
    Epidemiology
    Sites
    • Adenohypophysis
    Pathophysiology / etiology
    • Physiologic response
      • Most common: prolactin cell hyperplasia in pregnancy and lactation, peaks immediately postpartum
      • Puberty: pituitary height peaks in the 20 - 29 year age group (F > M)
      • Elderly: nonfunctional hyperplasia due to basophil cells
        • Pars intermedia derived proopiomelanocortin (POMC) cells invasion into the neurohypophysis, women in the 50 - 59 year age group (Hormones (Athens) 2003;2:149)
    • Pathologic hyperplasia due to end organ insufficiency
      • Longstanding primary hypothyroidism causes TSH cell hyperplasia, young females
      • Gonadotroph hyperplasia and sellar expansion in primary hypogonadism due to Klinefelter or Turner syndrome
      • Polycystic ovary syndrome: hyperprolactinemia and lactotroph hyperplasia
      • ACTH cell hyperplasia due to hypocortisolism in Addison disease
      • Rare: pituitary transcription factor gene, PROP1 mutations
    • Pathologic hyperplasia associated with ectopic excess of releasing hormones
      • GH releasing hormone (GHRH) or corticotropin releasing hormone (CRH) ectopic release causing somatotroph or corticotroph hyperplasia secreted by pancreatic islet cell tumor, pheochromocytoma, bronchial and thymic carcinoid tumors
      • ACTH cell hyperplasia due to corticotropin releasing hormone secretion from hypothalamic hamartoma or neuroendocrine tumors
    • Iatrogenic: treatment with antipsychotics and excess estrogen in transgender women are associated with increased secretion of prolactin (Int Clin Psychopharmacol 2019;34:89, Int J Mol Sci 2020;21:2024, BMJ Case Rep 2009;2009:bcr02.2009.1589)
    • Syndromic: mammosomatotroph hyperplasia in McCune-Albright syndrome, Carney complex, MEN1 related GHRH associated, X linked acrogigantism syndrome (XLAG)
    • Idiopathic
    • Hereditary (J Clin Endocrinol Metab 2011;96:E2078)
    Clinical features
    • Mass effect
      • Visual disturbance, bitemporal hemianopia, diplopia
      • Headaches
    • Hormone related hyperplasia
      • GH cell: gigantism or acromegaly
      • Prolactin cell: hyperprolactinemia
      • ACTH cell: Cushing disease
      • TSH cell:
      • LH / FSH: result of early onset hypogonadism
    Diagnosis
    Laboratory
    • Hormonal levels as indicated
    Radiology description
    Radiology images

    Images hosted on other servers:

    Sagittal and coronal MRI

    Prognostic factors
    Case reports
    Treatment
    Gross description
    • At autopsy or in the rare event the entire gland is removed, it is diffusely enlarged and tan-white with no discernible nodules
    • Lack of well defined lesion
    • Specimen may be fragmented
    Microscopic (histologic) description
    • Diffuse or nodular hyperplasia
      • Unevenly enlarged acini
      • Usually single cell type
      • Relative cellular monomorphism within affected acinus
      • Noncompressive; indistinct demarcation
      • Difficult to diagnose in fragmented specimen
      • Numerical increase in pituitary cells without alteration in architecture
      • Rare mitotic activity
    • GH cell hyperplasia: chromophobe to pale eosinophilic polygonal cells
    • PRL cell hyperplasia: chromophobe with rare microcalcifications
    • ACTH cell hyperplasia: amphophilic with large vacuoles (lysosomes) and Crooke cell change
    • TSH cell hyperplasia: chromophobe, occasional spindle cells and multiple large PAS+ lysosomes
    • LH / FSH hyperplasia: hypervacuolization (castration cells) (Endocrinol Diabetes Metab Case Rep 2015;2015:150017)
    Microscopic (histologic) images

    Images hosted on other servers:
    Pituitary hyperplasia microscopic

    Pituitary hyperplasia microscopic

    Cytology description
    • Hypercellular smear with heterogeneous cell populations
    Positive stains
    • Reticulin is essential for the diagnosis, highlights retention of acinar architecture and may show some expanded acini
    • Synaptophysin
    • Cell type routine cytoplasmic stains prolactin, GH, TSH, LH, FSH, ACTH
    • PAS highlights lysosomes in TSH hyperplasia
    • Ki67 labeling may be mildly increased (normal gland is completely negative)
    Negative stains
    Electron microscopy description
    • Same as normal pituitary:
      • GH producing somatotrophs:
        • Rough endoplasmic reticulum, well formed Golgi complexes and numerous large, dense secretory granules 100 - 250 nm
      • PRL producing lactotrophs:
        • Elaborate rough endoplasmic reticulum arranged in parallel arrays, occasionally forming concentric structures known as nebenkern formations
        • Prominent Golgi complexes and extrusion of secretory granules at the lateral cell borders known as misplaced exocytosis up to 700 nm
      • GH and PRL producing mammosomatotroph:
        • Pleomorphic heterogenous granules 150 - 1,000 nm and misplaced exocytosis
      • TSH producing thyrotrophs:
        • Short dilated rough endoplasmic reticulum and small secretory granules that align along the plasma membrane
      • ACTH producing corticotrophs:
        • Secretory granules are pleomorphic in shape and electron density with indentations and evaginations of granule membranes, resulting in heart and teardrop shapes
        • Also small bundles of intermediate (keratin) filaments throughout the cytoplasm
    • Reference: Ann Clin Lab Sci 1979;9:275, Microsc Res Tech 1992;20:107
    Molecular / cytogenetics description
    • No specific molecular / cytogenetics available
    • Reported germline mutations in the aryl hydrocarbon receptor interacting protein (AIP) gene in familiar cases
    • Some syndromic association: Carney complex, McCune-Albright syndrome, X linked acrogigantism (XLAG) and less likely, MEN1 related GHRH associated
    • Reference: J Clin Endocrinol Metab 2011;96:E2078
    Sample pathology report
    • Pituitary, transsphenoidal resection:
      • Consistent with pituitary hyperplasia (see comment)
      • Comment: Histological sections of the specimen show pituitary tissue composed of unevenly enlarged acini comprised of sheets of monomorphic cells with small round nuclei and moderate amounts of granular eosinophilic cytoplasm. Immunostains were performed that show the lesional tissue cells are strongly positive for synaptophysin, chromogranin, EMA. Ki67 stain labels a rare cell. Reticulin stain highlights retention of acinar architecture and expansion of acini. Prolactin hormone is positive. Other hormonal stains (ACTH, LH, FSH, GH) are negative. This immune profile is consistent with benign pituitary hyperplasia, prolactin hormone producing. Clinical and radiological correlation are recommended.
    Differential diagnosis
    • Normal adenohypophysis:
      • Normal size pituitary gland with heterogeneous cell populations within normal acini
      • ACTH cells normally aggregate at the lateral wings of anterior pituitary
    • Pituitary adenoma:
      • Enlarged mass, well defined from normal pituitary gland
      • Homogeneous cell populations with splayed and disrupted reticulin network
      • Compresses adjacent normal acini
      • Hyperplasia more likely to show strong cytoplasmic immunoreactivity to respective hormones throughout the fragments when compared with adenoma
    Board review style question #1
    Physiologic pituitary hyperplasia is mostly seen in

    1. Corticotrophin releasing hormone ectopic release
    2. Longstanding primary hypothyroidism
    3. Polycystic ovary syndrome
    4. Prolactin cell hyperplasia in pregnancy and lactation
    Board review style answer #1
    D. Prolactin cell hyperplasia in pregnancy and lactation

    Comment Here

    Reference: Pituitary hyperplasia
    Board review style question #2
    Which of the following is a characteristic microscopic feature of pituitary hyperplasia?

    1. Compresses adjacent normal acini
    2. Homogeneous cell populations with splayed and disrupted reticulin network
    3. Microcalcifications are never identified
    4. Mild acinar expansion which is noncompressive
    Board review style answer #2
    D. Mild acinar expansion which is noncompressive

    Comment Here

    Reference: Pituitary hyperplasia

    Pituitary neuroendocrine tumor (PitNET)
    Definition / general
    • Neuroendocrine tumors of the anterior pituitary gland that are composed of secretory cells with pituitary hormone production
    • Main tumor types include somatotroph, lactotroph, mammosomatotroph, thyrotroph, corticotroph, gonadotroph, null cell and plurihormonal tumors
    Essential features
    • Routine assessment of histology includes determination of:
      • Architecture: solid growth, nests, trabeculae or perivascular rosettes
      • Cytoplasmic features: acidophilia, basophilia, inclusions indicative of fibrous bodies
      • Nuclear features: mitoses, pleomorphism, giant cells
      • Other features: inflammatory changes, stroma, hemorrhage, vascular features
    • Determination of cell lineage and hormonal activity using stains for transcription factors, hormones and cytokeratins (Endocr Pathol 2022;33:6)
    • Evaluation of tumor proliferation potential by mitotic count and Ki67 labeling index
    • Evaluation of tumor invasion if possible to identify clinically aggressive tumors
    • Ultrastructural features of tumor cells may be important for classification of unusual tumors
    Terminology
    • Pituitary neuroendocrine tumor (preferred nomenclature)
    • Pituitary adenoma (still acceptable but not preferred terminology)
    • Metastatic pituitary neuroendocrine tumor (formerly known as pituitary carcinoma)
    ICD coding
    • ICD-O: 8272/3 - pituitary carcinoma, NOS
    • ICD-11: 2F9A & XH94U0 - neoplasms of unknown behavior of endocrine glands & pituitary adenoma, NOS
    Epidemiology
    • 10 - 15% of intracranial neoplasms
    • Mainly occurs in fourth to seventh decade
    • F > M
    • Incidental tumors are seen in ~14.4% of autopsies and 22.5% of radiologic studies (Cancer 2004;101:613)
    Sites
    Pathophysiology
    • Cell differentiation is driven by transcription factors leading to 3 main cell lineages
    • Transcription factors are (Endocr Pathol 2022;33:6):
      • Pit1 (pituitary specific POU class homeodomain transcription factor), which leads to differentiation of somatotrophs, mammosomatotrophs, lactotrophs and thyrotrophs
      • SF1 (steroidogenic factor 1), which regulates gonadotroph cell differentiation
      • Tpit (T box family member TBX19), which has a transcription factor driving proopiomelanocortin (POMC) lineage with differentiation of corticotrophs
    Etiology
    • Most tumors are of unknown etiology and lack mutations that are considered causative; epigenetic alterations are common (Endocr Pathol 2021;32:3)
    • Inherited syndromes:
      • MEN1 with involvement of the MEN1 gene and loss of menin
      • MEN4 with mutation of CDKN1B and loss of p27
      • MEN5 with mutation of MAX
      • Carney complex with mutation of PRKR1ɑ
      • Familial acromegaly with mutation of AIP
      • SDHx related disease due to mutation in any of the genes encoding the SDH complex
      • Hyperparathyroidism jaw tumor syndrome due to mutation of CDC73 and loss of parafibromin
      • Lynch syndrome due to mutation in MLH1 / MSH2 / MSH6 / PMS2
    • Noninherited germline predisposition syndromes include:
      • X linked acrogigantism due to Xq26 microduplication and GPR101 alterations
      • McCune-Albright syndrome with GNAS activating mutations
    Diagrams / tables
    Tumor type Transcription factor(s) Hormone(s) Other biomarkers Clinical features
    Tpit family
    Densely granulated corticotroph tumor Tpit ACTH PAS; intense keratins Florid Cushing disease or silent
    Sparsely granulated corticotroph tumor Tpit ACTH (weak) PAS (weak); strong keratins Cushing disease or silent
    Crooke cell tumor Tpit ACTH PAS; ring-like keratins Cushing disease or silent
    Pit1 family
    Densely granulated somatotroph tumor Pit1 GH Perinuclear keratins Acromegaly
    Sparsely granulated somatotroph tumor Pit1 GH (weak) Keratin fibrous bodies (> 70%) Acromegaly or silent
    Mammosomatotroph tumor Pit1, ER GH, PRL Perinuclear keratins Acromegaly and hyperprolactinemia
    Sparsely granulated lactotroph tumor Pit1, ER PRL (juxtanuclear) Variable keratins Hyperprolactinemia (tumor size correlates with PRL level)
    Densely granulated lactotroph tumor Pit1, ER PRL Variable keratins Hyperprolactinemia
    Acidophil stem cell tumor Pit1, ER PRL with or without GH (weak, focal) Variable keratins with or without fibrous bodies Hyperprolactinemia
    Thyrotroph tumor Pit1, GATA3 TSH Variable keratins Elevated TSH with or without hyperthyroidism
    Mature plurihormonal Pit1 lineage tumor Pit1, GATA3, ER Variable, often multiple Perinuclear keratins Acromegaly, hyperprolactinemia and hyperthyroidism
    Immature Pit1 lineage tumor Pit1 with or without GATA3 with or without ER Variable, often limited / focal Variable keratins with or without fibrous bodies Silent or acromegaly or hyperprolactinemia or hyperthyroidism
    SF1 family
    Gonadotroph tumor SF1, GATA3, ER FSH, LH Variable keratins, follicular cells Silent with or without elevated gonadotropins
    No lineage fidelity
    Null cell tumor None None Variable Silent
    Unclassified plurihormonal tumor Multiple combinations Multiple combinations Variable Variable
    Clinical features
    • Clinical features of hormone excess: acromegaly, gigantism, Cushing disease, sequelae of hyperprolactinemia, hyperthyroidism, rarely gonadotropin excess (Endocr Pathol 2022;33:6)
    • Larger tumors (> 1 cm) can be associated with mass effects such as headache, visual disturbance and hypopituitarism
    • Hemorrhagic necrosis of large tumors (pituitary apoplexy) may be a surgical emergency
    Diagnosis
    • Current consensus classifies tumors according to pituitary cell lineage and cell differentiation (see Diagrams / tables)
    • Classification requires immunohistochemical staining for pituitary transcription factors including the 3 main ones (Pit1, Tpit and SF1) as well as ER and GATA3
      • Main pituitary hormones (growth hormone, prolactin, adrenocorticotropic hormone [ACTH], beta thyroid stimulating hormone [TSH], beta luteinizing hormone [LH], beta follicle stimulating hormone [FSH] and alpha subunit of glycoproteins) and keratins (usually CAM 5.2 but also AE1 / AE3 or CK18) (Hum Pathol 2021;107:87)
    Laboratory
    • Biochemical assessment should be performed in all patients with pituitary tumors (Endocr Pathol 2022;33:6)
    • Serum prolactin can be elevated in any patient with a sellar lesion since it is under tonic inhibition and interruption of hypothalamic signaling can result in hyperprolactinemia
      • However, this so called stalk effect does not usually result in a level > 200 mcg/L; levels above this usually indicate a tumor producing prolactin
      • In patients with true lactotroph tumors, prolactin levels are usually proportional to tumor size
      • Patients with large tumors and prolactin levels of 500 mcg/L or less usually do not have a lactotroph tumor
    • Growth hormone and IGF1 are biomarkers of acromegaly or gigantism; since growth hormone secretion is pulsatile, IGF1 provides an integrated measure that is more reliable
      • Oral glucose tolerance test is a definitive confirmatory test for the diagnosis of acromegaly
    • Cortisol and ACTH are elevated in Cushing disease
      • The relationship between these is important; a high normal ACTH in a patient with high normal or elevated cortisol should be assessed for Cushing disease
      • High dose dexamethasone suppression testing can be useful
    • TSH secreting tumors cause increased TSH levels and are usually associated with elevated T3 and T4 levels, hyperthyroidism and goiter
    • Gonadotropin excess can be physiological in postmenopausal women; serum levels of FSH and LH as well as testosterone levels in men should be screened
    • Cerebrospinal fluid may be xanthochromic with crenated red blood cells and high protein levels in pituitary apoplexy
    Radiology description
    • Method of classification is based on tumor size and degree of invasion (Endocr Pathol 2022;33:6)
    • Important for planning of surgical resection
    • Microtumors measure < 1 cm in diameter; macrotumors are > 1 cm
    • Identification of tumor extension is important: suprasellar, lateral (cavernous sinus), inferior (sphenoid sinus), posterior fossa
    • Density on T2 weighted imaging distinguishes sparsely from densely granulated tumors
    Radiology images

    Images hosted on other servers:
    Missing Image

    Sellar and suprasellar masses

    Prognostic factors
    • Tumor subtypes are predictive of responsiveness to medical therapies
      • For example, densely granulated somatotroph tumors are usually responsive to long acting somatostatin analogues whereas sparsely granulated somatotroph tumors are not; densely granulated corticotroph tumors tend to respond to pasireotide therapy
    • Invasive, aggressive tumors adenomas often recur over several years; presurgical growth rates can predict temporal pattern of regrowth
    • Immature Pit1 lineage tumors tend to be aggressive (Mod Pathol 2016;29:131)
    • Crooke cell tumors are aggressive (Endocr Pathol 2022;33:6)
    • ATRX loss has been described in some aggressive corticotroph tumors (J Clin Endocrinol Metab 2021;106:1183, Endocrine 2022;76:228)
    Case reports
    Treatment
    • Surgical approaches
      • Commonly via transsphenoidal route (minimally invasive)
      • Transfrontal approach for larger invasive tumors
      • Partial versus gross total resection
    • Dopamine agonists can reduce prolactin levels and the size of sparsely granulated lactotroph tumors
    • Somatostatin analogs can reduce hormone (growth hormone and TSH) secretion but may cause little reduction in tumor size
    • Clinically nonfunctioning tumors may show little response to medical therapy
    • Radiation therapy may be used for tumors resistant to medical therapy or for surgical remnants from subtotal resections
    • Therapy may include linear accelerator radiotherapy, stereotactic radiotherapy or radiosurgery
    • Single fraction versus fractionated radiosurgery are options (Pituitary 2017;20:489)
    • Several studies have evaluated tumor response to temozolomide therapy, particularly in aggressive pituitary tumors (Front Neurol 2021;12:700007, Rev Endocr Metab Disord 2020;21:263)
    • Peptide receptor radionuclide therapy with lutetium oxodotreotide has been reported (Endocr Pathol 2019;30:118, Endocr Connect 2019;8:528, J Endocr Soc 2021;5:bvab133)
    Gross description
    • These tumors are usually resected as small fragments; however, occasionally they can be removed in a single piece that allows assessment of margins
    Frozen section description
    • Frozen sections are discouraged
    • Most cases do not require intraoperative consultation but in unusual situations (e.g., where the differential diagnosis is inflammation), touch preps can be used for cytologic assessment without compromising residual tissue that is retained for proper immunohistochemical analysis
    Microscopic (histologic) description
    • Architecture varies from solid sheets to nests within a fibrovascular stroma (Endocr Pathol 2022;33:6)
      • Some tumors, usually gonadotroph tumors, form pseudorosettes around vascular channels
      • Dyscohesive growth is characteristic of sparsely granulated somatotroph tumors (it is associated with loss of E-cadherin expression)
    • Most tumor cells are epithelioid but some can be spindle shaped; this is a feature of some immature Pit1 lineage tumors
    • Most tumors have a uniform nuclear morphology with stippled chromatin, inconspicuous nucleoli and moderately abundant cytoplasm
    • Cells may be classified as acidophilic, basophilic or chromophobic based on tinctorial differences; this usually correlates with content of hormone containing secretory cells (i.e., densely versus sparsely granulated)
    • Crooke hyaline change is characterized by large chromophobic or eosinophilic cells with a glassy hyaline appearance (due to accumulation of keratin filaments); it can be seen in neoplastic and nonneoplastic corticotrophs
    • Oncocytic change is a feature of some gonadotroph tumors where it tends to be variable; diffuse oncocytosis is a feature of acidophil stem cell tumors
    Microscopic (histologic) images

    Contributed by Sylvia L. Asa, M.D., Ph.D.
    Densely granulated corticotroph tumor

    Densely granulated corticotroph tumor

    Sparsely granulated corticotroph tumor

    Sparsely granulated corticotroph tumor

    p27 in silent corticotroph tumor

    p27 in silent corticotroph tumor

    Crooke cell tumor

    Crooke cell tumor

    Densely granulated somatotroph tumor

    Densely granulated somatotroph tumor


    Sparsely granulated somatotroph tumor

    Sparsely granulated somatotroph tumor

    Sparsely granulated lactotroph tumor

    Sparsely granulated lactotroph tumor

    Thyrotroph tumor

    Thyrotroph tumor

    Gonadotroph tumor

    Gonadotroph tumor

    Cytology description
    • Normal pituitary has mixed cell types on smear preparation whereas tumors show uniform morphology and cell type
    • Tumors produce cellular smear with discohesive small round blue cells
    • Some tumors have specific cytologic atypia (e.g., fibrous bodies of sparsely granulated somatotroph tumors, Crooke hyaline of Crooke cell tumors)
    Cytology images

    Contributed by Michael Punsoni, M.D.
    Touch prep showing a cellular population of discohesive small round blue cells Touch prep showing a cellular population of discohesive small round blue cells Touch prep showing a cellular population of discohesive small round blue cells

    Touch prep showing a cellular population of discohesive small round blue cells

    Positive stains
    • All pituitary neuroendocrine tumors express INSM1, synaptophysin and chromogranin; positivity for chromogranin A is variable
    • Stains for pituitary transcription factors (Pit1, Tpit, SF1, ER and GATA3) and hormones provide the basis for tumor classification (see Diagrams / tables)
    • Reticulin staining is useful to distinguish normal acinar architecture, enlarged acini of hyperplasia versus disrupted staining pattern in neoplasms
    • Perinuclear CAM 5.2 staining pattern is a feature of densely granulated somatotroph tumors, mammosomatotroph tumors and mature plurihormonal Pit1 lineage tumors
    • Paranuclear CAM 5.2 positive fibrous bodies are a conspicuous feature present in > 70% of tumor cells of sparsely granulated somatotroph tumors
      • Occasional fibrous bodies can be seen in poorly differentiated Pit1 lineage tumors and acidophil stem cell tumors
    • Diffuse and strong cytoplasmic CAM 5.2 staining is characteristic of corticotroph tumors
    • Ring-like CAM 5.2 positivity is characteristic of Crooke cell tumors (Brain Pathol 2012;22:443)
    • Ki67 immunolabeling often used to characterize tumors with elevated proliferative indices
    • Antimitochondrial antibody staining is a feature of oncocytic tumors, including some gonadotroph tumors and the characteristically oncocytic acidophil stem cell tumor
    Negative stains
    • GFAP: positive in ependymomas
    • CK7: positive in choroid plexus tumors and metastatic lesions
    • CK20: positive in metastatic lesions
    • TTF1: positive in tumors of posterior lobe pituicytes including conventional pituicytoma, granular cell pituicytoma, oncocytic pituicytoma (formerly known as spindle cell oncocytoma) and ependymal pituicytoma (formerly known as sellar ependymoma) (Am J Surg Pathol 2013;37:1694, Endocr Pathol 2014;25:436)
    Electron microscopy description
    • Mostly supplanted by immunohistochemistry; however, electron microscopy may still play a role in characterization of unusual pituitary tumors and identification of giant mitochondria in acidophil stem cell tumor and spheridia in immature Pit1 lineage tumors (formerly known as silent subtype III adenomas) (Brain Pathol 2012;22:443)
    Electron microscopy images

    Images hosted on other servers:

    Ultrastructural features of some rare pituitary adenomas

    Molecular / cytogenetics description
    • No specific molecular characteristics in routine clinical diagnostic workup
    • Most pituitary tumors are sporadic; minority are part of hereditary or familial syndromes
    • Somatic mutations in GNAS and USP8 genes have been found in ~40% of sporadic somatotroph tumors and 30 - 60% of sporadic corticotroph tumors, respectively (Endocr Pathol 2021;32:3)
    Sample pathology report
    • Pituitary, transsphenoidal resection specimen:
      • Mature plurihormonal Pit1 lineage tumor (see synoptic report)
      • Synoptic report:
        • Clinical features: functional
        • Hormone excess (specify): acromegaly, hyperthyroidism
        • Tumor size (from imaging): greatest dimension was 3.8 cm
          • Additional dimensions: 3.5 x 2.4 cm
        • Received: in formalin
        • Specimen integrity: fragmented
        • Specimen size: 2.4 x 2.2 x 0.4 cm
        • Histologic features:
          • Reticulin: disrupted
          • PAS: negative
          • Infiltrating tumor: cannot be determined
        • Immunohistochemistry:
          • Tpit is negative
          • Pit1 is positive
          • ER is focally positive
          • GATA3 is focally positive
          • SF1 is negative
          • ACTH is negative
          • Growth hormone is positive
          • Prolactin is focally positive
          • Beta TSH is focally positive
          • Alpha subunit is diffuse positive
          • Beta FSH is negative
          • Beta LH is negative
          • Keratin (CAM 5.2) is diffuse
          • Ki67 labeling index is 5%
        • Tumor type: pituitary neuroendocrine tumor
        • Subtype: mature plurihormonal Pit1 lineage tumor
        • Additional pathologic findings
        • Nontumorous adenohypophysis: present
        • Neurohypophysis: not identified
        • Gross description: Labeled with the patient's name and hospital number and received in formalin as multiple fragments of tan-pink friable tissue that measure 2.4 x 2.2 x 0.4 cm in aggregate. The specimen is submitted in toto in 1 cassette.
    Differential diagnosis
    Board review style question #1
    Which of the following is true of pituitary neuroendocrine tumors?

    1. Diabetes insipidus is often associated with PitNETs
    2. GNAS mutations have been described in pituitary tumors
    3. Most PitNETs are negative for pituitary hormones
    4. Most PitNETs are part of hereditary or familial syndromes
    5. The diagnosis of metastatic behavior can be predicted by morphological or immunohistochemical features
    Board review style answer #1
    B. GNAS mutations have been described in pituitary tumors. GNAS mutations have been described in a subset of pituitary somatotroph tumors.

    Comment Here

    Reference: Pituitary neuroendocrine tumor (PitNET)
    Board review style question #2

    The chromophobic tumor in the image above has the keratin pattern shown on the right. Which of the following is correct?

    1. This patient had acromegaly
    2. This patient had Cushing disease
    3. This patient would not have had hyperprolactinemia
    4. This tumor is likely to respond to first generation somatostatin analogues
    5. This tumor stains for Tpit
    Board review style answer #2
    A. This patient had acromegaly. This sparsely granulated somatotroph tumor stains for Pit1 (not Tpit) and causes acromegaly, which may be associated with hyperprolactinemia due to the stalk section effect since sparsely granulated tumors are often large at the time of diagnosis. These tumors generally do not respond to first generation somatostatin analogues. Somatotroph tumors do not cause Cushing disease.

    Comment Here

    Reference: Pituitary neuroendocrine tumor (PitNET)

    Pleomorphic xanthoastrocytoma
    Definition / general
    • Pleomorphic xanthoastrocytoma (PXA) is a circumscribed astrocytic glioma with large multinucleated cells, spindle cells and xanthomatous cells with frequent eosinophilic granular bodies and rich reticulin network
    • It is characterized by BRAF p.V600E mutation or other MAPK pathway gene alterations combined with homozygous CDKN2A / B deletion
    • Can be graded CNS WHO grade 2 or 3
    Essential features
    • Pleomorphic xanthoastrocytoma (PXA) is a circumscribed astrocytoma with superficial location often involving leptomeninges, most often located in temporal lobe
    • Classic histology with pleomorphic, xanthomatous and multinucleated tumor cells, often with eosinophilic granular bodies and rich reticulin network
    • It shows characteristic genetic alterations including BRAF p.V600E mutation and homozygous deletion of CDKN2A / B
    • Assigned CNS WHO grade 2 and 3
    Terminology
    • Not recommended: pleomorphic xanthoastrocytoma with anaplastic features; anaplastic pleomorphic xanthoastrocytoma for CNS WHO grade 3
    ICD coding
    • ICD-O: 9424/3 - pleomorphic xanthoastrocytoma
    • ICD-11: 2A00.0Y & XH99U2 - other specified gliomas of brain & pleomorphic xanthoastrocytoma
    Epidemiology
    • PXA accounts for < 0.3% of primary central nervous system (CNS) tumors, with an annual incidence of < 0.7 cases per 100,000 population (Neuro Oncol 2019;21:v1)
    • M = F; mean age at diagnosis is 26.3 years
    • It typically develops in children and young adults; however, older patients may be affected (J Neurooncol 2012;110:99)
    Sites
    Pathophysiology
    Etiology
    Clinical features
    • Many patients present with long history of epileptic seizures
    • Other symptoms that are related to raised intracranial pressure include headaches, nausea, vomiting, dizziness, diplopia and somnolence (CNS Oncol 2019;8:CNS39, Cureus 2023;15:e35975)
    Diagnosis
    • Magnetic resonance imaging (MRI) is the most effective imaging method
    • Diagnosis is confirmed by biopsy or surgical excision; gross total resection may not be achieved in deeply situated or widely infiltrative tumors
    • Diagnostic molecular DNA methylome profile
    • Next generation sequencing (NGS)
    • Chromosome microarray
    Radiology description
    • Computed tomography (CT) scan
      • Variable appearance (hypodense, hyperdense or mixed)
      • May be well or poorly demarcated
      • With strong and sometimes heterogenous contrast enhancement
      • Tumor cysts are usually hypodense
      • Calcification is rare
      • Due to superficial location, it may cause scalloping of overlying bone
    • MRI scan
      • Variable / heterogeneous appearance, often with cystic component
      • Solid component is either hypointense or isointense to gray matter on T1 weighted images
      • Hyperintense or mixed signal on T2 weighted and fluid attenuated inversion recovery (FLAIR) images
      • Cystic component is isointense to cerebrospinal fluid
      • Postcontrast enhancement is moderate to strong
      • Adjacent edema is not pronounced (Sci Rep 2018;8:14275)
    Radiology images

    Contributed by Shayan Anwar, M.B.B.S.
    Axial T2 weighted MRI

    Axial T2 weighted MRI

    Coronal FLAIR MRI

    Coronal FLAIR MRI

    Axial T1 weighted precontrast MRI

    Axial T1 weighted precontrast MRI

    Axial T1 weighted postcontrast MRI

    Axial T1 weighted postcontrast MRI

    Axial apparent diffusion coefficient (ADC)

    Axial apparent diffusion coefficient (ADC)

    Prognostic factors
    Case reports
    Treatment
    • For CNS WHO grade 2 PXAs, early intervention with complete surgical excision is treatment of choice and can be followed by wait and watch strategy (Lancet Oncol 2017;18:e315)
    • For CNS WHO grade 3 PXAs, postoperative radiotherapy may be offered in adults (Lancet Oncol 2017;18:e315)
    • Radiotherapy may play a role for residual or recurrent disease
    • Traditional systemic therapies have shown limited benefit in treating PXA
    • Targeted BRAF inhibition, alone or in combination with other treatments including MEK inhibition, has been associated with marked radiographic responses in BRAF mutated PXA (CNS Oncol 2019;8:CNS39)
    Frozen section description
    • Variable population of spindled to pleomorphic tumor cells with long and coarse cytoplasmic processes and fibrillary fragments with vessels
    • Multinucleated giant cells and tumor cells with microvacuolization resembling xanthomatous cells can be seen (Diagn Cytopathol 2017;45:339)
    • Sometimes eosinophilic granular bodies are observed
    Intraoperative frozen / smear cytology images

    Contributed by Mohammad Khurram Minhas, M.B.B.S.
    Glial tumor with pleomorphic cells Glial tumor with pleomorphic cells

    Glial tumor with pleomorphic cells

    Glial tumor exhibiting fibrillary processes Glial tumor exhibiting fibrillary processes

    Glial tumor exhibiting fibrillary processes

    Intranuclear inclusions

    Intranuclear inclusions

    Microscopic (histologic) description
    • Mostly solid, noninfiltrative growth pattern; however, microscopic infiltration can be seen at the periphery
    • Mixture of spindled, epithelioid, pleomorphic and multinucleated tumor cells, with some showing vacuolated cytoplasm (xanthomatous cells)
    • Intranuclear inclusions, prominent nucleoli and perivascular lymphocytic infiltration are common (Cancer 1999;85:2033)
    • Frequent eosinophilic granular bodies and dense reticulin deposition
    • CNS WHO grading
      • CNS WHO grade 2 is assigned to tumors with < 5 mitoses/10 high power fields (HPF)
      • Necrosis is commonly seen in tumors with high mitotic activity; however, its significance in isolation is unclear
      • Microvascular proliferation is uncommon (Cancer 1999;85:2033)
      • CNS WHO grade 3 PXAs may demonstrate less pleomorphism and more diffusely infiltrative pattern
      • In CNS WHO grade 3 tumors, a mean Ki67 labeling index of 15% has been reported, whereas it is generally < 1% in CNS WHO grade 2 tumors
    • Common cytological patterns of anaplasia include monomorphic small cells, fibrillary morphology, epithelioid and rhabdoid morphology (Brain Pathol 2018;28:172)
    Microscopic (histologic) images

    Contributed by Aisha Memon, M.B.B.S.
    Pleomorphic spindle cells

    Pleomorphic spindle cells

    Epithelioid tumor cells

    Epithelioid tumor cells

    Eosinophilic granular bodies

    Eosinophilic granular bodies

    Rosenthal fibers

    Rosenthal fibers


    Xanthomatous cells Xanthomatous cells

    Xanthomatous cells

    Multinucleated tumor cells Multinucleated tumor cells

    Multinucleated tumor cells


    GFAP

    GFAP

    CD34

    CD34

    S100

    S100

    Neurofilament

    Neurofilament

    Reticulin stain

    Reticulin stain

    Virtual slides

    Images hosted on other servers:
    Pleomorphic xanthoastrocytoma

    Pleomorphic xanthoastrocytoma

    Positive stains
    Negative stains
    Electron microscopy description
    • Presence of relatively few organelles, abundant filaments and microtubules, are characteristic of astrocytes
    • Existence of basal laminas in the extracellular space surrounding the surface of the tumor cells and the presence of localized cytoplasmic densities associated with the plasma membrane adjacent to the basal lamina resembling hemidesmosomes (Cancer 1983;52:2055)
    Molecular / cytogenetics description
    Videos

    Pleomorphic xanthoastrocytoma:
    a brief review

    Sample pathology report
    • Temporal lobe lesion, gross total resection:
      • Pleomorphic xanthoastrocytoma, BRAF V600E mutant, CNS WHO grade 2
      • Histological classification: pleomorphic xanthoastrocytoma
      • CNS WHO grade: 2
      • Molecular information: BRAF V600E mutation, combined with homozygous deletion of CDKN2A / B
    Differential diagnosis
    • Ganglioglioma:
      • Both show presence of eosinophilic granular bodies, lymphocytic infiltration, CD34 expression and BRAF p.V600E mutation
      • Presence of true ganglion cells in gangliogliomas and absence of xanthomatous cells
      • Absence of reticulin rich network surrounding individual tumor cells in gangliogliomas
      • Absence of homozygous deletion of CDKN2A / B in gangliogliomas
    • Giant cell glioblastoma:
      • Both show gross circumscription, reticulin deposition, marked pleomorphism, multinucleated giant cells and lymphocytic infiltration
      • Presence of neuronal antigens in PXAs, which are usually absent in giant cell glioblastoma
      • TP53 is mutated in most giant cell glioblastomas (Arch Pathol Lab Med 2003;127:1187)
    • Epithelioid glioblastoma:
      • Both can show presence of epithelioid cells, frank anaplasia and BRAF p.V600E mutation
      • Prior history of a CNS WHO grade 2 PXA can be helpful
      • There is a PXA-like epithelioid glioblastoma subset, which clusters by methylation profiling with canonical PXAs with BRAF p.V600E mutation (79%), CDKN2A homozygous deletion (61%) and TERT promoter mutations (30%) (Brain Pathol 2018;28:656)
    Board review style question #1

    A 25 year old man presented with history of recurrent headaches and seizures. Magnetic resonance imaging (MRI) scan revealed a solid circumscribed lesion in the temporal cortex with abnormal intensity. Biopsy revealed pleomorphic tumor cells with admixed xanthomatous cells and multinucleated giant cells. Eosinophilic granular bodies were noted along with reticulin rich network. There was no visible mitosis, microvascular proliferation or necrosis. On molecular testing, there was a BRAF p.V600E mutation. What is the most likely diagnosis?

    1. Epithelioid glioblastoma
    2. Ganglioglioma
    3. Giant cell glioblastoma
    4. Pleomorphic xanthoastrocytoma
    5. Subependymal giant cell astrocytoma
    Board review style answer #1
    D. Pleomorphic xanthoastrocytoma. The clinical scenario describes characteristic radiological, histological and genetic profile of pleomorphic xanthoastrocytoma. Answers A and C are incorrect because BRAF p.V600E mutation is reported in as many as 50% of glioblastoma with epithelioid morphology and rarely in giant cell glioblastoma; however, these show mitoses (including atypical mitoses) or microvascular proliferation and necrosis on histology. Answer B is incorrect because ganglioglioma also harbors BRAF p.V600E mutation but has distinctive morphology. Answer E is incorrect because subependymal giant cell astrocytoma (SEGA) may rarely show BRAF p.V600E mutation but has characteristic radiological and morphological features.

    Comment Here

    Reference: Pleomorphic xanthoastrocytoma
    Board review style question #2
    A 20 year old woman presented with history of epileptic seizures. Magnetic resonance imaging (MRI) scan revealed a solid circumscribed lesion in the temporal cortex with abnormal intensity. Biopsy revealed characteristic histological features of pleomorphic xanthoastrocytoma including pleomorphic tumor cells with admixed xanthomatous cells, multinucleated giant cells and eosinophilic granular bodies. What is the most frequently mutated gene?

    1. BRAF V600E
    2. EGFR
    3. Histone H3
    4. IDH1
    5. TERT promoter
    Board review style answer #2
    A. BRAF V600E. The most frequently found mutated gene in PXAs is BRAF, which encodes an intracellular component of the MAPK pathway. Answers B and E are incorrect because EGFR and TERT promoter mutations are rare in low grade gliomas. These are characteristically seen in high grade astrocytomas. Answer D is incorrect because IDH1 is characteristically mutated in adult type diffuse gliomas. Answer C is incorrect because H3 K27M is altered in diffuse midline gliomas.

    Comment Here

    Reference: Pleomorphic xanthoastrocytoma

    Polymorphous low grade neuroepithelial tumor of the young (PLNTY)
    Definition / general
    • Indolent cerebral neoplasm characterized by a strong association with seizures in young individuals, diffuse growth patterns, frequent presence of oligodendroglioma-like components, calcifications, CD34 immunoreactivity and MAPK pathway activating genetic abnormalities
    • CNS WHO grade 1
    • Recently introduced in WHO 2021 classification of CNS tumors
    Essential features
    • Recently described, frequently epileptogenic, low grade neuroepithelial tumor (Acta Neuropathol 2017;133:417)
    • Typically affects children and young adults
    • Characterized by diffuse growth patterns, oligodendroglioma-like cellular components, CD34 immunopositivity and MAPK pathway activating genetic abnormalities
    • Generally, exhibits a benign clinical course (Acta Neuropathol 2017;133:417)
    Terminology
    • Has been described under the generic designation of long term epilepsy associated tumors (Acta Neuropathol 2014;128:39)
    • Not recommended by WHO
      • Diffuse glioneuronal tumor
      • Diffuse or nonspecific form of dysembryoplastic neuroepithelial tumor
      • Massively calcified low grade glioma
    ICD coding
    • ICD-O: 9413/0 - polymorphous low grade neuroepithelial tumor of the young
    • ICD-11: 2A00.2Y - other specified tumors of neuroepithelial tissue of brain
    Epidemiology
    Sites
    Pathophysiology
    • Aberrant CD34 expression possibly reflects an origin from developmentally dysregulated neural precursors (Acta Neuropathol 1999;97:481)
    • Somatic MAPK pathway activating genetic abnormalities (particularly BRAF mutations or FGFR fusions) definitely drive the pathogenesis of PLNTY; however, the specific mechanisms by which these genetic alterations contribute to the development of PLNTY are not clear (Acta Neuropathol 2017;133:417)
    Etiology
    Diagrams / tables

    Images hosted on other servers:
    Prototypical PLNTY

    Prototypical PLNTY

    Clinical features
    Diagnosis
    • Neuroimaging: MRI and CT (Acta Neurol Belg 2023;123:327)
    • Biopsy
    • WHO essential and desirable diagnostic criteria
      • Essential diagnostic criteria (presence of the following 6 characteristics)
        • Diffuse growth pattern (at least regionally)
        • Oligodendroglioma-like components (may be minor)
        • Few (if any) mitotic figures
        • Regional CD34 expression by tumor cells and by ramified neural cells in associated cerebral cortex
        • IDH wild type status
        • Unequivocal expression of BRAF p.V600E on immunohistochemical assessment or molecular diagnostic evidence of BRAF V600E mutations, FGFR2 or FGFR3 fusions or potentially other MAPK pathway driving genetic abnormalities
      • Desirable diagnostic criteria
        • Conspicuous calcification (characteristic, although not constant)
        • Absence of 1p / 19q codeletion
    Radiology description
    Radiology images

    Images hosted on other servers:
    FLAIR, CT scan and T1 postcontrast

    FLAIR, CT scan and T1 postcontrast

    Radiological variability in CT and MRI

    Radiological variability in CT and MRI

    Prognostic factors
    Case reports
    Treatment
    Clinical images

    Images hosted on other servers:
    Operative view of PLNTY

    Operative view

    Gross description
    • Unencapsulated, indistinctly demarcated from normal brain
    • Soft to friable, gray-white masses
    Microscopic (histologic) description
    Microscopic (histologic) images

    Contributed by Eman Abdelzaher, M.D., Ph.D. and Jared T. Ahrendsen, M.D., Ph.D.
    Diffuse growth pattern

    Diffuse growth pattern

    Diffuse growth pattern of oligodendroglioma-like cells Diffuse growth pattern of oligodendroglioma-like cells

    Diffuse growth pattern of oligodendroglioma-like cells

    Oligodendroglioma-like cells with mild nuclear pleomorphism

    Oligodendroglioma-like cells with mild nuclear pleomorphism

    Calcification

    Calcification


    Oligo-like features

    Oligo-like features

    Dystrophic calcifications

    Dystrophic calcifications

    Polymorphic tumor cells Polymorphic tumor cells

    Polymorphic tumor cells

    Olig2 diffusely positive

    Olig2 diffusely positive

    Strong diffuse CD34 expression

    Strong, diffuse CD34 expression


    CD34 CD34

    CD34

    CD34 in tumor cells and ramified neural cells

    CD34 in tumor cells and ramified neural cells

    GFAP GFAP

    GFAP

    Ki67

    Ki67

    Cytology description
    • PLNTY cannot be identified on cytological grounds alone
    • Intraoperative smear preparations may show
      • Tumor cells with rounded nuclei and devoid of cytoplasm
      • Tumor cells with irregular nuclear contours and cytoplasmic processes
      • Tumor cells attached to blood vessels in a pseudorosette fashion (Acta Neuropathol 2017;133:417)
    Cytology images

    Images hosted on other servers:
    Vague perivascular pseudorosetting

    Vague perivascular pseudorosetting

    Positive stains
    Molecular / cytogenetics description
    Molecular / cytogenetics images

    Images hosted on other servers:
    Molecular and clinical findings

    Molecular and clinical findings

    FGFR fusions in PLNTY

    FGFR fusions

    Distinct methylation signature

    Distinct methylation signature

    Genome wide copy number changes

    Genome wide copy number changes

    Videos

    PLNTY, case 7

    Surgical resection of PLNTY
    (Acta Neurol Belg 2023;123:327)

    Sample pathology report
    • Brain mass lesion, temporal lobe, gross total resection:
      • Polymorphous low grade neuroepithelial tumor of the young (PLNTY), CNS WHO grade 1
      • Molecular genetics: evidence of BRAF V600E mutations, FGFR2 or FGFR3 fusions or potentially other MAPK pathway driving genetic abnormalities
    Differential diagnosis
    Board review style question #1

    A 14 year old boy presented with refractory epilepsy. MRI showed a right temporal cystic and solid nonenhancing mass lesion. The mass was surgically resected and histopathological examination showed a low grade tumor composed of oligodendroglioma-like tumor cells arranged in a diffuse fashion as shown in the image above. Immunohistochemistry for CD34 showed regional positivity associated with positive ramified neurons in the adjacent cortex. Which of the following molecular events is characteristic of this tumor?

    1. 1p / 19q codeletion
    2. IDH mutation
    3. MAPK pathway activating abnormalities
    4. PRKCA gene fusion
    Board review style answer #1
    C. MAPK pathway activating abnormalities. PLNTY is characterized by a strong association with seizures in young individuals, diffuse growth patterns, frequent presence of oligodendroglioma-like components and CD34 immunoreactivity. It is molecularly characterized by MAPK pathway activating genetic abnormalities. Answers A and B are incorrect because PLNTY does not harbor IDH mutations or 1p / 19q codeletion. Answer D is incorrect because PRKCA gene fusion (mainly SLC44A1::PRKCA) is the hallmark of papillary glioneuronal tumor (PGNT).

    Comment Here

    Reference: Polymorphous low grade neuroepithelial tumor of the young (PLNTY)
    Board review style question #2
    Polymorphous low grade neuroepithelial tumor of the young (PLNTY) is characterized by which of the following features?

    1. Absence of calcification
    2. CD34 negative immunostaining
    3. Clinical presentation of headache
    4. Oligodendroglioma-like components
    Board review style answer #2
    D. Oligodendroglioma-like components. PLNTY is characterized by frequent presence of oligodendroglioma-like components. Answer A is incorrect because calcification is a frequent finding in PLNTY. Answer B is incorrect because CD34 immunoreactivity is a characteristic finding in PLNTY. Answer C is incorrect because PLNTY is characterized by a strong association with seizures in young individuals.

    Comment Here

    Reference: Polymorphous low grade neuroepithelial tumor of the young (PLNTY)

    Primary CNS lymphoma
    Definition / general
    • Primary central nervous system (CNS) lymphoma occurs first in the brain, eye, spinal cord or leptomeninges, without systemic involvement and not arising in the setting of immunodeficiency
    • Majority are high grade diffuse large B cell lymphomas, with only rare cases representing T cell lymphomas, low grade B cell lymphomas or Burkitt lymphoma
    • This topic addresses primary diffuse large B cell lymphoma of the CNS and does not include cases that are associated with immunodeficiency (see separate category of immunodeficiency associated lymphoproliferative disorders)
    Essential features
    • Rare aggressive large B cell lymphoma occurs predominantly in elderly individuals, with an overall worse prognosis than systemic diffuse large B cell lymphoma
    • Composed of large, atypical basophilic lymphocytes with perivascular cuffing and frequent necrosis
    • Positive for CD45 and B cell markers and typically positive for BCL6, MUM1 and BCL2
    • Unlike CNS lymphomas associated with HIV infection, primary CNS lymphoma is not associated with Epstein-Barr virus infection
    Terminology
    • Previously known as reticulum cell sarcoma, diffuse histiocytic lymphoma or lymphomatosis cerebri (no longer recommended)
    • Primary CNS lymphoma is used interchangeably with primary diffuse large B cell lymphoma of the CNS
    • Excluded from this category are immunodeficiency associated lymphomas (see separate immunodeficiency associated topic)
    ICD coding
    • ICD-10: C83.8 - other nonfollicular lymphoma
    • ICD-10: C72.9 - malignant neoplasm of central nervous system, unspecified
    Epidemiology
    • Accounts for < 1% of all non-Hodgkin lymphoma cases and 2 - 3% of all CNS tumors (Cancer 2017;123:4314)
    • Overall incidence rate is < 1 cases per 100,000 population
    • Rates among men and women aged 65+ years have increased in recent decades (1992 - 2011), although the reason for the increase is unclear
    • Among immunocompetent individuals (HIV uninfected, nontransplant recipients), the median age at diagnosis among a U.S. population is 67 (range 52 - 77 years) with a similar male to female ratio (Br J Haematol 2016;174:417)
    Sites
    Pathophysiology
    • Not definitively understood
    • Central nervous system is an immune privileged site and the lymphoma itself may have genetic and phenotypic features that protect the tumor from ongoing immune surveillance (Virchows Arch 2020;476:647)
    • Neoplastic cells may arise from lymphocytes normally residing in the immune privileged milieu of the CNS or from lymphocytes that transform to a malignant clone outside the CNS and then home to the CNS and escape normal immune regulation in the immune privileged site (Virchows Arch 2020;476:647, Curr Oncol Rep 2015;17:60)
    Etiology
    • Unknown in immunocompetent patients
    • Viruses, including Epstein-Barr virus, do not play a role
    • Rising rates in elderly individuals (HIV uninfected, nontransplant recipients) may be related to increasing use of immunosuppressive medications to treat autoimmune conditions in the elderly (Br J Cancer 2011;105:1414)
    Clinical features
    • Symptoms vary depending on the CNS site involved and commonly include cognitive or behavioral changes, focal neurologic deficits and symptoms of increased intracranial pressure (Cancer 2017;123:4314)
    • Seizures are relatively uncommon and patients typically do not present with B symptoms (fever, night sweats, weight loss)
    • Ocular manifestations may present as decreased acuity, blurry vision or floaters (Hematol Oncol Clin North Am 2019;33:597)
    Diagnosis
    • Contrast enhanced MRI of the brain is the neuroimaging modality of choice for diagnosis; however, the diagnosis must be confirmed by review of sampled tumor tissue or fluid (Br J Haematol 2019;184:348)
    • Stereotactic needle biopsy is the gold standard for diagnosis and classification of CNS lymphoma (Neurooncol Pract 2019;6:415)
    • Cerebrospinal fluid (CSF) cytology and flow cytometry may be used when biopsy is not possible or to evaluate for leptomeningeal involvement
    Radiology description
    • On MRI, there is an isointense to hypointense lesion on T1 weighted images with homogenous gadolinium contrast enhancement
    • Lesions are usually solitary, involving the deep white matter and periventricular in location
    • In immunocompetent individuals, the lesions typically do not show hemorrhage, calcifications, necrosis or ring enhancement (Hematol Oncol Clin North Am 2019;33:597)
    Radiology images

    Images hosted on other servers:
    Brain MRI with contrast enhancing lesion

    Brain MRI with contrast enhancing lesion

    Solid and enhancing nodules on MRI

    Prognostic factors
    • Overall poor prognosis with considerably worse prognosis than systemic diffuse large B cell lymphoma
    • Age and performance status are the most important prognostic factors
    • In the Memorial Sloan Kettering Cancer Center prognostic model, only age and performance status were independent prognostic factors
    • The prognostic model from the International Extranodal Lymphoma Study Group includes age and performance status as well as serum lactate dehydrogenase level, CSF protein concentration and involvement of deep structures of the brain (Cancer 2017;123:4314)
    • In patients treated with methotrexate based regimens, BCL6 expression may be a predictor of improved progression free survival (Br J Haematol 2014;165:640)
    Case reports
    Treatment
    • High dose intravenous methotrexate based regimen is the first line induction therapy
    • Consolidation therapy may include conventional chemotherapy, radiotherapy, high dose chemotherapy with autologous stem cell transplantation or their combinations (Transl Oncol 2019;12:523)
    Gross description
    • From postmortem examination, tumors have a variable macroscopic appearance, including variable demarcation from the surrounding parenchyma
    • Lesions may be firm, homogenous, centrally necrotic, brownish, gray-tan or yellow with areas of hemorrhage (Clin Neuropathol 2008;27:13)
    Gross images

    Images hosted on other servers:

    Cerebral lymphoma
    spreading across
    the corpus callosum

    Frozen section description
    • Histologic features seen on frozen section are similar to those seen on permanent
    • Intraoperative cytology is an important component of intraoperative evaluation (Neuropathology 2016;36:313)
    Intraoperative frozen / smear cytology images

    Contributed by Genevieve M. Crane, M.D., Ph.D. and Lena Young, D.O.

    Squash preparation

    Frozen section

    Frozen section control

    Touch prep of
    monomorphic
    basophilic cells with
    crush artifact

    Touch prep

    Intraoperative squash preparation

    Microscopic (histologic) description
    • Characteristic angiocentric growth pattern with cuffs of neoplastic lymphocytes within and around blood vessels (Clin Neuropathol 2008;27:13)
    • Diffuse infiltration of brain parenchyma by small clusters or individual cells; necrosis is common
    • Accompanied by astrocytic and microglial activation and may be accompanied by a reactive inflammatory infiltrate
    • Neoplastic B cells are intermediate to large and frequently have vesicular chromatin and prominent nucleoli
    • Corticosteroid administration prior to biopsy may induce dramatic tumor shrinkage with increased macrophages and cellular debris
    Microscopic (histologic) images

    Contributed by Elizabeth Courville, M.D. and Lena Young, D.O.

    Perivascular infiltrate of atypical lymphoid cells

    Diffuse infiltrate
    of intermediate
    to large
    mononuclear cells

    Cytology of mononuclear cells

    Perivascular cuffing by neoplastic lymphocytes

    Infiltrate of neoplastic intermediate to large lymphocytes with background inflammation and necrosis


    Pleomorphic lymphocytes infiltrating brain

    Immunohisto-
    chemistry for B and T cells

    ISH shows lambda restriction

    Cytology description
    • Neoplastic lymphocytes in the cerebrospinal fluid are dispersed cells, increased in size compared to background reactive lymphocytes and frequently show irregular nuclear contours, abnormal chromatin and prominent nucleoli
    • Mostly discohesive groups of monomorphic, medium to large basophilic lymphoid cells with prominent smearing artifact and background reactive glial cells
    • Touch preparations show discohesive, large, atypical basophilic cells with a high nucleus to cytoplasm ratio and open chromatin with immature (lymphoblast) features; in post treatment tumors, only necrotic cells or reactive T cells may remain
    • Reference: Asian J Neurosurg 2013;8:195
    Negative stains
    • CD38, CD138 (plasma cell markers)
    • CD10 (< 10% are positive); CD10 positivity is seen more commonly in systemic diffuse large B cell lymphoma so should prompt careful search for systemic disease (Hematol Oncol 2014;32:57)
    • EBV positivity suggests underlying immunodeficiency
    • Pertinent negative stains include CD30, ALK1, HHV8, as they are helpful in the evaluation of mimicker neoplasms including systemic large B cell lymphoma, anaplastic large cell lymphoma and HHV8 associated lymphoproliferative disorders respectively
    Flow cytometry description
    • Specimen is not always sent for flow cytometry from stereotactic needle core biopsies due to limited material; flow cytometry of cerebrospinal fluid can aid in the diagnosis when needle core biopsy is not possible or to evaluate for leptomeningeal involvement
    • Flow cytometric immunophenotyping can identify abnormal B cell populations showing light chain restriction
    • References: J Exp Clin Cancer Res 2016;35:128, Cytometry B Clin Cytom 2018;94:928
    Molecular / cytogenetics description
    • MYC or BCL2 gene rearrangements are rare; BCL6 gene rearrangements are frequent (Blood Adv 2019;3:3953)
    • Oncogenic gain of function mutations in MYD88 (MYD88 L265P) are frequent (Blood 2016;127:869)
    Sample pathology report
    • Brain, biopsy:
      • Diffuse large B cell lymphoma (see comment)
      • Comment: In the absence of systemic disease, the morphologic and immunophenotypic findings are consistent with a diagnosis of primary diffuse large B cell lymphoma of the central nervous system. Clinical correlation is required, such as excluding an immunodeficiency associated lymphoproliferative disorder.
    Differential diagnosis
    • Systemic malignant lymphoma (diffuse large B cell lymphoma or other) involving the CNS:
      • CD10 expression by the lymphoma should strongly raise the possibility
      • Prior history and clinical workup is required to exclude systemic lymphoma involving the CNS
    • Lymphoma associated with HIV infection:
      • EBV positivity suggests this
      • Clinical workup is required
    • Posttransplant lymphoproliferative disorder (PTLD):
      • EBV positive
      • History of immunosuppression after allograft
    • High grade glioma:
      • Lacks the diffuse monomorphic lymphocyte infiltrate, neuronal morphology and IHC staining
      • CD45 negative
    • Infection including toxoplasma or cerebral abscess:
      • Occasionally multiloculate enhancement, generally encapsulated
      • Suppurative with mixed acute and chronic inflammation
    • Metastatic melanoma or carcinoma:
      • Cohesive groups of malignant cells that incite desmoplasia, CD45 negative, keratin or S100 positive, clinical correlation or history of malignancy
    • Demyelinating lesions:
      • May not enhance on imaging
      • Generally demarcated
      • Histiocytic as opposed to lymphoid infiltrate
    • Cerebral infarct:
      • Ischemic, "red dead" neurons
      • Slight to no lymphoid infiltrate
      • Histiocytes predominate
    Board review style question #1

    A 73 year old woman presents with progressive confusion and lethargy. Imaging shows a solitary lesion in the parietal lobe. A brain biopsy is performed and intraoperative touch preparations show discohesive, monomorphic, basophilic cells with crush artifact. Permanent sections show perivascular cuffing by intermediate to large mononuclear cells with scant cytoplasm, vesicular chromatin and prominent nucleoli. By immunohistochemical stains, the mononuclear cells are positive for CD20, BCL6 and MUM1 and negative for CD10 and EBV. What is the most likely diagnosis?

    1. Glioblastoma multiforme (GBM)
    2. Metastatic melanoma
    3. Metastatic neuroendocrine carcinoma
    4. Primary central nervous system lymphoma
    Board review style answer #1
    D. Primary central nervous system lymphoma. Based on the provided description, primary CNS lymphoma is the most likely diagnosis. This entity stains with B cells markers and is typically positive for MUM1 and negative for CD10. The entity primary CNS lymphoma excludes lymphomas associated with HIV infection or lymphomas arising after transplant. In contrast to the CNS lymphomas in those clinical scenarios, primary CNS lymphoma is almost always EBV negative.

    Comment Here

    Reference: Primary CNS lymphoma
    Board review style question #2
    Which of the following immunohistochemical stains is typically negative in the neoplastic lymphocytes in primary central nervous system (CNS) lymphoma?

    1. BCL6
    2. CD10
    3. CD20
    4. IRF4 / MUM1
    Board review style answer #2
    B. CD10. Less than 10% of cases of primary CNS lymphoma are positive for CD10. CD10 positivity is seen more commonly in systemic diffuse large B cell lymphoma so should prompt careful search for systemic disease. The other stains listed are typically positive in primary CNS lymphoma.

    Comment Here

    Reference: Primary CNS lymphoma

    Primary intracranial sarcoma, DICER1 mutant (pending)
    [Pending]

    Rathke cleft cyst
    Definition / general
    • Mucus containing cyst arising from Rathke cleft remnants
    • Intrasellar or suprasellar
    • Incidental postmortem finding
    • Larger cysts, usually in adults, may be symptomatic
    Gross description
    • Thin walled, mucinous contents
    Microscopic (histologic) description
    • Lined by columnar ciliated epithelium with goblet cells
    • Variable fragments of pituitary tissue, squamous metaplasia, xanthogranulomatous reaction
    Positive stains
    Differential diagnosis

    Rhabdoid meningioma
    Definition / general
    • WHO grade 3 due to aggressive behavior and metastatic potential
    • High rate of recurrence
    • Rhabdoid morphology may increase with recurrences
    Sites
    • May be supratentorial, infratentorial or spinal
    Radiology description
    • MRI may show irregular tumor borders, cystic component, heterogenous tumoral enhancement and peritumoral edema suggesting a high grade meningioma (AJNR Am J Neuroradiol 2007;28:1462)
    Radiology images

    Images hosted on other servers:

    Marked enhancing mass at left frontal lobe

    Hyperostosis, bony erosion in right anterior clinoid process

    Tumor in right infratemporal fossa

    Lesion was isointense on T1 and T2 weighted images

    Avidly enhancing, complex falcine meningioma

    Case reports
    Treatment
    Gross description
    • Dural based
    • May be well circumscribed or readily adherent to brain parenchyma
    • Size can vary
    Microscopic (histologic) description
    • Sheets of loosely cohesive cells with eccentric nuclei and hyaline, paranuclear inclusions
    • Nuclei may show pleomorphism, with vesicular chromatin and prominent nucleoli
    • May retain meningothelial features including whorl formation and nuclear pseudoinclusions
    • May have brain invasion
    • Often increased mitotic activity (greater than 4/10 HP)
    • May have papillary features
    Cytology description
    • Rhabdoid cells with abundant eosinophilic cytoplasm, cytoplasmic hyaline inclusions, eccentric nuclei
    • Stout processes consistent with meningioma; however distinct cell borders
    • Often vesicular nuclei with prominent nucleoli
    • May see mitoses on smear
    • May see pseudonuclear inclusions (Diagn Cytopathol 2010;38:594)
    Cytology images

    Images hosted on other servers:

    Squash prep 40x

    Positive stains
    Negative stains
    Electron microscopy description
    • Cytoplasmic whorls of intermediate filaments often entrapping lysosomes or other organelles
    • Interdigitating cell membranes and intercellular junctions

    Rosette forming glioneuronal tumor (RGNT)
    Definition / general
    Essential features
    • Rare, slow growing glioneuronal neoplasm (Neurooncol Adv 2020;2:vdaa116)
    • Preferentially affects young adults, adolescents and children
    • Occurs throughout CNS with predilection for fourth ventricle region
    • 2 distinct histological components: neurocytic with distinctive rosettes and glial areas
    • Recently, rosette forming glioneuronal tumors (RGNTs) with malignant behavior have been reported (Oncotarget 2017;8:109175)
    Terminology
    ICD coding
    • ICD-O: 9509/1 - rosette forming glioneuronal tumor
    • ICD-11: 2A00.21 & XH2JU8 - mixed neuronal glial tumors & rosette forming glioneuronal tumor
    Epidemiology
    • Rare; specific population based incidence rates are not available
    • Preferentially affects young adults, adolescents and children; mean age at diagnosis is 28 years (range: 2 - 81 years) (Oncotarget 2017;8:109175)
    • No gender predilection (Oncotarget 2017;8:109175)
    Sites
    Pathophysiology
    Etiology
    Clinical features
    Diagnosis
    • Neuroimaging: magnetic resonance imaging (MRI) is the preferred modality, computed tomography (CT) (Br J Neurosurg 2012;26:668)
    • Biopsy
    • WHO essential and desirable diagnostic criteria
      • Essential diagnostic criteria (presence of the following 3 characteristics)
        • Biphasic histomorphology with a neurocytic component and a glial component
        • Uniform neurocytes forming rosettes or perivascular pseudorosettes associated with synaptophysin expression
        • (For unresolved lesions) small biopsies showing only 1 tumor component (neurocytic or glial) and a methylation profile of rosette forming glioneuronal tumor
      • Desirable diagnostic criteria
        • FGFR1 mutation with co-occurring PIK3CA or NF1 mutation
    Radiology description
    • CT (Oncotarget 2017;8:109175)
      • Hypodense midline mass
      • Calcification (< 25%)
    • MRI (Oncotarget 2017;8:109175)
      • Relatively circumscribed midline mass, solid or cystic - solid pattern (47%), cystic pattern (35%) and mixed cystic solid pattern (18%)
      • T1 hypointense, T2 hyperintense
      • Variable contrast enhancement
        • Heterogeneous (44.7%), rim (23.7%) or focal (7.9%) enhancement
        • No enhancement (25%)
    Radiology images

    Contributed by Eman Abdelzaher, M.D., Ph.D.
    CT without contrast

    CT without contrast

    CT with contrast

    CT with contrast



    Images hosted on other servers:
    Cerebellar and fourth ventricular RGNT

    Cerebellar and fourth ventricular RGNT

    MRI of cerebellar vermis RGNT

    MRI of cerebellar vermis RGNT

    Third ventricle RGNT

    Third ventricle RGNT

    MRI of aqueduct and thalamic RGNT

    MRI of aqueduct and thalamic RGNT

    Postcontrast T1 weighted MRI image

    Postcontrast T1 weighted MRI

    MRI of pineal region and third ventricle RGNT

    MRI of pineal region and third ventricle RGNT

    Prognostic factors
    Case reports
    Treatment
    Clinical images

    Images hosted on other servers:
    Right frontal exophytic lesion

    Right frontal exophytic lesion

    Spinal RGNT, intraoperative picture

    Spinal RGNT, intraoperative picture

    Gross description
    • Well demarcated, soft, gelatinous
    Frozen section description
    Frozen section images

    Images hosted on other servers:
    Circle‐like arrangement of the small and uniform nuclei

    Moderately cellular and hypocellular areas

    Microscopic (histologic) description
    • Circumscribed, may show limited infiltration of surrounding parenchyma
    • Biphasic with 2 distinct histological components: neurocytic and glial
    • Neurocytic component
      • Consists of uniform neurocytes forming neurocytic rosettes or perivascular pseudorosettes (Clin Case Rep 2021;9:e04355)
      • Neurocytes have small round nuclei with fine stippled chromatin and scant cytoplasm
      • Neurocytic rosettes are small with ring-like arrangement of neurocytes around delicate eosinophilic neuropil cores; may be arrayed in a cribriform pattern (Brain Pathol 2007;17:308)
      • Perivascular pseudorosettes are narrow and formed of neurocytes with delicate cell processes radiating towards central vessels
      • Neurocytic elements may lie unanchored in partly microcystic, mucinous matrix and, when sectioned longitudinally, may show a linear arrangement (Brain Pathol 2007;17:308)
    • Glial component
      • Typically, dominant component and low grade
      • Resembles pilocytic astrocytoma with piloid and oligodendroglia-like cells
      • Astrocytic tumor cells are spindle to stellate in shape, with elongated or oval nuclei and cytoplasmic processes forming a compact to loosely textured fibrillary background (Acta Neuropathol 2019;138:497)
      • May show oligodendroglia-like cells with perinuclear haloes arranged in microcystic pattern or diffuse sheets (Brain Pathol 2007;17:308)
    • Mitoses and necrosis absent or rare, atypia absent or minimal (Oncotarget 2017;8:109175, Acta Neuropathol 2019;138:497)
    • Rosenthal fibers, eosinophilic granular bodies (EGBs), microcalcifications and hemosiderin deposits may be seen (Brain Pathol 2007;17:308, Acta Neuropathol 2019;138:497)
    • Blood vessels may be ectatic, hyalinized, thrombosed or show glomeruloid microvascular proliferation (Oncotarget 2017;8:109175)
    • Rarely, dysmorphic neurons may be seen in the glial regions (Brain Pathol 2007;17:308)
    Microscopic (histologic) images

    Contributed by Eman Abdelzaher, M.D., Ph.D.
    Biphasic neurocytic and glial components

    Biphasic neurocytic and glial components

    Biphasic neurocytic and pilocytic components

    Biphasic neurocytic and pilocytic components

    Neurocytic rosettes Neurocytic rosettesr

    Neurocytic rosettes

    Neurocytes with stippled chromatin

    Neurocytes with stippled chromatin


    Neurocytic rosettes and pseudorosettes Neurocytic rosettes and pseudorosettes

    Neurocytic rosettes and pseudorosettes

    Cribriform pattern Cribriform pattern

    Cribriform pattern

    Microcystic pattern

    Microcystic pattern


    Microcystic pattern

    Microcystic pattern

    Pilocytic-like area

    Pilocytic-like area

    Piloid and oligodendroglia-like cells

    Piloid and oligodendroglia-like cells


    Synaptophysin Synaptophysin

    Synaptophysin

    Synaptophysin Synaptophysin

    Synaptophysin

    Neurocytic cells of RGNT are negative for GFAP

    Negative neurocytes by GFAP


    Glial fibrillary acidic protein (GFAP) Glial fibrillary acidic protein (GFAP)

    Glial fibrillary acidic protein (GFAP)

    Epithelial membrane antigen (EMA) RGNT showing negative immunostaining for EMA

    Epithelial membrane antigen (EMA)

    Ki67

    Ki67

    Cytology description
    • Cellular smears
    • Neurocytic cells have uniform round nuclei with delicate chromatin, inconspicuous nucleoli and scant cytoplasm; neurocytic rosettes with fibrillar cores may be seen (Acta Cytol 2021;65:111)
    • Glial, fibrillary fragments showing piloid astrocytes with elongated nuclei and coarse bipolar processes
    • Dense extracellular mucoid substance
    • No atypia
    Cytology images

    Images hosted on other servers:
    Neurocytic rosettes

    Neurocytic rosettes

    Positive stains
    Electron microscopy description
    Molecular / cytogenetics description
    Molecular / cytogenetics images

    Images hosted on other servers:
    RGNT, DNA methylation profiling

    RGNT, DNA methylation profiling

    RGNT, recurrent genetic alterations

    RGNT, recurrent genetic alterations

    Videos

    WHO classification of CNS tumors: glioneuronal tumors

    Sample pathology report
    • Brain mass lesion, fourth ventricle, gross total resection:
      • Rosette forming glioneuronal tumor, CNS WHO grade 1
      • Molecular genetics: methylation profile of rosette forming glioneuronal tumor
    Differential diagnosis
    Board review style question #1

    A 15 year old girl presented with a headache. MRI showed a posterior fossa mass lesion epicentered on the cerebellar vermis. Resection of the tumor was done and the histopathological features of the tumor are shown above. Which of the following is a typical genetic finding in this tumor?

    1. 1p / 19q codeletion
    2. BRAF alterations
    3. FGFR1 mutations
    4. PRKCA gene fusion
    Board review style answer #1
    C. FGFR1 mutations. FGFR1 hotspot mutations are typical of rosette forming glioneuronal tumors (RGNTs). Answer B is incorrect because RGNTs lack BRAF alterations. Answer D is incorrect because PRKCA gene fusion (mainly SLC44A1::PRKCA) is the hallmark of papillary glioneuronal tumors (PGNTs). Answer A is incorrect because 1p / 19q codeletion is characteristic of oligodendroglioma.

    Comment Here

    Reference: Rosette forming glioneuronal tumor
    Board review style question #2
    Rosette forming glioneuronal tumor is characterized by which of the following features?

    1. Diffuse growth pattern
    2. Exclusive supratentorial location
    3. Neurocytic rosettes with synaptophysin positive neuropil cores
    4. Papillary architecture
    Board review style answer #2
    C. Neurocytic rosettes with synaptophysin positive neuropil cores. Neurocytic rosettes with synaptophysin positive neuropil cores are the histologic hallmark of rosette forming glioneuronal tumors (RGNTs). Answer A is incorrect because RGNTs are generally demarcated with occasional limited infiltration of the surrounding parenchyma. Answer B is incorrect because RGNTs occur throughout the CNS with predilection to posterior fossa. Answer D is incorrect because papillary architecture is not a feature of RGNTs.

    Comment Here

    Reference: Rosette forming glioneuronal tumor

    SHH activated
    Definition / general
    • An embryonal tumor of the cerebellum with molecular evidence of SHH pathway activation (2016 WHO definition)
    Essential features
    • WHO grade 4 tumors
    • Medulloblastomas which fall into the desmoplastic / nodular or medulloblastoma with extensive nodularity histologic categories almost invariably demonstrate SHH activation
    • Typically occur in children under 4 years of age, adolescents or young adults
    • Those with concurrent TP53 mutation have much worse clinical outcomes and often display large cell / anaplastic histology
    • SHH activation can be determined by molecular studies demonstrating mutations in this pathway, genome expression analysis, methylation classification or by immunohistochemistry demonstrating GAB1 expression
    ICD coding
    • ICD-10: C71.6 - malignant neoplasm of the cerebellum
    Epidemiology
    Sites
    • Cerebellum; can involve both the hemispheres or vermis with site of origin possibly age dependent, with 1 study showing hemispheric tumors are more common in older children and young adults and those centered in the vermis more often occurring in infants (Acta Neuropathol 2014;127:931)
    Pathophysiology
    Etiology
    • High proportion of TP53 mutant cases contain germline TP53 mutations (Li-Fraumeni syndrome) suggestive of a causal role (J Clin Oncol 2013;31:2927)
    Clinical features
    Diagnosis
    • Patients often present with signs and symptoms of increased cranial pressure (headaches, nausea, vomiting) secondary to obstruction of cerebrospinal fluid (CSF) flow from a large mass impinging on the fourth ventricle
    • Signs of cerebellar ataxia are common
    • Cases presenting with CSF dissemination may show more specific neurologic signs
    • Imaging of the head (CT, MRI) will show a large mass in the lateral hemisphere or involving midline structures
    • Diagnosis determined by biopsy or resection, with ancillary studies necessary for medulloblastoma molecular subtyping (immunohistochemistry or methylation profiling or sequencing)
    • References: Nature 2010;468:1095, Pediatr Blood Cancer 2013;60:1408
    Radiology description
    Radiology images

    Contributed by John DeWitt, M.D., Ph.D.
    CT scan with contrast

    CT scan with contrast

    Coronal T1 postcontrast

    Coronal T1 postcontrast

    Sagittal T1 postcontrast

    Sagittal T1 postcontrast

    Axial T1 postcontrast

    Axial T1 postcontrast

    Prognostic factors
    • TP53 mutated tumors typically have a much worse outcome
    • 5 year survival of 81% in TP53 wildtype tumors versus 41% in those harboring TP53 mutation (J Clin Oncol 2013;31:2927)
    • Presentation with CSF spread is poor prognostic factor
    • Histologic subtype can be important in predicting prognosis with those with desmoplastic histology (desmoplastic / nodular and medulloblastoma with extensive nodularity) having a relatively good prognosis versus those with large cell / anaplastic morphology performing poorly
    • Tumors with alterations in SMO or PTCH1 are sensitive to SMO inhibitors, while those with downstream alterations (SUFU, GLI2, MYCN) are refractory (Cancer Cell 2014;25:393, J Clin Oncol 2015;33:2646)
    Case reports
    Treatment
    • SMO inhibitors can be used in SHH activated medulloblastomas that are susceptible (those harboring SMO or PTCH1 mutations) (Cancer Cell 2014;25:393, J Clin Oncol 2015;33:2646)
    • Standard treatment includes surgery, radiation, and chemotherapy
    • Whole cranial - spinal radiation required in cases with CSF dissemination or drop metastasis
    Gross description
    • In general, medulloblastomas present grossly as pink to gray, soft and friable masses arising in cerebellar hemisphere or vermis (with expansion and filling of the fourth ventricle)
    • SHH activated medulloblastomas can be more firm and nodular due to increased desmoplastic stroma in many SHH activated tumors
    • Areas of gross necrosis may be seen but tend to be less than non-SHH activated tumors
    • In disseminated disease, nodules may be grossly evident throughout areas of CSF flow along the neuroaxis
    Frozen section description
    • On frozen section, a dominant population of undifferentiated cells with a high N/C ratio and frequent mitotic figures is often seen
    • In cases with desmoplastic / nodular or extensive nodularity histology, these features may be seen at frozen section as well
    Frozen section images

    Contributed by John DeWitt, M.D., Ph.D.
    Pleomorphic undifferentiated cells

    Pleomorphic undifferentiated cells

    Microscopic (histologic) description
    • Medulloblastomas in general have 4 distinct histologic patterns: classical, desmoplastic / nodular, medulloblastoma with extensive nodularity and large cell / anaplastic
    • SHH activated medulloblastomas can present with any of these histologic phenotypes; however, desmoplastic / nodular and extensive nodularity histologic patterns invariably fall into this molecular subtype
    • Classic medulloblastoma:
      • Small blue round cell tumor
      • Syncytial arrangement of densely packed undifferentiated cells (embryonal cells)
      • Mitosis with apoptotic bodies
      • Homer Wright rosettes
    • Desmoplastic / nodular medulloblastoma:
      • Densely packed, undifferentiated cells with hyperchromatic and pleomorphic nuclei, which produce dense intercellular reticulin fiber network with nodular reticulin free zones
    • Medulloblastoma with extensive nodularity:
      • Expanded lobular architecture as reticulin free nodular zones are enlarged and rich in neuropil-like tissue
    • Large cell / anaplastic medulloblastoma:
      • Anaplasia with marked nuclear pleomorphism, high mitotic count and apoptotic counts
      • Nuclear molding and cell wrapping
    Microscopic (histologic) images

    Contributed by John DeWitt, M.D., Ph.D.
    Sheets of undifferentiated cells

    Sheets of undifferentiated cells

    Vague nodular formation

    Vague nodular formation

    Vague nodular formation with reticulin

    Vague nodular formation with reticulin

    Neuron specific enolase (NSE)

    Neuron specific enolase (NSE)


    Ki67

    Ki67

    Synaptophysin

    Synaptophysin

    Beta catenin

    Beta catenin

    INI1

    INI1



    Contributed by Meaghan Morris, M.D., Ph.D.
    Anaplasia in medulloblastoma

    Anaplasia in medulloblastoma

    Virtual slides

    Images hosted on other servers
    Medulloblastoma, desmoplastic / nodular histologic subtype

    Medulloblastoma,
    desmoplastic / nodular
    histologic subtype

    Positive stains
    • Synaptophysin: positive at least focally in the majority of medulloblastomas
    • NeuN: positivity may be seen in nodules of neurocytic differentiation, particularly in cases with nodular / desmoplastic or extensive nodularity histology
    • p53: strong pathologic p53 positivity may be seen in some cases and is often associated with anaplasia; often correlated with underlying somatic or germline (Li-Fraumeni) TP53 mutation (J Clin Oncol 2010;28:1345, Cell 2012;148:59)
    • Ki67: many positive cells will be seen due to the highly proliferative nature of the tumor; in desmoplastic / nodular histology, Ki67 is higher in the internodular areas and lower in the pale islands
    • GAB1: positive in SHH activated medulloblastoma
    • YAP1: positive in SHH and WNT activated medulloblastoma
    • SMARCB1 / INI1 and SMARCA4 / BRG1: expression is maintained; loss of expression of either of these markers would be consistent with a diagnosis of atypical teratoid / rhabdoid tumor and not medulloblastoma
    • Beta catenin: should show cytoplasmic expression; nuclear expression would be consistent with medulloblastoma, WNT activated
    Negative stains
    • GFAP: may show focal positivity, which may represent entrapped astrocytes
    • Keratins
    Molecular / cytogenetics description
    • By definition, molecular studies will show activating mutations in the SHH signaling pathway, with a recent study identifying mutations in known pathway genes in 87% of cases (Cancer Cell 2014;25:393)
    • Associated with high level amplifications in other genes, including MYCL, GLI2, PPM1D, YAP1 and MDM4 (Nature 2012;488:49)
    • Homozygous deletions of PTCH1 and PTEN appear to occur overwhelmingly in the SHH activated subtype of medulloblastoma compared to other groups (Nature 2012;488:49)
    • Point mutations in TP53 are associated with chromothripsis and a very poor outcome (Cell 2012;148:59, J Clin Oncol 2013;31:2927)
    Sample pathology report
    • Brain, cerebellum, resection:
      • Medulloblastoma, classical type, SHH activated and TP53 wildtype, WHO grade 4 (see comment)
      • Comment:
        • Integrated diagnosis: medulloblastoma, classical type, SHH activated and TP53 wildtype
        • Histological diagnosis: medulloblastoma, classical type
        • WHO (histological) grade: 4 of 4
        • Molecular information:
          • P53: scattered positive cells (immunohistochemistry consistent with wildtype)
          • INI1: retained (immunohistochemistry consistent with wildtype)
          • Beta catenin: diffuse cytoplasmic staining (immunohistochemistry consistent with nonactivated)
          • YAP1: positive (immunohistochemistry)
          • GAB1: positive (immunohistochemistry)
        • Histological features in the tumor include high cell density, hyperchromasia and mitoses. Prominent anaplastic features, such as pleomorphism, prominent nucleoli or cell wrapping, are not appreciated. Desmoplastic or nodular areas are not present. Overall, the histologic and molecular characteristics are consistent with the classical histologic subtype and the SHH activated molecular group.
    Differential diagnosis
    • Other molecular subtypes of medulloblastoma:
      • WNT activated:
        • Will show strong nuclear positivity with beta catenin staining
      • Group 3 / 4:
        • Will not subtype into WNT activated or SHH activated by immunohistochemistry
        • Further expression profiling or methylation profiling can identify these specific subtypes
    • Atypical teratoid / rhabdoid tumor:
      • Will show loss of INI1 staining by immunohistochemistry
    • Other embryonal tumors:
      • CNS neuroblastoma:
        • Embryonal tumor with multilayered rosettes
      • C19MC altered:
        • These tumors will typically be found in the supratentorial region and will lack histologic and molecular features of medulloblastoma
    Board review style question #1

    A histologic section of a tumor resected from the posterior fossa of a young man is shown. What is the most likely diagnosis?

    1. Metastatic lung adenocarcinoma
    2. Medulloblastoma, desmoplastic / nodular, SHH activated
    3. Choroid plexus papilloma
    4. Pilocytic astrocytoma
    5. Hemangioblastoma
    Board review style answer #1
    B. Medulloblastoma, desmoplastic / nodular, SHH activated

    This is the best answer as the image shows sheets of undifferentiated cells consistent with medulloblastoma. Metastatic lung adenocarcinoma would commonly show gland formation and is unlikely in a young man. Choroid plexus papilloma would show well differentiated cells with a prominent papillary growth pattern. Pilocytic astrocytoma would show prominent glial cells with long thin processes. Hemangioblastoma would show a prominent vascular pattern with interspersed tumor cells and hemorrhage.

    Comment Here

    Reference: SHH activated
    Board review style question #2
    What additional genetic alteration portends a particularly poor prognosis in medulloblastoma, SHH activated?

    1. TP53 mutation
    2. EGFR amplification
    3. IDH mutation
    4. TERT promoter mutation
    5. 1p19q codeletion
    Board review style answer #2
    A. TP53 mutation

    This is the best answer as the presence of TP53 mutation in medulloblastoma, SHH activated is associated with a poor prognosis. EGFR amplification is often present in glioblastoma, IDH wildtype. IDH mutation is associated with improved prognosis in infiltrating gliomas. TERT promoter mutation is seen in oligodendroglioma, IDH mutant, 1p19q codeleted and often present in glioblastoma, IDH wildtype. 1p19q codeletion is seen in oligodendroglioma, IDH mutant, 1p19q codeleted.

    Comment Here

    Reference: SHH activated

    Schwannoma
    Definition / general
    • Schwannoma is a benign nerve sheath tumor composed of differentiated neoplastic Schwann cells
    • Classified as CNS WHO grade 1
    Essential features
    • CNS primary tumor mostly occurring in the spine / spinal nerve roots (intradural, extramedullary) and vestibular division of the eighth cranial nerve within the cerebellopontine angle
    • Multiple tumors in patients with neurofibromatosis type 2 (NF2, bilateral vestibular schwannomas) or schwannomatosis
    • Excellent prognosis with nearly absent recurrence after gross total resection
    • Malignant transformation is exceptionally rare
    • Biphasic, encapsulated tumor composed of compact areas with spindle cells (Antoni A) and palisading nuclei (Verocay bodies) and loose microcystic areas with lipid laden macrophages (Antoni B)
    Terminology
    • Neurilemmoma (not recommended)
    • Acoustic neuroma (not recommended)
    ICD coding
    • ICD-O: 9560/0 - schwannoma, NOS
    • ICD-11: 2A02.3 & XH98Z3 - benign neoplasm of cranial nerves & schwannoma, NOS
    Epidemiology
    Sites
    • Most common sites in the CNS:
      • Spine, where it is typically intradural and extramedullary
      • Vestibular division of the eighth cranial nerve (Neuro Oncol 2022;24:v1)
    Pathophysiology
    • NF2 inactivating mutation (frameshift and nonsense) and loss of the remaining wild type allele on chromosome 22, in 50 - 75% of sporadic schwannomas (J Neurosurg 2018;128:911)
    • Germline pathogenic NF2 mutation in schwannomas associated with neurofibromatosis type 2 (Genet Med 2022;24:1967)
    • Germline mutations of either SMARCB1 or LZTR1 on chromosome 22, followed by a somatic mutation in NF2 along with the deletion of the other chromosome 22 in schwannomas associated with schwannomatosis (Neurology 2015;84:141, Clin Genet 2020;97:376, Genet Med 2022;24:1967)
    Etiology
    Clinical features
    • Symptoms depend on the anatomical site of the tumor:
      • Tinnitus, unilateral sensorineural hearing loss, vertigo or disequilibrium in patients with vestibular schwannoma (J Neurosurg 2022 Dec 2 [Epub ahead of print])
      • Sensory or motor symptoms in patients with spinal schwannoma
    Diagnosis
    • Based on imaging (CT, MRI) / biopsy / resection specimen
    Radiology description
    • Well circumscribed masses that displace adjacent structures without direct invasion
    • On MRI, it may be
    Radiology images

    Contributed by Valeria Barresi, M.D., Ph.D.

    T1 weighted MRI

    FLAIR MRI

    T2 weighted MRI

    Prognostic factors
    Case reports
    Treatment
    • Observation, radiosurgery, surgery or surgery followed by radiosurgery, depending on symptoms, tumor size and association with neurofibromatosis type 2 (Neurooncol Pract 2016;3:281)
    • Bevacizumab is considered as an option in patients with neurofibromatosis type 2 (Neuro Oncol 2020;22:31)
    Gross description
    Gross images

    Images hosted on other servers:

    Surgically resected schwannoma

    Acoustic schwannoma

    Fish flesh tan cut surface

    Frozen section description
    • Admixture of compact and loose areas, Verocay bodies and wavy / spindled nuclei with pointed ends are useful diagnostic clues in the differential diagnosis versus other spindle cell tumors (e.g., meningioma, ependymoma, sarcoma, solitary fibrous tumor) (Arch Pathol Lab Med 2007;131:1532)
    • Additional features: hyalinized blood vessels, chronic inflammatory infiltrates along capsule, hemosiderin
    • Additional clues:
      • Differential diagnosis versus ependymoma: absence of fibrillary background and perivascular pseudorosettes
      • Differential diagnosis versus meningioma: lack of psammoma bodies, whorls or epithelioid syncytia
      • Differential diagnosis versus sarcoma: in malignant sarcomas, mitoses are more commonly encountered, nuclear atypia is more uniformly widespread and necrosis may be present (Arch Pathol Lab Med 2007;131:1532)
      • Differential diagnosis versus solitary fibrous tumor: lack of staghorn vessels
    Intraoperative frozen / smear cytology images

    Contributed by Valeria Barresi, M.D., Ph.D.

    Alternating compact and loose areas

    Compact areas composed of spindle cells

    Cohesive clusters with sharp borders

    Spindle cell nuclei in fibrillary matrix

    Microscopic (histologic) description
    • Spindle cell tumo (usually biphasic) and composed of compact hypercellular areas (Antoni A tissue) showing nuclear palisading (Verocay bodies) and loose microcystic areas (Antoni B tissue) with collection of lipid laden histiocytes and thick walled hyalinized blood vessels (Surg Neurol Int 2020;11:454)
    • Lymphoid aggregates maybe seen peripherally or in a subcapsular distribution
    • Mitoses or focal degenerative atypia may be present
    • Ancient subtype: characterized by scattered atypical or bizarre nuclei
    • Cellular subtype: composed exclusively or predominantly by Antoni A tissue and devoid of Verocay bodies
    • Epithelioid subtype: characterized by epithelioid cells with amphophilic or eosinophilic cytoplasm and uniform round nuclei with inconspicuous nucleoli
    Microscopic (histologic) images

    Contributed by Valeria Barresi, M.D., Ph.D.

    Compact and loose areas

    Atypical nuclei

    Lipid laden histiocytes

    Verocay bodies

    S100

    SOX10

    Cytology description
    Positive stains
    Molecular / cytogenetics description
    Sample pathology report
    • Spine, tumor, resection:
      • Schwannoma, CNS WHO grade 1 (see comment)
      • Comment: Spindle cell tumor showing loose areas with lipid laden macrophages and compact areas with nuclear palisading.
    Differential diagnosis
    Board review style question #1

    An intradural, extramedullary mass is found in the spine. Histological examination shows a tumor with compact areas composed of spindle cells alternating with loose areas with collection of lipid laden macrophages. What is the most likely diagnosis?

    1. Ependymoma
    2. Meningioma, fibrous subtype
    3. Schwannoma
    4. Solitary fibrous tumor
    Board review style answer #1
    C. Schwannoma. Fibrous meningioma may localize in the spine but it does not feature alternating compact and loose areas or lipid laden macrophages. Solitary fibrous tumor shows typical staghorn vessels, collagen fibers and neoplastic cells with oval nuclei. In spinal ependymoma, tumor cells have processes forming fibrillary areas around tumor vessels (pseudovascular rosettes) and they often dispose to form rosettes.

    Comment Here

    Reference: Schwannoma
    Board review style question #2
    An extra-axial mass is found in the cerebellopontine angle. Histological examination shows a tumor with compact areas composed of spindle cells with nuclear palisading and scattered atypical nuclei and loose areas. No mitoses or necrosis are seen. At immunohistochemistry, tumor cells are extensively positive for SOX10 and S100 and negative for EMA, STAT6, GFAP. Which is the most likely diagnosis?

    1. Malignant peripheral nerve sheath tumor
    2. Meningioma, fibrous subtype
    3. Schwannoma, ancient subtype
    4. Solitary fibrous tumor
    Board review style answer #2
    C. Schwannoma, ancient subtype. Schwannoma is typically SOX10 and S100 positive and EMA, STAT6 and GFAP negative. Ancient subtype shows scattered atypical nuclei. Meningioma, fibrous subtype may be negative for EMA but it is negative for SOX10 or S100 staining. Solitary fibrous tumor is STAT6 positive and negative for SOX10. Malignant peripheral nerve sheath tumor is negative or has only patchy positivity for SOX10 and S100 and it shows brisk mitotic activity and mitoses.

    Comment Here

    Reference: Schwannoma

    Simple cyst
    Definition / general
    Case reports
    Microscopic (histologic) description
    • Wall lined by gliosis, Rosenthal fibers present, no epithelial lining
    Positive stains

    Solitary fibrous tumor
    Definition / general
    • Solitary fibrous tumors (SFTs) of the CNS are spindle cell neoplasms with a wide range of histopathologic appearances and a pathognomonic NAB2::STAT6 fusion mutation with a characteristic immunophenotype
    Essential features
    • Spindle cell neoplasm that ranges in stromal collagenization and cellularity, often with a patternless pattern or dilated staghorn vessels
    • Most common in the dura (especially the tentorium) but very rare compared to other primary meningeal tumors (Acta Neuropathol 2019;137:307)
    • Pathognomonic NAB2::STAT6 fusion mutation that separates it from other tumors with similar microscopic appearances (Acta Neuropathol 2019;137:307)
    • Positive IHC includes STAT6 (nuclear), CD34 and CD99
    • Good prognosis but metastasis is possible, even with low grade tumors (Acta Neuropathol 2019;137:307)
    Terminology
    • Recent changes to the CNS WHO classification system have made solitary fibrous tumor the sole name for this entity (Neuro Oncol 2021;23:1231)
    • Formerly called solitary fibrous tumor / hemangiopericytoma (SFT / HPC) and angioblastic meningioma
    • Hemangiopericytoma has been removed from the most recent WHO CNS tumor classification
    ICD coding
    • ICD-O: 8815/1 - solitary fibrous tumor
    • ICD-10 codes no longer listed by the WHO
    • ICD-11: 2F7C & XH7E62 - neoplasms of uncertain behavior of connective or other soft tissue & solitary fibrous tumor, NOS
    • ICD-11: 2B5Y & XH1HP3 - other specified malignant mesenchymal neoplasms & solitary fibrous tumor, malignant
    Epidemiology
    Sites
    • Affects the dura most frequently (mainly found in the tentorium)
    • Other sites include falx, other dural sites, skull base, cerebellum and pineal gland
    Pathophysiology
    • Pathognomonic NAB2::STAT6 fusion mutation on 12q13
    • Exact pathogenesis of this neoplasm has not been fully elucidated apart from its spindle cell / fibroblastic nature
    Etiology
    • No known familial association
    Diagrams / tables

    Images hosted on other servers:

    Intracranial SFT vascular permeability

    Clinical features
    • Patients most commonly present with symptoms related to mass effect
    Diagnosis
    • Patients are given a preliminary, often nonspecific diagnosis from CNS imaging
    • Found either during a primary workup or incidentally as part of a separate workup
    • Typically made by (immuno)histologic examination but can be supplemented with molecular testing for the NAB2::STAT6 mutation
    Laboratory
    • Minority of cases present with hypoglycemia due to IGF1 production but this is nonspecific (Hum Pathol 1996;27:858)
    Radiology description
    • Common: dural tail sign (nonspecific) and clear demarcation from surrounding tissue
    • Some cases will show a lack of obvious demarcation
    • Isodense on noncontrast T1 weighted images but bright on T1 with contrast and T2 weighted images
    • Meningiomas may also show a dural tail sign and similar enhancement patterns but are more likely to show intratumoral calcification and spoked wheel vessels (Neuroradiology 2021;63:1215)
    Radiology images

    Images hosted on other servers:

    MRI of spinal SFT

    MRI of parieto-occipital SFT

    MRI of expansive SFT

    PET / CT after SFT resection


    Vividly enhancing mass, various images

    Extra-axial mass, various images

    Tumor mass in the left cerebellum

    Resection cavity with residual tumor tissue

    Prognostic factors
    Case reports
    Treatment
    • All tumors are treated with total resection
    • Higher grade tumor treatment is often supplemented with adjuvant radiation
    • Patients may be monitored for many years and even decades to evaluate for recurrence and metastasis
    Clinical images

    Images hosted on other servers:

    Spinal SFT during resection

    Gross description
    • Discrete mass, often contiguous with dura, white to brown
    • Gross appearance varies based on the relative amount of stromal collagen and tumor cellularity
    • Some tumors can lose their dural attachment and infiltrate surrounding tissue (Ann Diagn Pathol 2003;7:169)
    Gross images

    Images hosted on other servers:

    SFT metastatic to liver

    Frozen section description
    • Characteristic feature: spindled cells without a single / discrete architecture (patternless pattern)
    • Enlarged vascular spaces (staghorn vessels) can suggest SFT
    • Typical frozen diagnosis: spindle cell neoplasm, with differential to include SFT, meningioma and possibly other (schwannoma, etc.) depending on location
    Intraoperative frozen / smear cytology images

    Contributed by Brenndan Crumley, M.D., M.P.H. and Katherine Schwetye, M.D., Ph.D.
    Typical SFT

    Typical findings

    Spindle cells

    Spindle cells

    Spindled neoplastic cell aggregate

    Spindled neoplastic cell aggregate



    Images hosted on other servers:

    Squash prep

    Microscopic (histologic) description
    • Essential: patternless pattern of spindle cells, often solid but can also have papillary and other architectures, arranged around branching hyalinized vessels
    • Varying amounts of stromal collagen and overall cellularity
    • Nuclei are bland, without features seen in mimicking neoplasms
    • WHO grading criteria:
      • < 5 mitoses/10 high power fields (HPF) = grade 1
      • ≥ 5 mitoses/10 HPF = grade 2
      • ≥ 5 mitoses/10 HPF with necrosis = grade 3
    • Often has staghorn vasculature
    Microscopic (histologic) images

    Contributed by Brenndan Crumley, M.D., M.P.H. and Katherine Schwetye, M.D., Ph.D.
    Hypercellular neoplasm, patternless pattern

    Hypercellular neoplasm, patternless pattern

    Neoplasm, haphazard spindle cells

    Neoplasm, haphazard spindle cells

    Hypercellularity and staghorn vessels

    Hypercellularity and staghorn vessels

    Characteristic CD34 positivity

    CD34 positivity can be focal

    STAT6 nuclear positivity

    STAT6 nuclear positivity

    Virtual slides

    Images hosted on other servers:

    Fibrous dural mass

    Occipital dural mass

    Occipital dural mass, CD34

    Occipital dural mass, reticulin

    Cytology description
    • Lower grade tumors will show the characteristic patternless pattern but there is not a standardized list of cytology criteria for SFT diagnosis (J Pathol Transl Med 2019;53:192)
    • Higher grade tumors can show sheets of monotonous cells with heterochromatic nuclei, often with rhabdoid cells and eosinophilic cytoplasm (J Pathol Transl Med 2019;53:192)
    Positive stains
    Negative stains
    Molecular / cytogenetics description
    • NAB2::STAT6 fusion mutation can be detected by sequencing, RT PCR or a proximity ligation assay
    • NAB2 and STAT6 are close to each other on chromosome 12q, which can make genetic analysis difficult and the WHO considers a positive STAT6 immunostain to be sufficient for diagnosis in most cases (Nat Genet 2013;45:131)
    Molecular / cytogenetics images

    Images hosted on other servers:

    NAB2::STAT6 mutation analysis results

    NAB2::STAT6 RT PCR and sequencing

    Videos

    General overview of solitary fibrous tumors (not specific to CNS tumors) by Dr. Jerad Gardner

    Sample pathology report
    • Brain, tentorium, resection:
      • Solitary fibrous tumor, CNS WHO grade 1 (see comment)
      • Comment: Sections show a neoplasm composed of bland spindle cells within a heavily collagenized stroma, with 3 mitoses in 10 high power fields. A panel of immunohistochemical stains is strongly positive for CD34 and STAT6 (nuclear), with focal positivity for SMA and is negative for EMA and SSTR2A. These findings are consistent with a solitary fibrous tumor.
    Differential diagnosis
    Board review style question #1

    This STAT6 stained slide is taken from the excision of an intracranial neoplasm. Which of the following stains should be negative in this neoplasm?

    1. ALDH1
    2. CD34
    3. SMA
    4. SSTR2A
    Board review style answer #1
    D. SSTR2A. A is incorrect because ALDH1 stains are often positive in solitary fibrous tumors, with ALDH1 acting as a CD34 surrogate. B is incorrect because CD34 stains are positive in the vast majority of solitary fibrous tumors, with loss of expression seen in some higher grade tumors. C is incorrect because SMA can be expressed on the periphery of solitary fibrous tumors. SSTR2A is correct because this marker's positive staining in some meningiomas and complete lack of staining in solitary fibrous tumors can help distinguish between the two diagnostic entities.

    Comment Here

    Reference: Solitary fibrous tumor
    Board review style question #2

    What is the most common location of the extra-axial, STAT6 positive CNS neoplasm seen in this image?

    1. Cerebellum
    2. Pineal gland
    3. Spinal cord
    4. Tentorium
    Board review style answer #2
    D. Tentorium. A, B and C are incorrect because while solitary fibrous tumors can be found in the cerebellum, pineal gland and spinal cord, it is not their most common location. D is correct because solitary fibrous tumors are most common in the dura, and the dural site they are most common in is the tentorium.

    Comment Here

    Reference: Solitary fibrous tumor

    Subependymal giant cell astrocytoma
    Definition / general
    • Benign, slowly growing tumor typically arising in wall of lateral ventricles and composed of large ganglioid astrocytes
    • Usually associated with tuberous sclerosis, an autosomal dominant syndrome due to mutations in TSC1 gene on #9q34 (hamartin protein) and TSC2 gene on #16p13.3 (tuberin protein)
    • WHO grade 1
    Diagnosis

    Major features
    • Facial angiofibromas or forehead plaque
    • Nontraumatic ungual or periungual fibroma
    • Hypomelanotic macules (more than 3)
    • Shagreen patch (connective tissue nevus)
    • Multiple retinal nodular hamartomas
    • Cortical tuber
    • Subependymal nodule
    • Subependymal giant cell astrocytoma
    • Cardiac rhabdomyoma, single or multiple
    • Lymphangiomatosis
    • Renal angiomyolipoma

    Minor features
    • Multiple randomly distributed pits in dental enamel
    • Hamartomatous rectal polyps
    • Bone cysts
    • Cerebral white matter migration lines
    • Gingival fibromas
    • Nonrenal hamartomas
    • Retinal achromic patch
    • "Confetti" skin lesions
    • Multiple renal cysts

    • Definite TSC: either 2 major features or 1 major feature with 2 minor features
    • Probable TSC: 1 major feature and 1 minor feature
    • Possible TSC: either 1 major feature or 2 or more minor features

    • Tuberous sclerosis also associated with pulmonary and uterine lymphangiomyomatosis, renal angiomyolipoma, cardiac rhabdomyoma
    • SEGA present in 6% of tuberous sclerosis patients
    Radiology images

    Images hosted on other servers:

    Enhancing mass in the third ventricle

    Case reports
    • 20 year old woman with solitary subependymal giant cell astrocytoma and mutation of TSC2 gene in tumor but not in somatic cells (J Mol Diagn 2005;7:544)
    Gross description
    • Circumscribed, often calcified
    Microscopic (histologic) description
    • Composed mainly of large polygonal to elongate cells resembling astrocytes or ganglion cells with abundant, finely granular eosinophilic cytoplasm, bright pink cellular processes, large round / oval nuclei, prominent nucleoli
    • Perivascular pseudorosette formation is common
    • Infiltration of mast cells and lymphocytes is common
    • No Nissl substance in cytoplasm
    • Presence of mitoses, vascular proliferation or necrosis does NOT indicate anaplastic progression
    Positive stains
    • Mixed glioneuronal phenotype
    • GFAP+, S100+, also neurofilament proteins, neuronal associated class III β-tubulin
    Negative stains
    • HMB45 (unlike other tuberous sclerosis related lesions)
    Differential diagnosis

    Subependymoma
    Definition / general
    • Glioma characterized by the clustering of uniform to mildly pleomorphic tumor cell nuclei in an abundant fibrillary matrix prone to microcystic change
    • CNS WHO grade 1
    Essential features
    • Slow growing tumors that tend to occur within the ventricles of middle aged and elderly adults (Brain Pathol 2008;18:469)
    • Histologically characterized by clustered nuclei in an abundant fibrillary background
    • Benign biological behavior with excellent prognosis (J Neurosurg 2021;136:736)
    Terminology
    ICD coding
    • ICD-O: 9383/1 - subependymoma
    • ICD-11: 2A00.0Y & XH8FZ9 - other specified gliomas of brain & subependymoma
    Epidemiology
    Sites
    Pathophysiology
    • Embryological origin of these tumors remains uncertain; possible precursors include subependymal glia, astrocytes of the subependymal plate, ependymal cells and a mixture of astrocytes and ependymal cells (Brain Pathol 2008;18:469, J Neurosurg 2021;136:736, Arch Pathol Lab Med 1999;123:306)
    • Specific mechanisms by which the documented chromosomal or genetic abnormalities contribute to tumorigenesis are currently unknown
    Etiology
    Clinical features
    Diagnosis
    • Neuroimaging: MRI and CT (AJR Am J Roentgenol 1995;165:1245)
    • Biopsy
    • WHO essential diagnostic criteria
      • Circumscribed glioma with clustering of tumor cell nuclei within expansive, focally microcystic fibrillary matrix
      • Lack of conspicuous nuclear atypia
      • Absent or minimal mitotic activity
      • For unresolved lesions: DNA methylation profile aligned with subependymoma
    Radiology description
    Radiology images

    Contributed by Eman Abdelzaher, M.D., Ph.D.
    MRI, T2

    MRI, T2

    MRI, FLAIR

    MRI, FLAIR



    Images hosted on other servers:
    Lateral ventricle

    Lateral ventricle

    Spinal cord subependymoma, MRI

    Spinal cord subependymoma, MRI

    Intraparenchymal subependymoma, MRI

    Intraparenchymal subependymoma, MRI

    Fourth ventricle subependymoma

    Fourth ventricle subependymoma

    Prognostic factors
    Case reports
    Treatment
    Clinical images

    Images hosted on other servers:
    Smooth contoured lobulated mass

    Smooth contoured lobulated mass

    Spinal cord subependymoma

    Spinal cord
    subependymoma

    Intraparenchymal subependymoma

    Intraparenchymal
    subependymoma

    Gross description
    Gross images

    Contributed by Eman Abdelzaher, M.D., Ph.D.
    Circumscribed, lobulated mass

    Circumscribed, lobulated mass

    Frozen section description
    • Lobular pattern
    • Clusters of monomorphic nuclei with absent or mild pleomorphism; nuclei appear hyperchromatic and pointy with no prominent nucleoli or mitotic figures (Indian J Neurosurg 2019;8:64, OUHSC: Case 501-2 [Accessed 29 January 2024])
    • Dense, fine, fibrillary background
    • Microcystic formations impart a spongy appearance on low magnification; some bluish mucoid material is identified in some microcysts
    Frozen section images

    Images hosted on other servers:
    Monomorphic nuclei in fibrillary pattern

    Monomorphic nuclei in fibrillary pattern

    Spongy appearance

    Spongy appearance,
    clustering of nuclei,
    hyperchromatic /
    pointy nuclei

    Microscopic (histologic) description
    Microscopic (histologic) images

    Contributed by Eman Abdelzaher, M.D., Ph.D., David Taylor, M.D. and Nazila Azordegan, M.D.
    Nuclear clustering and microcystic change Nuclear clustering and microcystic change

    Nuclear clustering and microcystic change

    Nuclear clustering Nuclear clustering

    Nuclear clustering

    Nuclear clustering

    Nuclear clustering

    Nuclear clustering

    Nuclear clustering


    Microcystic change Microcystic change

    Microcystic change

    Microcystic change and nuclear pleomorphism

    Microcystic change and nuclear pleomorphism

    Uniform nuclei

    Uniform nuclei

    Uniform nuclei Uniform nuclei

    Uniform nuclei


    Lobulated tumor

    Lobulated tumor

    Clustered nuclei in fibrillary background Clustered nuclei in fibrillary background

    Clustered nuclei in fibrillary background

    Dot-like perinuclear EMA Dot-like perinuclear EMA

    Dot-like perinuclear EMA

    GFAP

    GFAP

    Virtual slides

    Images hosted on other servers:
    Subependymoma

    Subependymoma

    Cytology description
    Cytology images

    Images hosted on other servers:
    Cohesive tissue clumps

    Cohesive tissue
    clumps, elongated
    cytoplasmic
    processes

    Positive stains
    Electron microscopy description
    Electron microscopy images

    Images hosted on other servers:
    Ultrastructural findings in subependymoma

    Ultrastructural findings in subependymoma

    Molecular / cytogenetics description
    • Distinct DNA methylation profiles of subependymomas at different anatomical locations (supratentorial, posterior fossa and spinal) are documented
    • Chromosomal copy number variations are infrequent in subependymomas at different anatomical locations
      • Recurrent copy number abnormalities are loss of chromosome 19 (most frequently within posterior fossa subependymomas [79%] and less frequent within supratentorial [50%] and spinal [40%] subependymomas) and partial chromosome 6 loss in spinal and posterior fossa subependymomas (Neuro Oncol 2018;20:1616, Cancer Cell 2015;27:728, Brain Pathol 2008;18:469)
    • TRPS1 and PTPN1 gene mutations have been documented (J Neurosurg 2021;136:736)
    • Posterior fossa subependymomas express KIT at high levels (Cancer Cell 2015;27:728)
    • Brainstem subependymomas can have H3 K27M mutations but this does not carry the rapidly lethal prognosis of diffuse midline gliomas (Hum Pathol 2019;84:262)
    • Do not appear to be associated with NF2 mutations that are found in other ependymal neoplasms (J Neurooncol 2007;85:297)
    Molecular / cytogenetics images

    Images hosted on other servers:
    Key molecular and clinical characteristics

    Key molecular and clinical characteristics

    Methylation profiling

    Methylation profiling

    Copy number variations

    Copy number variations

    Videos

    Subependymoma

    Sample pathology report
    • Lateral ventricular mass lesion, gross total resection:
      • Subependymoma, CNS WHO grade 1
      • Molecular genetics: DNA methylation profile aligned with subependymoma
    Differential diagnosis
    Board review style question #1

    66 year old man presented with a headache. MRI of brain showed a nonenhancing mass lesion projecting into the right lateral ventricle. The mass was surgically excised. The image above shows the characteristic morphology of this lesion. What is the typical microscopic finding seen in this entity?

    1. High mitotic activity
    2. Nuclear clustering
    3. Palisading necrosis
    4. True ependymal rosettes
    Board review style answer #1
    B. Nuclear clustering. Nuclear clustering is a typical microscopic finding of subependymomas. Answer A is incorrect because mitoses are absent or rare in typical subependymomas. Answer C is incorrect because palisading necrosis is not a feature of subependymomas. Answer D is incorrect because true ependymal rosettes are not seen in pure subependymomas.

    Comment Here

    Reference: Subependymoma
    Board review style question #2
    The recently described loss of chromosome 19 in subependymomas is most frequent at which anatomical location?

    1. Intraparenchymal subependymoma
    2. Posterior fossa subependymomas
    3. Spinal subependymomas
    4. Supratentorial subependymomas
    Board review style answer #2
    B. Posterior fossa subependymomas. Loss of chromosome 19 is most frequently encountered within posterior fossa subependymomas (79%). Answers C and D are incorrect because loss of chromosome 19 is less frequent within supratentorial (50%) and spinal (40%) subependymomas. Answer A is incorrect because intraparencymal subependymomas are rare and chromosome 19 loss has not been documented in the studied cases (Int J Surg Pathol 2023;31:69).

    Comment Here

    Reference: Subependymoma

    Synovial cyst
    Definition / general
    • Extradural, commonly lumbar, spinal cysts
    • Associated with degenerative joint disease
    Clinical images

    Images hosted on other servers:

    Lumbar synovial cyst

    Gross description
    • Fibrous wall, mucoid fluid
    Microscopic (histologic) description
    • Lined by fibrous tissue with or without synovial lining

    Syrinx
    Definition / general
    • Fluid filled cavity within spinal cord (syringomyelia) or brain stem (syringobulbia)
    Terminology
    • Hydromyelia: accumulation of cerebrospinal fluid (CSF) causing simple distention of central canal of spinal cord lined by ependymal cells
    • Syringomyelia: accumulation of CSF dissecting into surrounding white matter to form a paracentral cavity, not lined by ependyma
    • Syringohydromyelia: combination of both, seen in most cases
    Etiology
    Radiology images

    Images hosted on other servers:

    Cervical spine MRI


    WHO classification
    Definition / general
    Major updates
    • 2021 WHO classification, 5th edition (WHO CNS5) summary
      • Builds upon the updated fourth edition published in 2016 and the subsequent recommendations of the Consortium to Inform Molecular and Practical Approaches to CNS Tumor Taxonomy (cIMPACT-NOW) (Brain Pathol 2017;27:851, Acta Neuropathol 2017;133:1, Acta Neuropathol 2018;135:481, Acta Neuropathol 2018;135:639, Acta Neuropathol 2018;136:805, Acta Neuropathol 2019;137:683, Acta Neuropathol 2020;139:603, Brain Pathol 2020;30:844, Brain Pathol 2020;30:863)
      • This fifth edition incorporates major changes including further advancing the role of molecular information in the diagnosis of CNS neoplasms, introducing the concept of CNS WHO grade distinct from WHO grade, grading within tumor type, endorsing the use of Arabic numerals for grading instead of Roman numerals and defining new entities based on molecular and histological characteristics (Neuro Oncol 2021;23:1231)
      • Taxonomy maintains a hybrid approach including histological and molecular features with some entities requiring specific molecular changes for a diagnosis (e.g., IDH mutant gliomas), while others display a looser association with molecular signatures that are not necessary for a diagnosis (e.g., pleomorphic xanthoastrocytoma and BRAF alteration); for each tumor type, diagnostic criteria are enumerated and listed as essential or desirable
      • Grading within tumor type and the use of Arabic (rather than Roman numerals) are both introduced for the first time in CNS classification to move closer to the grading approach of non-CNS neoplasms; however, some idiosyncrasies remain based on historical knowledge and practices, in addition to the use of molecular signatures to determine grade in some cases, which further warrants the introduction and use of CNS WHO grade instead of WHO grade
      • Grading within types in gliomas implies the elimination of the term anaplastic, which was previously linked to grade in these tumors
      • Anatomic site modifiers that were previously part of a tumor name are removed to make nomenclature simpler and more consistent (e.g., chordoid glioma of the third ventricle is now simply chordoid glioma)
      • In addition to NOS (not otherwise specified), NEC (not elsewhere classified) is introduced
        • NOS: terminology to be used when molecular information is insufficient, either because testing cannot be fully performed or because the results don't fit within a defined category
        • NEC: terminology to be used when necessary testing has been performed and results are available but the results do not fit perfectly in a defined tumor type
      • Glial, glioneuronal and neuronal tumors are entirely restructured into 6 different families in a scheme that introduces a distinction between adult type and pediatric type tumors
        • Adult type diffuse gliomas
        • Pediatric type diffuse low grade gliomas
        • Pediatric type diffuse high grade gliomas
        • Circumscribed astrocytic gliomas
        • Glioneuronal and neuronal tumors
        • Ependymomas
      • Classification of common adult type diffuse gliomas is drastically simplified to include only 3 types (with grading within tumor for the first 2): astrocytoma, IDH mutant (grade 2, 3 or 4), oligodendroglioma, IDH mutant and 1p / 19q codeleted (grade 2 or 3) and glioblastoma, IDH wild type (grade 4)
        • Consequently, glioblastoma is a term exclusively reserved for IDH wild type tumors and the nomenclature for IDH mutant astrocytomas is astrocytoma, IDH mutant, CNS WHO grade 2, 3 or 4
      • Classification of medulloblastomas remains largely similar to the 2016 classification, maintaining the incorporation of molecularly defined entities that determine prognostic categories, while adding novel subtypes (4 subgroups of SHH tumors and 8 subgroups of non-WNT / non-SHH medulloblastomas)
      • Ependymomas are restructured using a combination of histopathological features, molecular signatures and anatomic site (supratentorial, posterior fossa and spinal)
      • New tumor types have been added, such as diffuse hemispheric glioma, H3 G34 mutant or CNS tumor with BCOR internal tandem duplication
        • Some newly recognized types are considered provisional (e.g., diffuse glioneuronal tumor with oligodendroglioma-like features and nuclear clusters or intracranial mesenchymal tumor, FET::CREB fusion positive) while some lesions that were introduced in 2016 are no longer provisional (e.g., diffuse leptomeningeal glioneuronal tumor)
      • Meningioma is considered a single type with 15 subtypes; grading is significantly changed - while chordoid meningioma and clear cell meningioma are still considered CNS WHO grade 2 based on morphology (pending larger studies), the criteria that define atypical (grade 2) or anaplastic (grade 3) meningiomas should be applied to rhabdoid and papillary meningioma instead of assigning grade 3 based on morphology alone; furthermore, molecular alterations can influence grade, as the presence of TERT promoter mutation or homozygous deletion of CDKN2A/B is considered sufficient for a designation of CNS WHO grade 3 in meningiomas
      • New types of mesenchymal neoplasms are included, such as CIC rearranged sarcoma; hemangiopericytoma is now fully retired and solitary fibrous tumor is used instead, as it aligns with the soft tissue nomenclature (the 3 tiered CNS grading scheme remains)
      • Paragangliomas are understood as involving cells from the autonomic nervous system, therefore, they are now included in the chapter with nerve tumors and are renamed cauda equina neuroendocrine tumor; melanotic schwannoma is a term that is considered inadequate to reflect the unique features of this lesion, therefore this tumor type is now called malignant melanotic nerve sheath tumor
      • Some major changes are introduced to the tumors of the sellar region
        • Adamantinomatous craniopharyngioma and papillary craniopharyngioma are now considered 2 different tumor types (rather than subtypes) given their distinct clinicopathological and molecular characteristics
        • Tumors of the neurohypophysis are now considered to belong to 1 group, as they may represent morphologic variations of the same tumor
        • Pituitary neuroendocrine tumor (PitNET) is a newly introduced term, accompanying pituitary adenoma (pituitary adenoma / pituitary neuroendocrine tumor)
        • A new tumor type, characteristic of infancy and associated with DICER1 alterations, is included (pituitary blastoma)
    WHO (2021)







    Diagrams / tables

    Images hosted on other servers:

    Key diagnostic genes, molecules, pathways

    CNS WHO grades of selected types

    Newly recognized WHO 2021 tumor types

    Grading
    • Histological grading is still used based on morphology in many instances; however, in this classification, the presence of certain molecular alterations is incorporated in the grading algorithm
      • For example, IDH mutant astrocytomas can be designated as grade 4 based on histology (presence of microvascular proliferation or necrosis) or if there is a homozygous deletion CDKN2A/B independent of histological features
      • Similarly, glioblastoma, IDH wild type can now be diagnosed in the absence of the aforementioned microscopic features, if certain molecular alterations are present in a diffuse astrocytic glioma in adults (TERT promoter mutation, EGFR gene amplification or combined gain of entire chromosome 7 and loss of entire chromosome 10 [+7 / -10])
    • In the clinical setting, tumor grade remains a key factor influencing choice of therapy
    Board review style question #1
    Which of the following terms is the only acceptable diagnosis in the 2021 WHO classification of tumors of the central nervous system?

    1. Anaplastic astrocytoma, IDH mutant
    2. Anaplastic astrocytoma, IDH wild type
    3. Glioblastoma, IDH mutant
    4. Glioblastoma, IDH wild type
    Board review style answer #1
    D. Glioblastoma, IDH wild type. The 2021 CNS WHO classification simplifies the diagnosis of diffuse gliomas and reserves the term glioblastoma for IDH wild type tumors. Additionally, the term can now be used for infiltrating astrocytomas that lack the classic histological features of glioblastoma (microvascular proliferation and necrosis) but show certain molecular alterations (TERT promoter mutation, EGFR amplification or combined gain of chromosome 7 with loss of chromosome 10), which in the past could have been designated as anaplastic astrocytoma, IDH wild type. The new scheme also introduces grading within tumor type, so that the term anaplastic is eliminated from the diagnostic line for diffuse gliomas. A CNS WHO grade 3 tumor with IDH mutation and astrocytic identity would be correctly diagnosed as astrocytoma, IDH mutant, CNS WHO grade 3 (and no longer called anaplastic astrocytoma).

    Comment Here

    Reference: WHO classification of CNS tumors

    WHO grading of gliomas
    Definition / general
    • CNS WHO grading of gliomas according to the WHO Classification of Central Nervous System Tumors, 5th edition, published in 2021
    Major updates
    • Standardization with other fifth edition WHO classification systems (Neuro Oncol 2021;23:1231)
      • Switch from Roman to Arabic numeral system
      • The term "type" replaces "entity" and "subtype" replaces "variant"
      • Shift toward within tumor type grading
        • Removal of modifier terms, such as anaplastic
        • IDH mutant astrocytic gliomas were 3 separate entities under the 2016 WHO (diffuse astrocytoma, anaplastic astrocytoma and glioblastoma) but now fall under the single tumor type astrocytoma, IDH mutant, with CNS WHO grade ranging from 2 to 4
        • Oligodendroglioma, IDH mutant and 1p / 19q codeleted, is assigned a CNS WHO grade of 2 or 3
        • Pleomorphic xanthroastrocytoma is assigned a CNS WHO grade of 2 or 3
        • Terms that are no longer recommended include diffuse astrocytoma, anaplastic astrocytoma, glioblastoma IDH mutant, anaplastic oligodendroglioma and anaplastic pleomorphic xanthoastrocytoma
    • To emphasize difference of grading between CNS and non-CNS tumors, the term CNS WHO grade is endorsed
    • Allows designation of not otherwise specified (NOS) or not elsewhere classified (NEC) as modifiers
      • Not otherwise specified (NOS): applied when there is lack of diagnostic information (histological or molecular) necessary to classify glioma as a specific WHO diagnostic entity
      • Not elsewhere classified (NEC): applied when there is sufficient diagnostic information available (histological and molecular) but the results do not allow for the glioma to be classified as a specific WHO diagnostic entity - often ending up more as a descriptive diagnosis
    • New tumor type groupings
      • Adult type diffuse gliomas
      • Pediatric type diffuse low grade gliomas
      • Pediatric type diffuse high grade gliomas
      • Circumscribed astrocytic gliomas
    • New glioma types
      • Diffuse astrocytoma, MYB or MYBL1 altered
      • Polymorphous low grade neuroepithelial tumor of the young
      • Diffuse low grade glioma, MAPK pathway altered
      • Diffuse hemispheric glioma, H3 G34 mutant
      • Diffuse pediatric type high grade glioma, H3 wildtype and IDH wildtype
      • Infant type hemispheric glioma
      • High grade astrocytoma with piloid features
    • Removed glioma types
      • Diffuse astrocytoma, IDH wildtype
      • Anaplastic astrocytoma, IDH wildtype
    • Renamed glioma types
      • Astrocytoma, IDH mutant
        • Eliminates the following previously used terms: diffuse astrocytoma, IDH mutant; anaplastic astrocytoma, IDH mutant; and glioblastoma, IDH mutant
      • Diffuse midline glioma, H3 K27 altered
        • Terminology replaces the term mutant with altered, to include other mechanisms of histone alteration (e.g., EZHIP overexpression)
      • Chordoid glioma
        • Omits "of the third ventricle" anatomical designation
      • Astroblastoma, MN1 altered
        • Adds molecular criterion for diagnosis
    • Changes to existing grading criteria require that either histological or molecular features are present (adult type diffuse gliomas)
      • IDH mutant astrocytoma, CNS WHO grade 4 (Neuro Oncol 2021;23:1231)
        • IDH mutant diffuse astrocytic glioma with CDKN2A / CDKN2B homozygous deletion is assigned a CNS WHO grade 4, even in the absence of microvascular proliferation or necrosis
      • Glioblastoma, IDH wildtype, CNS WHO grade 4 (Acta Neuropathol 2018;136:805)
        • Changes require that either histological or molecular criteria are present in order to obtain diagnosis
        • Requirements: a diffuse astrocytic glioma with any one or more of the following: microvascular proliferation, necrosis, TERT promoter mutation, EGFR amplification, gain of chromosome 7 with concomitant loss of chromosome 10
    WHO (2021)



    Diagrams / tables

    Contributed by P.J. Cimino, M.D., Ph.D.
    Adult type IDH mutant gliomas

    Adult type IDH mutant gliomas

    Adult type IDH wildtype gliomas

    Adult type IDH wildtype gliomas

    Microscopic (histologic) images

    Contributed by P.J. Cimino, M.D., Ph.D.
    Astrocytoma lacking mitoses

    Astrocytoma lacking mitoses

    Increased mitotic activity

    Increased mitotic activity

    Pseudopalisading necrosis

    Pseudopalisading necrosis

    Microvascular proliferation

    Microvascular proliferation

    Molecular / cytogenetics images

    Contributed by P.J. Cimino, M.D., Ph.D.
    Glioblastoma EGFR amplification

    Glioblastoma EGFR amplification

    Glioblastoma copy number plot

    Glioblastoma copy number plot



    Images hosted on other servers:

    Glioblastoma TERT promoter mutation

    Videos

    2021 WHO classification of CNS tumors: update I - gliomas

    Board review style question #1

    This IDH mutant astrocytoma is from a 40 year old woman. There is no microvascular proliferation or necrosis identified. According to the 5th edition of the WHO classification of CNS tumors (2021), a CNS WHO grade 4 should be applied if which of the following genetic alterations is present?

    1. ATRX loss
    2. CDKN2A homozygous deletion
    3. Chromosome 1p / 19q codeletion
    4. EGFR amplification
    5. TERT promoter mutation
    Board review style answer #1
    B. CDKN2A homozygous deletion. The presence of CDKN2A or CDKN2B homozygous deletion in an astrocytoma, IDH mutant, necessitates a CNS WHO grade of 4, even in the absence of histologic features (i.e., lacking microvascular proliferation and spontaneous necrosis).

    Comment Here

    Reference: WHO grading of gliomas

    WHO grading of meningiomas
    Definition / general
    • Family of neoplasms most likely derived from meningothelial cells of the arachnoid mater
    Major updates
    WHO (2021)
    Diagrams / tables

    Contributed by Jared T. Ahrendsen, M.D., Ph.D.

    Criteria for meningioma classification

    Microscopic (histologic) images

    Contributed by Jared T. Ahrendsen, M.D., Ph.D. and Chunyu Cai, M.D., Ph.D.

    Brain invasion

    Necrosis

    Sheeting

    Increased cellularity

    Small cell transformation


    Prominent nucleoli

    Atypical meningioma

    Anaplastic meningioma

    Board review style question #1

    An intracranial neoplasm (histology shown in image) is resected from the lateral convexity of a 65 year old woman. Which of the following molecular alterations, if present, is compatible with a CNS WHO grade 3 designation for this tumor type?

    1. EGFR amplification
    2. Germline NF2 mutation
    3. IDH mutation
    4. Monosomy 22
    5. TERT promotor mutation
    Board review style answer #1
    E. TERT promotor mutation. The neoplasm shown is consistent with meningioma. CNS WHO grade 3 can be applied to meningiomas with either TERT promotor mutation or CDKN2A/B homozygous deletion. EGFR amplification (A) is a feature of IDH wildtype glioblastoma, not meningioma. While patients with germline NF2 mutations (B) (neurofibromatosis type 2) often have multiple meningiomas, this is not sufficient for CNS WHO grade 3 designation. IDH mutations (C) are present in diffusely infiltrating astrocytomas and oligodendrogliomas, not meningiomas. While monosomy 22 (D) is the most common genetic abnormality encountered in meningiomas, it is not sufficient for CNS WHO grade 3 designation.

    Comment Here

    Reference: WHO grading of meningiomas
    Back to top
    Recent CNS & pituitary tumors Pathology books

    Asa: 2020

    Ellison: 2013

    Gill: 2022

    Gray: 2018

    Gupta: 2016

    Husain: 2021

    IARC: 2022

    Kleinschmidt-DeMasters: 2022

    Love: 2015

    Perry: 2017

    Rodriguez: 2016



    Find related Pathology books: GU/adrenal, head & neck/endocrine, muscle and peripheral nerve nontumor, neuropathology, pediatric
    Image 01 Image 02